You are on page 1of 193

1

CHAPTER I
Republic of the Philippines
SUPREME COURT
FIRST DIVISION
G.R. No. 154514. July 28, 2005
WHITE GOLD MARINE SERVICES, INC., Petitioners,
vs.
PIONEER INSURANCE AND SURETY CORPORATION AND THE STEAMSHIP MUTUAL UNDERWRITING
ASSOCIATION (BERMUDA) LTD., Respondents.
DECISION
QUISUMBING, J.:
This petition for review assails the Decision1 dated July 30, 2002 of the Court of Appeals in CA-G.R. SP No. 60144,
affirming the Decision2 dated May 3, 2000 of the Insurance Commission in I.C. Adm. Case No. RD-277. Both
decisions held that there was no violation of the Insurance Code and the respondents do not need license as insurer
and insurance agent/broker.
The facts are undisputed.
White Gold Marine Services, Inc. (White Gold) procured a protection and indemnity coverage for its vessels from
The Steamship Mutual Underwriting Association (Bermuda) Limited (Steamship Mutual) through Pioneer Insurance
and Surety Corporation (Pioneer). Subsequently, White Gold was issued a Certificate of Entry and
Acceptance.3 Pioneer also issued receipts evidencing payments for the coverage. When White Gold failed to fully
pay its accounts, Steamship Mutual refused to renew the coverage.
Steamship Mutual thereafter filed a case against White Gold for collection of sum of money to recover the latters
unpaid balance. White Gold on the other hand, filed a complaint before the Insurance Commission claiming that
Steamship Mutual violated Sections 1864 and 1875 of the Insurance Code, while Pioneer violated Sections
299,63007 and 3018 in relation to Sections 302 and 303, thereof.
The Insurance Commission dismissed the complaint. It said that there was no need for Steamship Mutual to secure
a license because it was not engaged in the insurance business. It explained that Steamship Mutual was a
Protection and Indemnity Club (P & I Club). Likewise, Pioneer need not obtain another license as insurance agent
and/or a broker for Steamship Mutual because Steamship Mutual was not engaged in the insurance business.
Moreover, Pioneer was already licensed, hence, a separate license solely as agent/broker of Steamship Mutual was
already superfluous.
The Court of Appeals affirmed the decision of the Insurance Commissioner. In its decision, the appellate court
distinguished between P & I Clubs vis--vis conventional insurance. The appellate court also held that Pioneer
merely acted as a collection agent of Steamship Mutual.
In this petition, petitioner assigns the following errors allegedly committed by the appellate court,

2
FIRST ASSIGNMENT OF ERROR
THE COURT A QUO ERRED WHEN IT RULED THAT RESPONDENT STEAMSHIP IS NOT DOING BUSINESS IN
THE PHILIPPINES ON THE GROUND THAT IT COURSED . . . ITS TRANSACTIONS THROUGH ITS AGENT
AND/OR BROKER HENCE AS AN INSURER IT NEED NOT SECURE A LICENSE TO ENGAGE IN INSURANCE
BUSINESS IN THE PHILIPPINES.
SECOND ASSIGNMENT OF ERROR
THE COURT A QUO ERRED WHEN IT RULED THAT THE RECORD IS BEREFT OF ANY EVIDENCE THAT
RESPONDENT STEAMSHIP IS ENGAGED IN INSURANCE BUSINESS.
THIRD ASSIGNMENT OF ERROR
THE COURT A QUO ERRED WHEN IT RULED, THAT RESPONDENT PIONEER NEED NOT SECURE A LICENSE
WHEN CONDUCTING ITS AFFAIR AS AN AGENT/BROKER OF RESPONDENT STEAMSHIP.
FOURTH ASSIGNMENT OF ERROR
THE COURT A QUO ERRED IN NOT REVOKING THE LICENSE OF RESPONDENT PIONEER AND [IN NOT
REMOVING] THE OFFICERS AND DIRECTORS OF RESPONDENT PIONEER.9
Simply, the basic issues before us are (1) Is Steamship Mutual, a P & I Club, engaged in the insurance business in
the Philippines? (2) Does Pioneer need a license as an insurance agent/broker for Steamship Mutual?
The parties admit that Steamship Mutual is a P & I Club. Steamship Mutual admits it does not have a license to do
business in the Philippines although Pioneer is its resident agent. This relationship is reflected in the certifications
issued by the Insurance Commission.
Petitioner insists that Steamship Mutual as a P & I Club is engaged in the insurance business. To buttress its
assertion, it cites the definition of a P & I Club in Hyopsung Maritime Co., Ltd. v. Court of Appeals10 as "an
association composed of shipowners in general who band together for the specific purpose of providing insurance
cover on a mutual basis against liabilities incidental to shipowning that the members incur in favor of third parties." It
stresses that as a P & I Club, Steamship Mutuals primary purpose is to solicit and provide protection and indemnity
coverage and for this purpose, it has engaged the services of Pioneer to act as its agent.
Respondents contend that although Steamship Mutual is a P & I Club, it is not engaged in the insurance business in
the Philippines. It is merely an association of vessel owners who have come together to provide mutual protection
against liabilities incidental to shipowning.11 Respondents aver Hyopsung is inapplicable in this case because the
issue in Hyopsung was the jurisdiction of the court over Hyopsung.
Is Steamship Mutual engaged in the insurance business?
Section 2(2) of the Insurance Code enumerates what constitutes "doing an insurance business" or "transacting an
insurance business". These are:
(a) making or proposing to make, as insurer, any insurance contract;

3
(b) making, or proposing to make, as surety, any contract of suretyship as a vocation and not as merely incidental to
any other legitimate business or activity of the surety;
(c) doing any kind of business, including a reinsurance business, specifically recognized as constituting the doing of
an insurance business within the meaning of this Code;
(d) doing or proposing to do any business in substance equivalent to any of the foregoing in a manner designed to
evade the provisions of this Code.
...
The same provision also provides, the fact that no profit is derived from the making of insurance contracts,
agreements or transactions, or that no separate or direct consideration is received therefor, shall not preclude the
existence of an insurance business.12
The test to determine if a contract is an insurance contract or not, depends on the nature of the promise, the act
required to be performed, and the exact nature of the agreement in the light of the occurrence, contingency, or
circumstances under which the performance becomes requisite. It is not by what it is called. 13
Basically, an insurance contract is a contract of indemnity. In it, one undertakes for a consideration to indemnify
another against loss, damage or liability arising from an unknown or contingent event. 14
In particular, a marine insurance undertakes to indemnify the assured against marine losses, such as the losses
incident to a marine adventure.15 Section 9916 of the Insurance Code enumerates the coverage of marine insurance.
Relatedly, a mutual insurance company is a cooperative enterprise where the members are both the insurer and
insured. In it, the members all contribute, by a system of premiums or assessments, to the creation of a fund from
which all losses and liabilities are paid, and where the profits are divided among themselves, in proportion to their
interest.17 Additionally, mutual insurance associations, or clubs, provide three types of coverage, namely, protection
and indemnity, war risks, and defense costs.18
A P & I Club is "a form of insurance against third party liability, where the third party is anyone other than the P & I
Club and the members."19 By definition then, Steamship Mutual as a P & I Club is a mutual insurance association
engaged in the marine insurance business.
The records reveal Steamship Mutual is doing business in the country albeit without the requisite certificate of
authority mandated by Section 18720 of the Insurance Code. It maintains a resident agent in the Philippines to solicit
insurance and to collect payments in its behalf. We note that Steamship Mutual even renewed its P & I Club cover
until it was cancelled due to non-payment of the calls. Thus, to continue doing business here, Steamship Mutual or
through its agent Pioneer, must secure a license from the Insurance Commission.
Since a contract of insurance involves public interest, regulation by the State is necessary. Thus, no insurer or
insurance company is allowed to engage in the insurance business without a license or a certificate of authority from
the Insurance Commission.21
Does Pioneer, as agent/broker of Steamship Mutual, need a special license?
Pioneer is the resident agent of Steamship Mutual as evidenced by the certificate of registration 22 issued by the
Insurance Commission. It has been licensed to do or transact insurance business by virtue of the certificate of

4
authority23 issued by the same agency. However, a Certification from the Commission states that Pioneer does not
have a separate license to be an agent/broker of Steamship Mutual.24
Although Pioneer is already licensed as an insurance company, it needs a separate license to act as insurance
agent for Steamship Mutual. Section 299 of the Insurance Code clearly states:
SEC. 299 . . .
No person shall act as an insurance agent or as an insurance broker in the solicitation or procurement of
applications for insurance, or receive for services in obtaining insurance, any commission or other compensation
from any insurance company doing business in the Philippines or any agent thereof, without first procuring a license
so to act from the Commissioner, which must be renewed annually on the first day of January, or within six months
thereafter. . .
Finally, White Gold seeks revocation of Pioneers certificate of authority and removal of its directors and officers.
Regrettably, we are not the forum for these issues.
WHEREFORE, the petition is PARTIALLY GRANTED. The Decision dated July 30, 2002 of the Court of Appeals
affirming the Decision dated May 3, 2000 of the Insurance Commission is hereby REVERSED AND SET ASIDE.
The Steamship Mutual Underwriting Association (Bermuda) Ltd., and Pioneer Insurance and Surety Corporation are
ORDERED to obtain licenses and to secure proper authorizations to do business as insurer and insurance agent,
respectively. The petitioners prayer for the revocation of Pioneers Certificate of Authority and removal of its
directors and officers, is DENIED. Costs against respondents.
SO ORDERED.

5
Republic of the Philippines
SUPREME COURT
Manila
THIRD DIVISION

G.R. No. 75605 January 22, 1993


RAFAEL (REX) VERENDIA, petitioner,
vs.
COURT OF APPEALS and FIDELITY & SURETY CO. OF THE PHILIPPINES, respondents.
G.R. No. 76399 January 22, 1993
FIDELITY & SURETY CO. OF THE PHILIPPINES, INC., petitioner,
vs.
RAFAEL VERENDIA and THE COURT OF APPEALS, respondents.
B.L. Padilla for petitioner.
Sabino Padilla, Jr. for Fidelity & Surety, Co.

MELO, J.:
The two consolidated cases involved herein stemmed from the issuance by Fidelity and Surety Insurance
Company of the Philippines (Fidelity for short) of its Fire Insurance Policy No. F-18876 effective between
June 23, 1980 and June 23, 1981 covering Rafael (Rex) Verendia's residential building located at Tulip Drive,
Beverly Hills, Antipolo, Rizal in the amount of P385,000.00. Designated as beneficiary was the Monte de
Piedad & Savings Bank. Verendia also insured the same building with two other companies, namely, The
Country Bankers Insurance for P56,000.00 under Policy No. PDB-80-1913 expiring on May 12, 1981, and The
Development Insurance for P400,000.00 under Policy No. F-48867 expiring on June 30, 198l.
While the three fire insurance policies were in force, the insured property was completely destroyed by fire
on the early morning of December 28, 1980. Fidelity was accordingly informed of the loss and despite
demands, refused payment under its policy, thus prompting Verendia to file a complaint with the then Court
of First Instance of Quezon City, praying for payment of P385,000.00, legal interest thereon, plus attorney's
fees and litigation expenses. The complaint was later amended to include Monte de Piedad as an "unwilling
defendant" (P. 16, Record).
Answering the complaint, Fidelity, among other things, averred that the policy was avoided by reason of
over-insurance; that Verendia maliciously represented that the building at the time of the fire was leased
under a contract executed on June 25, 1980 to a certain Roberto Garcia, when actually it was a Marcelo
Garcia who was the lessee.

6
On May 24, 1983, the trial court rendered a decision, per Judge Rodolfo A. Ortiz, ruling in favor of Fidelity. In
sustaining the defenses set up by Fidelity, the trial court ruled that Paragraph 3 of the policy was also
violated by Verendia in that the insured failed to inform Fidelity of his other insurance coverages with
Country Bankers Insurance and Development Insurance.
Verendia appealed to the then Intermediate Appellate Court and in a decision promulgated on March 31,
1986, (CA-G.R. No. CV No. 02895, Coquia, Zosa, Bartolome, and Ejercito (P), JJ.), the appellate court
reversed for the following reasons: (a) there was no misrepresentation concerning the lease for the contract
was signed by Marcelo Garcia in the name of Roberto Garcia; and (b) Paragraph 3 of the policy contract
requiring Verendia to give notice to Fidelity of other contracts of insurance was waived by Fidelity as shown
by its conduct in attempting to settle the claim of Verendia (pp. 32-33, Rollo of G.R. No. 76399).
Fidelity received a copy of the appellate court's decision on April 4, 1986, but instead of directly filing a
motion for reconsideration within 15 days therefrom, Fidelity filed on April 21, 1986, a motion for extension
of 3 days within which to file a motion for reconsideration. The motion for extension was not filed on April
19, 1986 which was the 15th day after receipt of the decision because said 15th day was a Saturday and of
course, the following day was a Sunday (p. 14., Rollo of G.R. No. 75605). The motion for extension was
granted by the appellate court on April 30, 1986 (p. 15. ibid.), but Fidelity had in the meantime filed its
motion for reconsideration on April 24, 1986 (p. 16, ibid.).
Verendia filed a motion to expunge from the record Fidelity's motion for reconsideration on the ground that
the motion for extension was filed out of time because the 15th day from receipt of the decision which fell
on a Saturday was ignored by Fidelity, for indeed, so Verendia contended, the Intermediate Appellate Court
has personnel receiving pleadings even on Saturdays.
The motion to expunge was denied on June 17, 1986 (p. 27, ibid.) and after a motion for reconsideration was
similarly brushed aside on July 22, 1986 (p. 30, ibid .), the petition herein docketed as G.R. No. 75605 was
initiated. Subsequently, or more specifically on October 21, 1986, the appellate court denied Fidelity's
motion for reconsideration and account thereof. Fidelity filed on March 31, 1986, the petition for review
on certiorari now docketed as G.R. No. 76399. The two petitions, inter-related as they are, were consolidated
(p. 54, Rollo of G.R. No. 76399) and thereafter given due course.
Before we can even begin to look into the merits of the main case which is the petition for review
oncertiorari, we must first determine whether the decision of the appellate court may still be reviewed, or
whether the same is beyond further judicial scrutiny. Stated otherwise, before anything else, inquiry must
be made into the issue of whether Fidelity could have legally asked for an extension of the 15-day
reglementary period for appealing or for moving for reconsideration.
As early as 1944, this Court through Justice Ozaeta already pronounced the doctrine that the pendency of a
motion for extension of time to perfect an appeal does not suspend the running of the period sought to be
extended (Garcia vs. Buenaventura 74 Phil. 611 [1944]). To the same effect were the rulings in Gibbs vs. CFI
of Manila (80 Phil. 160 [1948]) Bello vs. Fernando (4 SCRA 138 [1962]), and Joe vs. King(20 SCRA 1120
[1967]).
The above cases notwithstanding and because the Rules of Court do not expressly prohibit the filing of a
motion for extension of time to file a motion for reconsideration in regard to a final order or judgment,
magistrates, including those in the Court of Appeals, held sharply divided opinions on whether the period
for appealing which also includes the period for moving to reconsider may be extended. The matter was not
definitely settled until this Court issued its Resolution in Habaluyas Enterprises, Inc. vs. Japson (142 SCRA
[1986]), declaring that beginning one month from the promulgation of the resolution on May 30, 1986

7
. . . the rule shall be strictly enforced that no motion for extension of time to file a motion for
new trial or reconsideration shall be filed . . . (at p. 212.)
In the instant case, the motion for extension was filed and granted before June 30, 1986, although, of
course, Verendia's motion to expunge the motion for reconsideration was not finally disposed until July 22,
1986, or after the dictum in Habaluyas had taken effect. Seemingly, therefore, the filing of the motion for
extension came before its formal proscription under Habaluyas, for which reason we now turn our attention
to G.R. No. 76399.
Reduced to bare essentials, the issues Fidelity raises therein are: (a) whether or not the contract of lease
submitted by Verendia to support his claim on the fire insurance policy constitutes a false declaration
which would forfeit his benefits under Section 13 of the policy and (b) whether or not, in submitting the
subrogation receipt in evidence, Fidelity had in effect agreed to settle Verendia's claim in the amount stated
in said receipt. 1
Verging on the factual, the issue of the veracity or falsity of the lease contract could have been better
resolved by the appellate court for, in a petition for review on certiorari under Rule 45, the jurisdiction of
this Court is limited to the review of errors of law. The appellate court's findings of fact are, therefore,
conclusive upon this Court except in the following cases: (1) when the conclusion is a finding grounded
entirely on speculation, surmises, or conjectures; (2) when the inference made is manifestly absurd,
mistaken, or impossible; (3) when there is grave abuse of discretion in the appreciation of facts; (4) when
the judgment is premised on a misapprehension of facts; (5) when the findings of fact are conflicting; and
(6) when the Court of Appeals in making its findings went beyond the issues of the case and the same are
contrary to the admissions of both appellant and appellee (Ronquillo v. Court of Appeals, 195 SCRA 433
[1991]). In view of the conflicting findings of the trial court and the appellate court on important issues in
these consolidated cases and it appearing that the appellate court judgment is based on a misapprehension
of facts, this Court shall review the evidence on record.
The contract of lease upon which Verendia relies to support his claim for insurance benefits, was entered
into between him and one Robert Garcia, married to Helen Cawinian, on June 25, 1980 (Exh. "1"), a couple
of days after the effectivity of the insurance policy. When the rented residential building was razed to the
ground on December 28, 1980, it appears that Robert Garcia (or Roberto Garcia) was still within the
premises. However, according to the investigation report prepared by Pat. Eleuterio M. Buenviaje of the
Antipolo police, the building appeared to have "no occupant" and that Mr. Roberto Garcia was "renting on
the otherside (sic) portion of said compound"
(Exh. "E"). These pieces of evidence belie Verendia's uncorroborated testimony that Marcelo Garcia, whom
he considered as the real lessee, was occupying the building when it was burned (TSN, July 27, 1982, p.10).
Robert Garcia disappeared after the fire. It was only on October 9, 1981 that an adjuster was able to locate
him. Robert Garcia then executed an affidavit before the National Intelligence and Security Authority (NISA)
to the effect that he was not the lessee of Verendia's house and that his signature on the contract of lease
was a complete forgery. Thus, on the strength of these facts, the adjuster submitted a report dated
December 4, 1981 recommending the denial of Verendia's claim (Exh. "2").
Ironically, during the trial, Verendia admitted that it was not Robert Garcia who signed the lease contract.
According to Verendia, it was signed by Marcelo Garcia, cousin of Robert, who had been paying the rentals
all the while. Verendia, however, failed to explain why Marcelo had to sign his cousin's name when he in fact
was paying for the rent and why he (Verendia) himself, the lessor, allowed such a ruse. Fidelity's
conclusions on these proven facts appear, therefore, to have sufficient bases; Verendia concocted the lease
contract to deflect responsibility for the fire towards an alleged "lessee", inflated the value of the property

8
by the alleged monthly rental of P6,500 when in fact, the Provincial Assessor of Rizal had assessed the
property's fair market value to be only P40,300.00, insured the same property with two other insurance
companies for a total coverage of around P900,000, and created a dead-end for the adjuster by the
disappearance of Robert Garcia.
Basically a contract of indemnity, an insurance contract is the law between the parties (Pacific Banking
Corporation vs. Court of Appeals 168 SCRA 1 [1988]). Its terms and conditions constitute the measure of the
insurer's liability and compliance therewith is a condition precedent to the insured's right to recovery from
the insurer (Oriental Assurance Corporation vs. Court of Appeals, 200 SCRA 459 [1991], citing Perla
Compania de Seguros, Inc. vs. Court of Appeals, 185 SCRA 741 [1991]). As it is also a contract of adhesion,
an insurance contract should be liberally construed in favor of the insured and strictly against the insurer
company which usually prepares it (Western Guaranty Corporation vs. Court of Appeals, 187 SCRA 652
[1980]).
Considering, however, the foregoing discussion pointing to the fact that Verendia used a false lease
contract to support his claim under Fire Insurance Policy No. F-18876, the terms of the policy should be
strictly construed against the insured. Verendia failed to live by the terms of the policy, specifically Section
13 thereof which is expressed in terms that are clear and unambiguous, that all benefits under the policy
shall be forfeited "If the claim be in any respect fraudulent, or if any false declaration be made or used in
support thereof, or if any fraudulent means or devises are used by the Insured or anyone acting in his
behalf to obtain any benefit under the policy". Verendia, having presented a false declaration to support his
claim for benefits in the form of a fraudulent lease contract, he forfeited all benefits therein by virtue of
Section 13 of the policy in the absence of proof that Fidelity waived such provision (Pacific Banking
Corporation vs. Court of Appeals, supra). Worse yet, by presenting a false lease contract, Verendia,
reprehensibly disregarded the principle that insurance contracts are uberrimae fidae and demand the most
abundant good faith (Velasco vs. Apostol, 173 SCRA 228 [1989]).
There is also no reason to conclude that by submitting the subrogation receipt as evidence in court, Fidelity
bound itself to a "mutual agreement" to settle Verendia's claims in consideration of the amount of
P142,685.77. While the said receipt appears to have been a filled-up form of Fidelity, no representative of
Fidelity had signed it. It is even incomplete as the blank spaces for a witness and his address are not filled
up. More significantly, the same receipt states that Verendia had received the aforesaid amount. However,
that Verendia had not received the amount stated therein, is proven by the fact that Verendia himself filed
the complaint for the full amount of P385,000.00 stated in the policy. It might be that there had been efforts
to settle Verendia's claims, but surely, the subrogation receipt by itself does not prove that a settlement had
been arrived at and enforced. Thus, to interpret Fidelity's presentation of the subrogation receipt in
evidence as indicative of its accession to its "terms" is not only wanting in rational basis but would be
substituting the will of the Court for that of the parties.
WHEREFORE, the petition in G.R. No. 75605 is DISMISSED. The petition in G.R. No. 76399 is GRANTED and
the decision of the then Intermediate Appellate Court under review is REVERSED and SET ASIDE and that
of the trial court is hereby REINSTATED and UPHELD.
SO ORDERED.

10

Republic of the Philippines


SUPREME COURT
Manila
THIRD DIVISION
G.R. No. 112360

July 18, 2000

RIZAL SURETY & INSURANCE COMPANY, petitioner,


vs.
COURT OF APPEALS and TRANSWORLD KNITTING MILLS, INC., respondents.
DECISION
PURISIMA, J.:
At bar is a Petition for Review on Certiorari under Rule 45 of the Rules of Court seeking to annul and set aside the
July 15, 1993 Decision1 and October 22, 1993 Resolution2 of the Court of Appeals3 in CA-G.R. CV NO. 28779, which
modified the Ruling4 of the Regional Trial Court of Pasig, Branch 161, in Civil Case No. 46106.
The antecedent facts that matter are as follows:
On March 13, 1980, Rizal Surety & Insurance Company (Rizal Insurance) issued Fire Insurance Policy No. 45727 in
favor of Transworld Knitting Mills, Inc. (Transworld), initially for One Million (P1,000,000.00) Pesos and eventually
increased to One Million Five Hundred Thousand (P1,500,000.00) Pesos, covering the period from August 14, 1980
to March 13, 1981.
Pertinent portions of subject policy on the buildings insured, and location thereof, read:
"On stocks of finished and/or unfinished products, raw materials and supplies of every kind and description, the
properties of the Insureds and/or held by them in trust, on commission or on joint account with others and/or for
which they (sic) responsible in case of loss whilst contained and/or stored during the currency of this Policy in the
premises occupied by them forming part of the buildings situate (sic) within own Compound at MAGDALO STREET,
BARRIO UGONG, PASIG, METRO MANILA, PHILIPPINES, BLOCK NO. 601.
xxx

xxx

xxx

Said building of four-span lofty one storey in height with mezzanine portions is constructed of reinforced concrete
and hollow blocks and/or concrete under galvanized iron roof and occupied as hosiery mills, garment and lingerie
factory, transistor-stereo assembly plant, offices, warehouse and caretaker's quarters.

11
'Bounds in front partly by one-storey concrete building under galvanized iron roof occupied as canteen and
guardhouse, partly by building of two and partly one storey constructed of concrete below, timber above
undergalvanized iron roof occupied as garage and quarters and partly by open space and/or tracking/ packing,
beyond which is the aforementioned Magdalo Street; on its right and left by driveway, thence open spaces, and at
the rear by open spaces.'"5
The same pieces of property insured with the petitioner were also insured with New India Assurance Company, Ltd.,
(New India).
On January 12, 1981, fire broke out in the compound of Transworld, razing the middle portion of its four-span
building and partly gutting the left and right sections thereof. A two-storey building (behind said four-span building)
where fun and amusement machines and spare parts were stored, was also destroyed by the fire.
Transworld filed its insurance claims with Rizal Surety & Insurance Company and New India Assurance Company
but to no avail.
On May 26, 1982, private respondent brought against the said insurance companies an action for collection of sum
of money and damages, docketed as Civil Case No. 46106 before Branch 161 of the then Court of First Instance of
Rizal; praying for judgment ordering Rizal Insurance and New India to pay the amount of P2,747, 867.00 plus legal
interest, P400,000.00 as attorney's fees, exemplary damages, expenses of litigation ofP50,000.00 and costs of suit.6
Petitioner Rizal Insurance countered that its fire insurance policy sued upon covered only the contents of the fourspan building, which was partly burned, and not the damage caused by the fire on the two-storey annex building. 7
On January 4, 1990, the trial court rendered its decision; disposing as follows:
"ACCORDINGLY, judgment is hereby rendered as follows:
(1)Dismissing the case as against The New India Assurance Co., Ltd.;
(2) Ordering defendant Rizal Surety And Insurance Company to pay Transwrold (sic) Knitting Mills, Inc. the
amount of P826, 500.00 representing the actual value of the losses suffered by it; and
(3) Cost against defendant Rizal Surety and Insurance Company.
SO ORDERED."8
Both the petitioner, Rizal Insurance Company, and private respondent, Transworld Knitting Mills, Inc., went to the
Court of Appeals, which came out with its decision of July 15, 1993 under attack, the decretal portion of which reads:
"WHEREFORE, and upon all the foregoing, the decision of the court below is MODIFIED in that defendant New
India Assurance Company has and is hereby required to pay plaintiff-appellant the amount of P1,818,604.19 while
the other Rizal Surety has to pay the plaintiff-appellant P470,328.67, based on the actual losses sustained by
plaintiff Transworld in the fire, totalling P2,790,376.00 as against the amounts of fire insurance coverages
respectively extended by New India in the amount of P5,800,000.00 and Rizal Surety and Insurance Company in
the amount of P1,500,000.00.
No costs.

12
SO ORDERED."9
On August 20, 1993, from the aforesaid judgment of the Court of Appeals New India appealed to this Court
theorizing inter alia that the private respondent could not be compensated for the loss of the fun and amusement
machines and spare parts stored at the two-storey building because it (Transworld) had no insurable interest in said
goods or items.
On February 2, 1994, the Court denied the appeal with finality in G.R. No. L-111118 (New India Assurance Company
Ltd. vs. Court of Appeals).
Petitioner Rizal Insurance and private respondent Transworld, interposed a Motion for Reconsideration before the
Court of Appeals, and on October 22, 1993, the Court of Appeals reconsidered its decision of July 15, 1993, as
regards the imposition of interest, ruling thus:
"WHEREFORE, the Decision of July 15, 1993 is amended but only insofar as the imposition of legal interest is
concerned, that, on the assessment against New India Assurance Company on the amount of P1,818,604.19 and
that against Rizal Surety & Insurance Company on the amount of P470,328.67, from May 26, 1982 when the
complaint was filed until payment is made. The rest of the said decision is retained in all other respects.
SO ORDERED."10
Undaunted, petitioner Rizal Surety & Insurance Company found its way to this Court via the present Petition,
contending that:
I.....SAID DECISION (ANNEX A) ERRED IN ASSUMING THAT THE ANNEX BUILDING WHERE THE BULK
OF THE BURNED PROPERTIES WERE STORED, WAS INCLUDED IN THE COVERAGE OF THE
INSURANCE POLICY ISSUED BY RIZAL SURETY TO TRANSWORLD.
II.....SAID DECISION AND RESOLUTION (ANNEXES A AND B) ERRED IN NOT CONSIDERING THE
PICTURES (EXHS. 3 TO 7-C-RIZAL SURETY), TAKEN IMMEDIATELY AFTER THE FIRE, WHICH
CLEARLY SHOW THAT THE PREMISES OCCUPIED BY TRANSWORLD, WHERE THE INSURED
PROPERTIES WERE LOCATED, SUSTAINED PARTIAL DAMAGE ONLY.
III. SAID DECISION (ANNEX A) ERRED IN NOT HOLDING THAT TRANSWORLD HAD ACTED IN
PALPABLE BAD FAITH AND WITH MALICE IN FILING ITS CLEARLY UNFOUNDED CIVIL ACTION, AND
IN NOT ORDERING TRANSWORLD TO PAY TO RIZAL SURETY MORAL AND PUNITIVE DAMAGES
(ART. 2205, CIVIL CODE), PLUS ATTORNEY'S FEES AND EXPENSES OF LITIGATION (ART. 2208 PARS.
4 and 11, CIVIL CODE).11
The Petition is not impressed with merit.
It is petitioner's submission that the fire insurance policy litigated upon protected only the contents of the main
building (four-span),12 and did not include those stored in the two-storey annex building. On the other hand, the
private respondent theorized that the so called "annex" was not an annex but was actually an integral part of the
four-span building13 and therefore, the goods and items stored therein were covered by the same fire insurance
policy.
Resolution of the issues posited here hinges on the proper interpretation of the stipulation in subject fire insurance
policy regarding its coverage, which reads:

13
"xxx contained and/or stored during the currency of this Policy in the premises occupied by them forming part of the
buildings situate (sic) within own Compound xxx"
Therefrom, it can be gleaned unerringly that the fire insurance policy in question did not limit its coverage to what
were stored in the four-span building. As opined by the trial court of origin, two requirements must concur in order
that the said fun and amusement machines and spare parts would be deemed protected by the fire insurance policy
under scrutiny, to wit:
"First, said properties must be contained and/or stored in the areas occupied by Transworld and second, said areas
must form part of the building described in the policy xxx"14
'Said building of four-span lofty one storey in height with mezzanine portions is constructed of reinforced concrete
and hollow blocks and/or concrete under galvanized iron roof and occupied as hosiery mills, garment and lingerie
factory, transistor-stereo assembly plant, offices, ware house and caretaker's quarter.'
The Court is mindful of the well-entrenched doctrine that factual findings by the Court of Appeals are conclusive on
the parties and not reviewable by this Court, and the same carry even more weight when the Court of Appeals has
affirmed the findings of fact arrived at by the lower court. 15
In the case under consideration, both the trial court and the Court of Appeals found that the so called "annex " was
not an annex building but an integral and inseparable part of the four-span building described in the policy and
consequently, the machines and spare parts stored therein were covered by the fire insurance in dispute. The letterreport of the Manila Adjusters and Surveyor's Company, which petitioner itself cited and invoked, describes the
"annex" building as follows:
"Two-storey building constructed of partly timber and partly concrete hollow blocks under g.i. roof which is adjoining
and intercommunicating with the repair of the first right span of the lofty storey building and thence by property fence
wall."16
Verily, the two-storey building involved, a permanent structure which adjoins and intercommunicates with the "first
right span of the lofty storey building",17 formed part thereof, and meets the requisites for compensability under the
fire insurance policy sued upon.
So also, considering that the two-storey building aforementioned was already existing when subject fire insurance
policy contract was entered into on January 12, 1981, having been constructed sometime in 1978, 18 petitioner
should have specifically excluded the said two-storey building from the coverage of the fire insurance if minded to
exclude the same but if did not, and instead, went on to provide that such fire insurance policy covers the products,
raw materials and supplies stored within the premises of respondent Transworld which was an integral part of the
four-span building occupied by Transworld, knowing fully well the existence of such building adjoining and
intercommunicating with the right section of the four-span building.
After a careful study, the Court does not find any basis for disturbing what the lower courts found and arrived at.
Indeed, the stipulation as to the coverage of the fire insurance policy under controversy has created a doubt
regarding the portions of the building insured thereby. Article 1377 of the New Civil Code provides:
"Art.1377. The interpretation of obscure words or stipulations in a contract shall not favor the party who caused the
obscurity"

14
Conformably, it stands to reason that the doubt should be resolved against the petitioner, Rizal Surety Insurance
Company, whose lawyer or managers drafted the fire insurance policy contract under scrutiny. Citing the aforecited
provision of law in point, the Court in Landicho vs. Government Service Insurance System,19 ruled:
"This is particularly true as regards insurance policies, in respect of which it is settled that the 'terms in an insurance
policy, which are ambiguous, equivocal, or uncertain x x x are to be construed strictly and most strongly against the
insurer, and liberally in favor of the insured so as to effect the dominant purpose of indemnity or payment to the
insured, especially where forfeiture is involved' (29 Am. Jur., 181), and the reason for this is that the 'insured usually
has no voice in the selection or arrangement of the words employed and that the language of the contract is
selected with great care and deliberation by experts and legal advisers employed by, and acting exclusively in the
interest of, the insurance company.' (44 C.J.S., p. 1174)."" 20
Equally relevant is the following disquisition of the Court in Fieldmen's Insurance Company, Inc. vs. Vda. De
Songco,21 to wit:
"'This rigid application of the rule on ambiguities has become necessary in view of current business practices. The
courts cannot ignore that nowadays monopolies, cartels and concentration of capital, endowed with overwhelming
economic power, manage to impose upon parties dealing with them cunningly prepared 'agreements' that the
weaker party may not change one whit, his participation in the 'agreement' being reduced to the alternative to 'take
it or leave it' labelled since Raymond Saleilles 'contracts by adherence' (contrats [sic] d'adhesion), in contrast to
these entered into by parties bargaining on an equal footing, such contracts (of which policies of insurance and
international bills of lading are prime example) obviously call for greater strictness and vigilance on the part of courts
of justice with a view to protecting the weaker party from abuses and imposition, and prevent their becoming traps
for the unwary (New Civil Code, Article 24; Sent. of Supreme Court of Spain, 13 Dec. 1934, 27 February 1942.)'" 22
1wphi1

The issue of whether or not Transworld has an insurable interest in the fun and amusement machines and spare
parts, which entitles it to be indemnified for the loss thereof, had been settled in G.R. No. L-111118, entitled New
India Assurance Company, Ltd., vs. Court of Appeals, where the appeal of New India from the decision of the Court
of Appeals under review, was denied with finality by this Court on February 2, 1994.
The rule on conclusiveness of judgment, which obtains under the premises, precludes the relitigation of a particular
fact or issue in another action between the same parties based on a different claim or cause of action. "xxx the
judgment in the prior action operates as estoppel only as to those matters in issue or points controverted, upon the
determination of which the finding or judgment was rendered. In fine, the previous judgment is conclusive in the
second case, only as those matters actually and directly controverted and determined and not as to matters merely
involved therein."23
Applying the abovecited pronouncement, the Court, in Smith Bell and Company (Phils.), Inc. vs. Court of
Appeals,24 held that the issue of negligence of the shipping line, which issue had already been passed upon in a
case filed by one of the insurers, is conclusive and can no longer be relitigated in a similar case filed by another
insurer against the same shipping line on the basis of the same factual circumstances. Ratiocinating further, the
Court opined:
"In the case at bar, the issue of which vessel ('Don Carlos' or 'Yotai Maru') had been negligent, or so negligent as to
have proximately caused the collision between them, was an issue that was actually, directly and expressly raised,
controverted and litigated in C.A.-G.R. No. 61320-R. Reyes, L.B., J., resolved that issue in his Decision and held the
'Don Carlos' to have been negligent rather than the 'Yotai Maru' and, as already noted, that Decision was affirmed
by this Court in G.R. No. L-48839 in a Resolution dated 6 December 1987. The Reyes Decision thus became final
and executory approximately two (2) years before the Sison Decision, which is assailed in the case at bar, was
promulgated. Applying the rule of conclusiveness of judgment, the question of which vessel had been negligent in

15
the collision between the two (2) vessels, had long been settled by this Court and could no longer be relitigated in
C.A.-G.R. No. 61206-R. Private respondent Go Thong was certainly bound by the ruling or judgment of Reyes, L.B.,
J. and that of this Court. The Court of Appeals fell into clear and reversible error when it disregarded the Decision of
this Court affirming the Reyes Decision."25
The controversy at bar is on all fours with the aforecited case. Considering that private respondent's insurable
interest in, and compensability for the loss of subject fun and amusement machines and spare parts, had been
adjudicated, settled and sustained by the Court of Appeals in CA-G.R. CV NO. 28779, and by this Court in G.R. No.
L-111118, in a Resolution, dated February 2, 1994, the same can no longer be relitigated and passed upon in the
present case. Ineluctably, the petitioner, Rizal Surety Insurance Company, is bound by the ruling of the Court of
Appeals and of this Court that the private respondent has an insurable interest in the aforesaid fun and amusement
machines and spare parts; and should be indemnified for the loss of the same.
So also, the Court of Appeals correctly adjudged petitioner liable for the amount of P470,328.67, it being the total
loss and damage suffered by Transworld for which petitioner Rizal Insurance is liable. 26
All things studiedly considered and viewed in proper perspective, the Court is of the irresistible conclusion, and so
finds, that the Court of Appeals erred not in holding the petitioner, Rizal Surety Insurance Company, liable for the
destruction and loss of the insured buildings and articles of the private respondent.
WHEREFORE, the Decision, dated July 15, 1993, and the Resolution, dated October 22, 1993, of the Court of
Appeals in CA-G.R. CV NO. 28779 are AFFIRMED in toto. No pronouncement as to costs.
SO ORDERED.

16

Republic of the Philippines


SUPREME COURT
Manila
FIRST DIVISION
G.R. No. 125678

March 18, 2002

17
PHILAMCARE HEALTH SYSTEMS, INC., petitioner,
vs.
COURT OF APPEALS and JULITA TRINOS, respondents.
YNARES-SANTIAGO, J.:
Ernani Trinos, deceased husband of respondent Julita Trinos, applied for a health care coverage with petitioner
Philamcare Health Systems, Inc. In the standard application form, he answered no to the following question:
Have you or any of your family members ever consulted or been treated for high blood pressure, heart
trouble, diabetes, cancer, liver disease, asthma or peptic ulcer? (If Yes, give details). 1
The application was approved for a period of one year from March 1, 1988 to March 1, 1989. Accordingly, he was
issued Health Care Agreement No. P010194. Under the agreement, respondents husband was entitled to avail of
hospitalization benefits, whether ordinary or emergency, listed therein. He was also entitled to avail of "out-patient
benefits" such as annual physical examinations, preventive health care and other out-patient services.
Upon the termination of the agreement, the same was extended for another year from March 1, 1989 to March 1,
1990, then from March 1, 1990 to June 1, 1990. The amount of coverage was increased to a maximum sum of
P75,000.00 per disability.2
During the period of his coverage, Ernani suffered a heart attack and was confined at the Manila Medical Center
(MMC) for one month beginning March 9, 1990. While her husband was in the hospital, respondent tried to claim the
benefits under the health care agreement. However, petitioner denied her claim saying that the Health Care
Agreement was void. According to petitioner, there was a concealment regarding Ernanis medical history. Doctors
at the MMC allegedly discovered at the time of Ernanis confinement that he was hypertensive, diabetic and
asthmatic, contrary to his answer in the application form. Thus, respondent paid the hospitalization expenses
herself, amounting to about P76,000.00.
After her husband was discharged from the MMC, he was attended by a physical therapist at home. Later, he was
admitted at the Chinese General Hospital. Due to financial difficulties, however, respondent brought her husband
home again. In the morning of April 13, 1990, Ernani had fever and was feeling very weak. Respondent was
constrained to bring him back to the Chinese General Hospital where he died on the same day.
On July 24, 1990, respondent instituted with the Regional Trial Court of Manila, Branch 44, an action for damages
against petitioner and its president, Dr. Benito Reverente, which was docketed as Civil Case No. 90-53795. She
asked for reimbursement of her expenses plus moral damages and attorneys fees. After trial, the lower court ruled
against petitioners, viz:
WHEREFORE, in view of the forgoing, the Court renders judgment in favor of the plaintiff Julita Trinos,
ordering:
1. Defendants to pay and reimburse the medical and hospital coverage of the late Ernani Trinos in the
amount of P76,000.00 plus interest, until the amount is fully paid to plaintiff who paid the same;
2. Defendants to pay the reduced amount of moral damages of P10,000.00 to plaintiff;
3. Defendants to pay the reduced amount of P10,000.00 as exemplary damages to plaintiff;

18
4. Defendants to pay attorneys fees of P20,000.00, plus costs of suit.
SO ORDERED.3
On appeal, the Court of Appeals affirmed the decision of the trial court but deleted all awards for damages and
absolved petitioner Reverente.4 Petitioners motion for reconsideration was denied. 5 Hence, petitioner brought the
instant petition for review, raising the primary argument that a health care agreement is not an insurance contract;
hence the "incontestability clause" under the Insurance Code 6 does not apply.
1wphi1.nt

Petitioner argues that the agreement grants "living benefits," such as medical check-ups and hospitalization which a
member may immediately enjoy so long as he is alive upon effectivity of the agreement until its expiration one-year
thereafter. Petitioner also points out that only medical and hospitalization benefits are given under the agreement
without any indemnification, unlike in an insurance contract where the insured is indemnified for his loss. Moreover,
since Health Care Agreements are only for a period of one year, as compared to insurance contracts which last
longer,7 petitioner argues that the incontestability clause does not apply, as the same requires an effectivity period of
at least two years. Petitioner further argues that it is not an insurance company, which is governed by the Insurance
Commission, but a Health Maintenance Organization under the authority of the Department of Health.
Section 2 (1) of the Insurance Code defines a contract of insurance as an agreement whereby one undertakes for a
consideration to indemnify another against loss, damage or liability arising from an unknown or contingent event. An
insurance contract exists where the following elements concur:
1. The insured has an insurable interest;
2. The insured is subject to a risk of loss by the happening of the designated peril;
3. The insurer assumes the risk;
4. Such assumption of risk is part of a general scheme to distribute actual losses among a large group of
persons bearing a similar risk; and
5. In consideration of the insurers promise, the insured pays a premium. 8
Section 3 of the Insurance Code states that any contingent or unknown event, whether past or future, which may
damnify a person having an insurable interest against him, may be insured against. Every person has an insurable
interest in the life and health of himself. Section 10 provides:
Every person has an insurable interest in the life and health:
(1) of himself, of his spouse and of his children;
(2) of any person on whom he depends wholly or in part for education or support, or in whom he has a
pecuniary interest;
(3) of any person under a legal obligation to him for the payment of money, respecting property or service, of
which death or illness might delay or prevent the performance; and
(4) of any person upon whose life any estate or interest vested in him depends.

19
In the case at bar, the insurable interest of respondents husband in obtaining the health care agreement was his
own health. The health care agreement was in the nature of non-life insurance, which is primarily a contract of
indemnity.9 Once the member incurs hospital, medical or any other expense arising from sickness, injury or other
stipulated contingent, the health care provider must pay for the same to the extent agreed upon under the contract.
Petitioner argues that respondents husband concealed a material fact in his application. It appears that in the
application for health coverage, petitioners required respondents husband to sign an express authorization for any
person, organization or entity that has any record or knowledge of his health to furnish any and all information
relative to any hospitalization, consultation, treatment or any other medical advice or examination. 10 Specifically, the
Health Care Agreement signed by respondents husband states:
We hereby declare and agree that all statement and answers contained herein and in any addendum
annexed to this application are full, complete and true and bind all parties in interest under the Agreement
herein applied for, that there shall be no contract of health care coverage unless and until an Agreement is
issued on this application and the full Membership Fee according to the mode of payment applied for is
actually paid during the lifetime and good health of proposed Members; that no information acquired by any
Representative of PhilamCare shall be binding upon PhilamCare unless set out in writing in the
application;that any physician is, by these presents, expressly authorized to disclose or give testimony at
anytime relative to any information acquired by him in his professional capacity upon any question affecting
the eligibility for health care coverage of the Proposed Members and that the acceptance of any Agreement
issued on this application shall be a ratification of any correction in or addition to this application as stated in
the space for Home Office Endorsement.11 (Underscoring ours)
In addition to the above condition, petitioner additionally required the applicant for authorization to inquire about the
applicants medical history, thus:
I hereby authorize any person, organization, or entity that has any record or knowledge of my health and/or
that of __________ to give to the PhilamCare Health Systems, Inc. any and all information relative to any
hospitalization, consultation, treatment or any other medical advice or examination. This authorization is in
connection with the application for health care coverage only. A photographic copy of this authorization shall
be as valid as the original.12 (Underscoring ours)
Petitioner cannot rely on the stipulation regarding "Invalidation of agreement" which reads:
Failure to disclose or misrepresentation of any material information by the member in the application or
medical examination, whether intentional or unintentional, shall automatically invalidate the Agreement from
the very beginning and liability of Philamcare shall be limited to return of all Membership Fees paid. An
undisclosed or misrepresented information is deemed material if its revelation would have resulted in the
declination of the applicant by Philamcare or the assessment of a higher Membership Fee for the benefit or
benefits applied for.13
The answer assailed by petitioner was in response to the question relating to the medical history of the applicant.
This largely depends on opinion rather than fact, especially coming from respondents husband who was not a
medical doctor. Where matters of opinion or judgment are called for, answers made in good faith and without intent
to deceive will not avoid a policy even though they are untrue.14 Thus,
(A)lthough false, a representation of the expectation, intention, belief, opinion, or judgment of the insured will
not avoid the policy if there is no actual fraud in inducing the acceptance of the risk, or its acceptance at a
lower rate of premium, and this is likewise the rule although the statement is material to the risk, if the
statement is obviously of the foregoing character, since in such case the insurer is not justified in relying

20
upon such statement, but is obligated to make further inquiry. There is a clear distinction between such a
case and one in which the insured is fraudulently and intentionally states to be true, as a matter of
expectation or belief, that which he then knows, to be actually untrue, or the impossibility of which is shown
by the facts within his knowledge, since in such case the intent to deceive the insurer is obvious and
amounts to actual fraud.15 (Underscoring ours)
The fraudulent intent on the part of the insured must be established to warrant rescission of the insurance
contract.16 Concealment as a defense for the health care provider or insurer to avoid liability is an affirmative
defense and the duty to establish such defense by satisfactory and convincing evidence rests upon the provider or
insurer. In any case, with or without the authority to investigate, petitioner is liable for claims made under the
contract. Having assumed a responsibility under the agreement, petitioner is bound to answer the same to the
extent agreed upon. In the end, the liability of the health care provider attaches once the member is hospitalized for
the disease or injury covered by the agreement or whenever he avails of the covered benefits which he has prepaid.
Under Section 27 of the Insurance Code, "a concealment entitles the injured party to rescind a contract of
insurance." The right to rescind should be exercised previous to the commencement of an action on the contract. 17 In
this case, no rescission was made. Besides, the cancellation of health care agreements as in insurance policies
require the concurrence of the following conditions:
1. Prior notice of cancellation to insured;
2. Notice must be based on the occurrence after effective date of the policy of one or more of the grounds
mentioned;
3. Must be in writing, mailed or delivered to the insured at the address shown in the policy;
4. Must state the grounds relied upon provided in Section 64 of the Insurance Code and upon request of insured, to
furnish facts on which cancellation is based.18
None of the above pre-conditions was fulfilled in this case. When the terms of insurance contract contain limitations
on liability, courts should construe them in such a way as to preclude the insurer from non-compliance with his
obligation.19 Being a contract of adhesion, the terms of an insurance contract are to be construed strictly against the
party which prepared the contract the insurer.20 By reason of the exclusive control of the insurance company over
the terms and phraseology of the insurance contract, ambiguity must be strictly interpreted against the insurer and
liberally in favor of the insured, especially to avoid forfeiture.21 This is equally applicable to Health Care Agreements.
The phraseology used in medical or hospital service contracts, such as the one at bar, must be liberally construed in
favor of the subscriber, and if doubtful or reasonably susceptible of two interpretations the construction conferring
coverage is to be adopted, and exclusionary clauses of doubtful import should be strictly construed against the
provider.22
Anent the incontestability of the membership of respondents husband, we quote with approval the following findings
of the trial court:
(U)nder the title Claim procedures of expenses, the defendant Philamcare Health Systems Inc. had twelve
months from the date of issuance of the Agreement within which to contest the membership of the patient if
he had previous ailment of asthma, and six months from the issuance of the agreement if the patient was
sick of diabetes or hypertension. The periods having expired, the defense of concealment or
misrepresentation no longer lie.23

21
Finally, petitioner alleges that respondent was not the legal wife of the deceased member considering that at the
time of their marriage, the deceased was previously married to another woman who was still alive. The health care
agreement is in the nature of a contract of indemnity. Hence, payment should be made to the party who incurred the
expenses. It is not controverted that respondent paid all the hospital and medical expenses. She is therefore entitled
to reimbursement. The records adequately prove the expenses incurred by respondent for the deceaseds
hospitalization, medication and the professional fees of the attending physicians. 24
WHEREFORE, in view of the foregoing, the petition is DENIED. The assailed decision of the Court of Appeals dated
December 14, 1995 is AFFIRMED.
SO ORDERED.

Republic of the Philippines


SUPREME COURT
Manila
FIRST DIVISION

22

G.R. No. 115278 May 23, 1995


FORTUNE INSURANCE AND SURETY CO., INC., petitioner,
vs.
COURT OF APPEALS and PRODUCERS BANK OF THE PHILIPPINES, respondents.

DAVIDE, JR., J.:


The fundamental legal issue raised in this petition for review on certiorari is whether the petitioner is liable under the
Money, Security, and Payroll Robbery policy it issued to the private respondent or whether recovery thereunder is
precluded under the general exceptions clause thereof. Both the trial court and the Court of Appeals held that there
should be recovery. The petitioner contends otherwise.
This case began with the filing with the Regional Trial Court (RTC) of Makati, Metro Manila, by private respondent
Producers Bank of the Philippines (hereinafter Producers) against petitioner Fortune Insurance and Surety Co., Inc.
(hereinafter Fortune) of a complaint for recovery of the sum of P725,000.00 under the policy issued by Fortune. The
sum was allegedly lost during a robbery of Producer's armored vehicle while it was in transit to transfer the money
from its Pasay City Branch to its head office in Makati. The case was docketed as Civil Case No. 1817 and assigned
to Branch 146 thereof.
After joinder of issues, the parties asked the trial court to render judgment based on the following stipulation of facts:
1. The plaintiff was insured by the defendants and an insurance policy was issued,
the duplicate original of which is hereto attached as Exhibit "A";
2. An armored car of the plaintiff, while in the process of transferring cash in the sum
of P725,000.00 under the custody of its teller, Maribeth Alampay, from its Pasay
Branch to its Head Office at 8737 Paseo de Roxas, Makati, Metro Manila on June 29,
1987, was robbed of the said cash. The robbery took place while the armored car
was traveling along Taft Avenue in Pasay City;
3. The said armored car was driven by Benjamin Magalong Y de Vera, escorted by
Security Guard Saturnino Atiga Y Rosete. Driver Magalong was assigned by PRC
Management Systems with the plaintiff by virtue of an Agreement executed on
August 7, 1983, a duplicate original copy of which is hereto attached as Exhibit "B";
4. The Security Guard Atiga was assigned by Unicorn Security Services, Inc. with the
plaintiff by virtue of a contract of Security Service executed on October 25, 1982, a
duplicate original copy of which is hereto attached as Exhibit "C";
5. After an investigation conducted by the Pasay police authorities, the driver
Magalong and guard Atiga were charged, together with Edelmer Bantigue Y Eulalio,
Reynaldo Aquino and John Doe, with violation of P.D. 532 (Anti-Highway Robbery
Law) before the Fiscal of Pasay City. A copy of the complaint is hereto attached as
Exhibit "D";

23
6. The Fiscal of Pasay City then filed an information charging the aforesaid persons
with the said crime before Branch 112 of the Regional Trial Court of Pasay City. A
copy of the said information is hereto attached as Exhibit "E." The case is still being
tried as of this date;
7. Demands were made by the plaintiff upon the defendant to pay the amount of the
loss of P725,000.00, but the latter refused to pay as the loss is excluded from the
coverage of the insurance policy, attached hereto as Exhibit "A," specifically under
page 1 thereof, "General Exceptions" Section (b), which is marked as Exhibit "A-1,"
and which reads as follows:
GENERAL EXCEPTIONS
The company shall not be liable under this policy in report of
xxx xxx xxx
(b) any loss caused by any dishonest, fraudulent or criminal act of the
insured or any officer, employee, partner, director, trustee or
authorized representative of the Insured whether acting alone or in
conjunction with others. . . .
8. The plaintiff opposes the contention of the defendant and contends that Atiga and
Magalong are not its "officer, employee, . . . trustee or authorized representative . . .
at the time of the robbery. 1
On 26 April 1990, the trial court rendered its decision in favor of Producers. The dispositive portion thereof reads as
follows:
WHEREFORE, premises considered, the Court finds for plaintiff and against defendant, and
(a) orders defendant to pay plaintiff the net amount of P540,000.00 as
liability under Policy No. 0207 (as mitigated by the P40,000.00
special clause deduction and by the recovered sum of P145,000.00),
with interest thereon at the legal rate, until fully paid;
(b) orders defendant to pay plaintiff the sum of P30,000.00 as and for
attorney's fees; and
(c) orders defendant to pay costs of suit.
All other claims and counterclaims are accordingly dismissed forthwith.
SO ORDERED. 2
The trial court ruled that Magalong and Atiga were not employees or representatives of Producers. It Said:
The Court is satisfied that plaintiff may not be said to have selected and engaged Magalong and
Atiga, their services as armored car driver and as security guard having been merely offered by PRC

24
Management and by Unicorn Security and which latter firms assigned them to plaintiff. The wages
and salaries of both Magalong and Atiga are presumably paid by their respective firms, which alone
wields the power to dismiss them. Magalong and Atiga are assigned to plaintiff in fulfillment of
agreements to provide driving services and property protection as such in a context which does
not impress the Court as translating into plaintiff's power to control the conduct of any assigned
driver or security guard, beyond perhaps entitling plaintiff to request are replacement for such driver
guard. The finding is accordingly compelled that neither Magalong nor Atiga were plaintiff's
"employees" in avoidance of defendant's liability under the policy, particularly the general exceptions
therein embodied.
Neither is the Court prepared to accept the proposition that driver Magalong and guard Atiga were
the "authorized representatives" of plaintiff. They were merely an assigned armored car driver and
security guard, respectively, for the June 29, 1987 money transfer from plaintiff's Pasay Branch to its
Makati Head Office. Quite plainly it was teller Maribeth Alampay who had "custody" of the
P725,000.00 cash being transferred along a specified money route, and hence plaintiff's then
designated "messenger" adverted to in the policy. 3
Fortune appealed this decision to the Court of Appeals which docketed the case as CA-G.R. CV No. 32946. In its
decision 4 promulgated on 3 May 1994, it affirmed in toto the appealed decision.
The Court of Appeals agreed with the conclusion of the trial court that Magalong and Atiga were neither employees
nor authorized representatives of Producers and ratiocinated as follows:
A policy or contract of insurance is to be construed liberally in favor of the insured and strictly against
the insurance company (New Life Enterprises vs. Court of Appeals, 207 SCRA 669; Sun Insurance
Office, Ltd. vs. Court of Appeals, 211 SCRA 554). Contracts of insurance, like other contracts, are to
be construed according to the sense and meaning of the terms which the parties themselves have
used. If such terms are clear and unambiguous, they must be taken and understood in their plain,
ordinary and popular sense (New Life Enterprises Case, supra, p. 676; Sun Insurance Office, Ltd.
vs. Court of Appeals, 195 SCRA 193).
The language used by defendant-appellant in the above quoted stipulation is plain, ordinary and
simple. No other interpretation is necessary. The word "employee" must be taken to mean in the
ordinary sense.
The Labor Code is a special law specifically dealing with/and specifically designed to protect labor
and therefore its definition as to employer-employee relationships insofar as the
application/enforcement of said Code is concerned must necessarily be inapplicable to an insurance
contract which defendant-appellant itself had formulated. Had it intended to apply the Labor Code in
defining what the word "employee" refers to, it must/should have so stated expressly in the
insurance policy.
Said driver and security guard cannot be considered as employees of plaintiff-appellee bank
because it has no power to hire or to dismiss said driver and security guard under the contracts
(Exhs. 8 and C) except only to ask for their replacements from the contractors. 5
On 20 June 1994, Fortune filed this petition for review on certiorari. It alleges that the trial court and the Court of
Appeals erred in holding it liable under the insurance policy because the loss falls within the general exceptions
clause considering that driver Magalong and security guard Atiga were Producers' authorized representatives or
employees in the transfer of the money and payroll from its branch office in Pasay City to its head office in Makati.

25
According to Fortune, when Producers commissioned a guard and a driver to transfer its funds from one branch to
another, they effectively and necessarily became its authorized representatives in the care and custody of the
money. Assuming that they could not be considered authorized representatives, they were, nevertheless, employees
of Producers. It asserts that the existence of an employer-employee relationship "is determined by law and being
such, it cannot be the subject of agreement." Thus, if there was in reality an employer-employee relationship
between Producers, on the one hand, and Magalong and Atiga, on the other, the provisions in the contracts of
Producers with PRC Management System for Magalong and with Unicorn Security Services for Atiga which state
that Producers is not their employer and that it is absolved from any liability as an employer, would not obliterate the
relationship.
Fortune points out that an employer-employee relationship depends upon four standards: (1) the manner of
selection and engagement of the putative employee; (2) the mode of payment of wages; (3) the presence or
absence of a power to dismiss; and (4) the presence and absence of a power to control the putative employee's
conduct. Of the four, the right-of-control test has been held to be the decisive factor. 6 It asserts that the power of
control over Magalong and Atiga was vested in and exercised by Producers. Fortune further insists that PRC
Management System and Unicorn Security Services are but "labor-only" contractors under Article 106 of the Labor Code
which provides:
Art. 106. Contractor or subcontractor. There is "labor-only" contracting where the person
supplying workers to an employer does not have substantial capital or investment in the form of
tools, equipment, machineries, work premises, among others, and the workers recruited and placed
by such persons are performing activities which are directly related to the principal business of such
employer. In such cases, the person or intermediary shall be considered merely as an agent of the
employer who shall be responsible to the workers in the same manner and extent as if the latter
were directly employed by him.
Fortune thus contends that Magalong and Atiga were employees of Producers, following the ruling in International
Timber Corp. vs. NLRC 7 that a finding that a contractor is a "labor-only" contractor is equivalent to a finding that there is
an employer-employee relationship between the owner of the project and the employees of the "labor-only" contractor.
On the other hand, Producers contends that Magalong and Atiga were not its employees since it had nothing to do
with their selection and engagement, the payment of their wages, their dismissal, and the control of their conduct.
Producers argued that the rule in International Timber Corp. is not applicable to all cases but only when it becomes
necessary to prevent any violation or circumvention of the Labor Code, a social legislation whose provisions may
set aside contracts entered into by parties in order to give protection to the working man.
Producers further asseverates that what should be applied is the rule in American President Lines vs. Clave, 8 to wit:
In determining the existence of employer-employee relationship, the following elements are
generally considered, namely: (1) the selection and engagement of the employee; (2) the payment of
wages; (3) the power of dismissal; and (4) the power to control the employee's conduct.
Since under Producers' contract with PRC Management Systems it is the latter which assigned Magalong as the
driver of Producers' armored car and was responsible for his faithful discharge of his duties and responsibilities, and
since Producers paid the monthly compensation of P1,400.00 per driver to PRC Management Systems and not to
Magalong, it is clear that Magalong was not Producers' employee. As to Atiga, Producers relies on the provision of
its contract with Unicorn Security Services which provides that the guards of the latter "are in no sense employees
of the CLIENT."
There is merit in this petition.

26
It should be noted that the insurance policy entered into by the parties is a theft or robbery insurance policy which is
a form of casualty insurance. Section 174 of the Insurance Code provides:
Sec. 174. Casualty insurance is insurance covering loss or liability arising from accident or mishap,
excluding certain types of loss which by law or custom are considered as falling exclusively within
the scope of insurance such as fire or marine. It includes, but is not limited to, employer's liability
insurance, public liability insurance, motor vehicle liability insurance, plate glass insurance, burglary
and theft insurance, personal accident and health insurance as written by non-life insurance
companies, and other substantially similar kinds of insurance. (emphases supplied)
Except with respect to compulsory motor vehicle liability insurance, the Insurance Code contains no other provisions
applicable to casualty insurance or to robbery insurance in particular. These contracts are, therefore, governed by
the general provisions applicable to all types of insurance. Outside of these, the rights and obligations of the parties
must be determined by the terms of their contract, taking into consideration its purpose and always in accordance
with the general principles of insurance law. 9
It has been aptly observed that in burglary, robbery, and theft insurance, "the opportunity to defraud the insurer
the moral hazard is so great that insurers have found it necessary to fill up their policies with countless
restrictions, many designed to reduce this hazard. Seldom does the insurer assume the risk of all losses due to the
hazards insured against." 10 Persons frequently excluded under such provisions are those in the insured's service and
employment. 11 The purpose of the exception is to guard against liability should the theft be committed by one having
unrestricted access to the property. 12 In such cases, the terms specifying the excluded classes are to be given their
meaning as understood in common speech. 13 The terms "service" and "employment" are generally associated with the
idea of selection, control, and compensation. 14
A contract of insurance is a contract of adhesion, thus any ambiguity therein should be resolved against the
insurer, 15 or it should be construed liberally in favor of the insured and strictly against the insurer. 16 Limitations of liability
should be regarded with extreme jealousy and must be construed
in such a way, as to preclude the insurer from non-compliance with its obligation. 17 It goes without saying then that if the
terms of the contract are clear and unambiguous, there is no room for construction and such terms cannot be enlarged or
diminished by judicial construction. 18
An insurance contract is a contract of indemnity upon the terms and conditions specified therein. 19 It is settled that
the terms of the policy constitute the measure of the insurer's liability. 20 In the absence of statutory prohibition to the
contrary, insurance companies have the same rights as individuals to limit their liability and to impose whatever conditions
they deem best upon their obligations not inconsistent with public policy.
With the foregoing principles in mind, it may now be asked whether Magalong and Atiga qualify as employees or
authorized representatives of Producers under paragraph (b) of the general exceptions clause of the policy which,
for easy reference, is again quoted:
GENERAL EXCEPTIONS
The company shall not be liable under this policy in respect of
xxx xxx xxx
(b) any loss caused by any dishonest, fraudulent or criminal act of the insured or any
officer, employee, partner, director, trustee or authorized representative of the
Insured whether acting alone or in conjunction with others. . . . (emphases supplied)

27
There is marked disagreement between the parties on the correct meaning of the terms "employee" and "authorized
representatives."
It is clear to us that insofar as Fortune is concerned, it was its intention to exclude and exempt from protection and
coverage losses arising from dishonest, fraudulent, or criminal acts of persons granted or having unrestricted
access to Producers' money or payroll. When it used then the term "employee," it must have had in mind any
person who qualifies as such as generally and universally understood, or jurisprudentially established in the light of
the four standards in the determination of the employer-employee relationship, 21 or as statutorily declared even in a
limited sense as in the case of Article 106 of the Labor Code which considers the employees under a "labor-only" contract
as employees of the party employing them and not of the party who supplied them to the employer. 22
Fortune claims that Producers' contracts with PRC Management Systems and Unicorn Security Services are "laboronly" contracts.
Producers, however, insists that by the express terms thereof, it is not the employer of Magalong.
Notwithstanding such express assumption of PRC Management Systems and Unicorn Security Services
that the drivers and the security guards each shall supply to Producers are not the latter's employees, it may,
in fact, be that it is because the contracts are, indeed, "labor-only" contracts. Whether they are is, in the light
of the criteria provided for in Article 106 of the Labor Code, a question of fact. Since the parties opted to
submit the case for judgment on the basis of their stipulation of facts which are strictly limited to the
insurance policy, the contracts with PRC Management Systems and Unicorn Security Services, the
complaint for violation of P.D. No. 532, and the information therefor filed by the City Fiscal of Pasay City,
there is a paucity of evidence as to whether the contracts between Producers and PRC Management
Systems and Unicorn Security Services are "labor-only" contracts.
But even granting for the sake of argument that these contracts were not "labor-only" contracts, and PRC
Management Systems and Unicorn Security Services were truly independent contractors, we are satisfied that
Magalong and Atiga were, in respect of the transfer of Producer's money from its Pasay City branch to its head
office in Makati, its "authorized representatives" who served as such with its teller Maribeth Alampay. Howsoever
viewed, Producers entrusted the three with the specific duty to safely transfer the money to its head office, with
Alampay to be responsible for its custody in transit; Magalong to drive the armored vehicle which would carry the
money; and Atiga to provide the needed security for the money, the vehicle, and his two other companions. In short,
for these particular tasks, the three acted as agents of Producers. A "representative" is defined as one who
represents or stands in the place of another; one who represents others or another in a special capacity, as an
agent, and is interchangeable with "agent." 23
In view of the foregoing, Fortune is exempt from liability under the general exceptions clause of the insurance policy.
WHEREFORE , the instant petition is hereby GRANTED. The decision of the Court of Appeals in CA-G.R. CV No.
32946 dated 3 May 1994 as well as that of Branch 146 of the Regional Trial Court of Makati in Civil Case No. 1817
are REVERSED and SET ASIDE. The complaint in Civil Case No. 1817 is DISMISSED.
No pronouncement as to costs.
SO ORDERED.

28

29
Republic of the Philippines
SUPREME COURT
Manila
SECOND DIVISION
G.R. No. 156167

May 16, 2005

GULF RESORTS, INC., petitioner,


vs.
PHILIPPINE CHARTER INSURANCE CORPORATION, respondent.
DECISION
PUNO, J.:
Before the Court is the petition for certiorari under Rule 45 of the Revised Rules of Court by petitioner GULF
RESORTS, INC., against respondent PHILIPPINE CHARTER INSURANCE CORPORATION. Petitioner assails the
appellate court decision1 which dismissed its two appeals and affirmed the judgment of the trial court.
For review are the warring interpretations of petitioner and respondent on the scope of the insurance companys
liability for earthquake damage to petitioners properties. Petitioner avers that, pursuant to its earthquake shock
endorsement rider, Insurance Policy No. 31944 covers all damages to the properties within its resort caused by
earthquake. Respondent contends that the rider limits its liability for loss to the two swimming pools of petitioner.
The facts as established by the court a quo, and affirmed by the appellate court are as follows:
[P]laintiff is the owner of the Plaza Resort situated at Agoo, La Union and had its properties in said resort
insured originally with the American Home Assurance Company (AHAC-AIU). In the first four insurance
policies issued by AHAC-AIU from 1984-85; 1985-86; 1986-1987; and 1987-88 (Exhs. "C", "D", "E" and "F";
also Exhs. "1", "2", "3" and "4" respectively), the risk of loss from earthquake shock was extended only to
plaintiffs two swimming pools, thus, "earthquake shock endt." (Item 5 only) (Exhs. "C-1"; "D-1," and "E" and
two (2) swimming pools only (Exhs. "C-1"; D-1", "E" and "F-1"). "Item 5" in those policies referred to the two
(2) swimming pools only (Exhs. "1-B", "2-B", "3-B" and "F-2"); that subsequently AHAC(AIU) issued in
plaintiffs favor Policy No. 206-4182383-0 covering the period March 14, 1988 to March 14, 1989 (Exhs. "G"
also "G-1") and in said policy the earthquake endorsement clause as indicated in Exhibits "C-1", "D-1",
Exhibits "E" and "F-1" was deleted and the entry under Endorsements/Warranties at the time of issue read
that plaintiff renewed its policy with AHAC (AIU) for the period of March 14, 1989 to March 14, 1990 under
Policy No. 206-4568061-9 (Exh. "H") which carried the entry under "Endorsement/Warranties at Time of
Issue", which read "Endorsement to Include Earthquake Shock (Exh. "6-B-1") in the amount of P10,700.00
and paid P42,658.14 (Exhs. "6-A" and "6-B") as premium thereof, computed as follows:

Item

P7,691,000.00 -

on the Clubhouse only


@ .392%;

1,500,000.00 -

393,000.00 -

on the furniture, etc. contained in the building


above-mentioned@ .490%;

on the two swimming pools, only (against the peril

30

of earthquake shock only) @ 0.100%

116,600.00

other buildings include as follows:

a) Tilter House

P19,800.00 -

0.551%

b) Power House

P41,000.00 -

0.551%

c) House Shed

P55,000.00 -

0.540%

P100,000.00 -

for furniture, fixtures, lines air-con and operating


equipment

that plaintiff agreed to insure with defendant the properties covered by AHAC (AIU) Policy No. 206-45680619 (Exh. "H") provided that the policy wording and rates in said policy be copied in the policy to be issued by
defendant; that defendant issued Policy No. 31944 to plaintiff covering the period of March 14, 1990 to
March 14, 1991 for P10,700,600.00 for a total premium of P45,159.92 (Exh. "I"); that in the computation of
the premium, defendants Policy No. 31944 (Exh. "I"), which is the policy in question, contained on the righthand upper portion of page 7 thereof, the following:

Rate-Various

Premium

P37,420.60 F/L

2,061.52

Typhoon

1,030.76

EC

393.00

ES

Doc. Stamps

3,068.10

F.S.T.

776.89

31

Prem. Tax

409.05

TOTAL

45,159.92;

that the above break-down of premiums shows that plaintiff paid only P393.00 as premium against
earthquake shock (ES); that in all the six insurance policies (Exhs. "C", "D", "E", "F", "G" and "H"), the
premium against the peril of earthquake shock is the same, that is P393.00 (Exhs. "C" and "1-B"; "2-B" and
"3-B-1" and "3-B-2"; "F-02" and "4-A-1"; "G-2" and "5-C-1"; "6-C-1"; issued by AHAC (Exhs. "C", "D", "E",
"F", "G" and "H") and in Policy No. 31944 issued by defendant, the shock endorsement provide(sic):
In consideration of the payment by the insured to the company of the sum included additional
premium the Company agrees, notwithstanding what is stated in the printed conditions of this policy
due to the contrary, that this insurance covers loss or damage to shock to any of the property insured
by this Policy occasioned by or through or in consequence of earthquake (Exhs. "1-D", "2-D", "3-A",
"4-B", "5-A", "6-D" and "7-C");
that in Exhibit "7-C" the word "included" above the underlined portion was deleted; that on July 16, 1990 an
earthquake struck Central Luzon and Northern Luzon and plaintiffs properties covered by Policy No. 31944
issued by defendant, including the two swimming pools in its Agoo Playa Resort were damaged. 2
After the earthquake, petitioner advised respondent that it would be making a claim under its Insurance Policy No.
31944 for damages on its properties. Respondent instructed petitioner to file a formal claim, then assigned the
investigation of the claim to an independent claims adjuster, Bayne Adjusters and Surveyors, Inc. 3 On July 30, 1990,
respondent, through its adjuster, requested petitioner to submit various documents in support of its claim. On August
7, 1990, Bayne Adjusters and Surveyors, Inc., through its Vice-President A.R. de Leon, 4 rendered a preliminary
report5 finding extensive damage caused by the earthquake to the clubhouse and to the two swimming pools. Mr. de
Leon stated that "except for the swimming pools, all affected items have no coverage for earthquake shocks." 6 On
August 11, 1990, petitioner filed its formal demand7 for settlement of the damage to all its properties in the Agoo
Playa Resort. On August 23, 1990, respondent denied petitioners claim on the ground that its insurance policy only
afforded earthquake shock coverage to the two swimming pools of the resort. 8Petitioner and respondent failed to
arrive at a settlement.9 Thus, on January 24, 1991, petitioner filed a complaint10 with the regional trial court of Pasig
praying for the payment of the following:
1.) The sum of P5,427,779.00, representing losses sustained by the insured properties, with interest
thereon, as computed under par. 29 of the policy (Annex "B") until fully paid;
2.) The sum of P428,842.00 per month, representing continuing losses sustained by plaintiff on account of
defendants refusal to pay the claims;
3.) The sum of P500,000.00, by way of exemplary damages;
4.) The sum of P500,000.00 by way of attorneys fees and expenses of litigation;
5.) Costs.11
Respondent filed its Answer with Special and Affirmative Defenses with Compulsory Counterclaims. 12
On February 21, 1994, the lower court after trial ruled in favor of the respondent, viz:
The above schedule clearly shows that plaintiff paid only a premium of P393.00 against the peril of
earthquake shock, the same premium it paid against earthquake shock only on the two swimming pools in

32
all the policies issued by AHAC(AIU) (Exhibits "C", "D", "E", "F" and "G"). From this fact the Court must
consequently agree with the position of defendant that the endorsement rider (Exhibit "7-C") means that only
the two swimming pools were insured against earthquake shock.
Plaintiff correctly points out that a policy of insurance is a contract of adhesion hence, where the language
used in an insurance contract or application is such as to create ambiguity the same should be resolved
against the party responsible therefor, i.e., the insurance company which prepared the contract. To the mind
of [the] Court, the language used in the policy in litigation is clear and unambiguous hence there is no need
for interpretation or construction but only application of the provisions therein.
From the above observations the Court finds that only the two (2) swimming pools had earthquake shock
coverage and were heavily damaged by the earthquake which struck on July 16, 1990. Defendant having
admitted that the damage to the swimming pools was appraised by defendants adjuster at P386,000.00,
defendant must, by virtue of the contract of insurance, pay plaintiff said amount.
Because it is the finding of the Court as stated in the immediately preceding paragraph that defendant is
liable only for the damage caused to the two (2) swimming pools and that defendant has made known to
plaintiff its willingness and readiness to settle said liability, there is no basis for the grant of the other
damages prayed for by plaintiff. As to the counterclaims of defendant, the Court does not agree that the
action filed by plaintiff is baseless and highly speculative since such action is a lawful exercise of the
plaintiffs right to come to Court in the honest belief that their Complaint is meritorious. The prayer, therefore,
of defendant for damages is likewise denied.
WHEREFORE, premises considered, defendant is ordered to pay plaintiffs the sum of THREE HUNDRED
EIGHTY SIX THOUSAND PESOS (P386,000.00) representing damage to the two (2) swimming pools, with
interest at 6% per annum from the date of the filing of the Complaint until defendants obligation to plaintiff is
fully paid.
No pronouncement as to costs.13
Petitioners Motion for Reconsideration was denied. Thus, petitioner filed an appeal with the Court of Appeals based
on the following assigned errors:14
A. THE TRIAL COURT ERRED IN FINDING THAT PLAINTIFF-APPELLANT CAN ONLY RECOVER FOR
THE DAMAGE TO ITS TWO SWIMMING POOLS UNDER ITS FIRE POLICY NO. 31944, CONSIDERING
ITS PROVISIONS, THE CIRCUMSTANCES SURROUNDING THE ISSUANCE OF SAID POLICY AND THE
ACTUATIONS OF THE PARTIES SUBSEQUENT TO THE EARTHQUAKE OF JULY 16, 1990.
B. THE TRIAL COURT ERRED IN DETERMINING PLAINTIFF-APPELLANTS RIGHT TO RECOVER
UNDER DEFENDANT-APPELLEES POLICY (NO. 31944; EXH "I") BY LIMITING ITSELF TO A
CONSIDERATION OF THE SAID POLICY ISOLATED FROM THE CIRCUMSTANCES SURROUNDING ITS
ISSUANCE AND THE ACTUATIONS OF THE PARTIES AFTER THE EARTHQUAKE OF JULY 16, 1990.
C. THE TRIAL COURT ERRED IN NOT HOLDING THAT PLAINTIFF-APPELLANT IS ENTITLED TO THE
DAMAGES CLAIMED, WITH INTEREST COMPUTED AT 24% PER ANNUM ON CLAIMS ON PROCEEDS
OF POLICY.
On the other hand, respondent filed a partial appeal, assailing the lower courts failure to award it attorneys fees
and damages on its compulsory counterclaim.
After review, the appellate court affirmed the decision of the trial court and ruled, thus:
However, after carefully perusing the documentary evidence of both parties, We are not convinced that the
last two (2) insurance contracts (Exhs. "G" and "H"), which the plaintiff-appellant had with AHAC (AIU) and
upon which the subject insurance contract with Philippine Charter Insurance Corporation is said to have

33
been based and copied (Exh. "I"), covered an extended earthquake shock insurance on all the insured
properties.
xxx
We also find that the Court a quo was correct in not granting the plaintiff-appellants prayer for the imposition
of interest 24% on the insurance claim and 6% on loss of income allegedly amounting toP4,280,000.00.
Since the defendant-appellant has expressed its willingness to pay the damage caused on the two (2)
swimming pools, as the Court a quo and this Court correctly found it to be liable only, it then cannot be said
that it was in default and therefore liable for interest.
Coming to the defendant-appellants prayer for an attorneys fees, long-standing is the rule that the award
thereof is subject to the sound discretion of the court. Thus, if such discretion is well-exercised, it will not be
disturbed on appeal (Castro et al. v. CA, et al., G.R. No. 115838, July 18, 2002). Moreover, being the award
thereof an exception rather than a rule, it is necessary for the court to make findings of facts and law that
would bring the case within the exception and justify the grant of such award (Country Bankers Insurance
Corp. v. Lianga Bay and Community Multi-Purpose Coop., Inc., G.R. No. 136914, January 25, 2002).
Therefore, holding that the plaintiff-appellants action is not baseless and highly speculative, We find that the
Court a quo did not err in granting the same.
WHEREFORE, in view of all the foregoing, both appeals are hereby DISMISSED and judgment of the Trial
Court hereby AFFIRMED in toto. No costs.15
Petitioner filed the present petition raising the following issues:16
A. WHETHER THE COURT OF APPEALS CORRECTLY HELD THAT UNDER RESPONDENTS
INSURANCE POLICY NO. 31944, ONLY THE TWO (2) SWIMMING POOLS, RATHER THAN ALL THE
PROPERTIES COVERED THEREUNDER, ARE INSURED AGAINST THE RISK OF EARTHQUAKE
SHOCK.
B. WHETHER THE COURT OF APPEALS CORRECTLY DENIED PETITIONERS PRAYER FOR
DAMAGES WITH INTEREST THEREON AT THE RATE CLAIMED, ATTORNEYS FEES AND EXPENSES
OF LITIGATION.
Petitioner contends:
First, that the policys earthquake shock endorsement clearly covers all of the properties insured and not only the
swimming pools. It used the words "any property insured by this policy," and it should be interpreted as all inclusive.
Second, the unqualified and unrestricted nature of the earthquake shock endorsement is confirmed in the body of
the insurance policy itself, which states that it is "[s]ubject to: Other Insurance Clause, Typhoon
Endorsement,Earthquake Shock Endt., Extended Coverage Endt., FEA Warranty & Annual Payment Agreement
On Long Term Policies."17
Third, that the qualification referring to the two swimming pools had already been deleted in the earthquake shock
endorsement.
Fourth, it is unbelievable for respondent to claim that it only made an inadvertent omission when it deleted the said
qualification.
Fifth, that the earthquake shock endorsement rider should be given precedence over the wording of the insurance
policy, because the rider is the more deliberate expression of the agreement of the contracting parties.
Sixth, that in their previous insurance policies, limits were placed on the endorsements/warranties enumerated at
the time of issue.

34
Seventh, any ambiguity in the earthquake shock endorsement should be resolved in favor of petitioner and against
respondent. It was respondent which caused the ambiguity when it made the policy in issue.
Eighth, the qualification of the endorsement limiting the earthquake shock endorsement should be interpreted as a
caveat on the standard fire insurance policy, such as to remove the two swimming pools from the coverage for the
risk of fire. It should not be used to limit the respondents liability for earthquake shock to the two swimming pools
only.
Ninth, there is no basis for the appellate court to hold that the additional premium was not paid under the extended
coverage. The premium for the earthquake shock coverage was already included in the premium paid for the policy.
Tenth, the parties contemporaneous and subsequent acts show that they intended to extend earthquake shock
coverage to all insured properties. When it secured an insurance policy from respondent, petitioner told respondent
that it wanted an exact replica of its latest insurance policy from American Home Assurance Company (AHAC-AIU),
which covered all the resorts properties for earthquake shock damage and respondent agreed. After the July 16,
1990 earthquake, respondent assured petitioner that it was covered for earthquake shock. Respondents insurance
adjuster, Bayne Adjusters and Surveyors, Inc., likewise requested petitioner to submit the necessary documents for
its building claims and other repair costs. Thus, under the doctrine of equitable estoppel, it cannot deny that the
insurance policy it issued to petitioner covered all of the properties within the resort.
Eleventh, that it is proper for it to avail of a petition for review by certiorari under Rule 45 of the Revised Rules of
Court as its remedy, and there is no need for calibration of the evidence in order to establish the facts upon which
this petition is based.
On the other hand, respondent made the following counter arguments: 18
First, none of the previous policies issued by AHAC-AIU from 1983 to 1990 explicitly extended coverage against
earthquake shock to petitioners insured properties other than on the two swimming pools. Petitioner admitted that
from 1984 to 1988, only the two swimming pools were insured against earthquake shock. From 1988 until 1990, the
provisions in its policy were practically identical to its earlier policies, and there was no increase in the premium
paid. AHAC-AIU, in a letter19 by its representative Manuel C. Quijano, categorically stated that its previous policy,
from which respondents policy was copied, covered only earthquake shock for the two swimming pools.
Second, petitioners payment of additional premium in the amount of P393.00 shows that the policy only covered
earthquake shock damage on the two swimming pools. The amount was the same amount paid by petitioner for
earthquake shock coverage on the two swimming pools from 1990-1991. No additional premium was paid to
warrant coverage of the other properties in the resort.
Third, the deletion of the phrase pertaining to the limitation of the earthquake shock endorsement to the two
swimming pools in the policy schedule did not expand the earthquake shock coverage to all of petitioners
properties. As per its agreement with petitioner, respondent copied its policy from the AHAC-AIU policy provided by
petitioner. Although the first five policies contained the said qualification in their riders title, in the last two policies,
this qualification in the title was deleted. AHAC-AIU, through Mr. J. Baranda III, stated that such deletion was a mere
inadvertence. This inadvertence did not make the policy incomplete, nor did it broaden the scope of the
endorsement whose descriptive title was merely enumerated. Any ambiguity in the policy can be easily resolved by
looking at the other provisions, specially the enumeration of the items insured, where only the two swimming pools
were noted as covered for earthquake shock damage.
Fourth, in its Complaint, petitioner alleged that in its policies from 1984 through 1988, the phrase "Item 5
P393,000.00 on the two swimming pools only (against the peril of earthquake shock only)" meant that only the
swimming pools were insured for earthquake damage. The same phrase is used in toto in the policies from 1989 to
1990, the only difference being the designation of the two swimming pools as "Item 3."
Fifth, in order for the earthquake shock endorsement to be effective, premiums must be paid for all the properties
covered. In all of its seven insurance policies, petitioner only paid P393.00 as premium for coverage of the

35
swimming pools against earthquake shock. No other premium was paid for earthquake shock coverage on the other
properties. In addition, the use of the qualifier "ANY" instead of "ALL" to describe the property covered was done
deliberately to enable the parties to specify the properties included for earthquake coverage.
Sixth, petitioner did not inform respondent of its requirement that all of its properties must be included in the
earthquake shock coverage. Petitioners own evidence shows that it only required respondent to follow the exact
provisions of its previous policy from AHAC-AIU. Respondent complied with this requirement. Respondents only
deviation from the agreement was when it modified the provisions regarding the replacement cost endorsement.
With regard to the issue under litigation, the riders of the old policy and the policy in issue are identical.
Seventh, respondent did not do any act or give any assurance to petitioner as would estop it from maintaining that
only the two swimming pools were covered for earthquake shock. The adjusters letter notifying petitioner to present
certain documents for its building claims and repair costs was given to petitioner before the adjuster knew the full
coverage of its policy.
Petitioner anchors its claims on AHAC-AIUs inadvertent deletion of the phrase "Item 5 Only" after the descriptive
name or title of the Earthquake Shock Endorsement. However, the words of the policy reflect the parties clear
intention to limit earthquake shock coverage to the two swimming pools.
Before petitioner accepted the policy, it had the opportunity to read its conditions. It did not object to any deficiency
nor did it institute any action to reform the policy. The policy binds the petitioner.
Eighth, there is no basis for petitioner to claim damages, attorneys fees and litigation expenses. Since respondent
was willing and able to pay for the damage caused on the two swimming pools, it cannot be considered to be in
default, and therefore, it is not liable for interest.
We hold that the petition is devoid of merit.
In Insurance Policy No. 31944, four key items are important in the resolution of the case at bar.
First, in the designation of location of risk, only the two swimming pools were specified as included, viz:
ITEM 3 393,000.00 On the two (2) swimming pools only (against the peril of earthquake shock only) 20
Second, under the breakdown for premium payments,21 it was stated that:

PREMIUM RECAPITULATION

ITEM NOS.

AMOUNT

RATES

PREMIUM

393,000.00

0.100%-E/S

393.0022]

xxx

Third, Policy Condition No. 6 stated:

36
6. This insurance does not cover any loss or damage occasioned by or through or in consequence, directly
or indirectly of any of the following occurrences, namely:-(a) Earthquake, volcanic eruption or other convulsion of nature.

23

Fourth, the rider attached to the policy, titled "Extended Coverage Endorsement (To Include the Perils of Explosion,
Aircraft, Vehicle and Smoke)," stated, viz:
ANNUAL PAYMENT AGREEMENT ON
LONG TERM POLICIES
THE INSURED UNDER THIS POLICY HAVING ESTABLISHED AGGREGATE SUMS INSURED IN
EXCESS OF FIVE MILLION PESOS, IN CONSIDERATION OF A DISCOUNT OF 5% OR 7 % OF THE
NET PREMIUM x x x POLICY HEREBY UNDERTAKES TO CONTINUE THE INSURANCE UNDER THE
ABOVE NAMED x x x AND TO PAY THE PREMIUM.
Earthquake Endorsement
In consideration of the payment by the Insured to the Company of the sum of P. . . . . . . . . . . . . . . . .
additional premium the Company agrees, notwithstanding what is stated in the printed conditions of this
Policy to the contrary, that this insurance covers loss or damage (including loss or damage by fire) to any of
the property insured by this Policy occasioned by or through or in consequence of Earthquake.
Provided always that all the conditions of this Policy shall apply (except in so far as they may be hereby
expressly varied) and that any reference therein to loss or damage by fire should be deemed to apply also to
loss or damage occasioned by or through or in consequence of Earthquake. 24
Petitioner contends that pursuant to this rider, no qualifications were placed on the scope of the earthquake shock
coverage. Thus, the policy extended earthquake shock coverage to all of the insured properties.
It is basic that all the provisions of the insurance policy should be examined and interpreted in consonance with
each other.25 All its parts are reflective of the true intent of the parties. The policy cannot be construed piecemeal.
Certain stipulations cannot be segregated and then made to control; neither do particular words or phrases
necessarily determine its character. Petitioner cannot focus on the earthquake shock endorsement to the exclusion
of the other provisions. All the provisions and riders, taken and interpreted together, indubitably show the intention of
the parties to extend earthquake shock coverage to the two swimming pools only.
A careful examination of the premium recapitulation will show that it is the clear intent of the parties to extend
earthquake shock coverage only to the two swimming pools. Section 2(1) of the Insurance Code defines a contract
of insurance as an agreement whereby one undertakes for a consideration to indemnify another against loss,
damage or liability arising from an unknown or contingent event. Thus, an insurance contract exists where the
following elements concur:
1. The insured has an insurable interest;
2. The insured is subject to a risk of loss by the happening of the designated peril;
3. The insurer assumes the risk;
4. Such assumption of risk is part of a general scheme to distribute actual losses among a large group of
persons bearing a similar risk; and
5. In consideration of the insurer's promise, the insured pays a premium.26 (Emphasis ours)

37
An insurance premium is the consideration paid an insurer for undertaking to indemnify the insured against a
specified peril.27 In fire, casualty, and marine insurance, the premium payable becomes a debt as soon as the risk
attaches.28 In the subject policy, no premium payments were made with regard to earthquake shock coverage,
except on the two swimming pools. There is no mention of any premium payable for the other resort properties with
regard to earthquake shock. This is consistent with the history of petitioners previous insurance policies from
AHAC-AIU. As borne out by petitioners witnesses:
CROSS EXAMINATION OF LEOPOLDO MANTOHAC TSN, November 25, 1991
pp. 12-13
Q. Now Mr. Mantohac, will it be correct to state also that insofar as your insurance policy during the period
from March 4, 1984 to March 4, 1985 the coverage on earthquake shock was limited to the two swimming
pools only?
A. Yes, sir. It is limited to the two swimming pools, specifically shown in the warranty, there is a provision
here that it was only for item 5.
Q. More specifically Item 5 states the amount of P393,000.00 corresponding to the two swimming pools
only?
A. Yes, sir.
CROSS EXAMINATION OF LEOPOLDO MANTOHAC TSN, November 25, 1991
pp. 23-26
Q. For the period from March 14, 1988 up to March 14, 1989, did you personally arrange for the
procurement of this policy?
A. Yes, sir.
Q. Did you also do this through your insurance agency?
A. If you are referring to Forte Insurance Agency, yes.
Q. Is Forte Insurance Agency a department or division of your company?
A. No, sir. They are our insurance agency.
Q. And they are independent of your company insofar as operations are concerned?
A. Yes, sir, they are separate entity.
Q. But insofar as the procurement of the insurance policy is concerned they are of course subject to your
instruction, is that not correct?
A. Yes, sir. The final action is still with us although they can recommend what insurance to take.
Q. In the procurement of the insurance police (sic) from March 14, 1988 to March 14, 1989, did you give
written instruction to Forte Insurance Agency advising it that the earthquake shock coverage must extend to
all properties of Agoo Playa Resort in La Union?
A. No, sir. We did not make any written instruction, although we made an oral instruction to that effect of
extending the coverage on (sic) the other properties of the company.

38
Q. And that instruction, according to you, was very important because in April 1987 there was an earthquake
tremor in La Union?
A. Yes, sir.
Q. And you wanted to protect all your properties against similar tremors in the [future], is that correct?
A. Yes, sir.
Q. Now, after this policy was delivered to you did you bother to check the provisions with respect to your
instructions that all properties must be covered again by earthquake shock endorsement?
A. Are you referring to the insurance policy issued by American Home Assurance Company marked Exhibit
"G"?
Atty. Mejia: Yes.
Witness:
A. I examined the policy and seeing that the warranty on the earthquake shock endorsement has no more
limitation referring to the two swimming pools only, I was contented already that the previous limitation
pertaining to the two swimming pools was already removed.
Petitioner also cited and relies on the attachment of the phrase "Subject to: Other Insurance Clause, Typhoon
Endorsement, Earthquake Shock Endorsement, Extended Coverage Endorsement, FEA Warranty & Annual
Payment Agreement on Long Term Policies"29 to the insurance policy as proof of the intent of the parties to
extend the coverage for earthquake shock. However, this phrase is merely an enumeration of the descriptive titles of
the riders, clauses, warranties or endorsements to which the policy is subject, as required under Section 50,
paragraph 2 of the Insurance Code.
We also hold that no significance can be placed on the deletion of the qualification limiting the coverage to the two
swimming pools. The earthquake shock endorsement cannot stand alone. As explained by the testimony of Juan
Baranda III, underwriter for AHAC-AIU:
DIRECT EXAMINATION OF JUAN BARANDA III30
TSN, August 11, 1992
pp. 9-12
Atty. Mejia:
We respectfully manifest that the same exhibits C to H inclusive have been previously marked by
counsel for defendant as Exhibit[s] 1-6 inclusive. Did you have occasion to review of (sic) these six
(6) policies issued by your company [in favor] of Agoo Playa Resort?
WITNESS:
Yes[,] I remember having gone over these policies at one point of time, sir.
Q. Now, wach (sic) of these six (6) policies marked in evidence as Exhibits C to H respectively carries an
earthquake shock endorsement[?] My question to you is, on the basis on (sic) the wordings indicated in
Exhibits C to H respectively what was the extent of the coverage [against] the peril of earthquake shock as
provided for in each of the six (6) policies?
xxx

39
WITNESS:
The extent of the coverage is only up to the two (2) swimming pools, sir.
Q. Is that for each of the six (6) policies namely: Exhibits C, D, E, F, G and H?
A. Yes, sir.
ATTY. MEJIA:
What is your basis for stating that the coverage against earthquake shock as provided for in each of
the six (6) policies extend to the two (2) swimming pools only?
WITNESS:
Because it says here in the policies, in the enumeration "Earthquake Shock Endorsement, in the
Clauses and Warranties: Item 5 only (Earthquake Shock Endorsement)," sir.
ATTY. MEJIA:
Witness referring to Exhibit C-1, your Honor.
WITNESS:
We do not normally cover earthquake shock endorsement on stand alone basis. For swimming pools
we do cover earthquake shock. For building we covered it for full earthquake coverage which
includes earthquake shock
COURT:
As far as earthquake shock endorsement you do not have a specific coverage for other things other
than swimming pool? You are covering building? They are covered by a general insurance?
WITNESS:
Earthquake shock coverage could not stand alone. If we are covering building or another we can
issue earthquake shock solely but that the moment I see this, the thing that comes to my mind is
either insuring a swimming pool, foundations, they are normally affected by earthquake but not by
fire, sir.
DIRECT EXAMINATION OF JUAN BARANDA III
TSN, August 11, 1992
pp. 23-25
Q. Plaintiffs witness, Mr. Mantohac testified and he alleged that only Exhibits C, D, E and F inclusive
[remained] its coverage against earthquake shock to two (2) swimming pools only but that Exhibits G and H
respectively entend the coverage against earthquake shock to all the properties indicated in the respective
schedules attached to said policies, what can you say about that testimony of plaintiffs witness?
WITNESS:
As I have mentioned earlier, earthquake shock cannot stand alone without the other half of it. I
assure you that this one covers the two swimming pools with respect to earthquake shock
endorsement. Based on it, if we are going to look at the premium there has been no change with

40
respect to the rates. Everytime (sic) there is a renewal if the intention of the insurer was to include
the earthquake shock, I think there is a substantial increase in the premium. We are not only going
to consider the two (2) swimming pools of the other as stated in the policy. As I see, there is no
increase in the amount of the premium. I must say that the coverage was not broaden (sic) to
include the other items.
COURT:
They are the same, the premium rates?
WITNESS:
They are the same in the sence (sic), in the amount of the coverage. If you are going to do some
computation based on the rates you will arrive at the same premiums, your Honor.
CROSS-EXAMINATION OF JUAN BARANDA III
TSN, September 7, 1992
pp. 4-6
ATTY. ANDRES:
Would you as a matter of practice [insure] swimming pools for fire insurance?
WITNESS:
No, we dont, sir.
Q. That is why the phrase "earthquake shock to the two (2) swimming pools only" was placed, is it not?
A. Yes, sir.
ATTY. ANDRES:
Will you not also agree with me that these exhibits, Exhibits G and H which you have pointed to
during your direct-examination, the phrase "Item no. 5 only" meaning to (sic) the two (2) swimming
pools was deleted from the policies issued by AIU, is it not?
xxx
ATTY. ANDRES:
As an insurance executive will you not attach any significance to the deletion of the qualifying phrase
for the policies?
WITNESS:
My answer to that would be, the deletion of that particular phrase is inadvertent. Being a company
underwriter, we do not cover. . it was inadvertent because of the previous policies that we have
issued with no specific attachments, premium rates and so on. It was inadvertent, sir.
The Court also rejects petitioners contention that respondents contemporaneous and subsequent acts to the
issuance of the insurance policy falsely gave the petitioner assurance that the coverage of the earthquake shock
endorsement included all its properties in the resort. Respondent only insured the properties as intended by the
petitioner. Petitioners own witness testified to this agreement, viz:

41
CROSS EXAMINATION OF LEOPOLDO MANTOHAC
TSN, January 14, 1992
pp. 4-5
Q. Just to be clear about this particular answer of yours Mr. Witness, what exactly did you tell Atty. Omlas
(sic) to copy from Exhibit "H" for purposes of procuring the policy from Philippine Charter Insurance
Corporation?
A. I told him that the insurance that they will have to get will have the same provisions as this American
Home Insurance Policy No. 206-4568061-9.
Q. You are referring to Exhibit "H" of course?
A. Yes, sir, to Exhibit "H".
Q. So, all the provisions here will be the same except that of the premium rates?
A. Yes, sir. He assured me that with regards to the insurance premium rates that they will be charging will be
limited to this one. I (sic) can even be lesser.
CROSS EXAMINATION OF LEOPOLDO MANTOHAC
TSN, January 14, 1992
pp. 12-14
Atty. Mejia:
Q. Will it be correct to state[,] Mr. Witness, that you made a comparison of the provisions and scope of
coverage of Exhibits "I" and "H" sometime in the third week of March, 1990 or thereabout?
A. Yes, sir, about that time.
Q. And at that time did you notice any discrepancy or difference between the policy wordings as well as
scope of coverage of Exhibits "I" and "H" respectively?
A. No, sir, I did not discover any difference inasmuch (sic) as I was assured already that the policy wordings
and rates were copied from the insurance policy I sent them but it was only when this case erupted that we
discovered some discrepancies.
Q. With respect to the items declared for insurance coverage did you notice any discrepancy at any time
between those indicated in Exhibit "I" and those indicated in Exhibit "H" respectively?
A. With regard to the wordings I did not notice any difference because it was exactly the same P393,000.00
on the two (2) swimming pools only against the peril of earthquake shock which I understood before that this
provision will have to be placed here because this particular provision under the peril of earthquake shock
only is requested because this is an insurance policy and therefore cannot be insured against fire, so this
has to be placed.
The verbal assurances allegedly given by respondents representative Atty. Umlas were not proved. Atty. Umlas
categorically denied having given such assurances.
Finally, petitioner puts much stress on the letter of respondents independent claims adjuster, Bayne Adjusters and
Surveyors, Inc. But as testified to by the representative of Bayne Adjusters and Surveyors, Inc., respondent never
meant to lead petitioner to believe that the endorsement for earthquake shock covered properties other than the two
swimming pools, viz:

42
DIRECT EXAMINATION OF ALBERTO DE LEON (Bayne Adjusters and Surveyors, Inc.)
TSN, January 26, 1993
pp. 22-26
Q. Do you recall the circumstances that led to your discussion regarding the extent of coverage of the policy
issued by Philippine Charter Insurance Corporation?
A. I remember that when I returned to the office after the inspection, I got a photocopy of the insurance
coverage policy and it was indicated under Item 3 specifically that the coverage is only for earthquake
shock. Then, I remember I had a talk with Atty. Umlas (sic), and I relayed to him what I had found out in the
policy and he confirmed to me indeed only Item 3 which were the two swimming pools have coverage for
earthquake shock.
xxx
Q. Now, may we know from you Engr. de Leon your basis, if any, for stating that except for the swimming
pools all affected items have no coverage for earthquake shock?
xxx
A. I based my statement on my findings, because upon my examination of the policy I found out that under
Item 3 it was specific on the wordings that on the two swimming pools only, then enclosed in parenthesis
(against the peril[s] of earthquake shock only), and secondly, when I examined the summary of premium
payment only Item 3 which refers to the swimming pools have a computation for premium payment for
earthquake shock and all the other items have no computation for payment of premiums.
In sum, there is no ambiguity in the terms of the contract and its riders. Petitioner cannot rely on the general rule
that insurance contracts are contracts of adhesion which should be liberally construed in favor of the insured and
strictly against the insurer company which usually prepares it.31 A contract of adhesion is one wherein a party,
usually a corporation, prepares the stipulations in the contract, while the other party merely affixes his signature or
his "adhesion" thereto. Through the years, the courts have held that in these type of contracts, the parties do not
bargain on equal footing, the weaker party's participation being reduced to the alternative to take it or leave it. Thus,
these contracts are viewed as traps for the weaker party whom the courts of justice must protect. 32Consequently,
any ambiguity therein is resolved against the insurer, or construed liberally in favor of the insured. 33
The case law will show that this Court will only rule out blind adherence to terms where facts and circumstances will
show that they are basically one-sided.34 Thus, we have called on lower courts to remain careful in scrutinizing the
factual circumstances behind each case to determine the efficacy of the claims of contending parties.
In Development Bank of the Philippines v. National Merchandising Corporation, et al.,35 the parties, who were
acute businessmen of experience, were presumed to have assented to the assailed documents with full knowledge.
We cannot apply the general rule on contracts of adhesion to the case at bar. Petitioner cannot claim it did not know
the provisions of the policy. From the inception of the policy, petitioner had required the respondent to
copyverbatim the provisions and terms of its latest insurance policy from AHAC-AIU. The testimony of Mr. Leopoldo
Mantohac, a direct participant in securing the insurance policy of petitioner, is reflective of petitioners
knowledge,viz:
DIRECT EXAMINATION OF LEOPOLDO MANTOHAC36
TSN, September 23, 1991
pp. 20-21
Q. Did you indicate to Atty. Omlas (sic) what kind of policy you would want for those facilities in Agoo Playa?

43
A. Yes, sir. I told him that I will agree to that renewal of this policy under Philippine Charter Insurance
Corporation as long as it will follow the same or exact provisions of the previous insurance policy we had
with American Home Assurance Corporation.
Q. Did you take any step Mr. Witness to ensure that the provisions which you wanted in the American Home
Insurance policy are to be incorporated in the PCIC policy?
A. Yes, sir.
Q. What steps did you take?
A. When I examined the policy of the Philippine Charter Insurance Corporation I specifically told him that the
policy and wordings shall be copied from the AIU Policy No. 206-4568061-9.
Respondent, in compliance with the condition set by the petitioner, copied AIU Policy No. 206-4568061-9 in drafting
its Insurance Policy No. 31944. It is true that there was variance in some terms, specifically in the replacement cost
endorsement, but the principal provisions of the policy remained essentially similar to AHAC-AIUs policy.
Consequently, we cannot apply the "fine print" or "contract of adhesion" rule in this case as the parties intent to limit
the coverage of the policy to the two swimming pools only is not ambiguous. 37
IN VIEW WHEREOF, the judgment of the Court of Appeals is affirmed. The petition for certiorari is dismissed. No
costs.
SO ORDERED.

44

Republic of the Philippines


SUPREME COURT
Manila
SECOND DIVISION
G.R. No. L-52756 October 12, 1987
MANILA MAHOGANY MANUFACTURING CORPORATION, petitioner,
vs.
COURT OF APPEALS AND ZENITH INSURANCE CORPORATION, respondents.

PADILLA, J:
Petition to review the decision * of the Court of Appeals, in CA-G.R. No. SP-08642, dated 21 March 1979, ordering petitioner Manila Mahogany
Manufacturing Corporation to pay private respondent Zenith Insurance Corporation the sum of Five Thousand Pesos (P5,000.00) with 6% annual interest from 18
January 1973, attorney's fees in the sum of five hundred pesos (P500.00), and costs of suit, and the resolution of the same Court, dated 8 February 1980, denying
petitioner's motion for reconsideration of it's decision.

From 6 March 1970 to 6 March 1971, petitioner insured its Mercedes Benz 4-door sedan with respondent insurance
company. On 4 May 1970 the insured vehicle was bumped and damaged by a truck owned by San Miguel
Corporation. For the damage caused, respondent company paid petitioner five thousand pesos (P5,000.00) in
amicable settlement. Petitioner's general manager executed a Release of Claim, subrogating respondent company
to all its right to action against San Miguel Corporation.
On 11 December 1972, respondent company wrote Insurance Adjusters, Inc. to demand reimbursement from San
Miguel Corporation of the amount it had paid petitioner. Insurance Adjusters, Inc. refused reimbursement, alleging
that San Miguel Corporation had already paid petitioner P4,500.00 for the damages to petitioner's motor vehicle, as
evidenced by a cash voucher and a Release of Claim executed by the General Manager of petitioner discharging
San Miguel Corporation from "all actions, claims, demands the rights of action that now exist or hereafter [sic]
develop arising out of or as a consequence of the accident."
Respondent insurance company thus demanded from petitioner reimbursement of the sum of P4,500.00 paid by
San Miguel Corporation. Petitioner refused; hence, respondent company filed suit in the City Court of Manila for the
recovery of P4,500.00. The City Court ordered petitioner to pay respondent P4,500.00. On appeal the Court of First
Instance of Manila affirmed the City Court's decision in toto, which CFI decision was affirmed by the Court of
Appeals, with the modification that petitioner was to pay respondent the total amount of P5,000.00 that it had earlier
received from the respondent insurance company.
Petitioner now contends it is not bound to pay P4,500.00, and much more, P5,000.00 to respondent company as the
subrogation in the Release of Claim it executed in favor of respondent was conditioned on recovery of the total
amount of damages petitioner had sustained. Since total damages were valued by petitioner at P9,486.43 and only
P5,000.00 was received by petitioner from respondent, petitioner argues that it was entitled to go after San Miguel
Corporation to claim the additional P4,500.00 eventually paid to it by the latter, without having to turn over said
amount to respondent. Respondent of course disputes this allegation and states that there was no qualification to its
right of subrogation under the Release of Claim executed by petitioner, the contents of said deed having expressed
all the intents and purposes of the parties.

45
To support its alleged right not to return the P4,500.00 paid by San Miguel Corporation, petitioner cites Art. 2207 of
the Civil Code, which states:
If the plaintiff's property has been insured, and he has received indemnity from the insurance
company for the injury or loss arising out of the wrong or breach of contract complained of the
insurance company shall be subrogated to the rights of the insured against the wrongdoer or the
person who has violated the contract. If the amount paid by the insurance company does not fully
cover the injury or loss the aggrieved party shall be entitled to recover the deficiency from the person
causing the loss or injury.
Petitioner also invokes Art. 1304 of the Civil Code, stating.
A creditor, to whom partial payment has been made, may exercise his right for the remainder, and he
shall be preferred to the person who has been subrogated in his place in virtue of the partial
payment of the same credit.
We find petitioners arguments to be untenable and without merit. In the absence of any other evidence to support its
allegation that a gentlemen's agreement existed between it and respondent, not embodied in the Release of Claim,
such ease of Claim must be taken as the best evidence of the intent and purpose of the parties. Thus, the Court of
Appeals rightly stated:
Petitioner argues that the release claim it executed subrogating Private respondent to any right of
action it had against San Miguel Corporation did not preclude Manila Mahogany from filing a
deficiency claim against the wrongdoer. Citing Article 2207, New Civil Code, to the effect that if the
amount paid by an insurance company does not fully cover the loss, the aggrieved party shall be
entitled to recover the deficiency from the person causing the loss, petitioner claims a preferred right
to retain the amount coming from San Miguel Corporation, despite the subrogation in favor of Private
respondent.
Although petitioners right to file a deficiency claim against San Miguel Corporation is with legal basis,
without prejudice to the insurer's right of subrogation, nevertheless when Manila Mahogany executed
another release claim (Exhibit K) discharging San Miguel Corporation from "all actions, claims,
demands and rights of action that now exist or hereafter arising out of or as a consequence of the
accident" after the insurer had paid the proceeds of the policy- the compromise agreement of
P5,000.00 being based on the insurance policy-the insurer is entitled to recover from the insured the
amount of insurance money paid (Metropolitan Casualty Insurance Company of New York vs.
Badler, 229 N.Y.S. 61, 132 Misc. 132 cited in Insurance Code and Insolvency Law with comments
and annotations, H.B. Perez 1976, p. 151). Since petitioner by its own acts released San Miguel
Corporation, thereby defeating private respondents, the right of subrogation, the right of action of
petitioner against the insurer was also nullified. (Sy Keng & Co. vs. Queensland Insurance Co., Ltd.,
54 O.G. 391) Otherwise stated: private respondent may recover the sum of P5,000.00 it had earlier
paid to petitioner. 1
As held in Phil. Air Lines v. Heald Lumber Co., 2
If a property is insured and the owner receives the indemnity from the insurer, it is provided in [Article
2207 of the New Civil Code] that the insurer is deemed subrogated to the rights of the insured
against the wrongdoer and if the amount paid by the insurer does not fully cover the loss, then the
aggrieved party is the one entitled to recover the deficiency. ... Under this legal provision, the real

46
party in interest with regard to the portion of the indemnity paid is the insurer and not the
insured 3(Emphasis supplied)
The decision of the respondent court ordering petitioner to pay respondent company, not the P4,500.00 as originally
asked for, but P5,000.00, the amount respondent company paid petitioner as insurance, is also in accord with law
and jurisprudence. In disposing of this issue, the Court of Appeals held:
... petitioner is entitled to keep the sum of P4,500.00 paid by San Miguel Corporation under its clear
right to file a deficiency claim for damages incurred, against the wrongdoer, should the insurance
company not fully pay for the injury caused (Article 2207, New Civil Code). However, when petitioner
released San Miguel Corporation from any liability, petitioner's right to retain the sum of P5,000.00
no longer existed, thereby entitling private respondent to recover the same. (Emphasis supplied)
As has been observed:
... The right of subrogation can only exist after the insurer has paid the otherwise the insured will be
deprived of his right to full indemnity. If the insurance proceeds are not sufficient to cover the
damages suffered by the insured, then he may sue the party responsible for the damage for the the
[sic] remainder. To the extent of the amount he has already received from the insurer enjoy's [sic] the
right of subrogation.
Since the insurer can be subrogated to only such rights as the insured may have, should the
insured, after receiving payment from the insurer, release the wrongdoer who caused the loss, the
insurer loses his rights against the latter. But in such a case, the insurer will be entitled to recover
from the insured whatever it has paid to the latter, unless the release was made with the consent of
the insurer. 4 (Emphasis supplied.)
And even if the specific amount asked for in the complaint is P4,500.00 only and not P5,000.00, still, the respondent
Court acted well within its discretion in awarding P5,000.00, the total amount paid by the insurer. The Court of
Appeals rightly reasoned as follows:
It is to be noted that private respondent, in its companies, prays for the recovery, not of P5,000.00 it
had paid under the insurance policy but P4,500.00 San Miguel Corporation had paid to petitioner. On
this score, We believe the City Court and Court of First Instance erred in not awarding the proper
relief. Although private respondent prays for the reimbursement of P4,500.00 paid by San Miguel
Corporation, instead of P5,000.00 paid under the insurance policy, the trial court should have
awarded the latter, although not prayed for, under the general prayer in the complaint "for such
further or other relief as may be deemed just or equitable, (Rule 6, Sec. 3, Revised Rules of Court;
Rosales vs. Reyes Ordoveza, 25 Phil. 495 ; Cabigao vs. Lim, 50 Phil. 844; Baguiro vs. Barrios
Tupas, 77 Phil 120).
WHEREFORE, premises considered, the petition is DENIED. The judgment appealed from is hereby AFFIRMED
with costs against petitioner.
SO ORDERED.

47

Republic of the Philippines


SUPREME COURT
Manila
THIRD DIVISION
G.R. No. 150094

August 18, 2004

48
FEDERAL EXPRESS CORPORATION, petitioner,
vs.
AMERICAN HOME ASSURANCE COMPANY and PHILAM INSURANCE COMPANY, INC., respondents.

DECISION

PANGANIBAN, J.:
Basic is the requirement that before suing to recover loss of or damage to transported goods, the plaintiff must give
the carrier notice of the loss or damage, within the period prescribed by the Warsaw Convention and/or the airway
bill.
The Case
Before us is a Petition for Review1 under Rule 45 of the Rules of Court, challenging the June 4, 2001 Decision 2and
the September 21, 2001 Resolution3 of the Court of Appeals (CA) in CA-GR CV No. 58208. The assailed Decision
disposed as follows:
"WHEREFORE, premises considered, the present appeal is hereby DISMISSED for lack of merit. The
appealed Decision of Branch 149 of the Regional Trial Court of Makati City in Civil Case No. 951219,entitled 'American Home Assurance Co. and PHILAM Insurance Co., Inc. v. FEDERAL EXPRESS
CORPORATION and/or CARGOHAUS, INC. (formerly U-WAREHOUSE, INC.),' is
hereby AFFIRMED andREITERATED.
"Costs against the [petitioner and Cargohaus, Inc.]." 4
The assailed Resolution denied petitioner's Motion for Reconsideration.
The Facts
The antecedent facts are summarized by the appellate court as follows:
"On January 26, 1994, SMITHKLINE Beecham (SMITHKLINE for brevity) of Nebraska, USA delivered to
Burlington Air Express (BURLINGTON), an agent of [Petitioner] Federal Express Corporation, a shipment of
109 cartons of veterinary biologicals for delivery to consignee SMITHKLINE and French Overseas Company
in Makati City, Metro Manila. The shipment was covered by Burlington Airway Bill No. 11263825 with the
words, 'REFRIGERATE WHEN NOT IN TRANSIT' and 'PERISHABLE' stamp marked on its face. That same
day, Burlington insured the cargoes in the amount of $39,339.00 with American Home Assurance Company
(AHAC). The following day, Burlington turned over the custody of said cargoes to Federal Express which
transported the same to Manila. The first shipment, consisting of 92 cartons arrived in Manila on January 29,
1994 in Flight No. 0071-28NRT and was immediately stored at [Cargohaus Inc.'s] warehouse. While the
second, consisting of 17 cartons, came in two (2) days later, or on January 31, 1994, in Flight No. 007130NRT which was likewise immediately stored at Cargohaus' warehouse. Prior to the arrival of the cargoes,
Federal Express informed GETC Cargo International Corporation, the customs broker hired by the
consignee to facilitate the release of its cargoes from the Bureau of Customs, of the impending arrival of its
client's cargoes.
"On February 10, 1994, DARIO C. DIONEDA ('DIONEDA'), twelve (12) days after the cargoes arrived in
Manila, a non-licensed custom's broker who was assigned by GETC to facilitate the release of the subject

49
cargoes, found out, while he was about to cause the release of the said cargoes, that the same [were]
stored only in a room with two (2) air conditioners running, to cool the place instead of a refrigerator. When
he asked an employee of Cargohaus why the cargoes were stored in the 'cool room' only, the latter told him
that the cartons where the vaccines were contained specifically indicated therein that it should not be
subjected to hot or cold temperature. Thereafter, DIONEDA, upon instructions from GETC, did not proceed
with the withdrawal of the vaccines and instead, samples of the same were taken and brought to the Bureau
of Animal Industry of the Department of Agriculture in the Philippines by SMITHKLINE for examination
wherein it was discovered that the 'ELISA reading of vaccinates sera are below the positive reference
serum.'
"As a consequence of the foregoing result of the veterinary biologics test, SMITHKLINE abandoned the
shipment and, declaring 'total loss' for the unusable shipment, filed a claim with AHAC through its
representative in the Philippines, the Philam Insurance Co., Inc. ('PHILAM') which recompensed
SMITHKLINE for the whole insured amount of THIRTY NINE THOUSAND THREE HUNDRED THIRTY
NINE DOLLARS ($39,339.00). Thereafter, [respondents] filed an action for damages against the [petitioner]
imputing negligence on either or both of them in the handling of the cargo.
"Trial ensued and ultimately concluded on March 18, 1997 with the [petitioner] being held solidarily liable for
the loss as follows:
'WHEREFORE, judgment is hereby rendered in favor of [respondents] and [petitioner and its CoDefendant Cargohaus] are directed to pay [respondents], jointly and severally, the following:
1. Actual damages in the amount of the peso equivalent of US$39,339.00 with interest from
the time of the filing of the complaint to the time the same is fully paid.
2. Attorney's fees in the amount of P50,000.00 and
3. Costs of suit.
'SO ORDERED.'
"Aggrieved, [petitioner] appealed to [the CA]."5
Ruling of the Court of Appeals
The Test Report issued by the United States Department of Agriculture (Animal and Plant Health Inspection Service)
was found by the CA to be inadmissible in evidence. Despite this ruling, the appellate court held that the shipping
Receipts were a prima facie proof that the goods had indeed been delivered to the carrier in good condition. We
quote from the ruling as follows:
"Where the plaintiff introduces evidence which shows prima facie that the goods were delivered to the carrier
in good condition [i.e., the shipping receipts], and that the carrier delivered the goods in a damaged
condition, a presumption is raised that the damage occurred through the fault or negligence of the
carrier,and this casts upon the carrier the burden of showing that the goods were not in good condition when
delivered to the carrier, or that the damage was occasioned by some cause excepting the carrier from
absolute liability. This the [petitioner] failed to discharge. x x x."6
Found devoid of merit was petitioner's claim that respondents had no personality to sue. This argument was
supposedly not raised in the Answer or during trial.
Hence, this Petition.7
The Issues

50
In its Memorandum, petitioner raises the following issues for our consideration:
"I.
Are the decision and resolution of the Honorable Court of Appeals proper subject for review by the
Honorable Court under Rule 45 of the 1997 Rules of Civil Procedure?
"II.
Is the conclusion of the Honorable Court of Appeals petitioner's claim that respondents have no
personality to sue because the payment was made by the respondents to Smithkline when the insured
under the policy is Burlington Air Express is devoid of merit correct or not?
"III.
Is the conclusion of the Honorable Court of Appeals that the goods were received in good condition, correct
or not?
"IV.
Are Exhibits 'F' and 'G' hearsay evidence, and therefore, not admissible?
"V.
Is the Honorable Court of Appeals correct in ignoring and disregarding respondents' own admission that
petitioner is not liable? and
"VI.
Is the Honorable Court of Appeals correct in ignoring the Warsaw Convention?" 8
Simply stated, the issues are as follows: (1) Is the Petition proper for review by the Supreme Court? (2) Is Federal
Express liable for damage to or loss of the insured goods?
This Court's Ruling
The Petition has merit.
Preliminary Issue:
Propriety of Review
The correctness of legal conclusions drawn by the Court of Appeals from undisputed facts is a question of law
cognizable by the Supreme Court.9
In the present case, the facts are undisputed. As will be shown shortly, petitioner is questioning the conclusions
drawn from such facts. Hence, this case is a proper subject for review by this Court.
Main Issue:
Liability for Damages
Petitioner contends that respondents have no personality to sue -- thus, no cause of action against it -- because the
payment made to Smithkline was erroneous.

51
Pertinent to this issue is the Certificate of Insurance10 ("Certificate") that both opposing parties cite in support of their
respective positions. They differ only in their interpretation of what their rights are under its terms. The determination
of those rights involves a question of law, not a question of fact. "As distinguished from a question of law which
exists 'when the doubt or difference arises as to what the law is on a certain state of facts' -- 'there is a question of
fact when the doubt or difference arises as to the truth or the falsehood of alleged facts'; or when the 'query
necessarily invites calibration of the whole evidence considering mainly the credibility of witnesses, existence and
relevancy of specific surrounding circumstance, their relation to each other and to the whole and the probabilities of
the situation.'"11
Proper Payee
The Certificate specifies that loss of or damage to the insured cargo is "payable to order x x x upon surrender of this
Certificate." Such wording conveys the right of collecting on any such damage or loss, as fully as if the property
were covered by a special policy in the name of the holder itself. At the back of the Certificate appears the signature
of the representative of Burlington. This document has thus been duly indorsed in blank and is deemed a bearer
instrument.
Since the Certificate was in the possession of Smithkline, the latter had the right of collecting or of being indemnified
for loss of or damage to the insured shipment, as fully as if the property were covered by a special policy in the
name of the holder. Hence, being the holder of the Certificate and having an insurable interest in the goods,
Smithkline was the proper payee of the insurance proceeds.
Subrogation
Upon receipt of the insurance proceeds, the consignee (Smithkline) executed a subrogation Receipt 12 in favor of
respondents. The latter were thus authorized "to file claims and begin suit against any such carrier, vessel, person,
corporation or government." Undeniably, the consignee had a legal right to receive the goods in the same condition
it was delivered for transport to petitioner. If that right was violated, the consignee would have a cause of action
against the person responsible therefor.
Upon payment to the consignee of an indemnity for the loss of or damage to the insured goods, the insurer's
entitlement to subrogation pro tanto -- being of the highest equity -- equips it with a cause of action in case of a
contractual breach or negligence.13 "Further, the insurer's subrogatory right to sue for recovery under the bill of
lading in case of loss of or damage to the cargo is jurisprudentially upheld." 14
In the exercise of its subrogatory right, an insurer may proceed against an erring carrier. To all intents and purposes,
it stands in the place and in substitution of the consignee. A fortiori, both the insurer and the consignee are bound by
the contractual stipulations under the bill of lading. 15
Prescription of Claim
From the initial proceedings in the trial court up to the present, petitioner has tirelessly pointed out that respondents'
claim and right of action are already barred. The latter, and even the consignee, never filed with the carrier any
written notice or complaint regarding its claim for damage of or loss to the subject cargo within the period required
by the Warsaw Convention and/or in the airway bill. Indeed, this fact has never been denied by respondents and is
plainly evident from the records.
Airway Bill No. 11263825, issued by Burlington as agent of petitioner, states:
"6. No action shall be maintained in the case of damage to or partial loss of the shipment unless a written
notice, sufficiently describing the goods concerned, the approximate date of the damage or loss, and the
details of the claim, is presented by shipper or consignee to an office of Burlington within (14) days from the
date the goods are placed at the disposal of the person entitled to delivery, or in the case of total loss
(including non-delivery) unless presented within (120) days from the date of issue of the [Airway Bill]." 16

52
Relevantly, petitioner's airway bill states:
"12./12.1 The person entitled to delivery must make a complaint to the carrier in writing in the case:
12.1.1 of visible damage to the goods, immediately after discovery of the damage and at the latest within
fourteen (14) days from receipt of the goods;
12.1.2 of other damage to the goods, within fourteen (14) days from the date of receipt of the goods;
12.1.3 delay, within twenty-one (21) days of the date the goods are placed at his disposal; and
12.1.4 of non-delivery of the goods, within one hundred and twenty (120) days from the date of the issue of
the air waybill.
12.2 For the purpose of 12.1 complaint in writing may be made to the carrier whose air waybill was used, or
to the first carrier or to the last carrier or to the carrier who performed the transportation during which the
loss, damage or delay took place."17
Article 26 of the Warsaw Convention, on the other hand, provides:
"ART. 26. (1) Receipt by the person entitled to the delivery of baggage or goods without complaint shall be
prima facie evidence that the same have been delivered in good condition and in accordance with the
document of transportation.
(2) In case of damage, the person entitled to delivery must complain to the carrier forthwith after the
discovery of the damage, and, at the latest, within 3 days from the date of receipt in the case of baggage
and 7 days from the date of receipt in the case of goods. In case of delay the complaint must be made at the
latest within 14 days from the date on which the baggage or goods have been placed at his disposal.
(3) Every complaint must be made in writing upon the document of transportation or by separate notice in
writing dispatched within the times aforesaid.
(4) Failing complaint within the times aforesaid, no action shall lie against the carrier, save in the case of
fraud on his part."18
Condition Precedent
In this jurisdiction, the filing of a claim with the carrier within the time limitation therefor actually constitutes a
condition precedent to the accrual of a right of action against a carrier for loss of or damage to the goods. 19 The
shipper or consignee must allege and prove the fulfillment of the condition. If it fails to do so, no right of action
against the carrier can accrue in favor of the former. The aforementioned requirement is a reasonable condition
precedent; it does not constitute a limitation of action. 20
The requirement of giving notice of loss of or injury to the goods is not an empty formalism. The fundamental
reasons for such a stipulation are (1) to inform the carrier that the cargo has been damaged, and that it is being
charged with liability therefor; and (2) to give it an opportunity to examine the nature and extent of the injury. "This
protects the carrier by affording it an opportunity to make an investigation of a claim while the matter is fresh and
easily investigated so as to safeguard itself from false and fraudulent claims." 21
When an airway bill -- or any contract of carriage for that matter -- has a stipulation that requires a notice of claim for
loss of or damage to goods shipped and the stipulation is not complied with, its enforcement can be prevented and
the liability cannot be imposed on the carrier. To stress, notice is a condition precedent, and the carrier is not liable if
notice is not given in accordance with the stipulation.22 Failure to comply with such a stipulation bars recovery for the
loss or damage suffered.23

53
Being a condition precedent, the notice must precede a suit for enforcement. 24 In the present case, there is neither
an allegation nor a showing of respondents' compliance with this requirement within the prescribed period. While
respondents may have had a cause of action then, they cannot now enforce it for their failure to comply with the
aforesaid condition precedent.
In view of the foregoing, we find no more necessity to pass upon the other issues raised by petitioner.
We note that respondents are not without recourse. Cargohaus, Inc. -- petitioner's co-defendant in respondents'
Complaint below -- has been adjudged by the trial court as liable for, inter alia, "actual damages in the amount of the
peso equivalent of US $39,339."25 This judgment was affirmed by the Court of Appeals and is already final and
executory.26
WHEREFORE, the Petition is GRANTED, and the assailed Decision REVERSED insofar as it pertains to Petitioner
Federal Express Corporation. No pronouncement as to costs.
SO ORDERED.

54
Republic of the Philippines
SUPREME COURT
Baguio City
SECOND DIVISION
G.R. No. 166245

April 9, 2008

ETERNAL GARDENS MEMORIAL PARK CORPORATION, petitioner,


vs.
THE PHILIPPINE AMERICAN LIFE INSURANCE COMPANY, respondent.
DECISION
VELASCO, JR., J.:
The Case
Central to this Petition for Review on Certiorari under Rule 45 which seeks to reverse and set aside the November
26, 2004 Decision1 of the Court of Appeals (CA) in CA-G.R. CV No. 57810 is the query: May the inaction of the
insurer on the insurance application be considered as approval of the application?
The Facts
On December 10, 1980, respondent Philippine American Life Insurance Company (Philamlife) entered into an
agreement denominated as Creditor Group Life Policy No. P-19202 with petitioner Eternal Gardens Memorial Park
Corporation (Eternal). Under the policy, the clients of Eternal who purchased burial lots from it on installment basis
would be insured by Philamlife. The amount of insurance coverage depended upon the existing balance of the
purchased burial lots. The policy was to be effective for a period of one year, renewable on a yearly basis.
The relevant provisions of the policy are:
ELIGIBILITY.
Any Lot Purchaser of the Assured who is at least 18 but not more than 65 years of age, is indebted to the
Assured for the unpaid balance of his loan with the Assured, and is accepted for Life Insurance coverage by
the Company on its effective date is eligible for insurance under the Policy.
EVIDENCE OF INSURABILITY.
No medical examination shall be required for amounts of insurance up to P50,000.00. However, a
declaration of good health shall be required for all Lot Purchasers as part of the application. The Company
reserves the right to require further evidence of insurability satisfactory to the Company in respect of the
following:
1. Any amount of insurance in excess of P50,000.00.
2. Any lot purchaser who is more than 55 years of age.

55
LIFE INSURANCE BENEFIT.
The Life Insurance coverage of any Lot Purchaser at any time shall be the amount of the unpaid balance of
his loan (including arrears up to but not exceeding 2 months) as reported by the Assured to the Company or
the sum of P100,000.00, whichever is smaller. Such benefit shall be paid to the Assured if the Lot Purchaser
dies while insured under the Policy.
EFFECTIVE DATE OF BENEFIT.
The insurance of any eligible Lot Purchaser shall be effective on the date he contracts a loan with the
Assured. However, there shall be no insurance if the application of the Lot Purchaser is not approved by the
Company.3
Eternal was required under the policy to submit to Philamlife a list of all new lot purchasers, together with a copy of
the application of each purchaser, and the amounts of the respective unpaid balances of all insured lot purchasers.
In relation to the instant petition, Eternal complied by submitting a letter dated December 29, 1982, 4containing a list
of insurable balances of its lot buyers for October 1982. One of those included in the list as "new business" was a
certain John Chuang. His balance of payments was PhP 100,000. On August 2, 1984, Chuang died.
Eternal sent a letter dated August 20, 19845 to Philamlife, which served as an insurance claim for Chuangs death.
Attached to the claim were the following documents: (1) Chuangs Certificate of Death; (2) Identification Certificate
stating that Chuang is a naturalized Filipino Citizen; (3) Certificate of Claimant; (4) Certificate of Attending Physician;
and (5) Assureds Certificate.
In reply, Philamlife wrote Eternal a letter on November 12, 1984, 6 requiring Eternal to submit the following
documents relative to its insurance claim for Chuangs death: (1) Certificate of Claimant (with form attached); (2)
Assureds Certificate (with form attached); (3) Application for Insurance accomplished and signed by the insured,
Chuang, while still living; and (4) Statement of Account showing the unpaid balance of Chuang before his death.
Eternal transmitted the required documents through a letter dated November 14, 1984, 7 which was received by
Philamlife on November 15, 1984.
After more than a year, Philamlife had not furnished Eternal with any reply to the latters insurance claim. This
prompted Eternal to demand from Philamlife the payment of the claim for PhP 100,000 on April 25, 1986. 8
In response to Eternals demand, Philamlife denied Eternals insurance claim in a letter dated May 20, 1986, 9 a
portion of which reads:
The deceased was 59 years old when he entered into Contract #9558 and 9529 with Eternal Gardens
Memorial Park in October 1982 for the total maximum insurable amount of P100,000.00 each. No
application for Group Insurance was submitted in our office prior to his death on August 2, 1984.
In accordance with our Creditors Group Life Policy No. P-1920, under Evidence of Insurability provision, "a
declaration of good health shall be required for all Lot Purchasers as party of the application." We cite further
the provision on Effective Date of Coverage under the policy which states that "there shall be no insurance if
the application is not approved by the Company." Since no application had been submitted by the
Insured/Assured, prior to his death, for our approval but was submitted instead on November 15, 1984, after
his death, Mr. John Uy Chuang was not covered under the Policy. We wish to point out that Eternal Gardens
being the Assured was a party to the Contract and was therefore aware of these pertinent provisions.

56
With regard to our acceptance of premiums, these do not connote our approval per se of the insurance
coverage but are held by us in trust for the payor until the prerequisites for insurance coverage shall have
been met. We will however, return all the premiums which have been paid in behalf of John Uy Chuang.
Consequently, Eternal filed a case before the Makati City Regional Trial Court (RTC) for a sum of money against
Philamlife, docketed as Civil Case No. 14736. The trial court decided in favor of Eternal, the dispositive portion of
which reads:
WHEREFORE, premises considered, judgment is hereby rendered in favor of Plaintiff ETERNAL, against
Defendant PHILAMLIFE, ordering the Defendant PHILAMLIFE, to pay the sum of P100,000.00, representing
the proceeds of the Policy of John Uy Chuang, plus legal rate of interest, until fully paid; and, to pay the sum
of P10,000.00 as attorneys fees.
SO ORDERED.
The RTC found that Eternal submitted Chuangs application for insurance which he accomplished before his death,
as testified to by Eternals witness and evidenced by the letter dated December 29, 1982, stating, among others:
"Encl: Phil-Am Life Insurance Application Forms & Cert." 10 It further ruled that due to Philamlifes inaction from the
submission of the requirements of the group insurance on December 29, 1982 to Chuangs death on August 2,
1984, as well as Philamlifes acceptance of the premiums during the same period, Philamlife was deemed to have
approved Chuangs application. The RTC said that since the contract is a group life insurance, once proof of death
is submitted, payment must follow.
Philamlife appealed to the CA, which ruled, thus:
WHEREFORE, the decision of the Regional Trial Court of Makati in Civil Case No. 57810 is REVERSED
and SET ASIDE, and the complaint is DISMISSED. No costs.
SO ORDERED.11
The CA based its Decision on the factual finding that Chuangs application was not enclosed in Eternals letter dated
December 29, 1982. It further ruled that the non-accomplishment of the submitted application form violated Section
26 of the Insurance Code. Thus, the CA concluded, there being no application form, Chuang was not covered by
Philamlifes insurance.
Hence, we have this petition with the following grounds:
The Honorable Court of Appeals has decided a question of substance, not therefore determined by this
Honorable Court, or has decided it in a way not in accord with law or with the applicable jurisprudence, in
holding that:
I. The application for insurance was not duly submitted to respondent PhilamLife before the death of
John Chuang;
II. There was no valid insurance coverage; and
III. Reversing and setting aside the Decision of the Regional Trial Court dated May 29, 1996.
The Courts Ruling

57
As a general rule, this Court is not a trier of facts and will not re-examine factual issues raised before the CA and
first level courts, considering their findings of facts are conclusive and binding on this Court. However, such rule is
subject to exceptions, as enunciated in Sampayan v. Court of Appeals:
(1) when the findings are grounded entirely on speculation, surmises or conjectures; (2) when the inference
made is manifestly mistaken, absurd or impossible; (3) when there is grave abuse of discretion; (4) when the
judgment is based on a misapprehension of facts; (5) when the findings of facts are conflicting; (6) when in
making its findings the [CA] went beyond the issues of the case, or its findings are contrary to the
admissions of both the appellant and the appellee; (7) when the findings [of the CA] are contrary to the
trial court; (8) when the findings are conclusions without citation of specific evidence on which they are
based; (9) when the facts set forth in the petition as well as in the petitioners main and reply briefs are not
disputed by the respondent; (10) when the findings of fact are premised on the supposed absence of
evidence and contradicted by the evidence on record; and (11) when the Court of Appeals manifestly
overlooked certain relevant facts not disputed by the parties, which, if properly considered, would justify a
different conclusion.12 (Emphasis supplied.)
In the instant case, the factual findings of the RTC were reversed by the CA; thus, this Court may review them.
Eternal claims that the evidence that it presented before the trial court supports its contention that it submitted a
copy of the insurance application of Chuang before his death. In Eternals letter dated December 29, 1982, a list of
insurable interests of buyers for October 1982 was attached, including Chuang in the list of new businesses. Eternal
added it was noted at the bottom of said letter that the corresponding "Phil-Am Life Insurance Application Forms &
Cert." were enclosed in the letter that was apparently received by Philamlife on January 15, 1983. Finally, Eternal
alleged that it provided a copy of the insurance application which was signed by Chuang himself and executed
before his death.
On the other hand, Philamlife claims that the evidence presented by Eternal is insufficient, arguing that Eternal must
present evidence showing that Philamlife received a copy of Chuangs insurance application.
The evidence on record supports Eternals position.
The fact of the matter is, the letter dated December 29, 1982, which Philamlife stamped as received, states that the
insurance forms for the attached list of burial lot buyers were attached to the letter. Such stamp of receipt has the
effect of acknowledging receipt of the letter together with the attachments. Such receipt is an admission by
Philamlife against its own interest.13 The burden of evidence has shifted to Philamlife, which must prove that the
letter did not contain Chuangs insurance application. However, Philamlife failed to do so; thus, Philamlife is deemed
to have received Chuangs insurance application.
To reiterate, it was Philamlifes bounden duty to make sure that before a transmittal letter is stamped as received,
the contents of the letter are correct and accounted for.
Philamlifes allegation that Eternals witnesses ran out of credibility and reliability due to inconsistencies is
groundless. The trial court is in the best position to determine the reliability and credibility of the witnesses, because
it has the opportunity to observe firsthand the witnesses demeanor, conduct, and attitude. Findings of the trial court
on such matters are binding and conclusive on the appellate court, unless some facts or circumstances of weight
and substance have been overlooked, misapprehended, or misinterpreted, 14 that, if considered, might affect the
result of the case.15
An examination of the testimonies of the witnesses mentioned by Philamlife, however, reveals no overlooked facts
of substance and value.

58
Philamlife primarily claims that Eternal did not even know where the original insurance application of Chuang was,
as shown by the testimony of Edilberto Mendoza:
Atty. Arevalo:
Q Where is the original of the application form which is required in case of new coverage?
[Mendoza:]
A It is [a] standard operating procedure for the new client to fill up two copies of this form and the original of
this is submitted to Philamlife together with the monthly remittances and the second copy is remained or
retained with the marketing department of Eternal Gardens.
Atty. Miranda:
We move to strike out the answer as it is not responsive as counsel is merely asking for the location and
does not [ask] for the number of copy.
Atty. Arevalo:
Q Where is the original?
[Mendoza:]
A As far as I remember I do not know where the original but when I submitted with that payment together
with the new clients all the originals I see to it before I sign the transmittal letter the originals are attached
therein.16
In other words, the witness admitted not knowing where the original insurance application was, but believed that the
application was transmitted to Philamlife as an attachment to a transmittal letter.
As to the seeming inconsistencies between the testimony of Manuel Cortez on whether one or two insurance
application forms were accomplished and the testimony of Mendoza on who actually filled out the application form,
these are minor inconsistencies that do not affect the credibility of the witnesses. Thus, we ruled in People v.
Paredes that minor inconsistencies are too trivial to affect the credibility of witnesses, and these may even serve to
strengthen their credibility as these negate any suspicion that the testimonies have been rehearsed. 17
We reiterated the above ruling in Merencillo v. People:
Minor discrepancies or inconsistencies do not impair the essential integrity of the prosecutions evidence as
a whole or reflect on the witnesses honesty. The test is whether the testimonies agree on essential facts
and whether the respective versions corroborate and substantially coincide with each other so as to make a
consistent and coherent whole.18
In the present case, the number of copies of the insurance application that Chuang executed is not at issue, neither
is whether the insurance application presented by Eternal has been falsified. Thus, the inconsistencies pointed out
by Philamlife are minor and do not affect the credibility of Eternals witnesses.
However, the question arises as to whether Philamlife assumed the risk of loss without approving the application.

59
This question must be answered in the affirmative.
As earlier stated, Philamlife and Eternal entered into an agreement denominated as Creditor Group Life Policy No.
P-1920 dated December 10, 1980. In the policy, it is provided that:
EFFECTIVE DATE OF BENEFIT.
The insurance of any eligible Lot Purchaser shall be effective on the date he contracts a loan with the
Assured. However, there shall be no insurance if the application of the Lot Purchaser is not approved by the
Company.
An examination of the above provision would show ambiguity between its two sentences. The first sentence appears
to state that the insurance coverage of the clients of Eternal already became effective upon contracting a loan with
Eternal while the second sentence appears to require Philamlife to approve the insurance contract before the same
can become effective.
It must be remembered that an insurance contract is a contract of adhesion which must be construed liberally in
favor of the insured and strictly against the insurer in order to safeguard the latters interest. Thus, in Malayan
Insurance Corporation v. Court of Appeals, this Court held that:
Indemnity and liability insurance policies are construed in accordance with the general rule of resolving any
ambiguity therein in favor of the insured, where the contract or policy is prepared by the insurer. A contract
of insurance, being a contract of adhesion, par excellence, any ambiguity therein should be resolved
against the insurer; in other words, it should be construed liberally in favor of the insured and strictly
against the insurer. Limitations of liability should be regarded with extreme jealousy and must be construed
in such a way as to preclude the insurer from noncompliance with its obligations. 19 (Emphasis supplied.)
In the more recent case of Philamcare Health Systems, Inc. v. Court of Appeals, we reiterated the above ruling,
stating that:
When the terms of insurance contract contain limitations on liability, courts should construe them in such a
way as to preclude the insurer from non-compliance with his obligation. Being a contract of adhesion, the
terms of an insurance contract are to be construed strictly against the party which prepared the contract, the
insurer. By reason of the exclusive control of the insurance company over the terms and phraseology of the
insurance contract, ambiguity must be strictly interpreted against the insurer and liberally in favor of the
insured, especially to avoid forfeiture.20
Clearly, the vague contractual provision, in Creditor Group Life Policy No. P-1920 dated December 10, 1980, must
be construed in favor of the insured and in favor of the effectivity of the insurance contract.
On the other hand, the seemingly conflicting provisions must be harmonized to mean that upon a partys purchase
of a memorial lot on installment from Eternal, an insurance contract covering the lot purchaser is created and the
same is effective, valid, and binding until terminated by Philamlife by disapproving the insurance application. The
second sentence of Creditor Group Life Policy No. P-1920 on the Effective Date of Benefit is in the nature of a
resolutory condition which would lead to the cessation of the insurance contract. Moreover, the mere inaction of the
insurer on the insurance application must not work to prejudice the insured; it cannot be interpreted as a termination
of the insurance contract. The termination of the insurance contract by the insurer must be explicit and
unambiguous.

60
As a final note, to characterize the insurer and the insured as contracting parties on equal footing is inaccurate at
best. Insurance contracts are wholly prepared by the insurer with vast amounts of experience in the industry
purposefully used to its advantage. More often than not, insurance contracts are contracts of adhesion containing
technical terms and conditions of the industry, confusing if at all understandable to laypersons, that are imposed on
those who wish to avail of insurance. As such, insurance contracts are imbued with public interest that must be
considered whenever the rights and obligations of the insurer and the insured are to be delineated. Hence, in order
to protect the interest of insurance applicants, insurance companies must be obligated to act with haste upon
insurance applications, to either deny or approve the same, or otherwise be bound to honor the application as a
valid, binding, and effective insurance contract.21
WHEREFORE, we GRANT the petition. The November 26, 2004 CA Decision in CA-G.R. CV No. 57810
isREVERSED and SET ASIDE. The May 29, 1996 Decision of the Makati City RTC, Branch 138 is MODIFIED.
Philamlife is hereby ORDERED:
(1) To pay Eternal the amount of PhP 100,000 representing the proceeds of the Life Insurance Policy of
Chuang;
(2) To pay Eternal legal interest at the rate of six percent (6%) per annum of PhP 100,000 from the time of
extra-judicial demand by Eternal until Philamlifes receipt of the May 29, 1996 RTC Decision on June 17,
1996;
(3) To pay Eternal legal interest at the rate of twelve percent (12%) per annum of PhP 100,000 from June
17, 1996 until full payment of this award; and
(4) To pay Eternal attorneys fees in the amount of PhP 10,000.
No costs.
SO ORDERED.

61

CHAPTER II
Republic of the Philippines
SUPREME COURT
Manila

62
EN BANC
G.R. No. L-15895

November 29, 1920

RAFAEL ENRIQUEZ, as administrator of the estate of the late Joaquin Ma. Herrer, plaintiff-appellant,
vs.
SUN LIFE ASSURANCE COMPANY OF CANADA, defendant-appellee.
Jose A. Espiritu for appellant.
Cohn, Fisher and DeWitt for appellee.

MALCOLM, J.:
This is an action brought by the plaintiff ad administrator of the estate of the late Joaquin Ma. Herrer to recover from
the defendant life insurance company the sum of pesos 6,000 paid by the deceased for a life annuity. The trial court
gave judgment for the defendant. Plaintiff appeals.
The undisputed facts are these: On September 24, 1917, Joaquin Herrer made application to the Sun Life
Assurance Company of Canada through its office in Manila for a life annuity. Two days later he paid the sum of
P6,000 to the manager of the company's Manila office and was given a receipt reading as follows:
MANILA, I. F., 26 de septiembre, 1917.
PROVISIONAL RECEIPT Pesos 6,000
Recibi la suma de seis mil pesos de Don Joaquin Herrer de Manila como prima dela Renta Vitalicia solicitada por
dicho Don Joaquin Herrer hoy, sujeta al examen medico y aprobacion de la Oficina Central de la Compaia.
The application was immediately forwarded to the head office of the company at Montreal, Canada. On November
26, 1917, the head office gave notice of acceptance by cable to Manila. (Whether on the same day the cable was
received notice was sent by the Manila office of Herrer that the application had been accepted, is a disputed point,
which will be discussed later.) On December 4, 1917, the policy was issued at Montreal. On December 18, 1917,
attorney Aurelio A. Torres wrote to the Manila office of the company stating that Herrer desired to withdraw his
application. The following day the local office replied to Mr. Torres, stating that the policy had been issued, and
called attention to the notification of November 26, 1917. This letter was received by Mr. Torres on the morning of
December 21, 1917. Mr. Herrer died on December 20, 1917.
As above suggested, the issue of fact raised by the evidence is whether Herrer received notice of acceptance of his
application. To resolve this question, we propose to go directly to the evidence of record.
The chief clerk of the Manila office of the Sun Life Assurance Company of Canada at the time of the trial testified
that he prepared the letter introduced in evidence as Exhibit 3, of date November 26, 1917, and handed it to the
local manager, Mr. E. E. White, for signature. The witness admitted on cross-examination that after preparing the
letter and giving it to he manager, he new nothing of what became of it. The local manager, Mr. White, testified to
having received the cablegram accepting the application of Mr. Herrer from the home office on November 26, 1917.
He said that on the same day he signed a letter notifying Mr. Herrer of this acceptance. The witness further said that
letters, after being signed, were sent to the chief clerk and placed on the mailing desk for transmission. The witness

63
could not tell if the letter had every actually been placed in the mails. Mr. Tuason, who was the chief clerk, on
November 26, 1917, was not called as a witness. For the defense, attorney Manuel Torres testified to having
prepared the will of Joaquin Ma. Herrer, that on this occasion, Mr. Herrer mentioned his application for a life annuity,
and that he said that the only document relating to the transaction in his possession was the provisional receipt.
Rafael Enriquez, the administrator of the estate, testified that he had gone through the effects of the deceased and
had found no letter of notification from the insurance company to Mr. Herrer.
Our deduction from the evidence on this issue must be that the letter of November 26, 1917, notifying Mr. Herrer
that his application had been accepted, was prepared and signed in the local office of the insurance company, was
placed in the ordinary channels for transmission, but as far as we know, was never actually mailed and thus was
never received by the applicant.
Not forgetting our conclusion of fact, it next becomes necessary to determine the law which should be applied to the
facts. In order to reach our legal goal, the obvious signposts along the way must be noticed.
Until quite recently, all of the provisions concerning life insurance in the Philippines were found in the Code of
Commerce and the Civil Code. In the Code of the Commerce, there formerly existed Title VIII of Book III and Section
III of Title III of Book III, which dealt with insurance contracts. In the Civil Code there formerly existed and
presumably still exist, Chapters II and IV, entitled insurance contracts and life annuities, respectively, of Title XII of
Book IV. On the after July 1, 1915, there was, however, in force the Insurance Act. No. 2427. Chapter IV of this Act
concerns life and health insurance. The Act expressly repealed Title VIII of Book II and Section III of Title III of Book
III of the code of Commerce. The law of insurance is consequently now found in the Insurance Act and the Civil
Code.
While, as just noticed, the Insurance Act deals with life insurance, it is silent as to the methods to be followed in
order that there may be a contract of insurance. On the other hand, the Civil Code, in article 1802, not only
describes a contact of life annuity markedly similar to the one we are considering, but in two other articles, gives
strong clues as to the proper disposition of the case. For instance, article 16 of the Civil Code provides that "In
matters which are governed by special laws, any deficiency of the latter shall be supplied by the provisions of this
Code." On the supposition, therefore, which is incontestable, that the special law on the subject of insurance is
deficient in enunciating the principles governing acceptance, the subject-matter of the Civil code, if there be any,
would be controlling. In the Civil Code is found article 1262 providing that "Consent is shown by the concurrence of
offer and acceptance with respect to the thing and the consideration which are to constitute the contract. An
acceptance made by letter shall not bind the person making the offer except from the time it came to his knowledge.
The contract, in such case, is presumed to have been entered into at the place where the offer was made." This
latter article is in opposition to the provisions of article 54 of the Code of Commerce.
If no mistake has been made in announcing the successive steps by which we reach a conclusion, then the only
duty remaining is for the court to apply the law as it is found. The legislature in its wisdom having enacted a new law
on insurance, and expressly repealed the provisions in the Code of Commerce on the same subject, and having
thus left a void in the commercial law, it would seem logical to make use of the only pertinent provision of law found
in the Civil code, closely related to the chapter concerning life annuities.
The Civil Code rule, that an acceptance made by letter shall bind the person making the offer only from the date it
came to his knowledge, may not be the best expression of modern commercial usage. Still it must be admitted that
its enforcement avoids uncertainty and tends to security. Not only this, but in order that the principle may not be
taken too lightly, let it be noticed that it is identical with the principles announced by a considerable number of
respectable courts in the United States. The courts who take this view have expressly held that an acceptance of an
offer of insurance not actually or constructively communicated to the proposer does not make a contract. Only the

64
mailing of acceptance, it has been said, completes the contract of insurance, as the locus poenitentiae is ended
when the acceptance has passed beyond the control of the party. (I Joyce, The Law of Insurance, pp. 235, 244.)
In resume, therefore, the law applicable to the case is found to be the second paragraph of article 1262 of the Civil
Code providing that an acceptance made by letter shall not bind the person making the offer except from the time it
came to his knowledge. The pertinent fact is, that according to the provisional receipt, three things had to be
accomplished by the insurance company before there was a contract: (1) There had to be a medical examination of
the applicant; (2) there had to be approval of the application by the head office of the company; and (3) this approval
had in some way to be communicated by the company to the applicant. The further admitted facts are that the head
office in Montreal did accept the application, did cable the Manila office to that effect, did actually issue the policy
and did, through its agent in Manila, actually write the letter of notification and place it in the usual channels for
transmission to the addressee. The fact as to the letter of notification thus fails to concur with the essential elements
of the general rule pertaining to the mailing and delivery of mail matter as announced by the American courts,
namely, when a letter or other mail matter is addressed and mailed with postage prepaid there is a rebuttable
presumption of fact that it was received by the addressee as soon as it could have been transmitted to him in the
ordinary course of the mails. But if any one of these elemental facts fails to appear, it is fatal to the presumption. For
instance, a letter will not be presumed to have been received by the addressee unless it is shown that it was
deposited in the post-office, properly addressed and stamped. (See 22 C.J., 96, and 49 L. R. A. [N. S.], pp. 458, et
seq., notes.)
We hold that the contract for a life annuity in the case at bar was not perfected because it has not been proved
satisfactorily that the acceptance of the application ever came to the knowledge of the applicant.
lawph!l.net

Judgment is reversed, and the plaintiff shall have and recover from the defendant the sum of P6,000 with legal
interest from November 20, 1918, until paid, without special finding as to costs in either instance. So ordered.
Mapa, C.J., Araullo, Avancea and Villamor, JJ., concur.
Johnson, J., dissents.

65

Republic of the Philippines


SUPREME COURT
Manila
FIRST DIVISION
G.R. No. L-31845 April 30, 1979

66
GREAT PACIFIC LIFE ASSURANCE COMPANY, petitioner,
vs.
HONORABLE COURT OF APPEALS, respondents.
G.R. No. L-31878 April 30, 1979
LAPULAPU D. MONDRAGON, petitioner,
vs.
HON. COURT OF APPEALS and NGO HING, respondents.
Siguion Reyna, Montecillo & Ongsiako and Sycip, Salazar, Luna & Manalo for petitioner Company.
Voltaire Garcia for petitioner Mondragon.
Pelaez, Pelaez & Pelaez for respondent Ngo Hing.

DE CASTRO, J.:
The two above-entitled cases were ordered consolidated by the Resolution of this Court dated April 29, 1970, (Rollo,
No. L-31878, p. 58), because the petitioners in both cases seek similar relief, through these petitions for certiorari by
way of appeal, from the amended decision of respondent Court of Appeals which affirmed in toto the decision of the
Court of First Instance of Cebu, ordering "the defendants (herein petitioners Great Pacific Ligfe Assurance Company
and Mondragon) jointly and severally to pay plaintiff (herein private respondent Ngo Hing) the amount of P50,000.00
with interest at 6% from the date of the filing of the complaint, and the sum of P1,077.75, without interest.
It appears that on March 14, 1957, private respondent Ngo Hing filed an application with the Great Pacific Life
Assurance Company (hereinafter referred to as Pacific Life) for a twenty-year endownment policy in the amount of
P50,000.00 on the life of his one-year old daughter Helen Go. Said respondent supplied the essential data which
petitioner Lapulapu D. Mondragon, Branch Manager of the Pacific Life in Cebu City wrote on the corresponding form
in his own handwriting (Exhibit I-M). Mondragon finally type-wrote the data on the application form which was signed
by private respondent Ngo Hing. The latter paid the annual premuim the sum of P1,077.75 going over to the
Company, but he reatined the amount of P1,317.00 as his commission for being a duly authorized agebt of Pacific
Life. Upon the payment of the insurance premuim, the binding deposit receipt (Exhibit E) was issued to private
respondent Ngo Hing. Likewise, petitioner Mondragon handwrote at the bottom of the back page of the application
form his strong recommendation for the approval of the insurance application. Then on April 30, 1957, Mondragon
received a letter from Pacific Life disapproving the insurance application (Exhibit 3-M). The letter stated that the said
life insurance application for 20-year endowment plan is not available for minors below seven years old, but Pacific
Life can consider the same under the Juvenile Triple Action Plan, and advised that if the offer is acceptable, the
Juvenile Non-Medical Declaration be sent to the company.
The non-acceptance of the insurance plan by Pacific Life was allegedly not communicated by petitioner Mondragon
to private respondent Ngo Hing. Instead, on May 6, 1957, Mondragon wrote back Pacific Life again strongly
recommending the approval of the 20-year endowment insurance plan to children, pointing out that since 1954 the
customers, especially the Chinese, were asking for such coverage (Exhibit 4-M).
It was when things were in such state that on May 28, 1957 Helen Go died of influenza with complication of
bronchopneumonia. Thereupon, private respondent sought the payment of the proceeds of the insurance, but

67
having failed in his effort, he filed the action for the recovery of the same before the Court of First Instance of Cebu,
which rendered the adverse decision as earlier refered to against both petitioners.
The decisive issues in these cases are: (1) whether the binding deposit receipt (Exhibit E) constituted a temporary
contract of the life insurance in question; and (2) whether private respondent Ngo Hing concealed the state of health
and physical condition of Helen Go, which rendered void the aforesaid Exhibit E.
1. At the back of Exhibit E are condition precedents required before a deposit is considered a BINDING RECEIPT.
These conditions state that:
A. If the Company or its agent, shan have received the premium deposit ... and the insurance
application, ON or PRIOR to the date of medical examination ... said insurance shan be in force and
in effect from the date of such medical examination, for such period as is covered by the
deposit ...,PROVIDED the company shall be satisfied that on said date the applicant was insurable
on standard rates under its rule for the amount of insurance and the kind of policy requested in the
application.
D. If the Company does not accept the application on standard rate for the amount of insurance
and/or the kind of policy requested in the application but issue, or offers to issue a policy for a
different plan and/or amount ..., the insurance shall not be in force and in effect until the applicant
shall have accepted the policy as issued or offered by the Company and shall have paid the full
premium thereof. If the applicant does not accept the policy, the deposit shall be refunded.
E. If the applicant shall not have been insurable under Condition A above, and the Company
declines to approve the application the insurance applied for shall not have been in force at any time
and the sum paid be returned to the applicant upon the surrender of this receipt. (Emphasis Ours).
The aforequoted provisions printed on Exhibit E show that the binding deposit receipt is intended to be merely a
provisional or temporary insurance contract and only upon compliance of the following conditions: (1) that the
company shall be satisfied that the applicant was insurable on standard rates; (2) that if the company does not
accept the application and offers to issue a policy for a different plan, the insurance contract shall not be binding
until the applicant accepts the policy offered; otherwise, the deposit shall be reftmded; and (3) that if the applicant is
not ble according to the standard rates, and the company disapproves the application, the insurance applied for
shall not be in force at any time, and the premium paid shall be returned to the applicant.
Clearly implied from the aforesaid conditions is that the binding deposit receipt in question is merely an
acknowledgment, on behalf of the company, that the latter's branch office had received from the applicant the
insurance premium and had accepted the application subject for processing by the insurance company; and that the
latter will either approve or reject the same on the basis of whether or not the applicant is "insurable on standard
rates." Since petitioner Pacific Life disapproved the insurance application of respondent Ngo Hing, the binding
deposit receipt in question had never become in force at any time.
Upon this premise, the binding deposit receipt (Exhibit E) is, manifestly, merely conditional and does not insure
outright. As held by this Court, where an agreement is made between the applicant and the agent, no liability shall
attach until the principal approves the risk and a receipt is given by the agent. The acceptance is merely conditional
and is subordinated to the act of the company in approving or rejecting the application. Thus, in life insurance, a
"binding slip" or "binding receipt" does not insure by itself (De Lim vs. Sun Life Assurance Company of Canada, 41
Phil. 264).

68
It bears repeating that through the intra-company communication of April 30, 1957 (Exhibit 3-M), Pacific Life
disapproved the insurance application in question on the ground that it is not offering the twenty-year endowment
insurance policy to children less than seven years of age. What it offered instead is another plan known as the
Juvenile Triple Action, which private respondent failed to accept. In the absence of a meeting of the minds between
petitioner Pacific Life and private respondent Ngo Hing over the 20-year endowment life insurance in the amount of
P50,000.00 in favor of the latter's one-year old daughter, and with the non-compliance of the abovequoted
conditions stated in the disputed binding deposit receipt, there could have been no insurance contract duly
perfected between thenl Accordingly, the deposit paid by private respondent shall have to be refunded by Pacific
Life.
As held in De Lim vs. Sun Life Assurance Company of Canada, supra, "a contract of insurance, like other contracts,
must be assented to by both parties either in person or by their agents ... The contract, to be binding from the date
of the application, must have been a completed contract, one that leaves nothing to be dione, nothing to be
completed, nothing to be passed upon, or determined, before it shall take effect. There can be no contract of
insurance unless the minds of the parties have met in agreement."
We are not impressed with private respondent's contention that failure of petitioner Mondragon to communicate to
him the rejection of the insurance application would not have any adverse effect on the allegedly perfected
temporary contract (Respondent's Brief, pp. 13-14). In this first place, there was no contract perfected between the
parties who had no meeting of their minds. Private respondet, being an authorized insurance agent of Pacific Life at
Cebu branch office, is indubitably aware that said company does not offer the life insurance applied for. When he
filed the insurance application in dispute, private respondent was, therefore, only taking the chance that Pacific Life
will approve the recommendation of Mondragon for the acceptance and approval of the application in question along
with his proposal that the insurance company starts to offer the 20-year endowment insurance plan for children less
than seven years. Nonetheless, the record discloses that Pacific Life had rejected the proposal and
recommendation. Secondly, having an insurable interest on the life of his one-year old daughter, aside from being
an insurance agent and an offense associate of petitioner Mondragon, private respondent Ngo Hing must have
known and followed the progress on the processing of such application and could not pretend ignorance of the
Company's rejection of the 20-year endowment life insurance application.
At this juncture, We find it fit to quote with approval, the very apt observation of then Appellate Associate Justice
Ruperto G. Martin who later came up to this Court, from his dissenting opinion to the amended decision of the
respondent court which completely reversed the original decision, the following:
Of course, there is the insinuation that neither the memorandum of rejection (Exhibit 3-M) nor the
reply thereto of appellant Mondragon reiterating the desire for applicant's father to have the
application considered as one for a 20-year endowment plan was ever duly communicated to Ngo;
Hing, father of the minor applicant. I am not quite conninced that this was so. Ngo Hing, as father of
the applicant herself, was precisely the "underwriter who wrote this case" (Exhibit H-1). The
unchallenged statement of appellant Mondragon in his letter of May 6, 1957) (Exhibit 4-M),
specifically admits that said Ngo Hing was "our associate" and that it was the latter who "insisted that
the plan be placed on the 20-year endowment plan." Under these circumstances, it is inconceivable
that the progress in the processing of the application was not brought home to his knowledge. He
must have been duly apprised of the rejection of the application for a 20-year endowment plan
otherwise Mondragon would not have asserted that it was Ngo Hing himself who insisted on the
application as originally filed, thereby implictly declining the offer to consider the application under
the Juvenile Triple Action Plan. Besides, the associate of Mondragon that he was, Ngo Hing should
only be presumed to know what kind of policies are available in the company for minors below 7
years old. What he and Mondragon were apparently trying to do in the premises was merely to prod
the company into going into the business of issuing endowment policies for minors just as other

69
insurance companies allegedly do. Until such a definite policy is however, adopted by the company,
it can hardly be said that it could have been bound at all under the binding slip for a plan of
insurance that it could not have, by then issued at all. (Amended Decision, Rollo, pp- 52-53).
2. Relative to the second issue of alleged concealment. this Court is of the firm belief that private respondent had
deliberately concealed the state of health and piysical condition of his daughter Helen Go. Wher private regpondeit
supplied the required essential data for the insurance application form, he was fully aware that his one-year old
daughter is typically a mongoloid child. Such a congenital physical defect could never be ensconced nor disguished.
Nonetheless, private respondent, in apparent bad faith, withheld the fact materal to the risk to be assumed by the
insurance compary. As an insurance agent of Pacific Life, he ought to know, as he surely must have known. his duty
and responsibility to such a material fact. Had he diamond said significant fact in the insurance application fom
Pacific Life would have verified the same and would have had no choice but to disapprove the application outright.
The contract of insurance is one of perfect good faith uberrima fides meaning good faith, absolute and perfect
candor or openness and honesty; the absence of any concealment or demotion, however slight [Black's Law
Dictionary, 2nd Edition], not for the alone but equally so for the insurer (Field man's Insurance Co., Inc. vs. Vda de
Songco, 25 SCRA 70). Concealment is a neglect to communicate that which a partY knows aDd Ought to
communicate (Section 25, Act No. 2427). Whether intentional or unintentional the concealment entitles the insurer to
rescind the contract of insurance (Section 26, Id.: Yu Pang Cheng vs. Court of Appeals, et al, 105 Phil 930;
Satumino vs. Philippine American Life Insurance Company, 7 SCRA 316). Private respondent appears guilty thereof.
We are thus constrained to hold that no insurance contract was perfected between the parties with the
noncompliance of the conditions provided in the binding receipt, and concealment, as legally defined, having been
comraitted by herein private respondent.
WHEREFORE, the decision appealed from is hereby set aside, and in lieu thereof, one is hereby entered absolving
petitioners Lapulapu D. Mondragon and Great Pacific Life Assurance Company from their civil liabilities as found by
respondent Court and ordering the aforesaid insurance company to reimburse the amount of P1,077.75, without
interest, to private respondent, Ngo Hing. Costs against private respondent.
SO ORDERED.

70

CHAPTER III

71
Republic of the Philippines
SUPREME COURT
Manila
FIRST DIVISION

G.R. No. 124520 August 18, 1997


Spouses NILO CHA and STELLA UY CHA, and UNITED INSURANCE CO., INC., petitioners,
vs.
COURT OF APPEALS and CKS DEVELOPMENT CORPORATION, respondents.

PADILLA, J.:
This petition for review on certiorari under Rule 45 of the Rules of Court seeks to set aside a decision of respondent
Court of Appeals.
The undisputed facts of the case are as follows:
1. Petitioner-spouses Nilo Cha and Stella Uy-Cha, as lessees, entered into a lease contract with private respondent
CKS Development Corporation (hereinafter CKS), as lessor, on 5 October 1988.
2. One of the stipulations of the one (1) year lease contract states:
18. . . . The LESSEE shall not insure against fire the chattels, merchandise, textiles, goods and effects
placed at any stall or store or space in the leased premises without first obtaining the written consent and
approval of the LESSOR. If the LESSEE obtain(s) the insurance thereof without the consent of the LESSOR
then the policy is deemed assigned and transferred to the LESSOR for its own benefit; . . . 1
3. Notwithstanding the above stipulation in the lease contract, the Cha spouses insured against loss by fire the
merchandise inside the leased premises for Five Hundred Thousand (P500,000.00) with the United Insurance Co.,
Inc. (hereinafter United) without the written consent of private respondent CKS.
4. On the day that the lease contract was to expire, fire broke out inside the leased premises.
5. When CKS learned of the insurance earlier procured by the Cha spouses (without its consent), it wrote the insurer
(United) a demand letter asking that the proceeds of the insurance contract (between the Cha spouses and United)
be paid directly to CKS, based on its lease contract with the Cha spouses.
6. United refused to pay CKS. Hence, the latter filed a complaint against the Cha spouses and United.
7. On 2 June 1992, the Regional Trial Court, Branch 6, Manila, rendered a decision * ordering therein defendant
United to pay CKS the amount of P335,063.11 and defendant Cha spouses to pay P50,000.00 as exemplary
damages, P20,000.00 as attorney's fees and costs of suit.

72
8. On appeal, respondent Court of Appeals in CA GR CV No. 39328 rendered a decision ** dated 11 January 1996,
affirming the trial court decision, deleting however the awards for exemplary damages and attorney's fees. A motion
for reconsideration by United was denied on 29 March 1996.
In the present petition, the following errors are assigned by petitioners to the Court of Appeals:
I
THE HONORABLE COURT OF APPEALS ERRED IN FAILING TO DECLARE THAT THE STIPULATION IN
THE CONTRACT OF LEASE TRANSFERRING THE PROCEEDS OF THE INSURANCE TO
RESPONDENT IS NULL AND VOID FOR BEING CONTRARY TO LAW, MORALS AND PUBLIC POLICY
II
THE HONORABLE COURT OF APPEALS ERRED IN FAILING TO DECLARE THE CONTRACT OF LEASE
ENTERED INTO AS A CONTRACT OF ADHESION AND THEREFORE THE QUESTIONABLE PROVISION
THEREIN TRANSFERRING THE PROCEEDS OF THE INSURANCE TO RESPONDENT MUST BE RULED
OUT IN FAVOR OF PETITIONER
III
THE HONORABLE COURT OF APPEALS ERRED IN AWARDING PROCEEDS OF AN INSURANCE
POLICY TO APPELLEE WHICH IS NOT PRIVY TO THE SAID POLICY IN CONTRAVENTION OF THE
INSURANCE LAW
IV
THE HONORABLE COURT OF APPEALS ERRED IN AWARDING PROCEEDS OF AN INSURANCE
POLICY ON THE BASIS OF A STIPULATION WHICH IS VOID FOR BEING WITHOUT CONSIDERATION
AND FOR BEING TOTALLY DEPENDENT ON THE WILL OF THE RESPONDENT CORPORATION. 2
The core issue to be resolved in this case is whether or not the aforequoted paragraph 18 of the lease contract
entered into between CKS and the Cha spouses is valid insofar as it provides that any fire insurance policy obtained
by the lessee (Cha spouses) over their merchandise inside the leased premises is deemed assigned or transferred
to the lessor (CKS) if said policy is obtained without the prior written consent of the latter.
It is, of course, basic in the law on contracts that the stipulations contained in a contract cannot be contrary to law,
morals, good customs, public order or public policy. 3
Sec. 18 of the Insurance Code provides:
Sec. 18. No contract or policy of insurance on property shall be enforceable except for the benefit of some
person having an insurable interest in the property insured.
A non-life insurance policy such as the fire insurance policy taken by petitioner-spouses over their merchandise is
primarily a contract of indemnity. Insurable interest in the property insured must exist at the time the insurance takes
effect and at the time the loss occurs. 4 The basis of such requirement of insurable interest in property insured is based
on sound public policy: to prevent a person from taking out an insurance policy on property upon which he has no
insurable interest and collecting the proceeds of said policy in case of loss of the property. In such a case, the contract of
insurance is a mere wager which is void under Section 25 of the Insurance Code, which provides:

73
Sec. 25. Every stipulation in a policy of Insurance for the payment of loss, whether the person insured has or
has not any interest in the property insured, or that the policy shall be received as proof of such interest, and
every policy executed by way of gaming or wagering, is void.
In the present case, it cannot be denied that CKS has no insurable interest in the goods and merchandise inside the
leased premises under the provisions of Section 17 of the Insurance Code which provide:
Sec. 17. The measure of an insurable interest in property is the extent to which the insured might be
damnified by loss of injury thereof.
Therefore, respondent CKS cannot, under the Insurance Code a special law be validly a beneficiary of the fire
insurance policy taken by the petitioner-spouses over their merchandise. This insurable interest over said
merchandise remains with the insured, the Cha spouses. The automatic assignment of the policy to CKS under the
provision of the lease contract previously quoted is void for being contrary to law and/or public policy. The proceeds
of the fire insurance policy thus rightfully belong to the spouses Nilo Cha and Stella Uy-Cha (herein co-petitioners).
The insurer (United) cannot be compelled to pay the proceeds of the fire insurance policy to a person (CKS) who
has no insurable interest in the property insured.
The liability of the Cha spouses to CKS for violating their lease contract in that the Cha spouses obtained a fire
insurance policy over their own merchandise, without the consent of CKS, is a separate and distinct issue which we
do not resolve in this case.
WHEREFORE, the decision of the Court of Appeals in CA-G.R. CV No. 39328 is SET ASIDE and a new decision is
hereby entered, awarding the proceeds of the fire insurance policy to petitioners Nilo Cha and Stella Uy-Cha. SO
ORDERED.
Republic of the Philippines
SUPREME COURT
Manila
FIRST DIVISION

G.R. No. 114427 February 6, 1995


ARMANDO GEAGONIA, petitioner,
vs.
COURT OF APPEALS and COUNTRY BANKERS INSURANCE CORPORATION, respondents.

DAVIDE, JR., J.:


Four our review under Rule 45 of the Rules of Court is the decision 1 of the Court of Appeals in CA-G.R. SP No. 31916,
entitled "Country Bankers Insurance Corporation versus Armando Geagonia," reversing the decision of the Insurance
Commission in I.C. Case No. 3340 which awarded the claim of petitioner Armando Geagonia against private respondent
Country Bankers Insurance Corporation.
The petitioner is the owner of Norman's Mart located in the public market of San Francisco, Agusan del Sur. On 22
December 1989, he obtained from the private respondent fire insurance policy No. F-14622 2 for P100,000.00. The

74
period of the policy was from 22 December 1989 to 22 December 1990 and covered the following: "Stock-in-trade
consisting principally of dry goods such as RTW's for men and women wear and other usual to assured's business."

The petitioner declared in the policy under the subheading entitled CO-INSURANCE that Mercantile Insurance Co.,
Inc. was the co-insurer for P50,000.00. From 1989 to 1990, the petitioner had in his inventory stocks amounting to
P392,130.50, itemized as follows:

Zenco Sales, Inc.

F. Legaspi Gen. Merchandise

Cebu Tesing Textiles

P55,698.00

86,432.50

250,000.00 (on credit)

P392,130.50

The policy contained the following condition:


3. The insured shall give notice to the Company of any insurance or insurances already affected, or
which may subsequently be effected, covering any of the property or properties consisting of stocks
in trade, goods in process and/or inventories only hereby insured, and unless such notice be given
and the particulars of such insurance or insurances be stated therein or endorsed in this policy
pursuant to Section 50 of the Insurance Code, by or on behalf of the Company before the
occurrence of any loss or damage, all benefits under this policy shall be deemed forfeited, provided
however, that this condition shall not apply when the total insurance or insurances in force at the
time of the loss or damage is not more than P200,000.00.
On 27 May 1990, fire of accidental origin broke out at around 7:30 p.m. at the public market of San Francisco,
Agusan del Sur. The petitioner's insured stock-in-trade were completely destroyed prompting him to file with the
private respondent a claim under the policy. On 28 December 1990, the private respondent denied the claim
because it found that at the time of the loss the petitioner's stocks-in-trade were likewise covered by fire insurance
policies No. GA-28146 and No. GA-28144, for P100,000.00 each, issued by the Cebu Branch of the Philippines
First Insurance Co., Inc. (hereinafter PFIC). 3 These policies indicate that the insured was "Messrs. Discount Mart (Mr.
Armando Geagonia, Prop.)" with a mortgage clause reading:
MORTGAGE: Loss, if any shall be payable to Messrs. Cebu Tesing Textiles, Cebu City as their
interest may appear subject to the terms of this policy. CO-INSURANCE DECLARED: P100,000.
Phils. First CEB/F 24758. 4
The basis of the private respondent's denial was the petitioner's alleged violation of Condition 3 of the policy.
The petitioner then filed a complaint 5 against the private respondent with the Insurance Commission (Case No. 3340)
for the recovery of P100,000.00 under fire insurance policy No. F-14622 and for attorney's fees and costs of litigation. He
attached as Annex "AM" 6 thereof his letter of 18 January 1991 which asked for the reconsideration of the denial. He
admitted in the said letter that at the time he obtained the private respondent's fire insurance policy he knew that the two
policies issued by the PFIC were already in existence; however, he had no knowledge of the provision in the private

75
respondent's policy requiring him to inform it of the prior policies; this requirement was not mentioned to him by the private
respondent's agent; and had it been mentioned, he would not have withheld such information. He further asserted that the
total of the amounts claimed under the three policies was below the actual value of his stocks at the time of loss, which
was P1,000,000.00.

In its answer, 7 the private respondent specifically denied the allegations in the complaint and set up as its principal
defense the violation of Condition 3 of the policy.
In its decision of 21 June 1993, 8 the Insurance Commission found that the petitioner did not violate Condition 3 as he
had no knowledge of the existence of the two fire insurance policies obtained from the PFIC; that it was Cebu Tesing
Textiles which procured the PFIC policies without informing him or securing his consent; and that Cebu Tesing Textile, as
his creditor, had insurable interest on the stocks. These findings were based on the petitioner's testimony that he came to
know of the PFIC policies only when he filed his claim with the private respondent and that Cebu Tesing Textile obtained
them and paid for their premiums without informing him thereof. The Insurance Commission then decreed:
WHEREFORE, judgment is hereby rendered ordering the respondent company to pay complainant
the sum of P100,000.00 with legal interest from the time the complaint was filed until fully satisfied
plus the amount of P10,000.00 as attorney's fees. With costs. The compulsory counterclaim of
respondent is hereby dismissed.
Its motion for the reconsideration of the decision 9 having been denied by the Insurance Commission in its resolution of
20 August 1993, 10 the private respondent appealed to the Court of Appeals by way of a petition for review. The petition
was docketed as CA-G.R. SP No. 31916.
In its decision of 29 December 1993, 11 the Court of Appeals reversed the decision of the Insurance Commission
because it found that the petitioner knew of the existence of the two other policies issued by the PFIC. It said:
It is apparent from the face of Fire Policy GA 28146/Fire Policy No. 28144 that the insurance was
taken in the name of private respondent [petitioner herein]. The policy states that "DISCOUNT MART
(MR. ARMANDO GEAGONIA, PROP)" was the assured and that "TESING TEXTILES" [was] only
the mortgagee of the goods.
In addition, the premiums on both policies were paid for by private respondent, not by the Tesing
Textiles which is alleged to have taken out the other insurance without the knowledge of private
respondent. This is shown by Premium Invoices nos. 46632 and 46630. (Annexes M and N). In both
invoices, Tesing Textiles is indicated to be only the mortgagee of the goods insured but the party to
which they were issued were the "DISCOUNT MART (MR. ARMANDO GEAGONIA)."
In is clear that it was the private respondent [petitioner herein] who took out the policies on the same
property subject of the insurance with petitioner. Hence, in failing to disclose the existence of these
insurances private respondent violated Condition No. 3 of Fire Policy No. 1462. . . .
Indeed private respondent's allegation of lack of knowledge of the provisions insurances is belied by
his letter to petitioner [of 18 January 1991. The body of the letter reads as follows;]
xxx xxx xxx
Please be informed that I have no knowledge of the provision requiring me to inform
your office about my
prior insurance under FGA-28146 and F-CEB-24758. Your representative did not
mention about said requirement at the time he was convincing me to insure with you.
If he only die or even inquired if I had other existing policies covering my
establishment, I would have told him so. You will note that at the time he talked to me
until I decided to insure with your company the two policies aforementioned were
already in effect. Therefore I would have no reason to withhold such information and

76
I would have desisted to part with my hard earned peso to pay the insurance
premiums [if] I know I could not recover anything.
Sir, I am only an ordinary businessman interested in protecting my investments. The
actual value of my stocks damaged by the fire was estimated by the Police
Department to be P1,000,000.00 (Please see xerox copy of Police Report Annex
"A"). My Income Statement as of December 31, 1989 or five months before the fire,
shows my merchandise inventory was already some P595,455.75. . . . These will
support my claim that the amount claimed under the three policies are much below
the value of my stocks lost.
xxx xxx xxx
The letter contradicts private respondent's pretension that he did not know that there were other
insurances taken on the stock-in-trade and seriously puts in question his credibility.
His motion to reconsider the adverse decision having been denied, the petitioner filed the instant petition. He
contends therein that the Court of Appeals acted with grave abuse of discretion amounting to lack or excess of
jurisdiction:
A . . . WHEN IT REVERSED THE FINDINGS OF FACTS OF THE INSURANCE COMMISSION, A
QUASI-JUDICIAL BODY CHARGED WITH THE DUTY OF DETERMINING INSURANCE CLAIM
AND WHOSE DECISION IS ACCORDED RESPECT AND EVEN FINALITY BY THE COURTS;
B . . . WHEN IT CONSIDERED AS EVIDENCE MATTERS WHICH WERE NOT PRESENTED AS
EVIDENCE DURING THE HEARING OR TRIAL; AND
C . . . WHEN IT DISMISSED THE CLAIM OF THE PETITIONER HEREIN AGAINST THE
PRIVATE RESPONDENT.
The chief issues that crop up from the first and third grounds are (a) whether the petitioner had prior knowledge of
the two insurance policies issued by the PFIC when he obtained the fire insurance policy from the private
respondent, thereby, for not disclosing such fact, violating Condition 3 of the policy, and (b) if he had, whether he is
precluded from recovering therefrom.
The second ground, which is based on the Court of Appeals' reliance on the petitioner's letter of reconsideration of
18 January 1991, is without merit. The petitioner claims that the said letter was not offered in evidence and thus
should not have been considered in deciding the case. However, as correctly pointed out by the Court of Appeals, a
copy of this letter was attached to the petitioner's complaint in I.C. Case No. 3440 as Annex "M" thereof and
made integral part of the complaint. 12 It has attained the status of a judicial admission and since its due execution and
authenticity was not denied by the other party, the petitioner is bound by it even if it were not introduced as an
independent evidence. 13
As to the first issue, the Insurance Commission found that the petitioner had no knowledge of the previous two
policies. The Court of Appeals disagreed and found otherwise in view of the explicit admission by the petitioner in
his letter to the private respondent of 18 January 1991, which was quoted in the challenged decision of the Court of
Appeals. These divergent findings of fact constitute an exception to the general rule that in petitions for review under
Rule 45, only questions of law are involved and findings of fact by the Court of Appeals are conclusive and binding
upon this Court. 14
We agree with the Court of Appeals that the petitioner knew of the prior policies issued by the PFIC. His letter of 18
January 1991 to the private respondent conclusively proves this knowledge. His testimony to the contrary before the
Insurance Commissioner and which the latter relied upon cannot prevail over a written admission made ante litem
motam. It was, indeed, incredible that he did not know about the prior policies since these policies were not new or

77
original. Policy No. GA-28144 was a renewal of Policy No. F-24758, while Policy No. GA-28146 had been renewed
twice, the previous policy being F-24792.
Condition 3 of the private respondent's Policy No. F-14622 is a condition which is not proscribed by law. Its
incorporation in the policy is allowed by Section 75 of the Insurance Code 15 which provides that "[a] policy may
declare that a violation of specified provisions thereof shall avoid it, otherwise the breach of an immaterial provision does
not avoid the policy." Such a condition is a provision which invariably appears in fire insurance policies and is intended to
prevent an increase in the moral hazard. It is commonly known as the additional or "other insurance" clause and has been
upheld as valid and as a warranty that no other insurance exists. Its violation would thus avoid the
policy. 16 However, in order to constitute a violation, the other insurance must be upon same subject matter, the same
interest therein, and the same risk. 17
As to a mortgaged property, the mortgagor and the mortgagee have each an independent insurable interest therein
and both interests may be one policy, or each may take out a separate policy covering his interest, either at the
same or at separate times. 18 The mortgagor's insurable interest covers the full value of the mortgaged property, even
though the mortgage debt is equivalent to the full value of the property. 19 The mortgagee's insurable interest is to the
extent of the debt, since the property is relied upon as security thereof, and in insuring he is not insuring the property but
his interest or lien thereon. His insurable interest is prima facie the value mortgaged and extends only to the amount of the
debt, not exceeding the value of the mortgaged property. 20 Thus, separate insurances covering different insurable
interests may be obtained by the mortgagor and the mortgagee.
A mortgagor may, however, take out insurance for the benefit of the mortgagee, which is the usual practice. The
mortgagee may be made the beneficial payee in several ways. He may become the assignee of the policy with the
consent of the insurer; or the mere pledgee without such consent; or the original policy may contain a mortgage
clause; or a rider making the policy payable to the mortgagee "as his interest may appear" may be attached; or a
"standard mortgage clause," containing a collateral independent contract between the mortgagee and insurer, may
be attached; or the policy, though by its terms payable absolutely to the mortgagor, may have been procured by a
mortgagor under a contract duty to insure for the mortgagee's benefit, in which case the mortgagee acquires an
equitable lien upon the proceeds. 21
In the policy obtained by the mortgagor with loss payable clause in favor of the mortgagee as his interest may
appear, the mortgagee is only a beneficiary under the contract, and recognized as such by the insurer but not made
a party to the contract himself. Hence, any act of the mortgagor which defeats his right will also defeat the right of
the mortgagee. 22 This kind of policy covers only such interest as the mortgagee has at the issuing of the policy.23
On the other hand, a mortgagee may also procure a policy as a contracting party in accordance with the terms of an
agreement by which the mortgagor is to pay the premiums upon such insurance. 24 It has been noted, however, that
although the mortgagee is himself the insured, as where he applies for a policy, fully informs the authorized agent of his
interest, pays the premiums, and obtains on the assurance that it insures him, the policy is in fact in the form used to
insure a mortgagor with loss payable clause. 25
The fire insurance policies issued by the PFIC name the petitioner as the assured and contain a mortgage clause
which reads:
Loss, if any, shall be payable to MESSRS. TESING TEXTILES, Cebu City as their interest may
appear subject to the terms of this policy.
This is clearly a simple loss payable clause, not a standard mortgage clause.
It must, however, be underscored that unlike the "other insurance" clauses involved in General Insurance and
Surety Corp. vs. Ng Hua 26 or in Pioneer Insurance & Surety Corp. vs. Yap, 27 which read:
The insured shall give notice to the company of any insurance or insurances already effected, or
which may subsequently be effected covering any of the property hereby insured, and unless such
notice be given and the particulars of such insurance or insurances be stated in or endorsed on this

78
Policy by or on behalf of the Company before the occurrence of any loss or damage, all benefits
under this Policy shall be forfeited.
or in the 1930 case of Santa Ana vs. Commercial Union Assurance
Co. 28 which provided "that any outstanding insurance upon the whole or a portion of the objects thereby assured
must be declared by the insured in writing and he must cause the company to add or insert it in the policy, without
which such policy shall be null and void, and the insured will not be entitled to indemnity in case of
loss," Condition 3in the private respondent's policy No. F-14622 does not absolutely declare void any violation
thereof. It expressly provides that the condition "shall not apply when the total insurance or insurances in force at
the time of the loss or damage is not more than P200,000.00."
It is a cardinal rule on insurance that a policy or insurance contract is to be interpreted liberally in favor of the
insured and strictly against the company, the reason being, undoubtedly, to afford the greatest protection which the
insured was endeavoring to secure when he applied for insurance. It is also a cardinal principle of law that
forfeitures are not favored and that any construction which would result in the forfeiture of the policy benefits for the
person claiming thereunder, will be avoided, if it is possible to construe the policy in a manner which would permit
recovery, as, for example, by finding a waiver for such forfeiture. 29 Stated differently, provisions, conditions or
exceptions in policies which tend to work a forfeiture of insurance policies should be construed most strictly against those
for whose benefits they are inserted, and most favorably toward those against whom they are intended to operate. 30 The
reason for this is that, except for riders which may later be inserted, the insured sees the contract already in its final form
and has had no voice in the selection or arrangement of the words employed therein. On the other hand, the language of
the contract was carefully chosen and deliberated upon by experts and legal advisers who had acted exclusively in the
interest of the insurers and the technical language employed therein is rarely understood by ordinary laymen. 31
With these principles in mind, we are of the opinion that Condition 3 of the subject policy is not totally free from
ambiguity and must, perforce, be meticulously analyzed. Such analysis leads us to conclude that (a) the prohibition
applies only to double insurance, and (b) the nullity of the policy shall only be to the extent exceeding P200,000.00
of the total policies obtained.
The first conclusion is supported by the portion of the condition referring to other insurance "covering any of the
property or properties consisting of stocks in trade, goods in process and/or inventories only hereby insured," and
the portion regarding the insured's declaration on the subheading CO-INSURANCE that the co-insurer is Mercantile
Insurance Co., Inc. in the sum of P50,000.00. A double insurance exists where the same person is insured by
several insurers separately in respect of the same subject and interest. As earlier stated, the insurable interests of a
mortgagor and a mortgagee on the mortgaged property are distinct and separate. Since the two policies of the PFIC
do not cover the same interest as that covered by the policy of the private respondent, no double insurance exists.
The non-disclosure then of the former policies was not fatal to the petitioner's right to recover on the private
respondent's policy.
Furthermore, by stating within Condition 3 itself that such condition shall not apply if the total insurance in force at
the time of loss does not exceed P200,000.00, the private respondent was amenable to assume a co-insurer's
liability up to a loss not exceeding P200,000.00. What it had in mind was to discourage over-insurance. Indeed, the
rationale behind the incorporation of "other insurance" clause in fire policies is to prevent over-insurance and thus
avert the perpetration of fraud. When a property owner obtains insurance policies from two or more insurers in a
total amount that exceeds the property's value, the insured may have an inducement to destroy the property for the
purpose of collecting the insurance. The public as well as the insurer is interested in preventing a situation in which
a fire would be profitable to the insured. 32
WHEREFORE, the instant petition is hereby GRANTED. The decision of the Court of Appeals in CA-G.R. SP No.
31916 is SET ASIDE and the decision of the Insurance Commission in Case No. 3340 is REINSTATED.
Costs against private respondent Country Bankers Insurance Corporation.
SO ORDERED.

79

Republic of the Philippines


SUPREME COURT
Manila
SECOND DIVISION

G.R. No. 128833 April 20, 1998


RIZAL COMMERCIAL BANKING CORPORATION, UY CHUN BING AND ELI D. LAO, petitioners,
vs.
COURT OF APPEALS and GOYU & SONS, INC., respondents.
G.R. No. 128834 April 20, 1998
RIZAL COMMERCIAL BANKING CORPORATION, petitioners,
vs.
COURT OF APPEALS, ALFREDO C. SEBASTIAN, GOYU & SONS, INC., GO SONG HIAP, SPOUSES GO TENG
KOK and BETTY CHIU SUK YING alias BETTY GO, respondents.
G.R. No. 128866 April 20, 1998
MALAYAN INSURANCE INC., petitioners,
vs.
GOYU & SONS, INC. respondent.
MELO, J.:
The issue relevant to the herein three consolidated petitions revolve around the fire loss claims of respondent Goyu
& Sons, Inc. (GOYU) with petitioner Malayan Insurance Company, Inc. (MICO) in connection with the mortgage
contracts entered into by and between Rizal Commercial Banking Corporation (RCBC) and GOYU.
The Court of Appeals ordered MICO to pay GOYU its claims in the total amount of P74,040,518.58, plus 37%
interest per annum commending July 27, 1992. RCBC was ordered to pay actual and compensatory damages in the
amount of P5,000,000.00. MICO and RCBC were held solidarily liable to pay GOYU P1,500,000.00 as exemplary
damages and P1,500,000.00 for attorney's fees. GOYU's obligation to RCBC was fixed at P68,785,069.04 as of
April 1992, without any interest, surcharges, and penalties. RCBC and MICO appealed separately but, in view of the
common facts and issues involved, their individual petitions were consolidated.
The undisputed facts may be summarized as follows:

80
GOYU applied for credit facilities and accommodations with RCBC at its Binondo Branch. After due evaluation,
RCBC Binondo Branch, through its key officers, petitioners Uy Chun Bing and Eli D. Lao, recommended GOYU's
application for approval by RCBC's executive committee. A credit facility in the amount of P30 million was initially
granted. Upon GOYU's application and Uy's and Lao's recommendation, RCBC's executive committee increased
GOYU's credit facility to P50 million, then to P90 million, and finally to P117 million.
As security for its credit facilities with RCBC, GOYU executed two real estate mortgages and two chattel mortgages
in favor of RCBC, which were registered with the Registry of Deeds at Valenzuela, Metro Manila. Under each of
these four mortgage contracts, GOYU committed itself to insure the mortgaged property with an insurance company
approved by RCBC, and subsequently, to endorse and deliver the insurance polices to RCBC.
GOYU obtained in its name a total of ten insurance policies from MICO. In February 1992, Alchester Insurance
Agency, Inc., the insurance agent where GOYU obtained the Malayan insurance policies, issued nine endorsements
in favor of RCBC seemingly upon instructions of GOYU (Exhibits "1-Malayan" to "9-Malayan").
On April 27, 1992, one of GOYU's factory buildings in Valenzuela was gutted by fire. Consequently, GOYU
submitted its claim for indemnity on account of the loss insured against. MICO denied the claim on the ground that
the insurance policies were either attached pursuant to writs of attachments/garnishments issued by various courts
or that the insurance proceeds were also claimed by other creditors of GOYU alleging better rights to the proceeds
than the insured. GOYU filed a complaint for specific performance and damages which was docketed at the
Regional Trial Court of the National Capital Judicial Region (Manila, Branch 3) as Civil Case No. 93-65442, now
subject of the present G.R. No. 128833 and 128866.
RCBC, one of GOYU's creditors, also filed with MICO its formal claim over the proceeds of the insurance policies,
but said claims were also denied for the same reasons that MICO denied GOYU's claims.
In an interlocutory order dated October 12, 1993 (Record, pp. 311-312), the Regional Trial Court of Manila (Branch
3), confirmed that GOYU's other creditors, namely, Urban Bank, Alfredo Sebastian, and Philippine Trust Company
obtained their respective writs of attachments from various courts, covering an aggregate amount of
P14,938,080.23, and ordered that the proceeds of the ten insurance policies be deposited with the said court minus
the aforementioned P14,938,080.23. Accordingly, on January 7, 1994, MICO deposited the amount of
P50,505,594.60 with Branch 3 of the Manila RTC.
In the meantime, another notice of garnishment was handed down by another Manila RTC sala (Branch 28) for the
amount of P8,696,838.75 (Exhibit "22-Malayan").
After trial, Branch 3 of the Manila RTC rendered judgment in favor of GOYU, disposing:
WHEREFORE, judgment is hereby rendered in favor of the plaintiff and against the defendant,
Malayan Insurance Company, Inc. and Rizal Commercial Banking Corporation, ordering the latter as
follows:
1. For defendant Malayan Insurance Co., Inc.:
a. To pay the plaintiff its fire loss claims in the total amount of
P74,040,518.58 less the amount of P50,000,000.00 which is
deposited with this Court;
b. To pay the plaintiff damages by was of interest for the duration of
the delay since July 27, 1992 (ninety days after defendant insurer's

81
receipt of the required proof of loss and notice of loss) at the rate of
twice the ceiling prescribed by the Monetary Board, on the following
amounts:
1) P50,000,000.00 from July 27, 1992 up to the
time said amount was deposited with this Court on
January 7, 1994;
2) P24,040,518.58 from July 27, 1992 up to the
time when the writs of attachments were received by
defendant Malayan;
2. For defendant Rizal Commercial Banking Corporation:
a. To pay the plaintiff actual and compensatory damages in the
amount of P2,000,000.00;
3. For both defendants Malayan and RCBC:
a. To pay the plaintiff, jointly and severally, the following amounts:
1) P1,000,000.00 as exemplary damages;
2) P1,000,000.00 as, and for, attorney's fees;
3) Costs of suit.
and on the Counterclaim of defendant RCBC, ordering the plaintiff to pay its loan
obligations with defendant RCBC in the amount of P68,785,069.04, as of April 27,
1992, with interest thereon at the rate stipulated in the respective promissory notes
(without surcharges and penalties) per computation, pp. 14-A, 14-B & 14-C.
FURTHER, the Clerk of Court of the Regional Trial Court of Manila is hereby ordered to release
immediately to the plaintiff the amount of P50,000,000.00 deposited with the Court by defendant
Malayan, together with all the interest earned thereon.
(Record, pp. 478-479.)
From this judgment, all parties interposed their respective appeals. GOYU was unsatisfied with the amount awarded
in its favor. MICO and RCBC disputed the trial court's findings of liability on their part. The Court of Appeals party
granted GOYU's appeal, but sustained the findings of the trial court with respect to MICO and RCBC's liabilities,
thusly:
WHEREFORE, the decision of the lower court dated June 29, 1994 is hereby modified as follows:
1. FOR DEFENDANT MALAYAN INSURANCE CO., INC:
a) To pay the plaintiff its fire loss claim in the total amount of
P74,040,518.58 less the amount of P50,505,594.60 (per O.R. No.

82
3649285) plus deposited in court and damages by way of interest
commencing July 27, 1992 until the time Goyu receives the said
amount at the rate of thirty-seven (37%) percent per annum which is
twice the ceiling prescribed by the Monetary Board.
2. FOR DEFENDANT RIZAL COMMERCIAL BANKING CORPORATION;
a) To pay the plaintiff actual and compensatory damages in the
amount of P5,000,000.00.
3. FOR DEFENDANTS MALAYAN INSURANCE CO., INC., RIZAL COMMERCIAL
BANKING CORPORATION, UY CHUN BING AND ELI D. LAO:
a) To pay the plaintiff jointly and severally the following amounts:
1. P1,500,000.00 as exemplary damages;
2. P1,500,000.00 as and for attorney's fees.
4. And on RCBC's Counterclaim, ordering the plaintiff Goyu & Sons, Inc. to pay its
loan obligation with RCBC in the amount of P68,785,069.04 as of April 27, 1992
without any interest, surcharges and penalties.
The Clerk of the Court of the Regional Trial Court of Manila is hereby ordered to immediately release
to Goyu & Sons, Inc. the amount of P50,505,594.60 (per O.R. No. 3649285) deposited with it by
Malayan Insurance Co., Inc., together with all the interests thereon.
(Rollo, p. 200.)
RCBC and MICO are now before us in G.R. No. 128833 and 128866, respectively, seeking review and consequent
reversal of the above dispositions of the Court of Appeals.
In G.R. No. 128834, RCBC likewise appeals from the decision in C.A. G.R. No. CV-48376, which case, by virtue of
the Court of Appeals' resolution dated August 7, 1996, was consolidated with C.A. G.R. No. CV-46162 (subject of
herein G.R. No. 128833). At issue in said petition is RCBC's right to intervene in the action between Alfredo C.
Sebastian (the creditor) and GOYU (the debtor), where the subject insurance policies were attached in favor of
Sebastian.
After a careful reviews of the material facts as found by the two courts below in relation to the pertinent and
applicable laws, we find merit in the submission of RCBC and MICO.
The several causes of action pursued below by GOYU gave rise to several related issues which are now submitted
in the petitions before us. This Court, however, discerns one primary and central issue, and this is, whether or not
RCBC, as mortgagee, has any right over the insurance policies taken by GOYU, the mortgagor, in case of the
occurrence of loss.
As earlier mentioned, accordant with the credit facilities extended by RCBC to GOYU, the latter executed several
mortgage contracts in favor of RCBC. It was expressly stipulated in these mortgage contracts that GOYU shall
insure the mortgaged property with any of the insurance companies acceptable to RCBC. GOYU indeed insured the

83
mortgaged property with MICO, an insurance company acceptable to RCBC. Bases on their stipulations in the
mortgage contracts, GOYU was supposed to endorse these insurance policies in favor of, and deliver them, to
RCBC. Alchester Insurance Agency, Inc., MICO's underwriter from whom GOYU obtained the subject insurance
policies, prepared the nine endorsements (see Exh. "1-Malayan" to "9-Malayan"; also Exh. "51-RCBC" to "59RCBC"), copies of which were delivered to GOYU, RCBC, and MICO. However, because these endorsements do
not bear the signature of any officer of GOYU, the trial court, as well as the Court of Appeals, concluded that the
endorsements are defective.
We do not quite agree.
It is settled that a mortgagor and a mortgagee have separated and distinct insurable interests in the same
mortgaged property, such that each one of them may insure the same property for his own sole benefit. There is no
question that GOYU could insure the mortgaged property for its own exclusive benefit. In the present case, although
it appears that GOYU obtained the subject insurance policies naming itself as the sole payee, the intentions of the
parties as shown by their contemporaneous acts, must be given due consideration in order to better serve the
interest of justice and equity.
It is to be noted that nine endorsement documents were prepared by Alchester in favor of RCBC. The Court is in a
quandary how Alchester could arrive at the idea of endorsing any specific insurance policy in favor of any particular
beneficiary or payee other than the insured had not such named payee or beneficiary been specifically disclosed by
the insured itself. It is also significant that GOYU voluntarily and purposely took the insurance policies from MICO, a
sister company of RCBC, and not just from any other insurance company. Alchester would not have found out that
the subject pieces of property were mortgaged to RCBC had not such information been voluntarily disclosed by
GOYU itself. Had it not been for GOYU, Alchester would not have known of GOYU's intention of obtaining insurance
coverage in compliance with its undertaking in the mortgage contracts with RCBC, and verily, Alchester would not
have endorsed the policies to RCBC had it not been so directed by GOYU.
On equitable principles, particularly on the ground of estoppel, the Court is constrained to rule in favor of mortgagor
RCBC. The basis and purpose of the doctrine was explained in Philippine National Bank vs. Court of Appeals (94
SCRA 357 [1979]), to wit:
The doctrine of estoppel is based upon the grounds of public, policy, fair dealing, good faith and
justice, and its purpose is to forbid one to speak against his own act, representations, or
commitments to the injury of one to whom they were directed and who reasonably relied thereon.
The doctrine of estoppel springs from equitable principles and the equities in the case. It is designed
to aid the law in the administration of justice where without its aid injustice might result. It has been
applied by this Court wherever and whenever special circumstances of a case so demand.
(p. 368.)
Evelyn Lozada of Alchester testified that upon instructions of Mr. Go, through a certain Mr. Yam, she prepared in
quadruplicate on February 11, 1992 the nine endorsement documents for GOYU's nine insurance policies in favor of
RCBC. The original copies of each of these nine endorsement documents were sent to GOYU, and the others were
sent to RCBC and MICO, while the fourth copies were detained for Alchester's file (tsn, February 23, pp. 7-8).
GOYU has not denied having received from Alchester the originals of these documents.
RCBC, in good faith, relied upon the endorsement documents sent to it as this was only pursuant to the stipulation
in the mortgage contracts. We find such reliance to be justified under the circumstances of the case. GOYU failed to
seasonably repudiate the authority of the person or persons who prepared such endorsements. Over and above
this, GOYU continued, in the meantime, to enjoy the benefits of the credit facilities extended to it by RCBC. After the

84
occurrence of the loss insure against, it was too late for GOYU to disown the endorsements for any imagined or
contrived lack of authority of Alchester to prepare and issue said endorsements. If there had not been actually an
implied ratification of said endorsements by virtue of GOYU's inaction in this case, GOYU is at the very least
estopped from assailing their operative effects. To permit GOYU to capitalize on its non-confirmation of these
endorsements while it continued to enjoy the benefits of the credit facilities of RCBC which believed in good faith
that there was due endorsement pursuant to their mortgage contracts, is to countenance grave contravention of
public policy, fair dealing, good faith, and justice. Such an unjust situation, the Court cannot sanction. Under the
peculiar circumstances obtaining in this case, the Court is bound to recognize RCBC's right to the proceeds of the
insurance polices if not for the actual endorsement of the policies, at least on the basis of the equitable principle of
estoppel.
GOYU cannot seek relief under Section 53 of the Insurance Code which provides that the proceeds of insurance
shall exclusively apply to the interest of the person in whose name or for whose benefit it is made. The peculiarity of
the circumstances obtaining in the instant case presents a justification to take exception to the strict application of
said provision, it having been sufficiently established that it was the intention of the parties to designate RCBC as
the party for whose benefit the insurance policies were taken out. Consider thus the following:
1. It is undisputed that the insured pieces of property were the subject of mortgage contracts entered into between
RCBC and GOYU in consideration of and for securing GOYU's credit facilities from RCBC. The mortgage contracts
contained common provisions whereby GOYU, as mortgagor, undertook to have the mortgaged property properly
covered against any loss by an insurance company acceptable to RCBC.
2. GOYU voluntarily procured insurance policies to cover the mortgaged property from MICO, no less than a sister
company of RCBC and definitely an acceptable insurance company to RCBC.
3. Endorsement documents were prepared by MICO's underwriter, Alchester Insurance Agency, Inc., and copies
thereof were sent to GOYU, MICO, and RCBC. GOYU did not assail, until of late, the validity of said endorsements.
4. GOYU continued until the occurrence of the fire, to enjoy the benefits of the credit facilities extended by RCBC
which was conditioned upon the endorsement of the insurance policies to be taken by GOYU to cover the
mortgaged properties.
This Court can not over stress the fact that upon receiving its copies of the endorsement documents prepared by
Alchester, GOYU, despite the absence of its written conformity thereto, obviously considered said endorsement to
be sufficient compliance with its obligation under the mortgage contracts since RCBC accordingly continued to
extend the benefits of its credits facilities and GOYU continued to benefit therefrom. Just as plain too is the intention
of the parties to constitute RCBC as the beneficiary of the various insurance policies obtained by GOYU. The
intention of the parties will have to be given full force and effect particular case. The insurance proceeds may,
therefore, be exclusively applied to RCBC, which under the factual circumstances of the case, is truly the person or
entity for whose benefit the polices were clearly intended.
Moreover, the law's evident intention to protect the interests of the mortgage upon the mortgaged property is
expressed in Article 2127 of the Civil Code which states:
Art. 2127. The mortgage extends to the natural accessions, to the improvements, growing fruits, and
the rents or income not yet received when the obligation becomes due, and to the amount of the
indemnity granted or owing to the proprietor from the insurers of the property mortgaged, or in virtue
of expropriation for public use, with the declarations, amplifications and limitations established by
law, whether the estate remains in the possession of the mortgagor, or it passes into the hands of a
third person.

85
Significantly, the Court notes that out of the 10 insurance policies subject of this case, only 8 of them appear to have
been subject of the endorsements prepared and delivered by Alchester for and upon instructions of GOYU as
shown below:
INSURANCE POLICY PARTICULARS ENDORSEMENT
a. Policy Number F-114-07795 None
Issue Date March 18, 1992
Expiry Date April 5, 1993
Amount P9,646,224.92

b. Policy Number ACIA/F-174-07660 Exhibit "1-Malayan"


Issue Date January 18, 1992
Expiry Date February 9, 1993
Amount P4,307,217.54
c. Policy Number ACIA/F-114-07661 Exhibit "2-Malayan"
Issue Date January 18, 1992
Expiry Date February 15, 1993
Amount P6,603,586.43
d. Policy Number ACIA/F-114-07662 Exhibit "3-Malayan"
Issue Date January 18, 1992
Expiry Date (not legible)
Amount P6,603,586.43
e. Policy Number ACIA/F-114-07663 Exhibit "4-Malayan"
Issue Date January 18, 1992
Expiry Date February 9, 1993
Amount P9,457,972.76
f. Policy Number ACIA/F-114-07623 Exhibit "7-Malayan"
Issue Date January 13, 1992
Expiry Date January 13, 1993
Amount P24,750,000.00
g. Policy Number ACIA/F-174-07223 Exhibit "6-Malayan"
Issue Date May 29, 1991
Expiry Date June 27, 1992
Amount P6,000,000.00
h. Policy Number CI/F-128-03341 None
Issue Date May 3, 1991
Expiry Date May 3, 1992
Amount P10,000,000.00
i. Policy Number F-114-07402 Exhibit "8-Malayan"
Issue Date September 16, 1991
Expiry Date October 19, 1992
Amount P32,252,125.20

86
j. Policy Number F-114-07525 Exhibit "9-Malayan"
Issue Date November 20, 1991
Expiry Date December 5, 1992
Amount P6,603,586.43
(pp. 456-457, Record; Folder of Exhibits for MICO.)
Policy Number F-114-07795 [(a) above] has not been endorsed. This fact was admitted by MICO's witness, Atty.
Farolan (tsn, February 16, 1994, p. 25). Likewise, the record shows no endorsement for Policy Number CI/F-12803341 [(h) above]. Also, one of the endorsement documents, Exhibit "5-Malayan", refers to a certain insurance
policy number ACIA-F-07066, which is not among the insurance policies involved in the complaint.
The proceeds of the 8 insurance policies endorsed to RCBC aggregate to P89,974,488.36. Being excessively
payable to RCBC by reason of the endorsement by Alchester to RCBC, which we already ruled to have the force
and effect of an endorsement by GOYU itself, these 8 policies can not be attached by GOYU's other creditors up to
the extent of the GOYU's outstanding obligation in RCBC's favor. Section 53 of the Insurance Code ordains that the
insurance proceeds of the endorsed policies shall be applied exclusively to the proper interest of the person for
whose benefit it was made. In this case, to the extent of GOYU's obligation with RCBC, the interest of GOYU in the
subject policies had been transferred to RCBC effective as of the time of the endorsement. These policies may no
longer be attached by the other creditors of GOYU, like Alfredo Sebastian in the present G.R. No. 128834, which
may nonetheless forthwith be dismissed for being moot and academic in view of the results reached herein. Only
the two other policies amounting to P19,646,224.92 may be validly attached, garnished, and levied upon by GOYU's
other creditors. To the extent of GOYU's outstanding obligation with RCBC, all the rest of the other insurance
policies above-listed which were endorsed to RCBC, are, therefore, to be released from attachment, garnishment,
and levy by the other creditors of GOYU.
This brings us to the next issue to be resolved, which is, the extent of GOYU's outstanding obligation with RCBC
which the proceeds of the 8 insurance policies will discharge and liquidate, or put differently, the actual amount of
GOYU's liability to RCBC.
The Court of Appeals simply echoed the declaration of the trial court finding that GOYU's total obligation to RCBC
was only P68,785,060.04 as of April 27, 1992, thus sanctioning the trial court's exclusion of Promissory Note No.
421-92 (renewal of Promissory Note No. 908-91) and Promissory Note No. 420-92 (renewal of Promissory Note No.
952-91) on the ground that their execution is highly questionable for not only are these dated after the fire, but also
because the signatures of either GOYU or any its representative are conspicuously absent. Accordingly, the Court of
Appeals speculated thusly:
. . . Hence, this Court is inclined to conclude that said promissory notes were pre-signed by plaintiff
in bank terms, as averred by plaintiff, in contemplation of the speedy grant of future loans, for the
same practice of procedure has always been adopted in its previous dealings with the bank.
(Rollo, pp. 181-182.)
The fact that the promissory notes bear dates posterior to the fire does not necessarily mean that the documents
are spurious, for it is presumed that the ordinary course of business had been followed (Metropolitan Bank and Trust
Company vs. Quilts and All, Inc., 22 SCRA 486 [1993]). The obligor and not the holder of the negotiable instrument
has the burden of proof of showing that he no longer owes the obligee any amount (Travel-On, Inc. vs. Court of
Appeals, 210 SCRA 351 [1992]).

87
Even casting aside the presumption of regularity of private transactions, receipt of the loan amounting to
P121,966,058.67 (Exhibits 1-29, RCBC) was admitted by GOYU as indicated in the testimony of Go Song Hiap
when he answered the queries of the trial court.
ATTY. NATIVIDAD
Q: But insofar as the amount stated in Exhibits 1 to 29-RCBC, you received all the
amounts stated therein?
A: Yes, sir, I received the amount.
COURT
He is asking if he received all the amounts stated in Exhibits 1 to 29-RCBC?
WITNESS:
Yes, Your Honor, I received all the amounts.
COURT
Indicated in the Promissory Notes?
WITNESS
A. The promissory Notes they did not give to me but the amount I asked which is
correct, Your Honor.
COURT
Q Your mean to say the amounts indicated in Exhibits 1 to 29-RCBC is correct?
A Yes, Your Honor.
(tsn, Jan. 14, 1994, p. 26.)
Furthermore, aside from its judicial admission of having received all the proceeds of the 29 promissory notes as
hereinabove quotes, GOYU also offered and admitted to RCBC that is obligation be fixed at P116,301,992.60 as
shown in its letter date March 9, 1993, which pertinently reads:
We wish to inform you, therefore that we are ready and willing to pay the current past due account of
this company in the amount of P116,301,992.60 as of 21 January 1993, specified in pars. 15, p. 10,
and 18, p. 13 of your affidavits of Third Party Claims in the Urban case at Makati, Metro Manila and
in the Zamboanga case at Zamboanga city, respectively, less the total of P8,851,519.71 paid from
the Seaboard and Equitable insurance companies and other legitimate deductions. We accept and
confirm this amount of P116,301,992.60 as stated as true and correct.
(Exhibit BB.)

88
The Court of Appeals erred in placing much significance on the fact that the excluded promissory notes are dated
after the fire. It failed to consider that said notes had for their origin transactions consummated prior to the fire. Thus,
careful attention must be paid to the fact that Promissory Notes No. 420-92 and 421-92 are mere renewalsof
Promissory Notes No. 908-91 and 952-91, loans already availed of by GOYU.
The two courts below erred in failing to see that the promissory notes which they ruled should be excluded for
bearing dates which are after that of the fire, are mere renewals of previous ones. The proceeds of the loan
represented by these promissory notes were admittedly received by GOYU. There is ample factual and legal basis
for giving GOYU's judicial admission of liability in the amount of P116,301,992.60 full force and effect.
It should, however, be quickly added that whatever amount RCBC may have recovered from the other insurers of
the mortgage property will, nonetheless, have to be applied as payment against GOYU's obligation. But, contrary to
the lower courts' findings, payments effected by GOYU prior to January 21, 1993 should no longer be deducted.
Such payments had obviously been duly considered by GOYU, in its aforequoted letter date March 9, 1993, wherein
it admitted that its past due account totaled P116,301,992.60 as of January 21, 1993.
The net obligation of GOYU, after deductions, is thus reduced to P107,246,887.90 as of January 21, 1993, to wit:
Total Obligation as admitted by GOYU
as of January 21, 1993: P116,301,992.60
Broken down as follows:
Principal 1 Interest
Regular 80,535,946.32
FDU 27,548,025.17
____________
Total 108,083,971.49 8,218,021.11 2
LESS:
1) Proceeds from
Seaboard Eastern
Insurance Company 6,095,145.81
2) Proceeds from
Equitable Insurance
Company 2,756,373.00
3) Payment from
foreign department
negotiation: 203,584.89
___________
9,055,104.70 3
================
NET AMOUNT as of January 21, 1993 P107,246,887.90

89
The need for the payment of interest due the principal amount of the obligation, which is the cost of money to
RCBC, the primary end and the ultimate reason for RCBC's existence and being, was duly recognized by the trial
court when it ruled favorably on RCBC's counterclaim, ordering GOYU "to pay its loan obligation with RCBC in the
amount of P68,785,069.04, as of April 27, 1992, with interest thereon at the rate stipulated in the respective
promissory notes (without surcharges and penalties) per computation, pp. 14-A, 14-B 14-C" (Record, p. 479).
Inexplicably, the Court of Appeals, without even laying down the factual or legal justification for its ruling, modified
the trial court's ruling and ordered GOYU "to pay the principal amount of P68,785,069.04 without any interest,
surcharges and penalties" (Rollo, p. 200).
It is to be noted in this regard that even the trial court hedgingly and with much uncertainty deleted the payment
ofadditional interest, penalties, and charges, in this manner:
Regarding defendant RCBC's commitment not to charge additional interest, penalties and
surcharges, the same does not require that it be embodied in a document or some form of writing to
be binding and enforceable. The principle is well known that generally a verbal agreement or
contract is no less binding and effective than a written one. And the existence of such a verbal
agreement has been amply established by the evidence in this case. In any event, regardless of the
existence of such verbal agreement, it would still be unjust and inequitable for defendant RCBC to
charge the plaintiff with surcharges and penalties considering the latter's pitiful situation. (Emphasis
supplied).
(Record, p. 476)
The essence or rationale for the payment of interest or cost of money is separate and distinct from that of
surcharges and penalties. What may justify a court in not allowing the creditor to charge surcharges and penalties
despite express stipulation therefor in a valid agreement, may not equally justify non-payment of interest. The
charging of interest for loans forms a very essential and fundamental element of the banking business, which may
truly be considered to be at the very core of its existence or being. It is inconceivable for a bank to grant loans for
which it will not charge any interest at all. We fail to find justification for the Court of Appeal's outright deletion of the
payment of interest as agreed upon in the respective promissory notes. This constitutes gross error.
For the computation of the interest due to be paid to RCBC, the following rules of thumb laid down by this Court
inEastern Shipping Lines, Inc. vs. Court of Appeals (234 SCRA 78 [1994]), shall apply, to wit:
I. When an obligation, regardless of its source, i.e., law, contracts, quasi-contracts, delicts or quasi-delicts is
breached, the contravenor can be held liable for damages. The provisions under Title XVIII on "Damages" of the
Civil Code govern in determining the measure of recoverable damages.
II. With regard particularly to an award of interest in the concept of actual and compensatory damages, the rate of
interest, as well as the actual thereof, is imposed, as follows:
1. When the obligation is breached, and it consists in the payment of a sum of money, i.e., a loan or
forbearance of money, the interest due should be that which may have been stipulated in writing.
Furthermore, the interest due shall itself earn legal interest from the time it is judicially demanded. In
the absence of stipulation, the rate of interest shall be 12% per annum to be computed from
default,i.e., from judicial or extrajudicial demand under and subject to the provisions of Article 1169
of the Civil Code.
2. When an obligation, not constituting a loan or forbearance of money, is breached, an interest on
the amount of damages awarded may be imposed at the discretion of the court at the rate of 6% per

90
annum. No interest, however, shall be adjudged on unliquidated claims or damages except when or
until the demand can be established with reasonable certainty. Accordingly, where the demand is
established with reasonable certainty, the interest shall begin to run from the time the claim is made
judicially or extrajudicially (Art. 1169, Civil Code) but when such certainty cannot be so reasonably
established at the time the demand is made, the interest shall begin to run only from the date of the
judgment of the court is made (at which time the quantification of damages may be deemed to have
been reasonably ascertained). The actual base for the computation of legal interest shall, in any
case, be on the amount finally adjudged.
3. When the judgment of the court awarding a sum of money becomes final and executory, the rate
of legal interest, whether the case falls under paragraph 1 or paragraph 2, above, shall be 12% per
annum from such finality until its satisfaction, this interim period being deemed to be by then an
equivalent to a forbearance of credit.
(pp. 95-97).
There being written stipulations as to the rate of interest owing on each specific promissory note as summarized and
tabulated by the trial court in its decision (pp. 470 and 471, Record) such agreed interest rates must be followed.
This is very clear from paragraph II, sub-paragraph 1 quoted above.
On the issue of payment of surcharges and penalties, we partly agree that GOYU's pitiful situation must be taken
into account. We do not agree, however, that payment of any amount as surcharges and penalties should altogether
be deleted. Even assuming that RCBC, through its responsible officers, herein petitioners Eli Lao and Uy Chun
Bing, may have relayed its assurance for assistance to GOYU immediately after the occurrence of the fire, we
cannot accept the lower courts' finding that RCBC had thereby ipso facto effectively waived collection of any
additional interests, surcharges, and penalties from GOYU. Assurances of assistance are one thing, but waiver of
additional interests, surcharges, and penalties is another.
Surcharges and penalties agreed to be paid by the debtor in case of default partake of the nature of liquidated
damages, covered by Section 4, Chapter 3, Title XVIII of the Civil Code. Article 2227 thereof provides:
Art. 2227. Liquidated damages, whether intended as a indemnity or penalty, shall be equitably
reduced if they are iniquitous and unconscionable.
In exercising this vested power to determine what is iniquitous and unconscionable, the Court must consider the
circumstances of each case. It should be stressed that the Court will not make any sweeping ruling that surcharges
and penalties imposed by banks for non-payment of the loans extended by them are generally iniquitous and
unconscionable. What may be iniquitous and unconscionable in one case, may be totally just and equitable in
another. This provision of law will have to be applied to the established facts of any given case. Given the
circumstance under which GOYU found itself after the occurrence of the fire, the Court rules the surcharges rates
ranging anywhere from 9% to 27%, plus the penalty charges of 36%, to be definitely iniquitous and unconscionable.
The Court tempers these rates to 2% and 3%, respectively. Furthermore, in the light of GOYU's offer to pay the
amount of P116,301,992.60 to RCBC as March 1993 (See: Exhibit "BB"), which RCBC refused, we find it more in
keeping with justice and equity for RCBC not to charge additional interest, surcharges, and penalties from that time
onward.
Given the factual milieu hereover, we rule that it was error to hold MICO liable in damages for denying or
withholding the proceeds of the insurance claim to GOYU.

91
Firstly, by virtue of the mortgage contracts as well as the endorsements of the insurance policies, RCBC has the
right to claim the insurance proceeds, in substitution of the property lost in the fire. Having assigned its rights, GOYU
lost its standing as the beneficiary of the said insurance policies.
Secondly, for an insurance company to be held liable for unreasonably delaying and withholding payment of
insurance proceeds, the delay must be wanton, oppressive, or malevolent (Zenith Insurance Corporation vs. CA.
185 SCRA 403 [1990]). It is generally agreed, however, that an insurer may in good faith and honesty entertain a
difference of opinion as to its liability. Accordingly, the statutory penalty for vexatious refusal of an insurer to pay a
claim should not be inflicted unless the evidence and circumstances show that such refusal was willful and without
reasonable cause as the facts appear to a reasonable and prudent man (Bufallo Ins. Co. vs. Bommarito [CCA 8th]
42 F [2d] 53, 70 ALR 1211; Phoenix Ins. Co. vs. Clay, 101 Ga. 331, 28 SE 853, 65 Am St. Rep 307; Kusnetsky vs.
Security Ins. Co., 313 Mo. 143, 281 SW 47, 45 ALR 189). The case at bar does not show that MICO wantonly and in
bad faith delayed the release of the proceeds. The problem in the determination of who is the actual beneficiary of
the insurance policies, aggravated by the claim of various creditors who wanted to partake of the insurance
proceeds, not to mention the importance of the endorsement to RCBC, to our mind, and as now borne out by the
outcome herein, justified MICO in withholding payment to GOYU.
In adjudging RCBC liable in damages to GOYU, the Court of Appeals said that RCBC cannot avail itself of two
simultaneous remedies in enforcing the claim of an unpaid creditor, one for specific performance and the other for
foreclosure. In doing so, said the appellate court, the second action is deemed barred, RCBC having split a single
cause of action (Rollo, pp. 195-199). The Court of Appeals was too accommodating in giving due consideration to
this argument of GOYU, for the foreclosure suit is still pending appeal before the same Court of Appeals in CA G.R.
CV No. 46247, the case having been elevated by RCBC.
In finding that the foreclosure suit cannot prosper, the Fifteenth Division of the Court of Appeals pre-empted the
resolution of said foreclosure case which is not before it. This is plain reversible error if not grave abuse of
discretion.
As held in Pea vs. Court of Appeals (245 SCRA 691 [1995]):
It should have been enough, nonetheless, for the appellate court to merely set aside the questioned
ordered of the trial court for having been issued by the latter with grave abuse of discretion. In
likewise enjoining permanently herein petitioner "from entering in and interfering with the use or
occupation and enjoyment of petitioner's (now private respondent) residential house and
compound," the appellate court in effect, precipitately resolved with finality the case for injunction
that was yet to be heard on the merits by the lower court. Elevated to the appellate court, it might be
stressed, were mere incidents of the principal case still pending with the trial court. In Municipality of
Bian, Laguna vs. Court of Appeals, 219 SCRA 69, we ruled that the Court of Appeals would have
"no jurisdiction in a certiorari proceeding involving an incident in a case to rule on the merits of the
main case itself which was not on appeal before it.
(pp. 701-702.)
Anent the right of RCBC to intervene in Civil Case No. 1073, before the Zamboanga Regional Trial Court, since it
has been determined that RCBC has the right to the insurance proceeds, the subject matter of intervention is
rendered moot and academic. Respondent Sebastian must, however, yield to the preferential right of RCBC over
the MICO insurance policies. It is basic and fundamental that the first mortgagee has superior rights over junior
mortgagees or attaching creditors (Alpha Insurance & Surety Co. vs. Reyes, 106 SCRA 274 [1981]; Sun Life
Assurance Co. of Canada vs. Gonzales Diaz, 52 Phil. 271 [1928]).

92
WHEREFORE, the petitions are hereby GRANTED and the decision and resolution of December 16, 1996 and April
3, 1997 in CA-G.R. CV No. 46162 are hereby REVERSED and SET ASIDE, and a new one entered:
1. Dismissing the Complaint of private respondent GOYU in Civil Case No. 93-65442 before Branch
3 of the Manila Trial Court for lack of merit;
2. Ordering Malayan Insurance Company, Inc. to deliver to Rizal Commercial Banking Corporation
the proceeds of the insurance policies in the amount of P51,862,390.94 (per report of adjuster Toplis
& Harding (Far East), Inc., Exhibits "2" and "2-1"), less the amount of P50,505,594.60 (per O.R. No.
3649285);
3. Ordering the Clerk of Court to release the amount of P50,505,594.60 including the interests
earned to Rizal Commercial Banking Corporation;
4. Ordering Goyu & Sons, Inc. to pay its loan obligation with Rizal Commercial Banking Corporation
in the principal amount of P107,246,887.90, with interest at the respective rates stipulated in each
promissory note from January 21, 1993 until finality of this judgment, and surcharges at 2% and
penalties at 3% from January 21, 1993 to March 9, 1993, minus payments made by Malayan
Insurance Company, Inc. and the proceeds of the amount deposited with the trial court and its
earned interest. The total amount due RCBC at the time of the finality of this judgment shall earn
interest at the legal rate of 12% in lieu of all other stipulated interests and charges until fully paid.
The petition of Rizal Commercial Banking Corporation against the respondent Court in CA-GR CV 48376 is
DISMISSED for being moot and academic in view of the results herein arrived at. Respondent Sebastian's right as
attaching creditor must yield to the preferential rights of Rizal Commercial Banking Corporation over the Malayan
insurance policies as first mortgagee.
SO ORDERED.

93

Republic of the Philippines


SUPREME COURT
Manila
FIRST DIVISION
G.R. No. 147839

June 8, 2006

GAISANO CAGAYAN, INC. Petitioner,


vs.
INSURANCE COMPANY OF NORTH AMERICA, Respondent.
DECISION
AUSTRIA-MARTINEZ, J.:
Before the Court is a petition for review on certiorari of the Decision1 dated October 11, 2000 of the Court of Appeals
(CA) in CA-G.R. CV No. 61848 which set aside the Decision dated August 31, 1998 of the Regional Trial Court,
Branch 138, Makati (RTC) in Civil Case No. 92-322 and upheld the causes of action for damages of Insurance
Company of North America (respondent) against Gaisano Cagayan, Inc. (petitioner); and the CA Resolution dated
April 11, 2001 which denied petitioner's motion for reconsideration.

94
The factual background of the case is as follows:
Intercapitol Marketing Corporation (IMC) is the maker of Wrangler Blue Jeans. Levi Strauss (Phils.) Inc. (LSPI) is the
local distributor of products bearing trademarks owned by Levi Strauss & Co.. IMC and LSPI separately obtained
from respondent fire insurance policies with book debt endorsements. The insurance policies provide for coverage
on "book debts in connection with ready-made clothing materials which have been sold or delivered to various
customers and dealers of the Insured anywhere in the Philippines." 2 The policies defined book debts as the "unpaid
account still appearing in the Book of Account of the Insured 45 days after the time of the loss covered under this
Policy."3 The policies also provide for the following conditions:
1. Warranted that the Company shall not be liable for any unpaid account in respect of the merchandise sold
and delivered by the Insured which are outstanding at the date of loss for a period in excess of six (6)
months from the date of the covering invoice or actual delivery of the merchandise whichever shall first
occur.
2. Warranted that the Insured shall submit to the Company within twelve (12) days after the close of every
calendar month all amount shown in their books of accounts as unpaid and thus become receivable item
from their customers and dealers. x x x4
xxxx
Petitioner is a customer and dealer of the products of IMC and LSPI. On February 25, 1991, the Gaisano Superstore
Complex in Cagayan de Oro City, owned by petitioner, was consumed by fire. Included in the items lost or destroyed
in the fire were stocks of ready-made clothing materials sold and delivered by IMC and LSPI.
On February 4, 1992, respondent filed a complaint for damages against petitioner. It alleges that IMC and LSPI filed
with respondent their claims under their respective fire insurance policies with book debt endorsements; that as of
February 25, 1991, the unpaid accounts of petitioner on the sale and delivery of ready-made clothing materials with
IMC was P2,119,205.00 while with LSPI it was P535,613.00; that respondent paid the claims of IMC and LSPI and,
by virtue thereof, respondent was subrogated to their rights against petitioner; that respondent made several
demands for payment upon petitioner but these went unheeded.5
In its Answer with Counter Claim dated July 4, 1995, petitioner contends that it could not be held liable because the
property covered by the insurance policies were destroyed due to fortuities event or force majeure; that
respondent's right of subrogation has no basis inasmuch as there was no breach of contract committed by it since
the loss was due to fire which it could not prevent or foresee; that IMC and LSPI never communicated to it that they
insured their properties; that it never consented to paying the claim of the insured. 6
At the pre-trial conference the parties failed to arrive at an amicable settlement. 7 Thus, trial on the merits ensued.
On August 31, 1998, the RTC rendered its decision dismissing respondent's complaint. 8 It held that the fire was
purely accidental; that the cause of the fire was not attributable to the negligence of the petitioner; that it has not
been established that petitioner is the debtor of IMC and LSPI; that since the sales invoices state that "it is further
agreed that merely for purpose of securing the payment of purchase price, the above-described merchandise
remains the property of the vendor until the purchase price is fully paid", IMC and LSPI retained ownership of the
delivered goods and must bear the loss.
Dissatisfied, petitioner appealed to the CA.9 On October 11, 2000, the CA rendered its decision setting aside the
decision of the RTC. The dispositive portion of the decision reads:

95
WHEREFORE, in view of the foregoing, the appealed decision is REVERSED and SET ASIDE and a new one is
entered ordering defendant-appellee Gaisano Cagayan, Inc. to pay:
1. the amount of P2,119,205.60 representing the amount paid by the plaintiff-appellant to the insured Inter
Capitol Marketing Corporation, plus legal interest from the time of demand until fully paid;
2. the amount of P535,613.00 representing the amount paid by the plaintiff-appellant to the insured Levi
Strauss Phil., Inc., plus legal interest from the time of demand until fully paid.
With costs against the defendant-appellee.
SO ORDERED.10
The CA held that the sales invoices are proofs of sale, being detailed statements of the nature, quantity and cost of
the thing sold; that loss of the goods in the fire must be borne by petitioner since the proviso contained in the sales
invoices is an exception under Article 1504 (1) of the Civil Code, to the general rule that if the thing is lost by a
fortuitous event, the risk is borne by the owner of the thing at the time the loss under the principle of res perit
domino; that petitioner's obligation to IMC and LSPI is not the delivery of the lost goods but the payment of its
unpaid account and as such the obligation to pay is not extinguished, even if the fire is considered a fortuitous
event; that by subrogation, the insurer has the right to go against petitioner; that, being a fire insurance with book
debt endorsements, what was insured was the vendor's interest as a creditor.11
Petitioner filed a motion for reconsideration12 but it was denied by the CA in its Resolution dated April 11, 2001. 13
Hence, the present petition for review on certiorari anchored on the following Assignment of Errors:
THE COURT OF APPEALS ERRED IN HOLDING THAT THE INSURANCE IN THE INSTANT CASE WAS ONE
OVER CREDIT.
THE COURT OF APPEALS ERRED IN HOLDING THAT ALL RISK OVER THE SUBJECT GOODS IN THE
INSTANT CASE HAD TRANSFERRED TO PETITIONER UPON DELIVERY THEREOF.
THE COURT OF APPEALS ERRED IN HOLDING THAT THERE WAS AUTOMATIC SUBROGATION UNDER ART.
2207 OF THE CIVIL CODE IN FAVOR OF RESPONDENT.14
Anent the first error, petitioner contends that the insurance in the present case cannot be deemed to be over credit
since an insurance "on credit" belies not only the nature of fire insurance but the express terms of the policies; that it
was not credit that was insured since respondent paid on the occasion of the loss of the insured goods to fire and
not because of the non-payment by petitioner of any obligation; that, even if the insurance is deemed as one over
credit, there was no loss as the accounts were not yet due since no prior demands were made by IMC and LSPI
against petitioner for payment of the debt and such demands came from respondent only after it had already paid
IMC and LSPI under the fire insurance policies.15
As to the second error, petitioner avers that despite delivery of the goods, petitioner-buyer IMC and LSPI assumed
the risk of loss when they secured fire insurance policies over the goods.
Concerning the third ground, petitioner submits that there is no subrogation in favor of respondent as no valid
insurance could be maintained thereon by IMC and LSPI since all risk had transferred to petitioner upon delivery of
the goods; that petitioner was not privy to the insurance contract or the payment between respondent and its

96
insured nor was its consent or approval ever secured; that this lack of privity forecloses any real interest on the part
of respondent in the obligation to pay, limiting its interest to keeping the insured goods safe from fire.
For its part, respondent counters that while ownership over the ready- made clothing materials was transferred upon
delivery to petitioner, IMC and LSPI have insurable interest over said goods as creditors who stand to suffer direct
pecuniary loss from its destruction by fire; that petitioner is liable for loss of the ready-made clothing materials since
it failed to overcome the presumption of liability under Article 1265 16 of the Civil Code; that the fire was caused
through petitioner's negligence in failing to provide stringent measures of caution, care and maintenance on its
property because electric wires do not usually short circuit unless there are defects in their installation or when there
is lack of proper maintenance and supervision of the property; that petitioner is guilty of gross and evident bad faith
in refusing to pay respondent's valid claim and should be liable to respondent for contracted lawyer's fees, litigation
expenses and cost of suit.17
As a general rule, in petitions for review, the jurisdiction of this Court in cases brought before it from the CA is limited
to reviewing questions of law which involves no examination of the probative value of the evidence presented by the
litigants or any of them.18 The Supreme Court is not a trier of facts; it is not its function to analyze or weigh evidence
all over again.19 Accordingly, findings of fact of the appellate court are generally conclusive on the Supreme Court. 20
Nevertheless, jurisprudence has recognized several exceptions in which factual issues may be resolved by this
Court, such as: (1) when the findings are grounded entirely on speculation, surmises or conjectures; (2) when the
inference made is manifestly mistaken, absurd or impossible; (3) when there is grave abuse of discretion; (4) when
the judgment is based on a misapprehension of facts; (5) when the findings of facts are conflicting; (6) when in
making its findings the CA went beyond the issues of the case, or its findings are contrary to the admissions of both
the appellant and the appellee; (7) when the findings are contrary to the trial court; (8) when the findings are
conclusions without citation of specific evidence on which they are based; (9) when the facts set forth in the petition
as well as in the petitioner's main and reply briefs are not disputed by the respondent; (10) when the findings of fact
are premised on the supposed absence of evidence and contradicted by the evidence on record; and (11) when the
CA manifestly overlooked certain relevant facts not disputed by the parties, which, if properly considered, would
justify a different conclusion.21 Exceptions (4), (5), (7), and (11) apply to the present petition.
At issue is the proper interpretation of the questioned insurance policy. Petitioner claims that the CA erred in
construing a fire insurance policy on book debts as one covering the unpaid accounts of IMC and LSPI since such
insurance applies to loss of the ready-made clothing materials sold and delivered to petitioner.
The Court disagrees with petitioner's stand.
It is well-settled that when the words of a contract are plain and readily understood, there is no room for
construction.22 In this case, the questioned insurance policies provide coverage for "book debts in connection with
ready-made clothing materials which have been sold or delivered to various customers and dealers of the Insured
anywhere in the Philippines."23 ; and defined book debts as the "unpaid account still appearing in the Book of
Account of the Insured 45 days after the time of the loss covered under this Policy." 24 Nowhere is it provided in the
questioned insurance policies that the subject of the insurance is the goods sold and delivered to the customers and
dealers of the insured.
Indeed, when the terms of the agreement are clear and explicit that they do not justify an attempt to read into it any
alleged intention of the parties, the terms are to be understood literally just as they appear on the face of the
contract.25 Thus, what were insured against were the accounts of IMC and LSPI with petitioner which remained
unpaid 45 days after the loss through fire, and not the loss or destruction of the goods delivered.

97
Petitioner argues that IMC bears the risk of loss because it expressly reserved ownership of the goods by stipulating
in the sales invoices that "[i]t is further agreed that merely for purpose of securing the payment of the purchase price
the above described merchandise remains the property of the vendor until the purchase price thereof is fully paid." 26
The Court is not persuaded.
The present case clearly falls under paragraph (1), Article 1504 of the Civil Code:
ART. 1504. Unless otherwise agreed, the goods remain at the seller's risk until the ownership therein is transferred
to the buyer, but when the ownership therein is transferred to the buyer the goods are at the buyer's risk whether
actual delivery has been made or not, except that:
(1) Where delivery of the goods has been made to the buyer or to a bailee for the buyer, in pursuance of the
contract and the ownership in the goods has been retained by the seller merely to secure performance by the buyer
of his obligations under the contract, the goods are at the buyer's risk from the time of such delivery; (Emphasis
supplied)
xxxx
Thus, when the seller retains ownership only to insure that the buyer will pay its debt, the risk of loss is borne by the
buyer.27 Accordingly, petitioner bears the risk of loss of the goods delivered.
IMC and LSPI did not lose complete interest over the goods. They have an insurable interest until full payment of
the value of the delivered goods. Unlike the civil law concept of res perit domino, where ownership is the basis for
consideration of who bears the risk of loss, in property insurance, one's interest is not determined by concept of title,
but whether insured has substantial economic interest in the property.28
Section 13 of our Insurance Code defines insurable interest as "every interest in property, whether real or personal,
or any relation thereto, or liability in respect thereof, of such nature that a contemplated peril might directly damnify
the insured." Parenthetically, under Section 14 of the same Code, an insurable interest in property may consist in:
(a) an existing interest; (b) an inchoate interest founded on existing interest; or (c) an expectancy, coupled with an
existing interest in that out of which the expectancy arises.
Therefore, an insurable interest in property does not necessarily imply a property interest in, or a lien upon, or
possession of, the subject matter of the insurance, and neither the title nor a beneficial interest is requisite to the
existence of such an interest, it is sufficient that the insured is so situated with reference to the property that he
would be liable to loss should it be injured or destroyed by the peril against which it is insured. 29 Anyone has an
insurable interest in property who derives a benefit from its existence or would suffer loss from its
destruction.30Indeed, a vendor or seller retains an insurable interest in the property sold so long as he has any
interest therein, in other words, so long as he would suffer by its destruction, as where he has a vendor's lien. 31 In
this case, the insurable interest of IMC and LSPI pertain to the unpaid accounts appearing in their Books of Account
45 days after the time of the loss covered by the policies.
The next question is: Is petitioner liable for the unpaid accounts?
Petitioner's argument that it is not liable because the fire is a fortuitous event under Article 1174 32 of the Civil Code is
misplaced. As held earlier, petitioner bears the loss under Article 1504 (1) of the Civil Code.
Moreover, it must be stressed that the insurance in this case is not for loss of goods by fire but for petitioner's
accounts with IMC and LSPI that remained unpaid 45 days after the fire. Accordingly, petitioner's obligation is for the

98
payment of money. As correctly stated by the CA, where the obligation consists in the payment of money, the failure
of the debtor to make the payment even by reason of a fortuitous event shall not relieve him of his liability.33 The
rationale for this is that the rule that an obligor should be held exempt from liability when the loss occurs thru a
fortuitous event only holds true when the obligation consists in the delivery of a determinate thing and there is no
stipulation holding him liable even in case of fortuitous event. It does not apply when the obligation is pecuniary in
nature.34
Under Article 1263 of the Civil Code, "[i]n an obligation to deliver a generic thing, the loss or destruction of anything
of the same kind does not extinguish the obligation." If the obligation is generic in the sense that the object thereof is
designated merely by its class or genus without any particular designation or physical segregation from all others of
the same class, the loss or destruction of anything of the same kind even without the debtor's fault and before he
has incurred in delay will not have the effect of extinguishing the obligation. 35This rule is based on the principle that
the genus of a thing can never perish. Genus nunquan perit.36 An obligation to pay money is generic; therefore, it is
not excused by fortuitous loss of any specific property of the debtor.37
Thus, whether fire is a fortuitous event or petitioner was negligent are matters immaterial to this case. What is
relevant here is whether it has been established that petitioner has outstanding accounts with IMC and LSPI.
With respect to IMC, the respondent has adequately established its claim. Exhibits "C" to "C-22" 38 show that
petitioner has an outstanding account with IMC in the amount of P2,119,205.00. Exhibit "E"39 is the check voucher
evidencing payment to IMC. Exhibit "F"40 is the subrogation receipt executed by IMC in favor of respondent upon
receipt of the insurance proceeds. All these documents have been properly identified, presented and marked as
exhibits in court. The subrogation receipt, by itself, is sufficient to establish not only the relationship of respondent as
insurer and IMC as the insured, but also the amount paid to settle the insurance claim. The right of subrogation
accrues simply upon payment by the insurance company of the insurance claim. 41Respondent's action against
petitioner is squarely sanctioned by Article 2207 of the Civil Code which provides:
Art. 2207. If the plaintiff's property has been insured, and he has received indemnity from the insurance company for
the injury or loss arising out of the wrong or breach of contract complained of, the insurance company shall be
subrogated to the rights of the insured against the wrongdoer or the person who has violated the contract. x x x
Petitioner failed to refute respondent's evidence.
As to LSPI, respondent failed to present sufficient evidence to prove its cause of action. No evidentiary weight can
be given to Exhibit "F Levi Strauss",42 a letter dated April 23, 1991 from petitioner's General Manager, Stephen S.
Gaisano, Jr., since it is not an admission of petitioner's unpaid account with LSPI. It only confirms the loss of Levi's
products in the amount of P535,613.00 in the fire that razed petitioner's building on February 25, 1991.
Moreover, there is no proof of full settlement of the insurance claim of LSPI; no subrogation receipt was offered in
evidence. Thus, there is no evidence that respondent has been subrogated to any right which LSPI may have
against petitioner. Failure to substantiate the claim of subrogation is fatal to petitioner's case for recovery of the
amount of P535,613.00.
WHEREFORE, the petition is partly GRANTED. The assailed Decision dated October 11, 2000 and Resolution
dated April 11, 2001 of the Court of Appeals in CA-G.R. CV No. 61848 are AFFIRMED with the MODIFICATIONthat
the order to pay the amount of P535,613.00 to respondent is DELETED for lack of factual basis.
No pronouncement as to costs.
SO ORDERED.

99

CHAPTER IV

100
Republic of the Philippines
SUPREME COURT
Manila
FIRST DIVISION

G.R. No. 95546 November 6, 1992


MAKATI TUSCANY CONDOMINIUM CORPORATION, petitioner,
vs.
THE COURT OF APPEALS, AMERICAN HOME ASSURANCE CO., represented by American International
Underwriters (Phils.), Inc., respondent.

BELLOSILLO, J.:
This case involves a purely legal question: whether payment by installment of the premiums due on an insurance
policy invalidates the contract of insurance, in view of Sec. 77 of P.D. 612, otherwise known as the Insurance Code,
as amended, which provides:
Sec. 77. An insurer is entitled to the payment of the premium as soon as the thing is exposed to the
peril insured against. Notwithstanding any agreement to the contrary, no policy or contract of
insurance issued by an insurance company is valid and binding unless and until the premium thereof
has been paid, except in the case of a life or an industrial life policy whenever the grace period
provision applies.
Sometime in early 1982, private respondent American Home Assurance Co. (AHAC), represented by American
International Underwriters (Phils.), Inc., issued in favor of petitioner Makati Tuscany Condominium Corporation
(TUSCANY) Insurance Policy No. AH-CPP-9210452 on the latter's building and premises, for a period beginning 1
March 1982 and ending 1 March 1983, with a total premium of P466,103.05. The premium was paid on installments
on 12 March 1982, 20 May 1982, 21 June 1982 and 16 November 1982, all of which were accepted by private
respondent.
On 10 February 1983, private respondent issued to petitioner Insurance Policy No. AH-CPP-9210596, which
replaced and renewed the previous policy, for a term covering 1 March 1983 to 1 March 1984. The premium in the
amount of P466,103.05 was again paid on installments on 13 April 1983, 13 July 1983, 3 August 1983, 9 September
1983, and 21 November 1983. All payments were likewise accepted by private respondent.
On 20 January 1984, the policy was again renewed and private respondent issued to petitioner Insurance Policy No.
AH-CPP-9210651 for the period 1 March 1984 to 1 March 1985. On this renewed policy, petitioner made two
installment payments, both accepted by private respondent, the first on 6 February 1984 for P52,000.00 and the
second, on 6 June 1984 for P100,000.00. Thereafter, petitioner refused to pay the balance of the premium.
Consequently, private respondent filed an action to recover the unpaid balance of P314,103.05 for Insurance Policy
No. AH-CPP-9210651.

101
In its answer with counterclaim, petitioner admitted the issuance of Insurance Policy No. AH-CPP-9210651. It
explained that it discontinued the payment of premiums because the policy did not contain a credit clause in its favor
and the receipts for the installment payments covering the policy for 1984-85, as well as the two (2) previous
policies, stated the following reservations:
2. Acceptance of this payment shall not waive any of the company rights to deny liability on any
claim under the policy arising before such payments or after the expiration of the credit clause of the
policy; and
3. Subject to no loss prior to premium payment. If there be any loss such is not covered.
Petitioner further claimed that the policy was never binding and valid, and no risk attached to the policy. It then
pleaded a counterclaim for P152,000.00 for the premiums already paid for 1984-85, and in its answer with amended
counterclaim, sought the refund of P924,206.10 representing the premium payments for 1982-85.
After some incidents, petitioner and private respondent moved for summary judgment.
On 8 October 1987, the trial court dismissed the complaint and the counterclaim upon the following findings:
While it is true that the receipts issued to the defendant contained the aforementioned reservations,
it is equally true that payment of the premiums of the three aforementioned policies (being sought to
be refunded) were made during the lifetime or term of said policies, hence, it could not be said,
inspite of the reservations, that no risk attached under the policies. Consequently, defendant's
counterclaim for refund is not justified.
As regards the unpaid premiums on Insurance Policy No. AH-CPP-9210651, in view of the
reservation in the receipts ordinarily issued by the plaintiff on premium payments the only plausible
conclusion is that plaintiff has no right to demand their payment after the lapse of the term of said
policy on March 1, 1985. Therefore, the defendant was justified in refusing to pay the same. 1
Both parties appealed from the judgment of the trial court. Thereafter, the Court of Appeals rendered a
decision 2modifying that of the trial court by ordering herein petitioner to pay the balance of the premiums due on Policy
No. AH-CPP-921-651, or P314,103.05 plus legal interest until fully paid, and affirming the denial of the counterclaim. The
appellate court thus explained
The obligation to pay premiums when due is ordinarily as indivisible obligation to pay the entire
premium. Here, the parties herein agreed to make the premiums payable in installments, and there
is no pretense that the parties never envisioned to make the insurance contract binding between
them. It was renewed for two succeeding years, the second and third policies being a
renewal/replacement for the previous one. And the insured never informed the insurer that it was
terminating the policy because the terms were unacceptable.
While it may be true that under Section 77 of the Insurance Code, the parties may not agree to make
the insurance contract valid and binding without payment of premiums, there is nothing in said
section which suggests that the parties may not agree to allow payment of the premiums in
installment, or to consider the contract as valid and binding upon payment of the first premium.
Otherwise, we would allow the insurer to renege on its liability under the contract, had a loss
incurred (sic) before completion of payment of the entire premium, despite its voluntary acceptance
of partial payments, a result eschewed by a basic considerations of fairness and equity.

102
To our mind, the insurance contract became valid and binding upon payment of the first premium,
and the plaintiff could not have denied liability on the ground that payment was not made in full, for
the reason that it agreed to accept installment payment. . . . 3
Petitioner now asserts that its payment by installment of the premiums for the insurance policies for 1982, 1983 and
1984 invalidated said policies because of the provisions of Sec. 77 of the Insurance Code, as amended, and by the
conditions stipulated by the insurer in its receipts, disclaiming liability for loss for occurring before payment of
premiums.
It argues that where the premiums is not actually paid in full, the policy would only be effective if there is an
acknowledgment in the policy of the receipt of premium pursuant to Sec. 78 of the Insurance Code. The absence of
an express acknowledgment in the policies of such receipt of the corresponding premium payments, and petitioner's
failure to pay said premiums on or before the effective dates of said policies rendered them invalid. Petitioner thus
concludes that there cannot be a perfected contract of insurance upon mere partial payment of the premiums
because under Sec. 77 of the Insurance Code, no contract of insurance is valid and binding unless the premium
thereof has been paid, notwithstanding any agreement to the contrary. As a consequence, petitioner seeks a refund
of all premium payments made on the alleged invalid insurance policies.
We hold that the subject policies are valid even if the premiums were paid on installments. The records clearly show
that petitioner and private respondent intended subject insurance policies to be binding and effective
notwithstanding the staggered payment of the premiums. The initial insurance contract entered into in 1982 was
renewed in 1983, then in 1984. In those three (3) years, the insurer accepted all the installment payments. Such
acceptance of payments speaks loudly of the insurer's intention to honor the policies it issued to petitioner. Certainly,
basic principles of equity and fairness would not allow the insurer to continue collecting and accepting the
premiums, although paid on installments, and later deny liability on the lame excuse that the premiums were not
prepared in full.
We therefore sustain the Court of Appeals. We quote with approval the well-reasoned findings and conclusion of the
appellate court contained in its Resolution denying the motion to reconsider its Decision
While the import of Section 77 is that prepayment of premiums is strictly required as a condition to
the validity of the contract, We are not prepared to rule that the request to make installment
payments duly approved by the insurer, would prevent the entire contract of insurance from going
into effect despite payment and acceptance of the initial premium or first installment. Section 78 of
the Insurance Code in effect allows waiver by the insurer of the condition of prepayment by making
an acknowledgment in the insurance policy of receipt of premium as conclusive evidence of payment
so far as to make the policy binding despite the fact that premium is actually unpaid. Section 77
merely precludes the parties from stipulating that the policy is valid even if premiums are not paid,
but does not expressly prohibit an agreement granting credit extension, and such an agreement is
not contrary to morals, good customs, public order or public policy (De Leon, the Insurance Code, at
p. 175). So is an understanding to allow insured to pay premiums in installments not so proscribed.
At the very least, both parties should be deemed in estoppel to question the arrangement they have
voluntarily accepted. 4
The reliance by petitioner on Arce vs. Capital Surety and Insurance
Co. 5 is unavailing because the facts therein are substantially different from those in the case at bar. In Arce, no payment
was made by the insured at all despite the grace period given. In the case before Us, petitioner paid the initial installment
and thereafter made staggered payments resulting in full payment of the 1982 and 1983 insurance policies. For the 1984
policy, petitioner paid two (2) installments although it refused to pay the balance.

103
It appearing from the peculiar circumstances that the parties actually intended to make three (3) insurance contracts
valid, effective and binding, petitioner may not be allowed to renege on its obligation to pay the balance of the
premium after the expiration of the whole term of the third policy (No. AH-CPP-9210651) in March 1985. Moreover,
as correctly observed by the appellate court, where the risk is entire and the contract is indivisible, the insured is not
entitled to a refund of the premiums paid if the insurer was exposed to the risk insured for any period, however brief
or momentary.
WHEREFORE, finding no reversible error in the judgment appealed from, the same is AFFIRMED. Costs against
petitioner.
SO ORDERED.

Republic of the Philippines


SUPREME COURT
Manila
FIRST DIVISION

G.R. No. 137172 June 15, 1999


UCPB GENERAL INSURANCE CO., INC., petitioner,
vs.
MASAGANA TELAMART, INC., respondent.

104

PARDO, J.:
The case is an appeal via certiorari seeking to set aside the decision of the Court of Appeals, 1 affirming with
modification that of the Regional Trial Court, Branch 58, Makati, ordering petitioner to pay respondent the sum of
P18,645,000.00, as the proceeds of the insurance coverage of respondent's property razed by fire; 25% of the total
amount due as attorney's fees and P25,000.00 as litigation expenses, and costs.
The facts are undisputed and may be related as follows:
On April 15, 1991, petitioner issued five (5) insurance policies covering respondent's various property described
therein against fire, for the period from May 22, 1991 to May 22, 1992.
In March 1992, petitioner evaluated the policies and decided not to renew them upon expiration of their terms on
May 22, 1992. Petitioner advised respondent's broker, Zuellig Insurance Brokers, Inc. of its intention not to renew
the policies.
On April 6, 1992, petitioner gave written notice to respondent of the non-renewal of the policies at the address
stated in the policies.
On June 13, 1992, fire razed respondent's property covered by three of the insurance policies petitioner issued.
On July 13, 1992, respondent presented to petitioner's cashier at its head office five (5) manager's checks in the
total amount of P225,753.95, representing premium for the renewal of the policies from May 22, 1992 to May 22,
1993. No notice of loss was filed by respondent under the policies prior to July 14, 1992.
On July 14, 1992, respondent filed with petitioner its formal claim for indemnification of the insured property razed
by fire.
On the same day, July 14, 1992, petitioner returned to respondent the five (5) manager's checks that it tendered,
and at the same time rejected respondent's claim for the reasons (a) that the policies had expired and were not
renewed, and (b) that the fire occurred on June 13, 1992, before respondent's tender of premium payment.
On July 21, 1992, respondent filed with the Regional Trial Court, Branch 58, Makati City, a civil complaint against
petitioner for recovery of P18,645,000.00, representing the face value of the policies covering respondent's insured
property razed by fire, and for attorney's fees. 2
On October 23, 1992, after its motion to dismiss had been denied, petitioner filed an answer to the complaint. It
alleged that the complaint "fails to state a cause of action"; that petitioner was not liable to respondent for insurance
proceeds under the policies because at the time of the loss of respondent's property due to fire, the policies had
long expired and were not renewed. 3
After due trial, on March 10, 1993, the Regional Trial Court, Branch 58, Makati, rendered decision, the dispositive
portion of which reads:
WHEREFORE, premises considered, judgment is hereby rendered in favor of the plaintiff and
against the defendant, as follows:

105
(1) Authorizing and allowing the plaintiff to consign/deposit with this Court the sum of P225,753.95
(refused by the defendant) as full payment of the corresponding premiums for the replacementrenewal policies for Exhibits A, B, C, D and E;
(2) Declaring plaintiff to have fully complied with its obligation to pay the premium thereby rendering
the replacement-renewal policy of Exhibits A, B, C, D and E effective and binding for the duration
May 22, 1992 until May 22, 1993; and, ordering defendant to deliver forthwith to plaintiff the said
replacement-renewal policies;
(3) Declaring Exhibits A & B, in force from August 22, 1991 up to August 23, 1992 and August 9,
1991 to August 9, 1992, respectively; and
(4) Ordering the defendant to pay plaintiff the sums of: (a) P18,645,000.00 representing the latter's
claim for indemnity under Exhibits A, B & C and/or its replacement-renewal policies; (b) 25% of the
total amount due as and for attorney's fees; (c) P25,000.00 as necessary litigation expenses; and,
(d) the costs of suit.
All other claims and counterclaims asserted by the parties are denied and/or dismissed, including
plaintiff's claim for interests.
SO ORDERED.
Makati, Metro-Manila, March 10, 1993.
ZOSIMO Z. ANGELES.
Judge. 4
In due time, petitioner appealed to the Court of Appeals. 5
On September 7, 1998, the Court of Appeals promulgated its decision 6 affirming that of the Regional Trial Court with
the modification that item No. 3 of the dispositive portion was deleted, and the award of attorney's fees was reduced to
10% of the total amount due. 7
The Court of Appeals held that following previous practise, respondent was allowed a sixty (60) to ninety (90) day
credit term for the renewal of its policies, and that the acceptance of the late premium payment suggested an
understanding that payment could be made later.
Hence, this appeal.
By resolution adopted on March 24, 1999, we required respondent to comment on the petition, not to file a motion to
dismiss within ten (10) days from notice. 8 On April 22, 1999, respondent filed its comment. 9
Respondent submits that the Court of Appeals correctly ruled that no timely notice of non-renewal was sent. The
notice of non-renewal sent to broker Zuellig which claimed that it verbally notified the insurance agency but not
respondent itself did not suffice. Respondent submits further that the Court of Appeals did not err in finding that
there existed a sixty (60) to ninety (90) days credit agreement between UCPB and Masagana, and that, finally, the
Supreme Court could not review factual findings of the lower court affirmed by the Court of Appeals. 10
We give due course to the appeal.

106
The basic issue raised is whether the fire insurance policies issued by petitioner to the respondent covering the
period May 22, 1991 to May 22, 1992, had expired on the latter date or had been extended or renewed by an
implied credit arrangement though actual payment of premium was tendered on a later date after the occurrence of
the risk (fire) insured against.
The answer is easily found in the Insurance Code. No, an insurance policy, other than life, issued originally or on
renewal, is not valid and binding until actual payment of the premium. Any agreement to the contrary is void. 11The
parties may not agree expressly or impliedly on the extension of creditor time to pay the premium and consider the policy
binding before actual payment.
The case of Malayan Insurance Co., Inc. vs. Cruz-Arnaldo, 12 cited by the Court of Appeals, is not applicable. In that
case, payment of the premium was in fact actually made on December 24, 1981, and the fire occurred on January 18,
1982. Here, the payment of the premium for renewal of the policies was tendered on July 13, 1992, a month after the fire
occurred on June 13, 1992. The assured did not even give the insurer a notice of loss within a reasonable time after
occurrence of the fire.
WHEREFORE, the Court hereby REVERSES and SETS ASIDE the decision of the Court of Appeals in CA-G.R. CV
No. 42321. In lieu thereof the Court renders judgment dismissing respondent's complaint and petitioner's
counterclaims thereto filed with the Regional Trial Court, Branch 58, Makati City, in Civil Case No. 92-2023. Without
costs. SO ORDERED.
1wphi1.nt

Republic of the Philippines


SUPREME COURT
Manila
EN BANC
G.R. No. 137172

April 4, 2001

UCPB GENERAL INSURANCE CO., INC., petitioner,


vs.
MASAGANA TELAMART, INC., respondent.
RESOLUTION
DAVIDE, JR., C.J.:
In our decision of 15 June 1999 in this case, we reversed and set aside the assailed decision 1 of the Court of
Appeals, which affirmed with modification the judgment of the trial court (a) allowing Respondent to consign the sum
of P225,753.95 as full payment of the premiums for the renewal of the five insurance policies on Respondent's
properties; (b) declaring the replacement-renewal policies effective and binding from 22 May 1992 until 22 May
1993; and (c) ordering Petitioner to pay Respondent P18,645,000.00 as indemnity for the burned properties covered
by the renewal-replacement policies. The modification consisted in the (1) deletion of the trial court's declaration that
three of the policies were in force from August 1991 to August 1992; and (2) reduction of the award of the attorney's
fees from 25% to 10% of the total amount due the Respondent.
The material operative facts upon which the appealed judgment was based are summarized by the Court of Appeals
in its assailed decision as follows:
Plaintiff [herein Respondent] obtained from defendant [herein Petitioner] five (5) insurance policies (Exhibits
"A" to "E", Record, pp. 158-175) on its properties [in Pasay City and Manila] . . . .

107
All five (5) policies reflect on their face the effectivity term: "from 4:00 P.M. of 22 May 1991 to 4:00 P.M. of 22
May 1992." On June 13, 1992, plaintiffs properties located at 2410-2432 and 2442-2450 Taft Avenue, Pasay
City were razed by fire. On July 13, 1992, plaintiff tendered, and defendant accepted, five (5) Equitable Bank
Manager's Checks in the total amount of P225,753.45 as renewal premium payments for which Official
Receipt Direct Premium No. 62926 (Exhibit "Q", Record, p. 191) was issued by defendant. On July 14, 1992,
Masagana made its formal demand for indemnification for the burned insured properties. On the same day,
defendant returned the five (5) manager's checks stating in its letter (Exhibit "R" / "8", Record, p. 192) that it
was rejecting Masagana's claim on the following grounds:
"a) Said policies expired last May 22, 1992 and were not renewed for another term;
b) Defendant had put plaintiff and its alleged broker on notice of non-renewal earlier; and
c) The properties covered by the said policies were burned in a fire that took place last June 13,
1992, or before tender of premium payment."

(Record, p. 5)

Hence Masagana filed this case.


The Court of Appeals disagreed with Petitioner's stand that Respondent's tender of payment of the premiums on 13
July 1992 did not result in the renewal of the policies, having been made beyond the effective date of renewal as
provided under Policy Condition No. 26, which states:
26. Renewal Clause. Unless the company at least forty five days in advance of the end of the policy
period mails or delivers to the assured at the address shown in the policy notice of its intention not to renew
the policy or to condition its renewal upon reduction of limits or elimination of coverages, the assured shall
be entitled to renew the policy upon payment of the premium due on the effective date of renewal.
Both the Court of Appeals and the trial court found that sufficient proof exists that Respondent, which had procured
insurance coverage from Petitioner for a number of years, had been granted a 60 to 90-day credit term for the
renewal of the policies. Such a practice had existed up to the time the claims were filed. Thus:
Fire Insurance Policy No. 34658 covering May 22, 1990 to May 22, 1991 was issued on May 7, 1990 but
premium was paid more than 90 days later on August 31, 1990 under O.R. No. 4771 (Exhs. "T" and "T-1").
Fire Insurance Policy No. 34660 for Insurance Risk Coverage from May 22, 1990 to May 22, 1991 was
issued by UCPB on May 4, 1990 but premium was collected by UCPB only on July 13, 1990 or more than
60 days later under O.R. No. 46487 (Exhs. "V" and "V-1"). And so were as other policies: Fire Insurance
Policy No. 34657 covering risks from May 22, 1990 to May 22, 1991 was issued on May 7, 1990 but
premium therefor was paid only on July 19, 1990 under O.R. No. 46583 (Exhs. "W" and "W-1"). Fire
Insurance Policy No. 34661 covering risks from May 22, 1990 to May 22, 1991 was issued on May 3, 1990
but premium was paid only on July 19, 1990 under O.R. No. 46582 (Exhs. "X" and "X-1"). Fire Insurance
Policy No. 34688 for insurance coverage from May 22, 1990 to May 22, 1991 was issued on May 7, 1990
but premium was paid only on July 19, 1990 under O.R. No. 46585 (Exhs. "Y" and "Y-1"). Fire Insurance
Policy No. 29126 to cover insurance risks from May 22, 1989 to May 22, 1990 was issued on May 22, 1989
but premium therefor was collected only on July 25, 1990[sic] under O.R. No. 40799 (Exhs. "AA" and "AA1"). Fire Insurance Policy No. HO/F-26408 covering risks from January 12, 1989 to January 12, 1990 was
issued to Intratrade Phils. (Masagana's sister company) dated December 10, 1988 but premium therefor
was paid only on February 15, 1989 under O.R. No. 38075 (Exhs. "BB" and "BB-1"). Fire Insurance Policy
No. 29128 was issued on May 22, 1989 but premium was paid only on July 25, 1989 under O.R. No. 40800
for insurance coverage from May 22, 1989 to May 22, 1990 (Exhs. "CC" and "CC-1"). Fire Insurance Policy
No. 29127 was issued on May 22, 1989 but premium was paid only on July 17, 1989 under O.R. No. 40682

108
for insurance risk coverage from May 22, 1989 to May 22, 1990 (Exhs. "DD" and "DD-1"). Fire Insurance
Policy No. HO/F-29362 was issued on June 15, 1989 but premium was paid only on February 13, 1990
under O.R. No. 39233 for insurance coverage from May 22, 1989 to May 22, 1990 (Exhs. "EE" and "EE-1").
Fire Insurance Policy No. 26303 was issued on November 22, 1988 but premium therefor was collected only
on March 15, 1989 under O.R. NO. 38573 for insurance risks coverage from December 15, 1988 to
December 15, 1989 (Exhs. "FF" and "FF-1").
Moreover, according to the Court of Appeals the following circumstances constitute preponderant proof that no
timely notice of non-renewal was made by Petitioner:
(1) Defendant-appellant received the confirmation (Exhibit "11", Record, p. 350) from Ultramar Reinsurance
Brokers that plaintiff's reinsurance facility had been confirmed up to 67.5% only on April 15, 1992 as
indicated on Exhibit "11". Apparently, the notice of non-renewal (Exhibit "7," Record, p. 320) was sent not
earlier than said date, or within 45 days from the expiry dates of the policies as provided under Policy
Condition No. 26; (2) Defendant insurer unconditionally accepted, and issued an official receipt for, the
premium payment on July 1[3], 1992 which indicates defendant's willingness to assume the risk despite only
a 67.5% reinsurance cover[age]; and (3) Defendant insurer appointed Esteban Adjusters and Valuers to
investigate plaintiff's claim as shown by the letter dated July 17, 1992 (Exhibit "11", Record, p. 254).
In our decision of 15 June 1999, we defined the main issue to be "whether the fire insurance policies issued by
petitioner to the respondent covering the period from May 22, 1991 to May 22, 1992 . . . had been extended or
renewed by an implied credit arrangement though actual payment of premium was tendered on a later date and
after the occurrence of the (fire) risk insured against." We resolved this issue in the negative in view of Section 77 of
the Insurance Code and our decisions in Valenzuela v. Court of Appeals; 2 South Sea Surety and Insurance Co., Inc.
v. Court of Appeals; 3 and Tibay v. Court of Appeals. 4 Accordingly, we reversed and set aside the decision of the
Court of Appeals.
Respondent seasonably filed a motion for the reconsideration of the adverse verdict. It alleges in the motion that we
had made in the decision our own findings of facts, which are not in accord with those of the trial court and the Court
of Appeals. The courts below correctly found that no notice of non-renewal was made within 45 days before 22 May
1992, or before the expiration date of the fire insurance policies. Thus, the policies in question were renewed by
operation of law and were effective and valid on 30 June 1992 when the fire occurred, since the premiums were
paid within the 60- to 90-day credit term.
Respondent likewise disagrees with our ruling that parties may neither agree expressly or impliedly on the extension
of credit or time to pay the premium nor consider a policy binding before actual payment. It urges the Court to take
judicial notice of the fact that despite the express provision of Section 77 of the Insurance Code, extension of credit
terms in premium payment has been the prevalent practice in the insurance industry. Most insurance companies,
including Petitioner, extend credit terms because Section 77 of the Insurance Code is not a prohibitive injunction but
is merely designed for the protection of the parties to an insurance contract. The Code itself, in Section 78,
authorizes the validity of a policy notwithstanding non-payment of premiums.
Respondent also asserts that the principle of estoppel applies to Petitioner. Despite its awareness of Section 77
Petitioner persuaded and induced Respondent to believe that payment of premium on the 60- to 90-day credit term
was perfectly alright; in fact it accepted payments within 60 to 90 days after the due dates. By extending credit and
habitually accepting payments 60 to 90 days from the effective dates of the policies, it has implicitly agreed to
modify the tenor of the insurance policy and in effect waived the provision therein that it would pay only for the loss
or damage in case the same occurred after payment of the premium.
Petitioner filed an opposition to the Respondent's motion for reconsideration. It argues that both the trial court and
the Court of Appeals overlooked the fact that on 6 April 1992 Petitioner sent by ordinary mail to Respondent a notice
of non-renewal and sent by personal delivery a copy thereof to Respondent's broker, Zuellig. Both courts likewise
ignored the fact that Respondent was fully aware of the notice of non-renewal. A reading of Section 66 of the
Insurance Code readily shows that in order for an insured to be entitled to a renewal of a non-life policy, payment of
the premium due on the effective date of renewal should first be made. Respondent's argument that Section 77 is
not a prohibitive provision finds no authoritative support.

109
Upon a meticulous review of the records and reevaluation of the issues raised in the motion for reconsideration and
the pleadings filed thereafter by the parties, we resolved to grant the motion for reconsideration. The following facts,
as found by the trial court and the Court of Appeals, are indeed duly established:
1. For years, Petitioner had been issuing fire policies to the Respondent, and these policies were annually
renewed.
2. Petitioner had been granting Respondent a 60- to 90-day credit term within which to pay the premiums on
the renewed policies.
3. There was no valid notice of non-renewal of the policies in question, as there is no proof at all that the
notice sent by ordinary mail was received by Respondent, and the copy thereof allegedly sent to Zuellig was
ever transmitted to Respondent.
4. The premiums for the policies in question in the aggregate amount of P225,753.95 were paid by
Respondent within the 60- to 90-day credit term and were duly accepted and received by Petitioner's
cashier.
The instant case has to rise or fall on the core issue of whether Section 77 of the Insurance Code of 1978 (P.D. No.
1460) must be strictly applied to Petitioner's advantage despite its practice of granting a 60- to 90-day credit term for
the payment of premiums.
Section 77 of the Insurance Code of 1978 provides:
SECTION 77. An insurer is entitled to payment of the premium as soon as the thing insured is exposed to
the peril insured against. Notwithstanding any agreement to the contrary, no policy or contract of insurance
issued by an insurance company is valid and binding unless and until the premium thereof has been paid,
except in the case of a life or an industrial life policy whenever the grace period provision applies.
This Section is a reproduction of Section 77 of P.D. No. 612 (The Insurance Code) promulgated on 18 December
1974. In turn, this Section has its source in Section 72 of Act No. 2427 otherwise known as the Insurance Act as
amended by R.A. No. 3540, approved on 21 June 1963, which read:
SECTION 72. An insurer is entitled to payment of premium as soon as the thing insured is exposed to the
peril insured against, unless there is clear agreement to grant the insured credit extension of the premium
due. No policy issued by an insurance company is valid and binding unless and until the premium thereof
has been paid. (Italic supplied)
It can be seen at once that Section 77 does not restate the portion of Section 72 expressly permitting an agreement
to extend the period to pay the premium. But are there exceptions to Section 77?
The answer is in the affirmative.
The first exception is provided by Section 77 itself, and that is, in case of a life or industrial life policy whenever the
grace period provision applies.
The second is that covered by Section 78 of the Insurance Code, which provides:
SECTION 78. Any acknowledgment in a policy or contract of insurance of the receipt of premium is
conclusive evidence of its payment, so far as to make the policy binding, notwithstanding any stipulation
therein that it shall not be binding until premium is actually paid.
A third exception was laid down in Makati Tuscany Condominium Corporation vs. Court of Appeals, 5 wherein we
ruled that Section 77 may not apply if the parties have agreed to the payment in installments of the premium and
partial payment has been made at the time of loss. We said therein, thus:

110
We hold that the subject policies are valid even if the premiums were paid on installments. The records
clearly show that the petitioners and private respondent intended subject insurance policies to be binding
and effective notwithstanding the staggered payment of the premiums. The initial insurance contract entered
into in 1982 was renewed in 1983, then in 1984. In those three years, the insurer accepted all the installment
payments. Such acceptance of payments speaks loudly of the insurer's intention to honor the policies it
issued to petitioner. Certainly, basic principles of equity and fairness would not allow the insurer to continue
collecting and accepting the premiums, although paid on installments, and later deny liability on the lame
excuse that the premiums were not prepaid in full.
Not only that. In Tuscany, we also quoted with approval the following pronouncement of the Court of Appeals in its
Resolution denying the motion for reconsideration of its decision:
While the import of Section 77 is that prepayment of premiums is strictly required as a condition to the
validity of the contract, We are not prepared to rule that the request to make installment payments duly
approved by the insurer would prevent the entire contract of insurance from going into effect despite
payment and acceptance of the initial premium or first installment. Section 78 of the Insurance Code in effect
allows waiver by the insurer of the condition of prepayment by making an acknowledgment in the insurance
policy of receipt of premium as conclusive evidence of payment so far as to make the policy binding despite
the fact that premium is actually unpaid. Section 77 merely precludes the parties from stipulating that the
policy is valid even if premiums are not paid, but does not expressly prohibit an agreement granting credit
extension, and such an agreement is not contrary to morals, good customs, public order or public policy (De
Leon, The Insurance Code, p. 175). So is an understanding to allow insured to pay premiums in installments
not so prescribed. At the very least, both parties should be deemed in estoppel to question the arrangement
they have voluntarily accepted.
By the approval of the aforequoted findings and conclusion of the Court of Appeals, Tuscany has provided a fourth
exception to Section 77, namely, that the insurer may grant credit extension for the payment of the premium. This
simply means that if the insurer has granted the insured a credit term for the payment of the premium and loss
occurs before the expiration of the term, recovery on the policy should be allowed even though the premium is paid
after the loss but within the credit term.
Moreover, there is nothing in Section 77 which prohibits the parties in an insurance contract to provide a credit term
within which to pay the premiums. That agreement is not against the law, morals, good customs, public order or
public policy. The agreement binds the parties. Article 1306 of the Civil Code provides:
ARTICLE 1306. The contracting parties may establish such stipulations clauses, terms and conditions as
they may deem convenient, provided they are not contrary to law, morals, good customs, public order, or
public policy.
Finally in the instant case, it would be unjust and inequitable if recovery on the policy would not be permitted against
Petitioner, which had consistently granted a 60- to 90-day credit term for the payment of premiums despite its full
awareness of Section 77. Estoppel bars it from taking refuge under said Section, since Respondent relied in good
faith on such practice. Estoppel then is the fifth exception to Section 77.
WHEREFORE, the Decision in this case of 15 June 1999 is RECONSIDERED and SET ASIDE, and a new
one is hereby entered DENYING the instant petition for failure of Petitioner to sufficiently show that a
reversible error was committed by the Court of Appeals in its challenged decision, which is hereby
AFFIRMED in toto.
No pronouncement as to cost.
SO ORDERED.

111

112

Republic of the Philippines


SUPREME COURT
Manila
FIRST DIVISION

G.R. No. 130421 June 28, 1999


AMERICAN HOME ASSURANCE COMPANY, petitioner,
vs.
ANTONIO CHUA, respondent.

DAVIDE, JR. C.J.:


In this petition for review on certiorari under Rule 45 of the 1997 Rules of Civil Procedure, petitioner seeks the
reversal of the decision 1 of the Court of Appeals in CA-G.R. CV No. 40751, which affirmed in toto the decision of the
Regional Trial Court, Makati City, Branch 150 (hereafter trial court), in Civil Case No. 91-1009.
Petitioner is a domestic corporation engaged in the insurance business. Sometime in 1990, respondent obtained
from petitioner a fire insurance covering the stock-in-trade of his business, Moonlight Enterprises, located at
Valencia, Bukidnon. The insurance was due to expire on 25 March 1990.
On 5 April 1990 respondent issued PCIBank Check No. 352123 in the amount of P2,983.50 to petitioner's agent,
James Uy, as payment for the renewal of the policy. In turn, the latter delivered Renewal Certificate No. 00099047 to
respondent. The check was drawn against a Manila bank and deposited in petitioner's bank account in Cagayan de
Oro City. The corresponding official receipt was issued on 10 April. Subsequently, a new insurance policy, Policy No.
206-4234498-7, was issued, whereby petitioner undertook to indemnify respondent for any damage or loss arising
from fire up to P200,000 for the period 25 March 1990 to 25 March 1991.
On 6 April 1990 Moonlight Enterprises was completely razed by fire. Total loss was estimated between P4,000,000
and P5,000,000. Respondent filed an insurance claim with petitioner and four other co-insurers, namely, Pioneer
Insurance and Surety Corporation, Prudential Guarantee and Assurance, Inc., Filipino Merchants Insurance Co. and
Domestic Insurance Company of the Philippines. Petitioner refused to honor the claim notwithstanding several
demands by respondent, thus, the latter filed an action against petitioner before the trial court.

113
In its defense, petitioner claimed there was no existing insurance contract when the fire occurred since respondent
did not pay the premium. It also alleged that even assuming there was a contract, respondent violated several
conditions of the policy, particularly: (1) his submission of fraudulent income tax return and financial statements; (2)
his failure to establish the actual loss, which petitioner assessed at P70,000; and (3) his failure to notify to petitioner
of any insurance already effected to cover the insured goods. These violations, petitioner insisted, justified the
denial of the claim.
The trial court ruled in favor of respondent. It found that respondent paid by way of check a day before the fire
occurred. The check, which was deposited in petitioner's bank account, was even acknowledged in the renewal
certificate issued by petitioner's agent. It declared that the alleged fraudulent documents were limited to the disparity
between the official receipts issued by the Bureau of Internal Revenue (BIR) and the income tax returns for the
years 1987 to 1989. All the other documents were found to be genuine. Nonetheless, it gave credence to the BIR
certification that respondent paid the corresponding taxes due for the questioned years.
As to respondent's failure to notify petitioner of the other insurance contracts covering the same goods, the trial
court held that petitioner failed to show that such omission was intentional and fraudulent. Finally, it noted that
petitioner's investigation of respondent's claim was done in collaboration with the representatives of other insurance
companies who found no irregularity therein. In fact, Pioneer Insurance and Surety Corporation and Prudential
Guarantee and Assurance, Inc. promptly paid the claims filed by respondent.
The trial court decreed as follows:
WHEREFORE, judgment is hereby rendered in favor of [respondent] and against the [petitioner]
ordering the latter to pay the former the following:
1. P200,000.00, representing the amount of the insurance, plus legal interest from
the date of filing of this case;
2. P200,000.00 as moral damages;
3. P200,000.00 as loss of profit;
4. P100,000.00 as exemplary damages;
5. P50,000.00 as attorney's fees; and
6. Cost of suit.
On appeal, the assailed decision was affirmed in toto by the Court of Appeals. The Court of Appeals found that
respondent's claim was substantially proved and petitioner's unjustified refusal to pay the claim entitled respondent
to the award of damages.
Its motion for reconsideration of the judgment having been denied, petitioner filed the petition in this case. Petitioner
reiterates its stand that there was no existing insurance contract between the parties. It invokes Section 77 of the
Insurance Code, which provides:
An insurer is entitled to payment of the premium as soon as the thing insured is exposed to the peril
insured against. Notwithstanding any agreement to the contrary, no policy or contract of insurance

114
issued by an insurance company is valid and binding unless and until the premium thereof has been
paid, except in the case of life or an industrial life policy whenever the grace period provision applies.
and cites the case of Arce v. Capital Insurance & Surety Co., Inc., 2 where we ruled that unless and until the
premium is paid there is no insurance.
Petitioner emphasizes that when the fire occurred on 6 April 1990 the insurance contract was not yet subsisting
pursuant to Article 1249 3 of the Civil Code, which recognizes that a check can only effect payment once it has been
cashed. Although respondent testified that he gave the check on 5 April to a certain James Uy, the check, drawn against a
Manila bank and deposited in a Cagayan de Oro City bank, could not have been cleared by 6 April, the date of the fire. In
fact, the official receipt issued for respondent's check payment was dated 10 April 1990, four days after the fire occurred.
Citing jurisprudence, 4 petitioner also contends that respondent's non-disclosure of the other insurance contracts
rendered the policy void. It underscores the trial court's neglect in considering the Commission on Audit's certification that
the BIR receipts submitted by respondent were, in effect, fake since they were issued to other persons. Finally, petitioner
argues that the award of damages was excessive and unreasonable considering that it did not act in bad faith in denying
respondent's claim.
Respondent counters that the issue of non-payment of premium is a question of fact which can no longer be
assailed. The trial court's finding on the matter, which was affirmed by the Court of Appeals, is conclusive.
Respondent refutes the reason for petitioner's denial of his claim. As found by the trial court, petitioner's loss
adjuster admitted prior knowledge of respondent's existing insurance contracts with the other insurance companies.
Nonetheless, the loss adjuster recommended the denial of the claim, not because of the said contracts, but because
he was suspicious of the authenticity of certain documents which respondent submitted in filing his claim.
To bolster his argument, respondent cites Section 66 of the Insurance Code, 5 which requires the insurer to give a
notice to the insured of its intention to terminate the policy forty-five days before the policy period ends. In the instant
case, petitioner opted not to terminate the policy. Instead, it renewed the policy by sending its agent to respondent, who
was issued a renewal certificate upon delivery of his check payment for the renewal of premium. At this precise moment
the contract of insurance was executed and already in effect. Respondent also claims that it is standard operating
procedure in the provinces to pay insurance premiums by check when collected by insurance agents.
On the issue of damages, respondent maintains that the amounts awarded were reasonable. He cites numerous
trips he had to make from Cagayan de Oro City to Manila to follow up his rightful claim. He imputes bad faith on
petitioner who made enforcement of his claim difficult in the hope that he would eventually abandon it. He further
emphasizes that the adjusters of the other insurance companies recommended payment of his claim, and they
complied therewith.
In its reply, petitioner alleges that the petition questions the conclusions of law made by the trial court and the Court
of Appeals.
Petitioner invokes respondent's admission that his check for the renewal of the policy was received only on 10 April
1990, taking into account that the policy period was 25 March 1990 to 25 March 1991. The official receipt was dated
10 April 1990. Anent respondent's testimony that the check was given to petitioner's agent, a certain James Uy, the
latter points out that even respondent was not sure if Uy was indeed its agent. It faults respondent for not producing
Uy as his witness and not taking any receipt from him upon presentment of the check. Even assuming that the
check was received a day before the concurrence of the fire, there still could not have been payment until the check
was cleared.

115
Moreover, petitioner denies respondent's allegation that it intended a renewal of the contract for the renewal
certificate clearly specified the following conditions:
Subject to the payment by the assured of the amount due prior to renewal date, the policy shall be
renewed for the period stated.
Any payment tendered other than in cash is received subject to actual cash collection.
Subject to no loss prior to premium and payment. If there be any loss, is not covered [sic].
Petitioner asserts that an insurance contract can only be enforced upon the payment of the premium, which
should have been made before the renewal period.
Finally, in assailing the excessive damages awarded to respondent petitioner stresses that the policy in issue was
limited to a liability of P200,000; but the trial court granted the following monetary awards: P200,000 as actual
damages; P200,000 as moral damages; P100,000 as exemplary damages; and P50,000 as attorney's fees.
The following issues must be resolved: first, whether there was a valid payment of premium, considering that
respondent's check was cashed after the occurrence of the fire; second, whether respondent violated the policy by
his submission of fraudulent documents and non-disclosure of the other existing insurance contracts; and finally,
whether respondent is entitled to the award of damages.
The general rule in insurance laws is that unless the premium is paid the insurance policy is not valid and binding.
The only exceptions are life and industrial life insurance. 6 Whether payment was indeed made is a question of fact
which is best determined by the trial court. The trial court found, as affirmed by the Court of Appeals, that there was a valid
check payment by respondent to petitioner. Well-settled is the rule that the factual findings and conclusions of the trial
court and the Court of Appeals are entitled to great weight and respect, and will not be disturbed on appeal in the absence
of any clear showing that the trial court overlooked certain facts or circumstances which would substantially affect the
disposition of the case. 7 We see no reason to depart from this ruling.
According to the trial court the renewal certificate issued to respondent contained the acknowledgment that premium
had been paid. It is not disputed that the check drawn by respondent in favor of petitioner and delivered to its agent
was honored when presented and petitioner forthwith issued its official receipt to respondent on 10 April 1990.
Section 306 of the Insurance Code provides that any insurance company which delivers a policy or contract of
insurance to an insurance agent or insurance broker shall be deemed to have authorized such agent or broker to
receive on its behalf payment of any premium which is due on such policy or contract of insurance at the time of its
issuance or delivery or which becomes due thereon. 8 In the instant case, the best evidence of such authority is the fact
that petitioner accepted the check and issued the official receipt for the payment. It is, as well, bound by its agent's
acknowledgment of receipt of payment.
Sec. 78 of the Insurance Code explicitly provides:
An acknowledgment in a policy or contract of insurance of the receipt of premium is conclusive
evidence of its payment, so far as to make the policy binding, notwithstanding any stipulation therein
that it shall not be binding until the premium is actually paid.
This Section establishes a legal fiction of payment and should be interpreted as an exception to Section 77. 9
Is respondent guilty of the policy violations imputed against him? We are not convinced by petitioner's arguments.
The submission of the alleged fraudulent documents pertained to respondent's income tax returns for 1987 to 1989.

116
Respondent, however, presented a BIR certification that he had paid the proper taxes for the said years. The trial
court and the Court of Appeals gave credence to the certification and it being a question of fact, we hold that said
finding is conclusive.
Ordinarily, where the insurance policy specifies as a condition the disclosure of existing co-insurers, non-disclosure
thereof is a violation that entitles the insurer to avoid the policy. This condition is common in fire insurance policies
and is known as the "other insurance clause." The purpose for the inclusion of this clause is to prevent an increase
in the moral hazard. We have ruled on its validity and the case of Geagonia v. Court of Appeals 10 clearly illustrates
such principle. However, we see an exception in the instant case.
Citing Section 29 11 of the Insurance Code, the trial court reasoned that respondent's failure to disclose was not
intentional and fraudulent. The application of Section 29 is misplaced. Section 29 concerns concealment which is
intentional. The relevant provision is Section 75, which provides that:
A policy may declare that a violation of specified provisions thereof shall avoid it, otherwise the
breach of an immaterial provision does not avoid the policy.
To constitute a violation the other existing insurance contracts must be upon the same subject matter and with the
same interest and risk. 12 Indeed, respondent acquired several co-insurers and he failed to disclose this information to
petitioner. Nonetheless, petitioner is estopped from must invoking this argument. The trial court cited the testimony of
petitioner's loss adjuster who admitted previous knowledge of the co-insurers. Thus,
COURT:
Q The matter of additional insurance of other companies, was that ever discussed in
your investigation?
A Yes, sir.
Q In other words, from the start, you were aware the insured was insured with other
companies like Pioneer and so on?
A Yes, Your Honor.
Q But in your report you never recommended the denial of the claim simply because
of the non-disclosure of other insurance? [sic]
A Yes, Your Honor.
Q In other words, to be emphatic about this, the only reason you recommended the
denial of the claim, you found three documents to be spurious. That is your only
basis?
A Yes, Your Honor. 13 [Emphasis supplied]
Indubitably, it cannot be said that petitioner was deceived by respondent by the latter's non-disclosure of the other
insurance contracts when petitioner actually had prior knowledge thereof. Petitioner's loss adjuster had known all
along of the other existing insurance contracts, yet, he did not use that as basis for his recommendation of denial.
The loss adjuster, being an employee of petitioner, is deemed a representative of the latter whose awareness of the

117
other insurance contracts binds petitioner. We, therefore, hold that there was no violation of the "other insurance"
clause by respondent.
Petitioner is liable to pay its share of the loss. The trial court and the Court of Appeals were correct in awarding
P200,000 for this. There is, however, merit in petitioner's grievance against the damages and attorney's fees
awarded.
There is no legal and factual basis for the award of P200,000 for loss of profit. It cannot be denied that the fire totally
gutted respondent's business; thus, respondent no longer had any business to operate. His loss of profit cannot be
shouldered by petitioner whose obligation is limited to the object of insurance, which was the stock-in-trade, and not
the expected loss in income or profit.
Neither can we approve the award of moral and exemplary damages. At the core of this case is petitioner's alleged
breach of its obligation under a contract of insurance. Under Article 2220 of the Civil Code, moral damages may be
awarded in breaches of contracts where the defendant acted fraudulently or in bad faith. We find no such fraud or
bad faith. It must again be stressed that moral damages are emphatically not intended to enrich a plaintiff at the
expense of the defendant. Such damages are awarded only to enable the injured party to obtain means, diversion
or amusements that will serve to obviate the moral suffering he has undergone, by reason of the defendant's
culpable action. Its award is aimed at the restoration, within the limits of the possible, of the spiritual status quo ante,
and it must be proportional to the suffering inflicted. 14 When awarded, moral damages must not be palpably and
scandalously excessive as to indicate that it was the result of passion, prejudice or corruption on the part of the trial court
judge. 15
The law 16 is likewise clear that in contracts and quasi-contracts the court may award exemplary damages if the defendant
acted in a wanton, fraudulent, reckless, oppressive, or malevolent manner. Nothing thereof can be attributed to petitioner
which merely tried to resist what it claimed to be an unfounded claim for enforcement of the fire insurance policy.
As to attorney's fees, the general rule is that attorney's fees cannot be recovered as part of damages because of the
policy that no premium should be placed on the right to litigate. 17 In short, the grant of attorney's fees as part of
damages is the exception rather than the rule; counsel's fees are not awarded every time a party prevails in a suit. It can
be awarded only in the cases enumerated in Article 2208 of the Civil Code, and in all cases it must be
reasonable. 18Thereunder, the trial court may award attorney's fees where it deems just and equitable that it be so
granted. While we respect the trial court's exercise of its discretion in this case, the award of P50,000 is unreasonable and
excessive. It should be reduced to P10,000.
WHEREFORE, the instant petition is partly GRANTED. The challenged decision of the Court of Appeals in CA-G.R.
No. 40751 is hereby MODIFIED by a) deleting the awards of P200,000 for loss of profit, P200,000 as moral
damages and P100,000 as exemplary damages, and b) reducing the award of attorney's fees from P50,000 to
P10,000.
No pronouncement as to costs.
Melo, Kapunan, Pardo and Santiago, JJ., concur.

118

Republic of the Philippines


SUPREME COURT
Manila
FIRST DIVISION

G.R. No. 119655 May 24, 1996


SPS. ANTONIO A. TIBAY and VIOLETA R. TIBAY and OFELIA M. RORALDO, VICTORINA M. RORALDO,
VIRGILIO M. RORALDO, MYRNA M. RORALDO and ROSABELLA M. RORALDO, petitioners,
vs.
COURT OF APPEALS and FORTUNE LIFE AND GENERAL INSURANCE CO., INC., respondents.

119

BELLOSILLO, J.:p
May a fire insurance policy be valid, binding and enforceable upon mere partial payment of premium?
On 22 January 1987 private respondent Fortune Life and General Insurance Co., Inc. (FORTUNE) issued Fire
Insurance Policy No. 136171 in favor of Violeta R. Tibay and/or Nicolas Roraldo on their two-storey residential
building located at 5855 Zobel Street, Makati City, together with all their personal effects therein. The insurance was
for P600,000.00 covering the period from 23 January 1987 to 23 January 1988. On 23 January 1987, of the total
premium of P2,983.50, petitioner Violeta Tibay only paid P600.00 thus leaving a considerable balance unpaid.
On 8 March 1987 the insured building was completely destroyed by fire. Two days later or on 10 March 1987 Violeta
Tibay paid the balance of the premium. On the same day, she filed with FORTUNE a claim on the fire insurance
policy. Her claim was accordingly referred to its adjuster, Goodwill Adjustment Services, Inc. (GASI), which
immediately wrote Violeta requesting her to furnish it with the necessary documents for the investigation and
processing of her claim. Petitioner forthwith complied. On 28 March 1987 she signed a non-waiver agreement with
GASI to the effect that any action taken by the companies or their representatives in investigating the claim made by
the claimant for his loss which occurred at 5855 Zobel Roxas, Makati on March 8, 1987, or in the investigating or
ascertainment of the amount of actual cash value and loss, shall not waive or invalidate any condition of the policies
of such companies held by said claimant, nor the rights of either or any of the parties to this agreement, and such
action shall not be, or be claimed to be, an admission of liability on the part of said companies or any of them. 1
In a letter dated 11 June 1987 FORTUNE denied the claim of Violeta for violation of Policy Condition No. 2 and of
Sec. 77 of the Insurance Code. Efforts to settle the case before the Insurance Commission proved futile. On 3
March 1988 Violets and the other petitioners sued FORTUNE for damages in the amount of P600,000.00
representing the total coverage of the fire insurance policy plus 12% interest per annum, P100,000.00 moral
damages, and attorney's fees equivalent to 20% of the total claim.
On 19 July 1990 the trial court ruled for petitioners and adjudged FORTUNE liable for the total value of the insured
building and personal properties in the amount of P600,000.00 plus interest at the legal rate of 6% per annum from
the filing of the complaint until full payment, and attorney's fees equivalent to 20% of the total amount claimed plus
costs of suit. 2
On 24 March 1995 the Court of Appeals reversed the court a quo by declaring FORTUNE not to be liable to plaintiffappellees therein but ordering defendant-appellant to return to the former the premium of P2,983.50 plus 12%
interest from 10 March 1987 until full payment. 3
Hence this petition for review with petitioners contending mainly that contrary to the conclusion of the appellate
court, FORTUNE remains liable under the subject fire insurance policy in spite of the failure of petitioners to pay
their premium in full.
We find no merit in the petition; hence, we affirm the Court of Appeals.
Insurance is a contract whereby one undertakes for a consideration to indemnify another against loss, damage or
liability arising from an unknown or contingent event. 4 The consideration is the premium, which must be paid at the time
and in the way and manner specified in the policy, and if not so paid, the policy will lapse and be forfeited by its own
terms. 5
The pertinent provisions in the Policy on premium read

120
THIS POLICY OF INSURANCE WITNISSETH THAT only after payment to the Company in
accordance with Policy Condition No. 2 of the total premiums by the insured as stipulated above for
the period aforementioned for insuring against Loss or Damage by Fire or Lightning as herein
appears, the Property herein described . . .
2. This policy including any renewal thereof and/or any endorsement thereon is not in force until the
premium has been fully paid to and duly receipted by the Company in the manner provided herein.
Any supplementary agreement seeking to amend this condition prepared by agent, broker or
Company official, shall be deemed invalid and of no effect.
xxx xxx xxx
Except only in those specific cases where corresponding rules and regulations which are or may
hereafter be in force provide for the payment of the stipulated premiums in periodic installments at
fixed percentage, it is hereby declared, agreed and warranted that this policy shall be deemed
effective, valid and binding upon the Company only when the premiums therefor have actually been
paid in full and duly acknowledged in a receipt signed by any authorized official or
representative/agent of the Company in such manner as provided herein. (emphasis supplied). 6
Clearly the Policy provides for payment of premium in full. Accordingly, where the premium has only been partially
paid and the balance paid only after the peril insured against has occurred, the insurance contract did not take effect
and the insured cannot collect at all on the policy. This is fully supported by Sec. 77 of the Insurance Code which
provides
Sec. 77. An insurer is entitled to payment of the premium as soon as the thing insured is exposed to
the peril insured against. Notwithstanding any agreement to the contrary, no policy or contract of
insurance issued by an insurance company is valid and binding unless and until the premium thereof
has been paid, except in the case of a life or an industrial life policy whenever the grace period
provision applies (emphasis supplied).
Apparently the crux of the controversy lies in the phrase "unless and until the premium thereof has been paid." This
leads us to the manner of payment envisioned by the law to make the insurance policy operative and binding. For
whatever judicial construction may be accorded the disputed phrase must ultimately yield to the clear mandate of
the law. The principle that where the law does not distinguish the court should neither distinguish assumes that the
legislature made no qualification on the use of a general word or expression. In Escosura v. San Miguel Brewery,
Inc., 7 the Court through Mr. Justice Jesus G. Barrera, interpreting the phrase "with pay" used in connection with leaves of
absence with pay granted to employees, ruled
. . . the legislative practice seems to be that when the intention is to distinguish between full and
partial payment, the modifying term is used . . .
Citing C.A. No. 647 governing maternity leaves of married women in government, R. A. No. 679 regulating
employment of women and children, R.A. No. 843 granting vacation and sick leaves to judges of municipal
courts and justices of the peace, and finally, Art. 1695 of the New Civil Code providing that every househelp
shall be allowed four (4) days vacation each month, which laws simply stated "with pay," the Court
concluded that it was undisputed that in all these laws the phrase "with pay" used without any qualifying
adjective meant that the employee was entitled to full compensation during his leave of absence.

121
Petitioners maintain otherwise. Insisting that FORTUNE is liable on the policy despite partial payment of the
premium due and the express stipulation thereof to the contrary, petitioners rely heavily on the 1967 case
ofPhilippine Phoenix and Insurance Co., Inc. v. Woodworks, Inc. 8 where the Court through Mr. Justice Arsenio P. Dizon
sustained the ruling of the trial court that partial payment of the premium made the policy effective during the whole period
of the policy. In that case, the insurance company commenced action against the insured for the unpaid balance on a fire
insurance policy. In its defense the insured claimed that nonpayment of premium produced the cancellation of the
insurance contract. Ruling otherwise the Court held
It is clear . . . that on April 1, 1960, Fire Insurance Policy No. 9652 was issued by appellee and
delivered to appellant, and that on September 22 of the same year, the latter paid to the former the
sum of P3,000.00 on account of the total premium of P6,051.95 due thereon. There is, consequently,
no doubt at all that, as between the insurer and the insured, there was not only a perfected contract
of insurance but a partially performed one as far as the payment of the agreed premium was
concerned. Thereafter the obligation of the insurer to pay the insured the amount, for which the
policy was issued in case the conditions therefor had been complied with, arose and became binding
upon it, while the obligation of the insured to pay the remainder of the total amount of the premium
due became demandable.
The 1967 Phoenix case is not persuasive; neither is it decisive of the instant dispute. For one, the factual scenario is
different. In Phoenix it was the insurance company that sued for the balance of the premium, i.e., it recognized and
admitted the existence of an insurance contract with the insured. In the case before us, there is, quite unlike in
Phoenix, a specific stipulation that (t)his policy . . . is not in force until the premium has been fully paid and duly
receipted by the Company . . . Resultantly, it is correct to say that in Phoenix a contract was perfected upon partial
payment of the premium since the parties had not otherwise stipulated that prepayment of the premium in full was a
condition precedent to the existence of a contract.
In Phoenix, by accepting the initial payment of P3,000.00 and then later demanding the remainder of the premium
without any other precondition to its enforceability as in the instant case, the insurer in effect had shown its intention
to continue with the existing contract of insurance, as in fact it was enforcing its right to collect premium, or exact
specific performance from the insured. This is not so here. By express agreement of the parties, novinculum juris or
bond of law was to be established until full payment was effected prior to the occurrence of the risk insured against.
In Makati Tuscany Condominium Corp. v. Court of Appeals 9 the parties mutually agreed that the premiums could be
paid in installments, which in fact they did for three (3) years, hence, this Court refused to invalidate the insurance policy.
In giving effect to the policy, the Court quoted with approval the Court of Appeals
The obligation to pay premiums when due is ordinarily an indivisible obligation to pay the entire
premium. Here, the parties . . . agreed to make the premiums payable in installments, and there is
no pretense that the parties never envisioned to make the insurance contract binding between them.
It was renewed for two succeeding years, the second and third policies being a renewal/replacement
for the previous one. And the insured never informed the insurer that it was terminating the policy
because the terms were unacceptable.
While it may be true that under Section 77 of the Insurance Code, the parties may not agree to make
the insurance contract valid and binding without payment of premiums, there is nothing in said
section which suggests that the parties may not agree to allow payment of the premiums in
installment, or to consider the contract as valid and binding upon
payment of the first premium. Otherwise we would allow the insurer to renege on its liability under
the contract, had a loss incurred (sic) before completion of payment of the entire premium, despite

122
its voluntary acceptance of partial payments, a result eschewed by basic considerations of fairness
and equity . . .
These two (2) cases, Phoenix and Tuscany, adequately demonstrate the waiver, either express or implied, of
prepayment in full by the insurer: impliedly, by suing for the balance of the premium as in Phoenix, and expressly, by
agreeing to make premiums payable in installments as in Tuscany. But contrary to the stance taken by petitioners,
there is no waiver express or implied in the case at bench. Precisely, the insurer and the insured expressly
stipulated that (t)his policy including any renewal thereof and/or any indorsement thereon is not in force until the
premium has been fully paid to and duly receipted by the Company . . . and that this policy shall be deemed
effective, valid and binding upon the Company only when the premiums therefor have actually been paid in full and
duly acknowledged.
Conformably with the aforesaid stipulations explicitly worded and taken in conjunction with Sec. 77 of the Insurance
Code the payment of partial premium by the assured in this particular instance should not be considered the
payment required by the law and the stipulation of the parties. Rather, it must be taken in the concept of a deposit to
be held in trust by the insurer until such time that the full amount has been tendered and duly receipted for. In other
words, as expressly agreed upon in the contract, full payment must be made before the risk occurs for the policy to
be considered effective and in force.
Thus, no vinculum juris whereby the insurer bound itself to indemnify the assured according to law ever resulted
from the fractional payment of premium. The insurance contract itself expressly provided that the policy would be
effective only when the premium was paid in full. It would have been altogether different were it not so stipulated.
Ergo, petitioners had absolute freedom of choice whether or not to be insured by FORTUNE under the terms of its
policy and they freely opted to adhere thereto.
Indeed, and far more importantly, the cardinal polestar in the construction of an insurance contract is the intention of
the parties as expressed in the
policy. 10 Courts have no other function but to enforce the same. The rule that contracts of insurance will be construed in
favor of the insured and most strongly against the insurer should not be permitted to have the effect of making a plain
agreement ambiguous and then construe it in favor of the insured. 11 Verily, it is elemental law that the payment of
premium is requisite to keep the policy of insurance in force. If the premium is not paid in the manner prescribed in the
policy as intended by the parties the policy is ineffective. Partial payment even when accepted as a partial payment will
not keep the policy alive even for such fractional part of the year as the part payment bears to the whole
payment. 12
Applying further the rules of statutory construction, the position maintained by petitioners becomes even more
untenable. The case of South Sea Surety and Insurance Company, Inc. v. Court Of Appeals, 13 speaks only of two (2)
statutory exceptions to the requirement of payment of the entire premium as a prerequisite to the validity of the insurance
contract. These exceptions are: (a) in case the insurance coverage relates to life or industrial life (health) insurance when
a grace period applies, and (b) when the insurer makes a written acknowledgment of the receipt of premium, this
acknowledgment being declared by law to be then conclusive evidence of the premium payment. 14
A maxim of recognized practicality is the rule that the expressed exception or exemption excludes others.Exceptio
firmat regulim in casibus non exceptis. The express mention of exceptions operates to exclude other exceptions;
conversely, those which are not within the enumerated exceptions are deemed included in the general rule. Thus,
under Sec. 77, as well as Sec. 78, until the premium is paid, and the law has not expressly excepted partial
payments, there is no valid and binding contract. Hence, in the absence of clear waiver of prepayment in full by the
insurer, the insured cannot collect on the proceeds of the policy.

123
In the desire to safeguard the interest of the assured, it must not be ignored that the contract of insurance is
primarily a risk distributing device, a mechanism by which all members of a group exposed to a particular risk
contribute premiums to an insurer. From these contributory funds are paid whatever losses occur due to exposure to
the peril insured against. Each party therefore takes a risk: the insurer, that of being compelled upon the happening
of the contingency to pay the entire sum agreed upon, and the insured, that of parting with the amount required as
premium, without receiving anything therefor in case the contingency does not happen. To ensure payment for these
losses, the law mandates all insurance companies to maintain a legal reserve fund in favor of those claiming under
their policies. 15 It should be understood that the integrity of this fund cannot be secured and maintained if by judicial fiat
partial offerings of premiums were to be construed as a legal nexus between the applicant and the insurer despite an
express agreement to the contrary. For what could prevent the insurance applicant from deliberately or wilfully holding
back full premium payment and wait for the risk insured against to transpire and then conveniently pass on the balance of
the premium to be deducted from the proceeds of the insurance? Worse, what if the insured makes an initial payment of
only 10%, or even 1%, of the required premium, and when the risk occurs simply points to the proceeds from where to
source the balance? Can an insurance company then exist and survive upon the payment of 1%, or even 10%, of the
premium stipulated in the policy on the basis that, after all, the insurer can deduct from the proceeds of the insurance
should the risk insured against occur?
Interpreting the contract of insurance stringently against the insurer but liberally in favor of the insured despite
clearly defined obligations of the parties to the policy can be carried out to extremes that there is the danger that we
may, so to speak, "kill the goose that lays the golden egg." We are well aware of insurance companies falling into
the despicable habit of collecting premiums promptly yet resorting to all kinds of excuses to deny or delay payment
of just insurance claims. But, in this case, the law is manifestly on the side of the insurer. For as long as the current
Insurance Code remains unchanged and partial payment of premiums is not mentioned at all as among the
exceptions provided in Sees. 77 and 78, no policy of insurance can ever pretend to be efficacious or effective until
premium has been fully paid.
And so it must be. For it cannot be disputed that premium is the elixir vitae of the insurance business because by
law the insurer must maintain a legal reserve fund to meet its contingent obligations to the public, hence, the
imperative need for its prompt payment and full satisfaction. 16 It must be emphasized here that all actuarial
calculations and various tabulations of probabilities of losses under the risks insured against are based on the sound
hypothesis of prompt payment of premiums. Upon this bedrock insurance firms are enabled to offer the assurance of
security to the public at favorable rates. But once payment of premium is left to the whim and caprice of the insured, as
when the courts tolerate the payment of a mere P600.00 as partial undertaking out of the stipulated total premium of
P2,983.50 and the balance to be paid even after the risk insured against has occurred, as petitioners have done in this
case, on the principle that the strength of the vinculum juris is not measured by any specific amount of premium payment,
we will surely wreak havoc on the business and set to naught what has taken actuarians centuries to devise to arrive at a
fair and equitable distribution of risks and benefits between the insurer and the insured.
The terms of the insurance policy constitute the measure of the insurer's liability. In the absence of statutory
prohibition to the contrary, insurance companies have the same rights as individuals to limit their liability and to
impose whatever conditions they deem best upon their obligations not inconsistent with public policy. 17 The validity
of these limitations is by law passed upon by the Insurance Commissioner who is empowered to approve all forms of
policies, certificates or contracts of insurance which insurers intend to issue or deliver. That the policy contract in the case
at bench was approved and allowed issuance simply reaffirms the validity of such policy, particularly the provision in
question.
WHEREFORE, the petition is DENIED and the assailed Decision of the Court of Appeals dated 24 March 1995 is
AFFIRMED.
SO ORDERED.

124

Republic of the Philippines


SUPREME COURT
Manila

125
FIRST DIVISION
G.R. No. L-25317 August 6, 1979
PHILIPPINE PHOENIX SURETY & INSURANCE COMPANY, plaintiff-appellee,
vs.
WOODWORKS, INC., defendant-appellant.
Zosimo Rivas for appellant.
Manuel O. Chan for appellee.

MELENCIO-HERRERA, J.:
This case was certified to this Tribunal by the Court of Appeals in its Resolution of October 4, 1965 on a pure
question of law and "because the issues raised are practically the same as those in CA-G.R. No. 32017-R" between
the same parties, which case had been forwarded to us on April 1, 1964. The latter case, "Philippine Phoenix Surety
& Insurance Inc. vs. Woodworks, Inc.," docketed in this Court as L-22684, was decided on August 31, 1967 and has
been reported in 20 SCRA 1270.
Specifically, this action is for recovery of unpaid premium on a fire insurance policy issued by plaintiff, Philippine
Phoenix Surety & Insurance Company, in favor of defendant Woodworks, Inc.
The following are the established facts:
On July 21, 1960, upon defendant's application, plaintiff issued in its favor Fire Insurance Policy No. 9749 for
P500,000.00 whereby plaintiff insured defendant's building, machinery and equipment for a term of one year from
July 21, 1960 to July 21, 1961 against loss by fire. The premium and other charges including the margin fee
surcharge of P590.76 and the documentary stamps in the amount of P156.60 affixed on the Policy, amounted to
P10,593.36.
It is undisputed that defendant did not pay the premium stipulated in the Policy when it was issued nor at any time
thereafter.
On April 19, 1961, or before the expiration of the one-year term, plaintiff notified defendant, through its Indorsement
No. F-6963/61, of the cancellation of the Policy allegedly upon request of defendant. 1 The latter has denied having
made such a request. In said Indorsement, plaintiff credited defendant with the amount of P3,110.25 for the unexpired
period of 94 days, and claimed the balance of P7,483.11 representing ,learned premium from July 21, 1960 to 18th April
1961 or, say 271 days." On July 6, 1961, plaintiff demanded in writing for the payment of said amount. 2Defendant,
through counsel, disclaimed any liability in its reply- letter of August 15, 1961, contending, in essence, that it need not pay
premium "because the Insurer did not stand liable for any indemnity during the period the premiums were not paid." 3
On January 30, 1962, plaintiff commenced action in the Court of First Instance of Manila, Branch IV (Civil Case No.
49468), to recover the amount of P7,483.11 as "earned premium." Defendant controverted basically on the theory
that its failure "to pay the premium after the issuance of the policy put an end to the insurance contract and rendered
the policy unenforceable." 4

126
On September 13, 1962, judgment was rendered in plaintiff's favor "ordering defendant to pay plaintiff the sum of
P7,483.11, with interest thereon at the rate of 6%, per annum from January 30, 1962, until the principal shall have
been fully paid, plus the sum of P700.00 as attorney's fees of the plaintiff, and the costs of the suit." From this
adverse Decision, defendant appealed to the Court of Appeals which, as heretofore stated, certified the case to us
on a question of law.
The errors assigned read:
1. The lower court erred in sustaining that Fire Insurance Policy, Exhibit A, was a binding contract
even if the premium stated in the policy has not been paid.
2. That the lower court erred in sustaining that the premium in Insurance Policy, Exhibit B, became
an obligation which was demandable even after the period in the Policy has expired.
3. The lower court erred in not deciding that a premium not paid is not a debt enforceable by action
of the insurer.
We find the appeal meritorious.
Insurance is "a contract whereby one undertakes for a consideration to indemnify another against loss, damage or
liability arising from an unknown or contingent event." 5 The consideration is the "premium". "The premium must be paid
at the time and in the way and manner specified in the policy and, if not so paid, the policy will lapse and be forfeited by its
own terms." 6
The provisions on premium in the subject Policy read:
THIS POLICY OF INSURANCE WITNESSETH, THAT in consideration of MESSRS.
WOODWORKS, INC. hereinafter called the Insured, paying to the PHILIPPINE PHOENIX
SURETY AND INSURANCE, INC., hereinafter called the Company, the sum of PESOS NINE
THOUSAND EIGHT HUNDRED FORTY SIX ONLY the Premium for the first period hereinafter
mentioned. ...
xxx xxx xxx
THE COMPANY HEREBY AGREES with the Insured ... that if the Property above described, or any
part thereof, shall be destroyed or damaged by Fire or Lightning after payment of Premium, at any
time between 4:00 o'clock in the afternoon of the TWENTY FIRST day of JULY One Thousand Nine
Hundred and SIXTY and 4:00 o'clock in the afternoon of the TWENTY FIRST day of JULY One
Thousand Nine Hundred and SIXTY ONE. ... (Emphasis supplied)
Paragraph "2" of the Policy further contained the following condition:
2. No payment in respect of any premium shall be deemed to be payment to the Company unless a
printed form of receipt for the same signed by an Official or duly-appointed Agent of the Company
shall have been given to the Insured.
Paragraph "10" of the Policy also provided:
10. This insurance may be terminated at any time at the request of the Insured, in which case the
Company will retain the customary short period rate for the time the policy has been in force. This

127
insurance may also at any time be terminated at the option of the Company, on notice to that effect
being given to the Insured, in which case the Company shall be liable to repay on demand a ratable
proportion of the premium for the unexpired term from the date of the cancelment.
Clearly, the Policy provides for pre-payment of premium. Accordingly; "when the policy is tendered the insured must
pay the premium unless credit is given or there is a waiver, or some agreement obviating the necessity for
prepayment." 7 To constitute an extension of credit there must be a clear and express agreement therefor." 8
From the Policy provisions, we fail to find any clear agreement that a credit extension was accorded defendant. And
even if it were to be presumed that plaintiff had extended credit from the circumstances of the unconditional delivery
of the Policy without prepayment of the premium, yet it is obvious that defendant had not accepted the insurer's offer
to extend credit, which is essential for the validity of such agreement.
An acceptance of an offer to allow credit, if one was made, is as essential to make a valid agreement
for credit, to change a conditional delivery of an insurance policy to an unconditional delivery, as it is
to make any other contract. Such an acceptance could not be merely a mental act or state of mind,
but would require a promise to pay made known in some manner to defendant. 9
In this respect, the instant case differs from that involving the same parties entitled Philippine Phoenix Surety &
Insurance Inc. vs. Woodworks, Inc., 10 where recovery of the balance of the unpaid premium was allowed inasmuch as
in that case "there was not only a perfected contract of insurance but a partially performed one as far as the payment of
the agreed premium was concerned." This is not the situation obtaining here where no partial payment of premiums has
been made whatsoever.
Since the premium had not been paid, the policy must be deemed to have lapsed.
The non-payment of premiums does not merely suspend but put, an end to an insurance contract,
since the time of the payment is peculiarly of the essence of the contract. 11
... the rule is that under policy provisions that upon the failure to make a payment of a premium or
assessment at the time provided for, the policy shall become void or forfeited, or the obligation of the
insurer shall cease, or words to like effect, because the contract so prescribes and because such a
stipulation is a material and essential part of the contract. This is true, for instance, in the case of life,
health and accident, fire and hail insurance policies. 12
In fact, if the peril insured against had occurred, plaintiff, as insurer, would have had a valid defense against
recovery under the Policy it had issued. Explicit in the Policy itself is plaintiff's agreement to indemnify defendant for
loss by fire only "after payment of premium," supra. Compliance by the insured with the terms of the contract is a
condition precedent to the right of recovery.
The burden is on an insured to keep a policy in force by the payment of premiums, rather than on
the insurer to exert every effort to prevent the insured from allowing a policy to elapse through a
failure to make premium payments. The continuance of the insurer's obligation is conditional upon
the payment of premiums, so that no recovery can be had upon a lapsed policy, the contractual
relation between the parties having ceased. 13
Moreover, "an insurer cannot treat a contract as valid for the purpose of collecting premiums and invalid for the
purpose of indemnity." 14

128
The foregoing findings are buttressed by section 77 of the Insurance Code (Presidential Decree No. 612,
promulgated on December 18, 1974), which now provides that no contract of insurance issued by an insurance
company is valid and binding unless and until the premium thereof has been paid, notwithstanding any agreement
to the contrary.
WHEREFORE, the judgment appealed from is reversed, and plaintiff's complaint hereby dismissed.
Teehankee (Chairman), Fernandez, Guerrero and De Castro, JJ., concur.
Makasiar, J., is on leave.

CHAPTER V

129
Republic of the Philippines
SUPREME COURT
Manila
FIRST DIVISION
G.R. No. L-38613 February 25, 1982
PACIFIC TIMBER EXPORT CORPORATION, petitioner,
vs.
THE HONORABLE COURT OF APPEALS and WORKMEN'S INSURANCE COMPANY, INC., respondents.

DE CASTRO, ** J.:
This petition seeks the review of the decision of the Court of Appeals reversing the decision of the Court of First
Instance of Manila in favor of petitioner and against private respondent which ordered the latter to pay the sum of
Pll,042.04 with interest at the rate of 12% interest from receipt of notice of loss on April 15, 1963 up to the complete
payment, the sum of P3,000.00 as attorney's fees and the costs 1 thereby dismissing petitioner s complaint with
costs. 2
The findings of the of fact of the Court of Appeals, which are generally binding upon this Court, Except as shall be
indicated in the discussion of the opinion of this Court the substantial correctness of still particular finding having
been disputed, thereby raising a question of law reviewable by this Court 3 are as follows:
March 19, l963, the plaintiff secured temporary insurance from the defendant for its exportation of
1,250,000 board feet of Philippine Lauan and Apitong logs to be shipped from the Diapitan. Bay,
Quezon Province to Okinawa and Tokyo, Japan. The defendant issued on said date Cover Note No.
1010, insuring the said cargo of the plaintiff "Subject to the Terms and Conditions of the
WORKMEN'S INSURANCE COMPANY, INC. printed Marine Policy form as filed with and approved
by the Office of the Insurance Commissioner (Exhibit A).
The regular marine cargo policies were issued by the defendant in favor of the plaintiff on April 2,
1963. The two marine policies bore the numbers 53 HO 1032 and 53 HO 1033 (Exhibits B and C,
respectively). Policy No. 53 H0 1033 (Exhibit B) was for 542 pieces of logs equivalent to 499,950
board feet. Policy No. 53 H0 1033 was for 853 pieces of logs equivalent to 695,548 board feet
(Exhibit C). The total cargo insured under the two marine policies accordingly consisted of 1,395
logs, or the equivalent of 1,195.498 bd. ft.
After the issuance of Cover Note No. 1010 (Exhibit A), but before the issuance of the two marine
policies Nos. 53 HO 1032 and 53 HO 1033, some of the logs intended to be exported were lost
during loading operations in the Diapitan Bay. The logs were to be loaded on the 'SS Woodlock'
which docked about 500 meters from the shoreline of the Diapitan Bay. The logs were taken from the
log pond of the plaintiff and from which they were towed in rafts to the vessel. At about 10:00 o'clock
a. m. on March 29, 1963, while the logs were alongside the vessel, bad weather developed resulting
in 75 pieces of logs which were rafted together co break loose from each other. 45 pieces of logs
were salvaged, but 30 pieces were verified to have been lost or washed away as a result of the
accident.

130
In a letter dated April 4, 1963, the plaintiff informed the defendant about the loss of 'appropriately 32 pieces of log's
during loading of the 'SS Woodlock'. The said letter (Exhibit F) reads as follows:
April 4, 1963
Workmen's Insurance Company, Inc. Manila, Philippines
Gentlemen:
This has reference to Insurance Cover Note No. 1010 for shipment of 1,250,000 bd. ft. Philippine
Lauan and Apitong Logs. We would like to inform you that we have received advance preliminary
report from our Office in Diapitan, Quezon that we have lost approximately 32 pieces of logs during
loading of the SS Woodlock.
We will send you an accurate report all the details including values as soon as same will be reported
to us.
Thank you for your attention, we wish to remain.
Very respectfully yours,
PACIFIC TIMBER EXPORT CORPORATION
(Sgd.) EMMANUEL S. ATILANO Asst. General Manager.
Although dated April 4, 1963, the letter was received in the office of the defendant only on April 15,
1963, as shown by the stamp impression appearing on the left bottom corner of said letter. The
plaintiff subsequently submitted a 'Claim Statement demanding payment of the loss under Policies
Nos. 53 HO 1032 and 53 HO 1033, in the total amount of P19,286.79 (Exhibit G).
On July 17, 1963, the defendant requested the First Philippine Adjustment Corporation to inspect the
loss and assess the damage. The adjustment company submitted its 'Report on August 23, 1963
(Exhibit H). In said report, the adjuster found that 'the loss of 30 pieces of logs is not covered by
Policies Nos. 53 HO 1032 and 1033 inasmuch as said policies covered the actual number of logs
loaded on board the 'SS Woodlock' However, the loss of 30 pieces of logs is within the 1,250,000 bd.
ft. covered by Cover Note 1010 insured for $70,000.00.
On September 14, 1963, the adjustment company submitted a computation of the defendant's
probable liability on the loss sustained by the shipment, in the total amount of Pl1,042.04 (Exhibit 4).
On January 13, 1964, the defendant wrote the plaintiff denying the latter's claim, on the ground they
defendant's investigation revealed that the entire shipment of logs covered by the two marines
policies No. 53 110 1032 and 713 HO 1033 were received in good order at their point of destination.
It was further stated that the said loss may be considered as covered under Cover Note No. 1010
because the said Note had become 'null and void by virtue of the issuance of Marine Policy Nos. 53
HO 1032 and 1033'(Exhibit J-1). The denial of the claim by the defendant was brought by the plaintiff
to the attention of the Insurance Commissioner by means of a letter dated March 21, 1964 (Exhibit
K). In a reply letter dated March 30, 1964, Insurance Commissioner Francisco Y. Mandanas

131
observed that 'it is only fair and equitable to indemnify the insured under Cover Note No. 1010', and
advised early settlement of the said marine loss and salvage claim (Exhibit L).
On June 26, 1964, the defendant informed the Insurance Commissioner that, on advice of their
attorneys, the claim of the plaintiff is being denied on the ground that the cover note is null and void
for lack of valuable consideration (Exhibit M). 4
Petitioner assigned as errors of the Court of Appeals, the following:
I
THE COURT OF APPEALS ERRED IN HOLDING THAT THE COVER NOTE WAS NULL AND VOID
FOR LACK OF VALUABLE CONSIDERATION BECAUSE THE COURT DISREGARDED THE
PROVEN FACTS THAT PREMIUMS FOR THE COMPREHENSIVE INSURANCE COVERAGE
THAT INCLUDED THE COVER NOTE WAS PAID BY PETITIONER AND THAT INCLUDED THE
COVER NOTE WAS PAID BY PETITIONER AND THAT NO SEPARATE PREMIUMS ARE
COLLECTED BY PRIVATE RESPONDENT ON ALL ITS COVER NOTES.
II
THE COURT OF APPEALS ERRED IN HOLDING THAT PRIVATE RESPONDENT WAS
RELEASED FROM LIABILITY UNDER THE COVER NOTE DUE TO UNREASONABLE DELAY IN
GIVING NOTICE OF LOSS BECAUSE THE COURT DISREGARDED THE PROVEN FACT THAT
PRIVATE RESPONDENT DID NOT PROMPTLY AND SPECIFICALLY OBJECT TO THE CLAIM ON
THE GROUND OF DELAY IN GIVING NOTICE OF LOSS AND, CONSEQUENTLY, OBJECTIONS
ON THAT GROUND ARE WAIVED UNDER SECTION 84 OF THE INSURANCE ACT. 5
1. Petitioner contends that the Cover Note was issued with a consideration when, by express stipulation, the cover
note is made subject to the terms and conditions of the marine policies, and the payment of premiums is one of the
terms of the policies. From this undisputed fact, We uphold petitioner's submission that the Cover Note was not
without consideration for which the respondent court held the Cover Note as null and void, and denied recovery
therefrom. The fact that no separate premium was paid on the Cover Note before the loss insured against occurred,
does not militate against the validity of petitioner's contention, for no such premium could have been paid, since by
the nature of the Cover Note, it did not contain, as all Cover Notes do not contain particulars of the shipment that
would serve as basis for the computation of the premiums. As a logical consequence, no separate premiums are
intended or required to be paid on a Cover Note. This is a fact admitted by an official of respondent company, Juan
Jose Camacho, in charge of issuing cover notes of the respondent company (p. 33, tsn, September 24, 1965).
At any rate, it is not disputed that petitioner paid in full all the premiums as called for by the statement issued by
private respondent after the issuance of the two regular marine insurance policies, thereby leaving no account
unpaid by petitioner due on the insurance coverage, which must be deemed to include the Cover Note. If the Note is
to be treated as a separate policy instead of integrating it to the regular policies subsequently issued, the purpose
and function of the Cover Note would be set at naught or rendered meaningless, for it is in a real sense a contract,
not a mere application for insurance which is a mere offer. 6
It may be true that the marine insurance policies issued were for logs no longer including those which had been lost
during loading operations. This had to be so because the risk insured against is not for loss during operations
anymore, but for loss during transit, the logs having already been safely placed aboard. This would make no
difference, however, insofar as the liability on the cover note is concerned, for the number or volume of logs lost can

132
be determined independently as in fact it had been so ascertained at the instance of private respondent itself when
it sent its own adjuster to investigate and assess the loss, after the issuance of the marine insurance policies.
The adjuster went as far as submitting his report to respondent, as well as its computation of respondent's liability
on the insurance coverage. This coverage could not have been no other than what was stipulated in the Cover Note,
for no loss or damage had to be assessed on the coverage arising from the marine insurance policies. For obvious
reasons, it was not necessary to ask petitioner to pay premium on the Cover Note, for the loss insured against
having already occurred, the more practical procedure is simply to deduct the premium from the amount due the
petitioner on the Cover Note. The non-payment of premium on the Cover Note is, therefore, no cause for the
petitioner to lose what is due it as if there had been payment of premium, for non-payment by it was not chargeable
against its fault. Had all the logs been lost during the loading operations, but after the issuance of the Cover Note,
liability on the note would have already arisen even before payment of premium. This is how the cover note as a
"binder" should legally operate otherwise, it would serve no practical purpose in the realm of commerce, and is
supported by the doctrine that where a policy is delivered without requiring payment of the premium, the
presumption is that a credit was intended and policy is valid. 7
2. The defense of delay as raised by private respondent in resisting the claim cannot be sustained. The law requires
this ground of delay to be promptly and specifically asserted when a claim on the insurance agreement is made. The
undisputed facts show that instead of invoking the ground of delay in objecting to petitioner's claim of recovery on
the cover note, it took steps clearly indicative that this particular ground for objection to the claim was never in its
mind. The nature of this specific ground for resisting a claim places the insurer on duty to inquire when the loss took
place, so that it could determine whether delay would be a valid ground upon which to object to a claim against it.
As already stated earlier, private respondent's reaction upon receipt of the notice of loss, which was on April 15,
1963, was to set in motion from July 1963 what would be necessary to determine the cause and extent of the loss,
with a view to the payment thereof on the insurance agreement. Thus it sent its adjuster to investigate and assess
the loss in July, 1963. The adjuster submitted his report on August 23, 1963 and its computation of respondent's
liability on September 14, 1963. From April 1963 to July, 1963, enough time was available for private respondent to
determine if petitioner was guilty of delay in communicating the loss to respondent company. In the proceedings that
took place later in the Office of the Insurance Commissioner, private respondent should then have raised this ground
of delay to avoid liability. It did not do so. It must be because it did not find any delay, as this Court fails to find a real
and substantial sign thereof. But even on the assumption that there was delay, this Court is satisfied and convinced
that as expressly provided by law, waiver can successfully be raised against private respondent. Thus Section 84 of
the Insurance Act provides:
Section 84.Delay in the presentation to an insurer of notice or proof of loss is waived if caused by
any act of his or if he omits to take objection promptly and specifically upon that ground.
From what has been said, We find duly substantiated petitioner's assignments of error.
ACCORDINGLY, the appealed decision is set aside and the decision of the Court of First Instance is reinstated in
toto with the affirmance of this Court. No special pronouncement as to costs.
SO ORDERED.

133

134
Republic of the Philippines
SUPREME COURT
Manila
FIRST DIVISION
G.R. No. L-31845 April 30, 1979
GREAT PACIFIC LIFE ASSURANCE COMPANY, petitioner,
vs.
HONORABLE COURT OF APPEALS, respondents.
G.R. No. L-31878 April 30, 1979
LAPULAPU D. MONDRAGON, petitioner,
vs.
HON. COURT OF APPEALS and NGO HING, respondents.
Siguion Reyna, Montecillo & Ongsiako and Sycip, Salazar, Luna & Manalo for petitioner Company.
Voltaire Garcia for petitioner Mondragon.
Pelaez, Pelaez & Pelaez for respondent Ngo Hing.

DE CASTRO, J.:
The two above-entitled cases were ordered consolidated by the Resolution of this Court dated April 29, 1970, (Rollo,
No. L-31878, p. 58), because the petitioners in both cases seek similar relief, through these petitions for certiorari by
way of appeal, from the amended decision of respondent Court of Appeals which affirmed in toto the decision of the
Court of First Instance of Cebu, ordering "the defendants (herein petitioners Great Pacific Ligfe Assurance Company
and Mondragon) jointly and severally to pay plaintiff (herein private respondent Ngo Hing) the amount of P50,000.00
with interest at 6% from the date of the filing of the complaint, and the sum of P1,077.75, without interest.
It appears that on March 14, 1957, private respondent Ngo Hing filed an application with the Great Pacific Life
Assurance Company (hereinafter referred to as Pacific Life) for a twenty-year endownment policy in the amount of
P50,000.00 on the life of his one-year old daughter Helen Go. Said respondent supplied the essential data which
petitioner Lapulapu D. Mondragon, Branch Manager of the Pacific Life in Cebu City wrote on the corresponding form
in his own handwriting (Exhibit I-M). Mondragon finally type-wrote the data on the application form which was signed
by private respondent Ngo Hing. The latter paid the annual premuim the sum of P1,077.75 going over to the
Company, but he reatined the amount of P1,317.00 as his commission for being a duly authorized agebt of Pacific
Life. Upon the payment of the insurance premuim, the binding deposit receipt (Exhibit E) was issued to private
respondent Ngo Hing. Likewise, petitioner Mondragon handwrote at the bottom of the back page of the application
form his strong recommendation for the approval of the insurance application. Then on April 30, 1957, Mondragon
received a letter from Pacific Life disapproving the insurance application (Exhibit 3-M). The letter stated that the said
life insurance application for 20-year endowment plan is not available for minors below seven years old, but Pacific
Life can consider the same under the Juvenile Triple Action Plan, and advised that if the offer is acceptable, the
Juvenile Non-Medical Declaration be sent to the company.

135
The non-acceptance of the insurance plan by Pacific Life was allegedly not communicated by petitioner Mondragon
to private respondent Ngo Hing. Instead, on May 6, 1957, Mondragon wrote back Pacific Life again strongly
recommending the approval of the 20-year endowment insurance plan to children, pointing out that since 1954 the
customers, especially the Chinese, were asking for such coverage (Exhibit 4-M).
It was when things were in such state that on May 28, 1957 Helen Go died of influenza with complication of
bronchopneumonia. Thereupon, private respondent sought the payment of the proceeds of the insurance, but
having failed in his effort, he filed the action for the recovery of the same before the Court of First Instance of Cebu,
which rendered the adverse decision as earlier refered to against both petitioners.
The decisive issues in these cases are: (1) whether the binding deposit receipt (Exhibit E) constituted a temporary
contract of the life insurance in question; and (2) whether private respondent Ngo Hing concealed the state of health
and physical condition of Helen Go, which rendered void the aforesaid Exhibit E.
1. At the back of Exhibit E are condition precedents required before a deposit is considered a BINDING RECEIPT.
These conditions state that:
A. If the Company or its agent, shan have received the premium deposit ... and the insurance
application, ON or PRIOR to the date of medical examination ... said insurance shan be in force and
in effect from the date of such medical examination, for such period as is covered by the
deposit ...,PROVIDED the company shall be satisfied that on said date the applicant was insurable
on standard rates under its rule for the amount of insurance and the kind of policy requested in the
application.
D. If the Company does not accept the application on standard rate for the amount of insurance
and/or the kind of policy requested in the application but issue, or offers to issue a policy for a
different plan and/or amount ..., the insurance shall not be in force and in effect until the applicant
shall have accepted the policy as issued or offered by the Company and shall have paid the full
premium thereof. If the applicant does not accept the policy, the deposit shall be refunded.
E. If the applicant shall not have been insurable under Condition A above, and the Company
declines to approve the application the insurance applied for shall not have been in force at any time
and the sum paid be returned to the applicant upon the surrender of this receipt. (Emphasis Ours).
The aforequoted provisions printed on Exhibit E show that the binding deposit receipt is intended to be merely a
provisional or temporary insurance contract and only upon compliance of the following conditions: (1) that the
company shall be satisfied that the applicant was insurable on standard rates; (2) that if the company does not
accept the application and offers to issue a policy for a different plan, the insurance contract shall not be binding
until the applicant accepts the policy offered; otherwise, the deposit shall be reftmded; and (3) that if the applicant is
not ble according to the standard rates, and the company disapproves the application, the insurance applied for
shall not be in force at any time, and the premium paid shall be returned to the applicant.
Clearly implied from the aforesaid conditions is that the binding deposit receipt in question is merely an
acknowledgment, on behalf of the company, that the latter's branch office had received from the applicant the
insurance premium and had accepted the application subject for processing by the insurance company; and that the
latter will either approve or reject the same on the basis of whether or not the applicant is "insurable on standard
rates." Since petitioner Pacific Life disapproved the insurance application of respondent Ngo Hing, the binding
deposit receipt in question had never become in force at any time.

136
Upon this premise, the binding deposit receipt (Exhibit E) is, manifestly, merely conditional and does not insure
outright. As held by this Court, where an agreement is made between the applicant and the agent, no liability shall
attach until the principal approves the risk and a receipt is given by the agent. The acceptance is merely conditional
and is subordinated to the act of the company in approving or rejecting the application. Thus, in life insurance, a
"binding slip" or "binding receipt" does not insure by itself (De Lim vs. Sun Life Assurance Company of Canada, 41
Phil. 264).
It bears repeating that through the intra-company communication of April 30, 1957 (Exhibit 3-M), Pacific Life
disapproved the insurance application in question on the ground that it is not offering the twenty-year endowment
insurance policy to children less than seven years of age. What it offered instead is another plan known as the
Juvenile Triple Action, which private respondent failed to accept. In the absence of a meeting of the minds between
petitioner Pacific Life and private respondent Ngo Hing over the 20-year endowment life insurance in the amount of
P50,000.00 in favor of the latter's one-year old daughter, and with the non-compliance of the abovequoted
conditions stated in the disputed binding deposit receipt, there could have been no insurance contract duly
perfected between thenl Accordingly, the deposit paid by private respondent shall have to be refunded by Pacific
Life.
As held in De Lim vs. Sun Life Assurance Company of Canada, supra, "a contract of insurance, like other contracts,
must be assented to by both parties either in person or by their agents ... The contract, to be binding from the date
of the application, must have been a completed contract, one that leaves nothing to be dione, nothing to be
completed, nothing to be passed upon, or determined, before it shall take effect. There can be no contract of
insurance unless the minds of the parties have met in agreement."
We are not impressed with private respondent's contention that failure of petitioner Mondragon to communicate to
him the rejection of the insurance application would not have any adverse effect on the allegedly perfected
temporary contract (Respondent's Brief, pp. 13-14). In this first place, there was no contract perfected between the
parties who had no meeting of their minds. Private respondet, being an authorized insurance agent of Pacific Life at
Cebu branch office, is indubitably aware that said company does not offer the life insurance applied for. When he
filed the insurance application in dispute, private respondent was, therefore, only taking the chance that Pacific Life
will approve the recommendation of Mondragon for the acceptance and approval of the application in question along
with his proposal that the insurance company starts to offer the 20-year endowment insurance plan for children less
than seven years. Nonetheless, the record discloses that Pacific Life had rejected the proposal and
recommendation. Secondly, having an insurable interest on the life of his one-year old daughter, aside from being
an insurance agent and an offense associate of petitioner Mondragon, private respondent Ngo Hing must have
known and followed the progress on the processing of such application and could not pretend ignorance of the
Company's rejection of the 20-year endowment life insurance application.
At this juncture, We find it fit to quote with approval, the very apt observation of then Appellate Associate Justice
Ruperto G. Martin who later came up to this Court, from his dissenting opinion to the amended decision of the
respondent court which completely reversed the original decision, the following:
Of course, there is the insinuation that neither the memorandum of rejection (Exhibit 3-M) nor the
reply thereto of appellant Mondragon reiterating the desire for applicant's father to have the
application considered as one for a 20-year endowment plan was ever duly communicated to Ngo;
Hing, father of the minor applicant. I am not quite conninced that this was so. Ngo Hing, as father of
the applicant herself, was precisely the "underwriter who wrote this case" (Exhibit H-1). The
unchallenged statement of appellant Mondragon in his letter of May 6, 1957) (Exhibit 4-M),
specifically admits that said Ngo Hing was "our associate" and that it was the latter who "insisted that
the plan be placed on the 20-year endowment plan." Under these circumstances, it is inconceivable
that the progress in the processing of the application was not brought home to his knowledge. He

137
must have been duly apprised of the rejection of the application for a 20-year endowment plan
otherwise Mondragon would not have asserted that it was Ngo Hing himself who insisted on the
application as originally filed, thereby implictly declining the offer to consider the application under
the Juvenile Triple Action Plan. Besides, the associate of Mondragon that he was, Ngo Hing should
only be presumed to know what kind of policies are available in the company for minors below 7
years old. What he and Mondragon were apparently trying to do in the premises was merely to prod
the company into going into the business of issuing endowment policies for minors just as other
insurance companies allegedly do. Until such a definite policy is however, adopted by the company,
it can hardly be said that it could have been bound at all under the binding slip for a plan of
insurance that it could not have, by then issued at all. (Amended Decision, Rollo, pp- 52-53).
2. Relative to the second issue of alleged concealment. this Court is of the firm belief that private respondent had
deliberately concealed the state of health and piysical condition of his daughter Helen Go. Wher private regpondeit
supplied the required essential data for the insurance application form, he was fully aware that his one-year old
daughter is typically a mongoloid child. Such a congenital physical defect could never be ensconced nor disguished.
Nonetheless, private respondent, in apparent bad faith, withheld the fact materal to the risk to be assumed by the
insurance compary. As an insurance agent of Pacific Life, he ought to know, as he surely must have known. his duty
and responsibility to such a material fact. Had he diamond said significant fact in the insurance application fom
Pacific Life would have verified the same and would have had no choice but to disapprove the application outright.
The contract of insurance is one of perfect good faith uberrima fides meaning good faith, absolute and perfect
candor or openness and honesty; the absence of any concealment or demotion, however slight [Black's Law
Dictionary, 2nd Edition], not for the alone but equally so for the insurer (Field man's Insurance Co., Inc. vs. Vda de
Songco, 25 SCRA 70). Concealment is a neglect to communicate that which a partY knows aDd Ought to
communicate (Section 25, Act No. 2427). Whether intentional or unintentional the concealment entitles the insurer to
rescind the contract of insurance (Section 26, Id.: Yu Pang Cheng vs. Court of Appeals, et al, 105 Phil 930;
Satumino vs. Philippine American Life Insurance Company, 7 SCRA 316). Private respondent appears guilty thereof.
We are thus constrained to hold that no insurance contract was perfected between the parties with the
noncompliance of the conditions provided in the binding receipt, and concealment, as legally defined, having been
comraitted by herein private respondent.
WHEREFORE, the decision appealed from is hereby set aside, and in lieu thereof, one is hereby entered absolving
petitioners Lapulapu D. Mondragon and Great Pacific Life Assurance Company from their civil liabilities as found by
respondent Court and ordering the aforesaid insurance company to reimburse the amount of P1,077.75, without
interest, to private respondent, Ngo Hing. Costs against private respondent.
SO ORDERED.

138

139

Republic of the Philippines


SUPREME COURT
Manila
FIRST DIVISION
G.R. No. 139776

August 1, 2002

PHILIPPINE AMERICAN LIFE AND GENERAL INSURANCE COMPANY, petitioner,


vs.
JUDGE LORE R. VALENCIA-BAGALACSA, Regional Trial Court of Libmanan, Camarines Sur, Branch 56, and
EDUARDO Z. LUMANIOG, CELSO Z. LUMANIOG and RUBEN Z. LUMANIOG, respondents.
DECISION
AUSTRIA-MARTINEZ, J.:
Before us is a petition for review on certiorari under Rule 45 of the Rules of Court. Petitioner Philippine American
Life and General Insurance Company prays that the decision of the Court of Appeals promulgated on April 30, 1999
be reversed and set aside and that the Complaint filed against it by private respondents Eduardo Z. Lumaniog,
Celso Z. Lumaniog and Ruben Z. Lumaniog before the Regional Trial Court of Libmanan, Camarines Sur, docketed
as Civil Case No. L-787 be ordered dismissed on ground of prescription of action.
The facts of the case:
On June 20, 1995, private respondents, as legitimate children and forced heirs of their late father, Faustino
Lumaniog, filed with the aforesaid RTC, a complaint for recovery of sum of money against petitioner alleging that:
their father was insured by petitioner under Life Insurance Policy No. 1305486 with a face value of P50,000.00; their
father died of "coronary thrombosis" on November 25, 1980; on June 22, 1981, they claimed and continuously
claimed for all the proceeds and interests under the life insurance policy in the amount of P641,000.00, despite
repeated demands for payment and/or settlement of the claim due from petitioner, the last of which is on December
1, 1994, petitioner finally refused or disallowed said claim on February 14, 1995; 1 and so, they filed their complaint
on June 20, 1995.
Petitioner filed an Answer with Counterclaim and Motion to Dismiss, contending that: the cause of action of private
respondents had prescribed and they are guilty of laches; it had denied private respondents claim in a letter dated
March 12, 1982, signed by its then Assistant Vice President, Amado Dimalanta, on ground of concealment on the
part of the deceased insured Faustino when he asserted in his application for insurance coverage that he had not
been treated for indication of "chest pain, palpitation, high blood pressure, rheumatic fever, heart murmur, heart
attack or other disorder of the heart or blood vessel" when in fact he was a known hypertensive since 1974; private
respondents sent a letter dated May 25, 19832 requesting for reconsideration of the denial; in a letter dated July 11,
1983, it reiterated its decision to deny the claim for payment of the proceeds; 3 more than ten (10) years later, or on
December 1, 1994, it received a letter from Jose C. Claro, a provincial board member of the province of Camarines
Sur, reiterating the early request for reconsideration which it denied in a letter dated February 14, 1995. 4
Private respondents opposed the motion to dismiss. 5

140
On June 7, 1996, the RTC issued an Order which reads:
"After a perusal of the motion to dismiss filed by defendants counsel and the objection submitted by plaintiffs
counsel, the Court finds that the matters treated in their respective pleadings are evidentiary in nature, hence, the
necessity of a trial on the merits.
"Set therefore the hearing in this case on August 1, 1996 at 8:30 a.m., considering that the calendar of the Court is
already filled up until the end of July. Notify parties and counsels.
"SO ORDERED."6
Petitioners motion for reconsideration was denied by the RTC in its Order dated December 12, 1997 upholding
however in the same Order the claim of private respondents counsel that the running of the 10-year period was
"stopped" on May 25, 1983 when private respondents requested for a reconsideration of the denial and it was only
on February 14, 1995 when petitioner finally decided to deny their claim that the 10-year period began to run. 7
Petitioner filed a petition for certiorari (docketed as CA-G.R. SP No. 47885) under Rule 65 of the Rules of Court in
the Court of Appeals and after the comment of the private respondents and reply of petitioner, the appellate court
rendered its Decision, dated April 30, 1999, portions of which read as follows:
"Thus, this Court of the opinion and so holds that the prescriptive period to bring the present action commences to
run only on February 14, 1995 (Rollo, pp. 25-26), the date when the petitioner finally rejected the claim of private
respondents and not in 1983. The ten year period should instead be counted from the date of rejection by the
insurer in this case February 14, 1995 since this is the time when the cause of action accrues.
"This fact was supported further by the letter of the petitioner to Atty. Claro dated December 20, 1994, stating that
they were reviewing the claim and shall advise Atty. Claro of their action regarding his request for reconsideration
(Id., p. 53).
"In the case of Summit Guaranty and Insurance Co., Inc. Vs. De Guzman (151 SCRA 389, 397-398), citing the case
of Eagle Star Insurance Co., Ltd., et al. vs. Chia Yu, the Supreme Court held that:
The plaintiffs cause of action did not accrue until his claim was finally rejected by the insurance company. This is
because, before such final rejection, there was no real necessity for bringing suit.
"In the same case, the case of ACCFA vs. Alpha Insurance and Surety Co., was likewise cited where the Supreme
Court ruled in this wise:
Since a cause of action requires, as essential elements, not only a legal right of the plaintiff and a correlative of the
defendant but also an act or omission of the defendant in violation of said legal right, the cause of action does not
accrue until the party obligated refuses, expressly or impliedly, to comply with its duty.
"Hence, We find no grave abuse of discretion committed by the court a quo when it issued the Orders dated June 7,
1996 and dated December 12, 1997.
"WHEREFORE, the instant petition for certiorari with prayer for issuance of temporary restraining order and/or
preliminary injunction is DENIED DUE COURSE and is accordingly DISMISSED by this Court for lack of merit.
"Costs against the petitioner.

141
"SO ORDERED."8
Hence, the present petition for review. Petitioner posits the following issues:
"A. Whether or not the complaint filed by private respondents for payment of life insurance proceeds is already
barred by prescription of action.
"B. Whether or not an extrajudicial demand made after an action has prescribed shall cause the revival of the
action."9
Private respondents filed their Comment and petitioners, their Reply.
Before we determine whether the Court of Appeals had committed any reversible error, we must necessarily first
ascertain whether or not the RTC committed grave abuse of discretion in issuing the Orders dated June 7, 1996 and
December 12, 1997.
Notably, the RTC was initially correct in issuing the Order dated June 7, 1996 when it set the case below for hearing
as there are matters in the respective pleadings of the parties "that are evidentiary in nature, hence the necessity of
a trial on the merits"10, in effect, denying the motion to dismiss, pursuant to the then prevailing Section 3, Rule 16, of
the Rules of Court, to wit:
"Sec. 3. Hearing and order. - After hearing the court may deny or grant the motion or allow amendment of pleading,
or may defer the hearing and determination of the motion until the trial if the ground alleged therein does not appear
to be indubitable."
before it was amended by the 1997 Rules of Civil Procedure, effective July 1, 1997. 11
It must be emphasized that petitioner had specifically alleged in the Answer that it had denied private respondents
claim per its letter dated July 11, 1983.12 Hence, due process demands that it be given the opportunity to prove that
private respondents had received said letter, dated July 11, 1983. Said letter is crucial to petitioners defense that
the filing of the complaint for recovery of sum of money in June, 1995 is beyond the 10-year prescriptive period 13.
It is for the above reason that the RTC committed a grave abuse of discretion when, in resolving the motion for
reconsideration of petitioner, it arbitrarily ruled in its Order dated December 12, 1997, that the period of ten (10)
years had not yet lapsed. It based its finding on a mere explanation of the private respondents counsel and not on
evidence presented by the parties as to the date when to reckon the prescriptive period. Portions of the Order dated
December 12, 1997 read:
"A perusal of the record will likewise reveal that plaintiffs counsel explained that the running of the ten (10) year
period was stopped on May 25, 1983, upon demand of Celso Lomaniog for the compliance of the contract and
reconsideration of the decision. Counsel also wrote the President of the Company on December 1, 1994, asking for
reconsideration. The letter was answered by the Assistant Vice President of the Claims Department of Philamlife,
with the advise that the company is reviewing the claim. On February 14, 1995, Atty. Abis sent a letter to counsel,
finally deciding the plaintiffs claim. Thus, the period of prescription should commence to run only from February 14,
1995, when Atty. Abis finally decided plaintiffs claim.
1wphi1

"It is evident from the foregoing that the ten (10) year period for plaintiffs to claim the insurance proceeds has not yet
prescribed. The final determination denying the claim was made only on February 14, 1995. Hence, when the
instant case was filed on June 20, 1995, the ten year period has not yet lapsed. Moreover, defendants counsel
failed to comply with the requirements of the Rules in filing his motion for reconsideration." 14 (emphasis supplied)

142
The ruling of the RTC that the cause of action of private respondents had not prescribed, is arbitrary and patently
erroneous for not being founded on evidence on record, and therefore, the same is void. 15
Consequently, while the Court of Appeals did not err in upholding the June 7, 1986 Order of the RTC, it committed a
reversible error when it declared that the RTC did not commit any grave abuse of discretion in issuing the Order
dated December 12, 1997.
The appellate court should have granted the petition for certiorari assailing said Order of December 12, 1997.
Certiorari is an appropriate remedy to assail an interlocutory order (1) when the tribunal issued such order without or
in excess of jurisdiction or with grave abuse of discretion and (2) when the assailed interlocutory order is patently
erroneous and the remedy of appeal would not afford adequate and expeditious relief. 16 Said Order was issued with
grave abuse of discretion for being patently erroneous and arbitrary, thus, depriving petitioner of due process, as
discussed earlier.
WHEREFORE, the petition is partly GRANTED. The assailed decision of the Court of Appeals dated April 30, 1999
insofar only as it upheld the Order dated December 12, 1997 is REVERSED and SET ASIDE. A new judgment is
entered reversing and setting aside the Order dated December 12, 1997 of the Regional Trial Court of Libmanan,
Camarines Sur (Branch 56) and affirming its Order dated June 20, 1995. Said RTC is directed to proceed with
dispatch with Civil Case No. L-787.
No costs.
SO ORDERED.

CHPATER VI

143
Republic of the Philippines
SUPREME COURT
Manila
SECOND DIVISION

G.R. No. 113899 October 13, 1999


GREAT PACIFIC LIFE ASSURANCE CORP., petitioner,
vs.
COURT OF APPEALS AND MEDARDA V. LEUTERIO, respondents.
QUISUMBING, J.:
This petition for review, under Rule 45 of the Rules of Court, assails the Decision 1 dated May 17, 1993, of the Court
of Appeals and its Resolution 2 dated January 4, 1994 in CA-G.R. CV No. 18341. The appellate court affirmed in toto the
judgment of the Misamis Oriental Regional Trial Court, Branch 18, in an insurance claim filed by private respondent
against Great Pacific Life Assurance Co. The dispositive portion of the trial court's decision reads:
WHEREFORE, judgment is rendered adjudging the defendant GREAT PACIFIC LIFE ASSURANCE
CORPORATION as insurer under its Group policy No. G-1907, in relation to Certification B-18558
liable and ordered to pay to the DEVELOPMENT BANK OF THE PHILIPPINES as creditor of the
insured Dr. Wilfredo Leuterio, the amount of EIGHTY SIX THOUSAND TWO HUNDRED PESOS
(P86,200.00); dismissing the claims for damages, attorney's fees and litigation expenses in the
complaint and counterclaim, with costs against the defendant and dismissing the complaint in
respect to the plaintiffs, other than the widow-beneficiary, for lack of cause of action. 3
The facts, as found by the Court of Appeals, are as follows:
A contract of group life insurance was executed between petitioner Great Pacific Life Assurance Corporation
(hereinafter Grepalife) and Development Bank of the Philippines (hereinafter DBP). Grepalife agreed to insure the
lives of eligible housing loan mortgagors of DBP.
On November 11, 1983, Dr. Wilfredo Leuterio, a physician and a housing debtor of DBP applied for membership in
the group life insurance plan. In an application form, Dr. Leuterio answered questions concerning his health
condition as follows:
7. Have you ever had, or consulted, a physician for a heart condition, high blood
pressure, cancer, diabetes, lung; kidney or stomach disorder or any other physical
impairment?
Answer: No. If so give details _____________.
8. Are you now, to the best of your knowledge, in good health?
Answer: [x] Yes [ ] NO. 4

144
On November 15, 1983, Grepalife issued Certificate No. B-18558, as insurance coverage of Dr. Leuterio, to the
extent of his DBP mortgage indebtedness amounting to eighty-six thousand, two hundred (P86,200.00) pesos.

1wphi1.nt

On August 6, 1984, Dr. Leuterio died due to "massive cerebral hemorrhage." Consequently, DBP submitted a death
claim to Grepalife. Grepalife denied the claim alleging that Dr. Leuterio was not physically healthy when he applied
for an insurance coverage on November 15, 1983. Grepalife insisted that Dr. Leuterio did not disclose he had been
suffering from hypertension, which caused his death. Allegedly, such non-disclosure constituted concealment that
justified the denial of the claim.
On October 20, 1986, the widow of the late Dr. Leuterio, respondent Medarda V. Leuterio, filed a complaint with the
Regional Trial Court of Misamis Oriental, Branch 18, against Grepalife for "Specific Performance with
Damages." 5 During the trial, Dr. Hernando Mejia, who issued the death certificate, was called to testify. Dr. Mejia's
findings, based partly from the information given by the respondent widow, stated that Dr. Leuterio complained of
headaches presumably due to high blood pressure. The inference was not conclusive because Dr. Leuterio was not
autopsied, hence, other causes were not ruled out.
On February 22, 1988, the trial court rendered a decision in favor of respondent widow and against Grepalife. On
May 17, 1993, the Court of Appeals sustained the trial court's decision. Hence, the present petition. Petitioners
interposed the following assigned errors:
1. THE LOWER COURT ERRED IN HOLDING DEFENDANT-APPELLANT LIABLE
TO THE DEVELOPMENT BANK OF THE PHILIPPINES (DBP) WHICH IS NOT A
PARTY TO THE CASE FOR PAYMENT OF THE PROCEEDS OF A MORTGAGE
REDEMPTION INSURANCE ON THE LIFE OF PLAINTIFF'S HUSBAND
WILFREDO LEUTERIO ONE OF ITS LOAN BORROWERS, INSTEAD OF
DISMISSING THE CASE AGAINST DEFENDANT-APPELLANT [Petitioner Grepalife]
FOR LACK OF CAUSE OF ACTION.
2. THE LOWER COURT ERRED IN NOT DISMISSING THE CASE FOR WANT OF
JURISDICTION OVER THE SUBJECT OR NATURE OF THE ACTION AND OVER
THE PERSON OF THE DEFENDANT.
3. THE LOWER COURT ERRED IN ORDERING DEFENDANT-APPELLANT TO
PAY TO DBP THE AMOUNT OF P86,200.00 IN THE ABSENCE OF ANY EVIDENCE
TO SHOW HOW MUCH WAS THE ACTUAL AMOUNT PAYABLE TO DBP IN
ACCORDANCE WITH ITS GROUP INSURANCE CONTRACT WITH DEFENDANTAPPELLANT.
4. THE LOWER COURT ERRED IN HOLDING THAT THERE WAS NO
CONCEALMENT OF MATERIAL INFORMATION ON THE PART OF WILFREDO
LEUTERIO IN HIS APPLICATION FOR MEMBERSHIP IN THE GROUP LIFE
INSURANCE PLAN BETWEEN DEFENDANT-APPELLANT OF THE INSURANCE
CLAIM ARISING FROM THE DEATH OF WILFREDO LEUTERIO. 6
Synthesized below are the assigned errors for our resolution:
1. Whether the Court of Appeals erred in holding petitioner liable to DBP as
beneficiary in a group life insurance contract from a complaint filed by the widow of
the decedent/mortgagor?

145
2. Whether the Court of Appeals erred in not finding that Dr. Leuterio concealed that
he had hypertension, which would vitiate the insurance contract?
3. Whether the Court of Appeals erred in holding Grepalife liable in the amount of
eighty six thousand, two hundred (P86,200.00) pesos without proof of the actual
outstanding mortgage payable by the mortgagor to DBP.
Petitioner alleges that the complaint was instituted by the widow of Dr. Leuterio, not the real party in interest, hence
the trial court acquired no jurisdiction over the case. It argues that when the Court of Appeals affirmed the trial
court's judgment, Grepalife was held liable to pay the proceeds of insurance contract in favor of DBP, the
indispensable party who was not joined in the suit.
To resolve the issue, we must consider the insurable interest in mortgaged properties and the parties to this type of
contract. The rationale of a group insurance policy of mortgagors, otherwise known as the "mortgage redemption
insurance," is a device for the protection of both the mortgagee and the mortgagor. On the part of the mortgagee, it
has to enter into such form of contract so that in the event of the unexpected demise of the mortgagor during the
subsistence of the mortgage contract, the proceeds from such insurance will be applied to the payment of the
mortgage debt, thereby relieving the heirs of the mortgagor from paying the obligation. 7 In a similar vein, ample
protection is given to the mortgagor under such a concept so that in the event of death; the mortgage obligation will be
extinguished by the application of the insurance proceeds to the mortgage indebtedness. 8 Consequently, where the
mortgagor pays the insurance premium under the group insurance policy, making the loss payable to the mortgagee, the
insurance is on the mortgagor's interest, and the mortgagor continues to be a party to the contract. In this type of policy
insurance, the mortgagee is simply an appointee of the insurance fund, such loss-payable clause does not make the
mortgagee a party to the contract. 9
Sec. 8 of the Insurance Code provides:
Unless the policy provides, where a mortgagor of property effects insurance in his own name
providing that the loss shall be payable to the mortgagee, or assigns a policy of insurance to a
mortgagee, the insurance is deemed to be upon the interest of the mortgagor, who does not cease
to be a party to the original contract, and any act of his, prior to the loss, which would otherwise
avoid the insurance, will have the same effect, although the property is in the hands of the
mortgagee, but any act which, under the contract of insurance, is to be performed by the mortgagor,
may be performed by the mortgagee therein named, with the same effect as if it had been performed
by the mortgagor.
The insured private respondent did not cede to the mortgagee all his rights or interests in the insurance, the policy
stating that: "In the event of the debtor's death before his indebtedness with the Creditor [DBP] shall have been fully
paid, an amount to pay the outstanding indebtedness shall first be paid to the creditor and the balance of sum
assured, if there is any, shall then be paid to the beneficiary/ies designated by the debtor." 10 When DBP submitted
the insurance claim against petitioner, the latter denied payment thereof, interposing the defense of concealment
committed by the insured. Thereafter, DBP collected the debt from the mortgagor and took the necessary action of
foreclosure on the residential lot of private respondent. 11 In Gonzales La O vs. Yek Tong Lin Fire & Marine Ins. Co. 12 we
held:
Insured, being the person with whom the contract was made, is primarily the proper person to bring
suit thereon. * * * Subject to some exceptions, insured may thus sue, although the policy is taken
wholly or in part for the benefit of another person named or unnamed, and although it is expressly
made payable to another as his interest may appear or otherwise. * * * Although a policy issued to a
mortgagor is taken out for the benefit of the mortgagee and is made payable to him, yet the

146
mortgagor may sue thereon in his own name, especially where the mortgagee's interest is less than
the full amount recoverable under the policy, * * *.
And in volume 33, page 82, of the same work, we read the following:
Insured may be regarded as the real party in interest, although he has assigned the policy for the
purpose of collection, or has assigned as collateral security any judgment he may obtain. 13
And since a policy of insurance upon life or health may pass by transfer, will or succession to any person, whether
he has an insurable interest or not, and such person may recover it whatever the insured might have
recovered,14 the widow of the decedent Dr. Leuterio may file the suit against the insurer, Grepalife.
The second assigned error refers to an alleged concealment that the petitioner interposed as its defense to annul
the insurance contract. Petitioner contends that Dr. Leuterio failed to disclose that he had hypertension, which might
have caused his death. Concealment exists where the assured had knowledge of a fact material to the risk, and
honesty, good faith, and fair dealing requires that he should communicate it to the assured, but he designedly and
intentionally withholds the same. 15
Petitioner merely relied on the testimony of the attending physician, Dr. Hernando Mejia, as supported by the
information given by the widow of the decedent. Grepalife asserts that Dr. Mejia's technical diagnosis of the cause of
death of Dr. Leuterio was a duly documented hospital record, and that the widow's declaration that her husband had
"possible hypertension several years ago" should not be considered as hearsay, but as part of res gestae.
On the contrary the medical findings were not conclusive because Dr. Mejia did not conduct an autopsy on the body
of the decedent. As the attending physician, Dr. Mejia stated that he had no knowledge of Dr. Leuterio's any
previous hospital confinement. 16 Dr. Leuterio's death certificate stated that hypertension was only "the possible cause of
death." The private respondent's statement, as to the medical history of her husband, was due to her unreliable
recollection of events. Hence, the statement of the physician was properly considered by the trial court as hearsay.
The question of whether there was concealment was aptly answered by the appellate court, thus:
The insured, Dr. Leuterio, had answered in his insurance application that he was in good health and
that he had not consulted a doctor or any of the enumerated ailments, including hypertension; when
he died the attending physician had certified in the death certificate that the former died of cerebral
hemorrhage, probably secondary to hypertension. From this report, the appellant insurance
company refused to pay the insurance claim. Appellant alleged that the insured had concealed the
fact that he had hypertension.
Contrary to appellant's allegations, there was no sufficient proof that the insured had suffered from
hypertension. Aside from the statement of the insured's widow who was not even sure if the
medicines taken by Dr. Leuterio were for hypertension, the appellant had not proven nor produced
any witness who could attest to Dr. Leuterio's medical history . . .
xxx xxx xxx
Appellant insurance company had failed to establish that there was concealment made by the
insured, hence, it cannot refuse payment of the claim. 17
The fraudulent intent on the part of the insured must be established to entitle the insurer to rescind the
contract.18 Misrepresentation as a defense of the insurer to avoid liability is an affirmative defense and the duty to

147
establish such defense by satisfactory and convincing evidence rests upon the insurer. 19 In the case at bar, the petitioner
failed to clearly and satisfactorily establish its defense, and is therefore liable to pay the proceeds of the
insurance.1wphi1.nt
And that brings us to the last point in the review of the case at bar. Petitioner claims that there was no evidence as to the
amount of Dr. Leuterio's outstanding indebtedness to DBP at the time of the mortgagor's death. Hence, for private
respondent's failure to establish the same, the action for specific performance should be dismissed. Petitioner's claim is
without merit. A life insurance policy is a valued policy. 20 Unless the interest of a person insured is susceptible of exact
pecuniary measurement, the measure of indemnity under a policy of insurance upon life or health is the sum fixed in the
policy. 21 The mortgagor paid the premium according to the coverage of his insurance, which states that:
The policy states that upon receipt of due proof of the Debtor's death during the terms of this insurance, a
death benefit in the amount of P86,200.00 shall be paid.
In the event of the debtor's death before his indebtedness with the creditor shall have been fully paid, an
amount to pay the outstanding indebtedness shall first be paid to the Creditor and the balance of the Sum
Assured, if there is any shall then be paid to the beneficiary/ies designated by the debtor." 22 (Emphasis
omitted)
However, we noted that the Court of Appeals' decision was promulgated on May 17, 1993. In private respondent's
memorandum, she states that DBP foreclosed in 1995 their residential lot, in satisfaction of mortgagor's outstanding loan.
Considering this supervening event, the insurance proceeds shall inure to the benefit of the heirs of the deceased person
or his beneficiaries. Equity dictates that DBP should not unjustly enrich itself at the expense of another (Nemo cum
alterius detrimenio protest). Hence, it cannot collect the insurance proceeds, after it already foreclosed on the mortgage.
The proceeds now rightly belong to Dr. Leuterio's heirs represented by his widow, herein private respondent Medarda
Leuterio.
WHEREFORE, the petition is hereby DENIED. The Decision and Resolution of the Court of Appeals in CA-G.R. CV 18341
is AFFIRMED with MODIFICATION that the petitioner is ORDERED to pay the insurance proceeds amounting to Eightysix thousand, two hundred (P86,200.00) pesos to the heirs of the insured, Dr. Wilfredo Leuterio (deceased), upon
presentation of proof of prior settlement of mortgagor's indebtedness to Development Bank of the Philippines. Costs
against petitioner.
1wphi1.nt

SO ORDERED.

Republic of the Philippines


SUPREME COURT
Manila
FIRST DIVISION

G.R. No. 105135 June 22, 1995


SUNLIFE ASSURANCE COMPANY OF CANADA, petitioner,
vs.
The Hon. COURT OF APPEALS and Spouses ROLANDO and BERNARDA BACANI, respondents.

148
QUIASON, J.:
This is a petition for review for certiorari under Rule 45 of the Revised Rules of Court to reverse and set aside the
Decision dated February 21, 1992 of the Court of Appeals in CA-G.R. CV No. 29068, and its Resolution dated April
22, 1992, denying reconsideration thereof.
We grant the petition.
I
On April 15, 1986, Robert John B. Bacani procured a life insurance contract for himself from petitioner. He was
issued Policy No. 3-903-766-X valued at P100,000.00, with double indemnity in case of accidental death. The
designated beneficiary was his mother, respondent Bernarda Bacani.
On June 26, 1987, the insured died in a plane crash. Respondent Bernarda Bacani filed a claim with petitioner,
seeking the benefits of the insurance policy taken by her son. Petitioner conducted an investigation and its findings
prompted it to reject the claim.
In its letter, petitioner informed respondent Bernarda Bacani, that the insured did not disclose material facts relevant
to the issuance of the policy, thus rendering the contract of insurance voidable. A check representing the total
premiums paid in the amount of P10,172.00 was attached to said letter.
Petitioner claimed that the insured gave false statements in his application when he answered the following
questions:
5. Within the past 5 years have you:
a) consulted any doctor or other health practitioner?
b) submitted to:
EGG?
X-rays?
blood tests?
other tests?
c) attended or been admitted to any hospital or other medical facility?
6. Have you ever had or sought advice for:
xxx xxx xxx
b) urine, kidney or bladder disorder? (Rollo, p. 53)
The deceased answered question No. 5(a) in the affirmative but limited his answer to a consultation with a certain
Dr. Reinaldo D. Raymundo of the Chinese General Hospital on February 1986, for cough and flu complications. The
other questions were answered in the negative (Rollo, p. 53).

149
Petitioner discovered that two weeks prior to his application for insurance, the insured was examined and confined
at the Lung Center of the Philippines, where he was diagnosed for renal failure. During his confinement, the
deceased was subjected to urinalysis, ultra-sonography and hematology tests.
On November 17, 1988, respondent Bernarda Bacani and her husband, respondent Rolando Bacani, filed an action
for specific performance against petitioner with the Regional Trial Court, Branch 191, Valenzuela, Metro Manila.
Petitioner filed its answer with counterclaim and a list of exhibits consisting of medical records furnished by the Lung
Center of the Philippines.
On January 14, 1990, private respondents filed a "Proposed Stipulation with Prayer for Summary Judgment" where
they manifested that they "have no evidence to refute the documentary evidence of concealment/misrepresentation
by the decedent of his health condition (Rollo, p. 62).
Petitioner filed its Request for Admissions relative to the authenticity and due execution of several documents as
well as allegations regarding the health of the insured. Private respondents failed to oppose said request or reply
thereto, thereby rendering an admission of the matters alleged.
Petitioner then moved for a summary judgment and the trial court decided in favor of private respondents. The
dispositive portion of the decision is reproduced as follows:
WHEREFORE, judgment is hereby rendered in favor of the plaintiffs and against the defendant,
condemning the latter to pay the former the amount of One Hundred Thousand Pesos (P100,000.00)
the face value of insured's Insurance Policy No. 3903766, and the Accidental Death Benefit in the
amount of One Hundred Thousand Pesos (P100,000.00) and further sum of P5,000.00 in the
concept of reasonable attorney's fees and costs of suit.
Defendant's counterclaim is hereby Dismissed (Rollo, pp. 43-44).
In ruling for private respondents, the trial court concluded that the facts concealed by the insured were made in
good faith and under a belief that they need not be disclosed. Moreover, it held that the health history of the insured
was immaterial since the insurance policy was "non-medical".
Petitioner appealed to the Court of Appeals, which affirmed the decision of the trial court. The appellate court ruled
that petitioner cannot avoid its obligation by claiming concealment because the cause of death was unrelated to the
facts concealed by the insured. It also sustained the finding of the trial court that matters relating to the health
history of the insured were irrelevant since petitioner waived the medical examination prior to the approval and
issuance of the insurance policy. Moreover, the appellate court agreed with the trial court that the policy was "nonmedical" (Rollo, pp. 4-5).
Petitioner's motion for reconsideration was denied; hence, this petition.
II
We reverse the decision of the Court of Appeals.
The rule that factual findings of the lower court and the appellate court are binding on this Court is not absolute and
admits of exceptions, such as when the judgment is based on a misappreciation of the facts (Geronimo v. Court of
Appeals, 224 SCRA 494 [1993]).

150
In weighing the evidence presented, the trial court concluded that indeed there was concealment and
misrepresentation, however, the same was made in "good faith" and the facts concealed or misrepresented were
irrelevant since the policy was "non-medical". We disagree.
Section 26 of The Insurance Code is explicit in requiring a party to a contract of insurance to communicate to the
other, in good faith, all facts within his knowledge which are material to the contract and as to which he makes no
warranty, and which the other has no means of ascertaining. Said Section provides:
A neglect to communicate that which a party knows and ought to communicate, is called
concealment.
Materiality is to be determined not by the event, but solely by the probable and reasonable influence of the facts
upon the party to whom communication is due, in forming his estimate of the disadvantages of the proposed
contract or in making his inquiries (The Insurance Code, Sec. 31).
The terms of the contract are clear. The insured is specifically required to disclose to the insurer matters relating to
his health.
The information which the insured failed to disclose were material and relevant to the approval and issuance of the
insurance policy. The matters concealed would have definitely affected petitioner's action on his application, either
by approving it with the corresponding adjustment for a higher premium or rejecting the same. Moreover, a
disclosure may have warranted a medical examination of the insured by petitioner in order for it to reasonably
assess the risk involved in accepting the application.
In Vda. de Canilang v. Court of Appeals, 223 SCRA 443 (1993), we held that materiality of the information withheld
does not depend on the state of mind of the insured. Neither does it depend on the actual or physical events which
ensue.
Thus, "goad faith" is no defense in concealment. The insured's failure to disclose the fact that he was hospitalized
for two weeks prior to filing his application for insurance, raises grave doubts about his bonafides. It appears that
such concealment was deliberate on his part.
The argument, that petitioner's waiver of the medical examination of the insured debunks the materiality of the facts
concealed, is untenable. We reiterate our ruling in Saturnino v. Philippine American Life Insurance Company, 7
SCRA 316 (1963), that " . . . the waiver of a medical examination [in a non-medical insurance contract] renders even
more material the information required of the applicant concerning previous condition of health and diseases
suffered, for such information necessarily constitutes an important factor which the insurer takes into consideration
in deciding whether to issue the policy or not . . . "
Moreover, such argument of private respondents would make Section 27 of the Insurance Code, which allows the
injured party to rescind a contract of insurance where there is concealment, ineffective (See Vda. de Canilang v.
Court of Appeals, supra).
Anent the finding that the facts concealed had no bearing to the cause of death of the insured, it is well settled that
the insured need not die of the disease he had failed to disclose to the insurer. It is sufficient that his non-disclosure
misled the insurer in forming his estimates of the risks of the proposed insurance policy or in making inquiries
(Henson v. The Philippine American Life Insurance Co., 56 O.G. No. 48 [1960]).

151
We, therefore, rule that petitioner properly exercised its right to rescind the contract of insurance by reason of the
concealment employed by the insured. It must be emphasized that rescission was exercised within the two-year
contestability period as recognized in Section 48 of The Insurance Code.
WHEREFORE, the petition is GRANTED and the Decision of the Court of Appeals is REVERSED and SET ASIDE.
SO ORDERED.

Republic of the Philippines


SUPREME COURT
Manila
FIRST DIVISION
G.R. No. 125678

March 18, 2002

152
PHILAMCARE HEALTH SYSTEMS, INC., petitioner,
vs.
COURT OF APPEALS and JULITA TRINOS, respondents.
YNARES-SANTIAGO, J.:
Ernani Trinos, deceased husband of respondent Julita Trinos, applied for a health care coverage with petitioner
Philamcare Health Systems, Inc. In the standard application form, he answered no to the following question:
Have you or any of your family members ever consulted or been treated for high blood pressure, heart
trouble, diabetes, cancer, liver disease, asthma or peptic ulcer? (If Yes, give details). 1
The application was approved for a period of one year from March 1, 1988 to March 1, 1989. Accordingly, he was
issued Health Care Agreement No. P010194. Under the agreement, respondents husband was entitled to avail of
hospitalization benefits, whether ordinary or emergency, listed therein. He was also entitled to avail of "out-patient
benefits" such as annual physical examinations, preventive health care and other out-patient services.
Upon the termination of the agreement, the same was extended for another year from March 1, 1989 to March 1,
1990, then from March 1, 1990 to June 1, 1990. The amount of coverage was increased to a maximum sum of
P75,000.00 per disability.2
During the period of his coverage, Ernani suffered a heart attack and was confined at the Manila Medical Center
(MMC) for one month beginning March 9, 1990. While her husband was in the hospital, respondent tried to claim the
benefits under the health care agreement. However, petitioner denied her claim saying that the Health Care
Agreement was void. According to petitioner, there was a concealment regarding Ernanis medical history. Doctors
at the MMC allegedly discovered at the time of Ernanis confinement that he was hypertensive, diabetic and
asthmatic, contrary to his answer in the application form. Thus, respondent paid the hospitalization expenses
herself, amounting to about P76,000.00.
After her husband was discharged from the MMC, he was attended by a physical therapist at home. Later, he was
admitted at the Chinese General Hospital. Due to financial difficulties, however, respondent brought her husband
home again. In the morning of April 13, 1990, Ernani had fever and was feeling very weak. Respondent was
constrained to bring him back to the Chinese General Hospital where he died on the same day.
On July 24, 1990, respondent instituted with the Regional Trial Court of Manila, Branch 44, an action for damages
against petitioner and its president, Dr. Benito Reverente, which was docketed as Civil Case No. 90-53795. She
asked for reimbursement of her expenses plus moral damages and attorneys fees. After trial, the lower court ruled
against petitioners, viz:
WHEREFORE, in view of the forgoing, the Court renders judgment in favor of the plaintiff Julita Trinos,
ordering:
1. Defendants to pay and reimburse the medical and hospital coverage of the late Ernani Trinos in the
amount of P76,000.00 plus interest, until the amount is fully paid to plaintiff who paid the same;
2. Defendants to pay the reduced amount of moral damages of P10,000.00 to plaintiff;
3. Defendants to pay the reduced amount of P10,000.00 as exemplary damages to plaintiff;

153
4. Defendants to pay attorneys fees of P20,000.00, plus costs of suit.
SO ORDERED.3
On appeal, the Court of Appeals affirmed the decision of the trial court but deleted all awards for damages and
absolved petitioner Reverente.4 Petitioners motion for reconsideration was denied. 5 Hence, petitioner brought the
instant petition for review, raising the primary argument that a health care agreement is not an insurance contract;
hence the "incontestability clause" under the Insurance Code 6 does not apply.
1wphi1.nt

Petitioner argues that the agreement grants "living benefits," such as medical check-ups and hospitalization which a
member may immediately enjoy so long as he is alive upon effectivity of the agreement until its expiration one-year
thereafter. Petitioner also points out that only medical and hospitalization benefits are given under the agreement
without any indemnification, unlike in an insurance contract where the insured is indemnified for his loss. Moreover,
since Health Care Agreements are only for a period of one year, as compared to insurance contracts which last
longer,7 petitioner argues that the incontestability clause does not apply, as the same requires an effectivity period of
at least two years. Petitioner further argues that it is not an insurance company, which is governed by the Insurance
Commission, but a Health Maintenance Organization under the authority of the Department of Health.
Section 2 (1) of the Insurance Code defines a contract of insurance as an agreement whereby one undertakes for a
consideration to indemnify another against loss, damage or liability arising from an unknown or contingent event. An
insurance contract exists where the following elements concur:
1. The insured has an insurable interest;
2. The insured is subject to a risk of loss by the happening of the designated peril;
3. The insurer assumes the risk;
4. Such assumption of risk is part of a general scheme to distribute actual losses among a large group of
persons bearing a similar risk; and
5. In consideration of the insurers promise, the insured pays a premium. 8
Section 3 of the Insurance Code states that any contingent or unknown event, whether past or future, which may
damnify a person having an insurable interest against him, may be insured against. Every person has an insurable
interest in the life and health of himself. Section 10 provides:
Every person has an insurable interest in the life and health:
(1) of himself, of his spouse and of his children;
(2) of any person on whom he depends wholly or in part for education or support, or in whom he has a
pecuniary interest;
(3) of any person under a legal obligation to him for the payment of money, respecting property or service, of
which death or illness might delay or prevent the performance; and
(4) of any person upon whose life any estate or interest vested in him depends.

154
In the case at bar, the insurable interest of respondents husband in obtaining the health care agreement was his
own health. The health care agreement was in the nature of non-life insurance, which is primarily a contract of
indemnity.9 Once the member incurs hospital, medical or any other expense arising from sickness, injury or other
stipulated contingent, the health care provider must pay for the same to the extent agreed upon under the contract.
Petitioner argues that respondents husband concealed a material fact in his application. It appears that in the
application for health coverage, petitioners required respondents husband to sign an express authorization for any
person, organization or entity that has any record or knowledge of his health to furnish any and all information
relative to any hospitalization, consultation, treatment or any other medical advice or examination. 10 Specifically, the
Health Care Agreement signed by respondents husband states:
We hereby declare and agree that all statement and answers contained herein and in any addendum
annexed to this application are full, complete and true and bind all parties in interest under the Agreement
herein applied for, that there shall be no contract of health care coverage unless and until an Agreement is
issued on this application and the full Membership Fee according to the mode of payment applied for is
actually paid during the lifetime and good health of proposed Members; that no information acquired by any
Representative of PhilamCare shall be binding upon PhilamCare unless set out in writing in the
application;that any physician is, by these presents, expressly authorized to disclose or give testimony at
anytime relative to any information acquired by him in his professional capacity upon any question affecting
the eligibility for health care coverage of the Proposed Members and that the acceptance of any Agreement
issued on this application shall be a ratification of any correction in or addition to this application as stated in
the space for Home Office Endorsement.11 (Underscoring ours)
In addition to the above condition, petitioner additionally required the applicant for authorization to inquire about the
applicants medical history, thus:
I hereby authorize any person, organization, or entity that has any record or knowledge of my health and/or
that of __________ to give to the PhilamCare Health Systems, Inc. any and all information relative to any
hospitalization, consultation, treatment or any other medical advice or examination. This authorization is in
connection with the application for health care coverage only. A photographic copy of this authorization shall
be as valid as the original.12 (Underscoring ours)
Petitioner cannot rely on the stipulation regarding "Invalidation of agreement" which reads:
Failure to disclose or misrepresentation of any material information by the member in the application or
medical examination, whether intentional or unintentional, shall automatically invalidate the Agreement from
the very beginning and liability of Philamcare shall be limited to return of all Membership Fees paid. An
undisclosed or misrepresented information is deemed material if its revelation would have resulted in the
declination of the applicant by Philamcare or the assessment of a higher Membership Fee for the benefit or
benefits applied for.13
The answer assailed by petitioner was in response to the question relating to the medical history of the applicant.
This largely depends on opinion rather than fact, especially coming from respondents husband who was not a
medical doctor. Where matters of opinion or judgment are called for, answers made in good faith and without intent
to deceive will not avoid a policy even though they are untrue.14 Thus,
(A)lthough false, a representation of the expectation, intention, belief, opinion, or judgment of the insured will
not avoid the policy if there is no actual fraud in inducing the acceptance of the risk, or its acceptance at a
lower rate of premium, and this is likewise the rule although the statement is material to the risk, if the
statement is obviously of the foregoing character, since in such case the insurer is not justified in relying

155
upon such statement, but is obligated to make further inquiry. There is a clear distinction between such a
case and one in which the insured is fraudulently and intentionally states to be true, as a matter of
expectation or belief, that which he then knows, to be actually untrue, or the impossibility of which is shown
by the facts within his knowledge, since in such case the intent to deceive the insurer is obvious and
amounts to actual fraud.15 (Underscoring ours)
The fraudulent intent on the part of the insured must be established to warrant rescission of the insurance
contract.16 Concealment as a defense for the health care provider or insurer to avoid liability is an affirmative
defense and the duty to establish such defense by satisfactory and convincing evidence rests upon the provider or
insurer. In any case, with or without the authority to investigate, petitioner is liable for claims made under the
contract. Having assumed a responsibility under the agreement, petitioner is bound to answer the same to the
extent agreed upon. In the end, the liability of the health care provider attaches once the member is hospitalized for
the disease or injury covered by the agreement or whenever he avails of the covered benefits which he has prepaid.
Under Section 27 of the Insurance Code, "a concealment entitles the injured party to rescind a contract of
insurance." The right to rescind should be exercised previous to the commencement of an action on the contract. 17 In
this case, no rescission was made. Besides, the cancellation of health care agreements as in insurance policies
require the concurrence of the following conditions:
1. Prior notice of cancellation to insured;
2. Notice must be based on the occurrence after effective date of the policy of one or more of the grounds
mentioned;
3. Must be in writing, mailed or delivered to the insured at the address shown in the policy;
4. Must state the grounds relied upon provided in Section 64 of the Insurance Code and upon request of insured, to
furnish facts on which cancellation is based.18
None of the above pre-conditions was fulfilled in this case. When the terms of insurance contract contain limitations
on liability, courts should construe them in such a way as to preclude the insurer from non-compliance with his
obligation.19 Being a contract of adhesion, the terms of an insurance contract are to be construed strictly against the
party which prepared the contract the insurer.20 By reason of the exclusive control of the insurance company over
the terms and phraseology of the insurance contract, ambiguity must be strictly interpreted against the insurer and
liberally in favor of the insured, especially to avoid forfeiture.21 This is equally applicable to Health Care Agreements.
The phraseology used in medical or hospital service contracts, such as the one at bar, must be liberally construed in
favor of the subscriber, and if doubtful or reasonably susceptible of two interpretations the construction conferring
coverage is to be adopted, and exclusionary clauses of doubtful import should be strictly construed against the
provider.22
Anent the incontestability of the membership of respondents husband, we quote with approval the following findings
of the trial court:
(U)nder the title Claim procedures of expenses, the defendant Philamcare Health Systems Inc. had twelve
months from the date of issuance of the Agreement within which to contest the membership of the patient if
he had previous ailment of asthma, and six months from the issuance of the agreement if the patient was
sick of diabetes or hypertension. The periods having expired, the defense of concealment or
misrepresentation no longer lie.23

156
Finally, petitioner alleges that respondent was not the legal wife of the deceased member considering that at the
time of their marriage, the deceased was previously married to another woman who was still alive. The health care
agreement is in the nature of a contract of indemnity. Hence, payment should be made to the party who incurred the
expenses. It is not controverted that respondent paid all the hospital and medical expenses. She is therefore entitled
to reimbursement. The records adequately prove the expenses incurred by respondent for the deceaseds
hospitalization, medication and the professional fees of the attending physicians. 24
WHEREFORE, in view of the foregoing, the petition is DENIED. The assailed decision of the Court of Appeals dated
December 14, 1995 is AFFIRMED.
SO ORDERED.

Republic of the Philippines


SUPREME COURT
Manila
THIRD DIVISION

157

G.R. No. 92492 June 17, 1993


THELMA VDA. DE CANILANG, petitioner,
vs.
HON. COURT OF APPEALS and GREAT PACIFIC LIFE ASSURANCE CORPORATION, respondents.
Simeon C. Sato for petitioner.
FELICIANO, J.:
On 18 June 1982, Jaime Canilang consulted Dr. Wilfredo B. Claudio and was diagnosed as suffering from "sinus
tachycardia." The doctor prescribed the following fro him: Trazepam, a tranquilizer; and Aptin, a beta-blocker drug.
Mr. Canilang consulted the same doctor again on 3 August 1982 and this time was found to have "acute bronchitis."
On next day, 4 August 1982, Jaime Canilang applied for a "non-medical" insurance policy with respondent Great
Pacific Life Assurance Company ("Great Pacific") naming his wife, Thelma Canilang, as his beneficiary. 1 Jaime
Canilang was issued ordinary life insurance Policy No. 345163, with the face value of P19,700, effective as of 9 August
1982.
On 5 August 1983, Jaime Canilang died of "congestive heart failure," "anemia," and "chronic anemia." 2 Petitioner,
widow and beneficiary of the insured, filed a claim with Great Pacific which the insurer denied on 5 December 1983 upon
the ground that the insured had concealed material information from it.
Petitioner then filed a complaint against Great Pacific with the Insurance Commission for recovery of the insurance
proceeds. During the hearing called by the Insurance Commissioner, petitioner testified that she was not aware of
any serious illness suffered by her late husband 3 and that, as far as she knew, her husband had died because of a
kidney disorder. 4 A deposition given by Dr. Wilfredo Claudio was presented by petitioner. There Dr. Claudio stated that he
was the family physician of the deceased Jaime Canilang 5 and that he had previously treated him for "sinus tachycardia"
and "acute bronchitis." 6 Great Pacific for its part presented Dr. Esperanza Quismorio, a physician
and a medical underwriter working for Great Pacific. 7 She testified that the deceased's insurance application had been
approved on the basis of his medical declaration. 8 She explained that as a rule, medical examinations are required only in
cases where the applicant has indicated in his application for insurance coverage that he has previously undergone
medical consultation and hospitalization. 9
In a decision dated 5 November 1985, Insurance Commissioner Armando Ansaldo ordered Great Pacific to pay
P19,700 plus legal interest and P2,000.00 as attorney's fees after holding that:
1. the ailment of Jaime Canilang was not so serious that, even if it had been disclosed, it would not
have affected Great Pacific's decision to insure him;
2. Great Pacific had waived its right to inquire into the health condition of the applicant by the
issuance of the policy despite the lack of answers to "some of the pertinent questions" in the
insurance application;
3. there was no intentional concealment on the part of the insured Jaime Canilang as he had
thought that he was merely suffering from a minor ailment and simple cold; 10 and

158
4. Batas Pambansa Blg. 847 which voids an insurance contract, whether or not concealment was
intentionally made, was not applicable to Canilang's case as that law became effective only on 1
June 1985.
On appeal by Great Pacific, the Court of Appeals reversed and set aside the decision of the Insurance
Commissioner and dismissed Thelma Canilang's complaint and Great Pacific's counterclaim. The Court of Appealed
found that the use of the word "intentionally" by the Insurance Commissioner in defining and resolving the issue
agreed upon by the parties at pre-trial before the Insurance Commissioner was not supported by the evidence; that
the issue agreed upon by the parties had been whether the deceased insured, Jaime Canilang, made a material
concealment as the state of his health at the time of the filing of insurance application, justifying respondent's denial
of the claim. The Court of Appeals also found that the failure of Jaime Canilang to disclose previous medical
consultation and treatment constituted material information which should have been communicated to Great Pacific
to enable the latter to make proper inquiries. The Court of Appeals finally held that the Ng Gan Zee case which had
involved misrepresentation was not applicable in respect of the case at bar which involves concealment.
Petitioner Thelma Canilang is now before this Court on a Petition for Review on Certiorari alleging that:
1. . . . the Honorable Court of Appeals, speaking with due respect, erred in not holding that the issue
in the case agreed upon between the parties before the Insurance Commission is whether or not
Jaime Canilang "intentionally" made material concealment in stating his state of health;
2. . . . at any rate, the non-disclosure of certain facts about his previous health conditions does not
amount to fraud and private respondent is deemed to have waived inquiry thereto. 11
The medical declaration which was set out in the application for insurance executed by Jaime Canilang read as
follows:
MEDICAL DECLARATION
I hereby declare that:
(1) I have not been confined in any hospital, sanitarium or infirmary, nor receive any medical or
surgical advice/attention within the last five (5) years.
(2) I have never been treated nor consulted a physician for a heart condition, high blood pressure,
cancer, diabetes, lung, kidney, stomach disorder, or any other physical impairment.
(3) I am, to the best of my knowledge, in good health.
EXCEPTIONS:
_______________________________________________________________________________
_
GENERAL DECLARATION
I hereby declare that all the foregoing answers and statements are complete, true and correct. I
hereby agree that if there be any fraud or misrepresentation in the above statements material to the
risk, the INSURANCE COMPANY upon discovery within two (2) years from the effective date of

159
insurance shall have the right to declare such insurance null and void. That the liabilities of the
Company under the said Policy/TA/Certificate shall accrue and begin only from the date of
commencement of risk stated in the Policy/TA/Certificate, provided that the first premium is paid and
the Policy/TA/Certificate is delivered to, and accepted by me in person, when I am in actual good
health.
Signed at Manila his 4th day of August, 1992.
Illegibl
e

Signat
ure of
Applica
nt. 12
We note that in addition to the negative statements made by Mr. Canilang in paragraph 1 and 2 of the medical
declaration, he failed to disclose in the appropriate space, under the caption "Exceptions," that he had twice
consulted Dr. Wilfredo B. Claudio who had found him to be suffering from "sinus tachycardia" and "acute bronchitis."
The relevant statutory provisions as they stood at the time Great Pacific issued the contract of insurance and at the
time Jaime Canilang died, are set out in P.D. No. 1460, also known as the Insurance Code of 1978, which went into
effect on 11 June 1978. These provisions read as follows:
Sec. 26. A neglect to communicate that which a party knows and ought to communicate, is called a
concealment.
xxx xxx xxx
Sec. 28. Each party to a contract of insurance must communicate to the other, in good faith, all
factorswithin his knowledge which are material to the contract and as to which he makes no
warranty, and which the other has not the means of ascertaining. (Emphasis supplied)
Under the foregoing provisions, the information concealed must be information which the concealing party knew and
"ought to [have] communicate[d]," that is to say, information which was "material to the contract." The test of
materiality is contained in Section 31 of the Insurance Code of 1978 which reads:
Sec. 31. Materially is to be determined not by the event, but solely by the probable and reasonable
influence of the facts upon the party to whom the communication is due, in forming his estimate of
the disadvantages of the proposed contract, or in making his inquiries. (Emphasis supplied)
"Sinus tachycardia" is considered present "when the heart rate exceeds 100 beats per minute." 13 The symptoms of
this condition include pounding in the chest and sometimes faintness and weakness of the person affected. The following
elaboration was offered by Great Pacific and set out by the Court of Appeals in its Decision:
Sinus tachycardia is defined as sinus-initiated; heart rate faster than 100 beats per minute.
(Harrison' s Principles of Internal Medicine, 8th ed. [1978], p. 1193.) It is, among others, a common

160
reaction to heart disease, including myocardial infarction, and heart failure per se. (Henry J.L.
Marriot, M.D.,Electrocardiography, 6th ed., [1977], p. 127.) The medication prescribed by Dr. Claudio
for treatment of Canilang's ailment on June 18, 1982, indicates the condition that said physician was
trying to manage. Thus, he prescribed Trazepam, (Philippine Index of Medical Specialties (PIMS),
Vol. 14, No. 3, Dec. 1985, p. 112) which is anti-anxiety, anti-convulsant, muscle-relaxant; and Aptin,
(Idem, p. 36) a cardiac drug, for palpitations and nervous heart. Such treatment could have been a
very material information to the insurer in determining the action to be take on Canilang's application
for life insurance coverage. 14
We agree with the Court of Appeals that the information which Jaime Canilang failed to disclose was material to the
ability of Great Pacific to estimate the probable risk he presented as a subject of life insurance. Had Canilang
disclosed his visits to his doctor, the diagnosis made and medicines prescribed by such doctor, in the insurance
application, it may be reasonably assumed that Great Pacific would have made further inquiries and would have
probably refused to issue a non-medical insurance policy or, at the very least, required a higher premium for the
same coverage. 15 The materiality of the information withheld by Great Pacific did not depend upon the state of mind of
Jaime Canilang. A man's state of mind or subjective belief is not capable of proof in our judicial process, except through
proof of external acts or failure to act from which inferences as to his subjective belief may be reasonably drawn. Neither
does materiality depend upon the actual or physical events which ensue. Materiality relates rather to the "probable and
reasonable influence of the facts" upon the party to whom the communication should have been made, in assessing the
risk involved in making or omitting to make further inquiries and in accepting the application for insurance; that "probable
and reasonable influence of the facts" concealed must, of course, be determined objectively, by the judge ultimately.
The insurance Great Pacific applied for was a "non-medical" insurance policy. In Saturnino v. Philippine-American
Life Insurance Company, 16 this Court held that:
. . . if anything, the waiver of medical examination [in a non-medical insurance contract] renders
even more material the information required of the applicant concerning previous condition of health
and diseases suffered, for such information necessarily constitutes an important factor which the
insurer takes into consideration in deciding whether to issue the policy or not . . . . 17 (Emphasis
supplied)
The Insurance Commissioner had also ruled that the failure of Great Pacific to convey certain information to the
insurer was not "intentional" in nature, for the reason that Jaime Canilang believed that he was suffering from minor
ailment like a common cold. Section 27 of the Insurance Code of 1978 as it existed from 1974 up to 1985, that is,
throughout the time range material for present purposes, provided that:
Sec. 27. A concealment entitles the injured party to rescind a contract of insurance.
The preceding statute, Act No. 2427, as it stood from 1914 up to 1974, had provided:
Sec. 26. A concealment, whether intentional or unintentional, entitles the injured party to rescind a
contract of insurance. (Emphasis supplied)
Upon the other hand, in 1985, the Insurance Code of 1978 was amended by
B.P. Blg. 874. This subsequent statute modified Section 27 of the Insurance Code of 1978 so as to read as follows:
Sec. 27. A concealment whether intentional or unintentional entitles the injured party to rescind a
contract of insurance. (Emphasis supplied)

161
The unspoken theory of the Insurance Commissioner appears to have been that by deleting the phrase "intentional
or unintentional," the Insurance Code of 1978 (prior to its amendment by B.P. Blg. 874) intended to limit the kinds of
concealment which generate a right to rescind on the part of the injured party to "intentional concealments." This
argument is not persuasive. As a simple matter of grammar, it may be noted that "intentional" and "unintentional"
cancel each other out. The net result therefore of the phrase "whether intentional or unitentional" is precisely to
leave unqualified the term "concealment." Thus, Section 27 of the Insurance Code of 1978 is properly read as
referring to "any concealment" without regard to whether such concealment is intentional or unintentional. The
phrase "whether intentional or unintentional" was in fact superfluous. The deletion of the phrase "whether intentional
or unintentional" could not have had the effect of imposing an affirmative requirement that a concealment must be
intentional if it is to entitle the injured party to rescind a contract of insurance. The restoration in 1985 by B.P. Blg.
874 of the phrase "whether intentional or unintentional" merely underscored the fact that all throughout (from 1914
to 1985), the statute did not require proof that concealment must be "intentional" in order to authorize rescission by
the injured party.
In any case, in the case at bar, the nature of the facts not conveyed to the insurer was such that the failure to
communicate must have been intentional rather than merely inadvertent. For Jaime Canilang could not have been
unaware that his heart beat would at times rise to high and alarming levels and that he had consulted a doctor twice
in the two (2) months before applying for non-medical insurance. Indeed, the last medical consultation took place
just the day before the insurance application was filed. In all probability, Jaime Canilang went to visit his doctor
precisely because of the discomfort and concern brought about by his experiencing "sinus tachycardia."
We find it difficult to take seriously the argument that Great Pacific had waived inquiry into the concealment by
issuing the insurance policy notwithstanding Canilang's failure to set out answers to some of the questions in the
insurance application. Such failure precisely constituted concealment on the part of Canilang. Petitioner's argument,
if accepted, would obviously erase Section 27 from the Insurance Code of 1978.
It remains only to note that the Court of Appeals finding that the parties had not agreed in the pretrial before the
Insurance Commission that the relevant issue was whether or not Jaime Canilang had intentionally concealed
material information from the insurer, was supported by the evidence of record, i.e., the Pre-trial Order itself dated
17 October 1984 and the Minutes of the Pre-trial Conference dated 15 October 1984, which "readily shows that the
word "intentional" does not appear in the statement or definition of the issue in the said Order and Minutes." 18
WHEREFORE, the Petition for Review is DENIED for lack of merit and the Decision of the Court of Appeals dated
16 October 1989 in C.A.-G.R. SP No. 08696 is hereby AFFIRMED. No pronouncement as to the costs.
SO ORDERED.

162

Republic of the Philippines


SUPREME COURT
Manila
THIRD DIVISION
G.R. No. 48049 June 29, 1989
EMILIO TAN, JUANITO TAN, ALBERTO TAN and ARTURO TAN, petitioners,
vs.
THE COURT OF APPEALS and THE PHILIPPINE AMERICAN LIFE INSURANCE COMPANY, respondents.

163
O.F. Santos & P.C. Nolasco for petitioners.
Ferry, De la Rosa and Associates for private respondent.

GUTIERREZ, JR., J.:


This is a petition for review on certiorari of the Court of Appeals' decision affirming the decision of the Insurance
Commissioner which dismissed the petitioners' complaint against respondent Philippine American Life Insurance
Company for the recovery of the proceeds from their late father's policy. The facts of the case as found by the Court
of Appeals are:
Petitioners appeal from the Decision of the Insurance Commissioner dismissing herein petitioners'
complaint against respondent Philippine American Life Insurance Company for the recovery of the
proceeds of Policy No. 1082467 in the amount of P 80,000.00.
On September 23,1973, Tan Lee Siong, father of herein petitioners, applied for life insurance in the
amount of P 80,000.00 with respondent company. Said application was approved and Policy No.
1082467 was issued effective November 6,1973, with petitioners the beneficiaries thereof (Exhibit
A).
On April 26,1975, Tan Lee Siong died of hepatoma (Exhibit B). Petitioners then filed with respondent
company their claim for the proceeds of the life insurance policy. However, in a letter dated
September 11, 1975, respondent company denied petitioners' claim and rescinded the policy by
reason of the alleged misrepresentation and concealment of material facts made by the deceased
Tan Lee Siong in his application for insurance (Exhibit 3). The premiums paid on the policy were
thereupon refunded .
Alleging that respondent company's refusal to pay them the proceeds of the policy was unjustified
and unreasonable, petitioners filed on November 27, 1975, a complaint against the former with the
Office of the Insurance Commissioner, docketed as I.C. Case No. 218.
After hearing the evidence of both parties, the Insurance Commissioner rendered judgment on
August 9, 1977, dismissing petitioners' complaint. (Rollo, pp. 91-92)
The Court of Appeals dismissed ' the petitioners' appeal from the Insurance Commissioner's decision for lack of
merit
Hence, this petition.
The petitioners raise the following issues in their assignment of errors, to wit:
A. The conclusion in law of respondent Court that respondent insurer has the right to rescind the
policy contract when insured is already dead is not in accordance with existing law and applicable
jurisprudence.

164
B. The conclusion in law of respondent Court that respondent insurer may be allowed to avoid the
policy on grounds of concealment by the deceased assured, is contrary to the provisions of the
policy contract itself, as well as, of applicable legal provisions and established jurisprudence.
C. The inference of respondent Court that respondent insurer was misled in issuing the policy are
manifestly mistaken and contrary to admitted evidence. (Rollo, p. 7)
The petitioners contend that the respondent company no longer had the right to rescind the contract of insurance as
rescission must allegedly be done during the lifetime of the insured within two years and prior to the commencement
of action.
The contention is without merit.
The pertinent section in the Insurance Code provides:
Section 48. Whenever a right to rescind a contract of insurance is given to the insurer by any
provision of this chapter, such right must be exercised previous to the commencement of an action
on the contract.
After a policy of life insurance made payable on the death of the insured shall have been in force
during the lifetime of the insured for a period of two years from the date of its issue or of its last
reinstatement, the insurer cannot prove that the policy is void ab initio or is rescindable by reason of
the fraudulent concealment or misrepresentation of the insured or his agent.
According to the petitioners, the Insurance Law was amended and the second paragraph of Section 48 added to
prevent the insurance company from exercising a right to rescind after the death of the insured.
The so-called "incontestability clause" precludes the insurer from raising the defenses of false representations or
concealment of material facts insofar as health and previous diseases are concerned if the insurance has been in
force for at least two years during the insured's lifetime. The phrase "during the lifetime" found in Section 48 simply
means that the policy is no longer considered in force after the insured has died. The key phrase in the second
paragraph of Section 48 is "for a period of two years."
As noted by the Court of Appeals, to wit:
The policy was issued on November 6,1973 and the insured died on April 26,1975. The policy was
thus in force for a period of only one year and five months. Considering that the insured died before
the two-year period had lapsed, respondent company is not, therefore, barred from proving that the
policy is void ab initio by reason of the insured's fraudulent concealment or misrepresentation.
Moreover, respondent company rescinded the contract of insurance and refunded the premiums
paid on September 11, 1975, previous to the commencement of this action on November 27,1975.
(Rollo, pp. 99-100)
xxx xxx xxx
The petitioners contend that there could have been no concealment or misrepresentation by their late father
because Tan Lee Siong did not have to buy insurance. He was only pressured by insistent salesmen to do so. The
petitioners state:

165
Here then is a case of an assured whose application was submitted because of repeated visits and
solicitations by the insurer's agent. Assured did not knock at the door of the insurer to buy insurance.
He was the object of solicitations and visits.
Assured was a man of means. He could have obtained a bigger insurance, not just P 80,000.00. If
his purpose were to misrepresent and to conceal his ailments in anticipation of death during the twoyear period, he certainly could have gotten a bigger insurance. He did not.
Insurer Philamlife could have presented as witness its Medical Examiner Dr. Urbano Guinto. It was
he who accomplished the application, Part II, medical. Philamlife did not.
Philamlife could have put to the witness stand its Agent Bienvenido S. Guinto, a relative to Dr.
Guinto, Again Philamlife did not. (pp. 138139, Rollo)
xxx xxx xxx
This Honorable Supreme Court has had occasion to denounce the pressure and practice indulged in
by agents in selling insurance. At one time or another most of us have been subjected to that
pressure, that practice. This court took judicial cognizance of the whirlwind pressure of insurance
selling-especially of the agent's practice of 'supplying the information, preparing and answering the
application, submitting the application to their companies, concluding the transactions and
otherwisesmoothing out all difficulties.
We call attention to what this Honorable Court said in Insular Life v. Feliciano, et al., 73 Phil. 201; at page 205:
It is of common knowledge that the selling of insurance today is subjected to the
whirlwind pressureof modern salesmanship.
Insurance companies send detailed instructions to their agents to solicit and procure applications.
These agents are to be found all over the length and breadth of the land. They are stimulated to
more active efforts by contests and by the keen competition offered by the other rival insurance
companies.
They supply all the information, prepare and answer the applications, submit the applications to their
companies, conclude the transactions, and otherwise smooth out all difficulties.
The agents in short do what the company set them out to do.
The Insular Life case was decided some forty years ago when the pressure of insurance
salesmanship was not overwhelming as it is now; when the population of this country was less than
one-fourth of what it is now; when the insurance companies competing with one another could be
counted by the fingers. (pp. 140-142, Rollo)
xxx xxx xxx
In the face of all the above, it would be unjust if, having been subjected to the whirlwind pressure of
insurance salesmanship this Court itself has long denounced, the assured who dies within the twoyear period, should stand charged of fraudulent concealment and misrepresentation." (p. 142, Rollo)

166
The legislative answer to the arguments posed by the petitioners is the "incontestability clause" added by the
second paragraph of Section 48.
The insurer has two years from the date of issuance of the insurance contract or of its last reinstatement within
which to contest the policy, whether or not, the insured still lives within such period. After two years, the defenses of
concealment or misrepresentation, no matter how patent or well founded, no longer lie. Congress felt this was a
sufficient answer to the various tactics employed by insurance companies to avoid liability. The petitioners'
interpretation would give rise to the incongruous situation where the beneficiaries of an insured who dies right after
taking out and paying for a life insurance policy, would be allowed to collect on the policy even if the insured
fraudulently concealed material facts.
The petitioners argue that no evidence was presented to show that the medical terms were explained in a layman's
language to the insured. They state that the insurer should have presented its two medical field examiners as
witnesses. Moreover, the petitioners allege that the policy intends that the medical examination must be conducted
before its issuance otherwise the insurer "waives whatever imperfection by ratification."
We agree with the Court of Appeals which ruled:
On the other hand, petitioners argue that no evidence was presented by respondent company to
show that the questions appearing in Part II of the application for insurance were asked, explained to
and understood by the deceased so as to prove concealment on his part. The same is not well
taken. The deceased, by affixing his signature on the application form, affirmed the correctness of all
the entries and answers appearing therein. It is but to be expected that he, a businessman, would
not have affixed his signature on the application form unless he clearly understood its significance.
For, the presumption is that a person intends the ordinary consequence of his voluntary act and
takes ordinary care of his concerns. [Sec. 5(c) and (d), Rule 131, Rules of Court].
The evidence for respondent company shows that on September 19,1972, the deceased was
examined by Dr. Victoriano Lim and was found to be diabetic and hypertensive; that by January,
1973, the deceased was complaining of progressive weight loss and abdominal pain and was
diagnosed to be suffering from hepatoma, (t.s.n. August 23, 1976, pp. 8-10; Exhibit 2). Another
physician, Dr. Wenceslao Vitug, testified that the deceased came to see him on December 14, 1973
for consolation and claimed to have been diabetic for five years. (t.s.n., Aug. 23,1976, p. 5; Exhibit 6)
Because of the concealment made by the deceased of his consultations and treatments for
hypertension, diabetes and liver disorders, respondent company was thus misled into accepting the
risk and approving his application as medically standard (Exhibit 5- C) and dispensing with further
medical investigation and examination (Exhibit 5-A). For as long as no adverse medical history is
revealed in the application form, an applicant for insurance is presumed to be healthy and physically
fit and no further medical investigation or examination is conducted by respondent company. (t.s.n.,
April 8,1976, pp. 6-8). (Rollo, pp. 96-98)
There is no strong showing that we should apply the "fine print" or "contract of adhesion" rule in this case. (Sweet
Lines, Inc. v. Teves, 83 SCRA 361 [1978]). The petitioners cite:
It is a matter of common knowledge that large amounts of money are collected from ignorant
persons by companies and associations which adopt high sounding titles and print the amount of
benefits they agree to pay in large black-faced type, following such undertakings by fine print
conditions which destroy the substance of the promise. All provisions, conditions, or exceptions
which in any way tend to work a forfeiture of the policy should be construed most strongly against

167
those for whose benefit they are inserted, and most favorably toward those against whom they are
meant to operate. (Trinidad v. Orient Protective Assurance Assn., 67 Phil. 184)
There is no showing that the questions in the application form for insurance regarding the insured's medical history
are in smaller print than the rest of the printed form or that they are designed in such a way as to conceal from the
applicant their importance. If a warning in bold red letters or a boxed warning similar to that required for cigarette
advertisements by the Surgeon General of the United States is necessary, that is for Congress or the Insurance
Commission to provide as protection against high pressure insurance salesmanship. We are limited in this petition
to ascertaining whether or not the respondent Court of Appeals committed reversible error. It is the petitioners'
burden to show that the factual findings of the respondent court are not based on substantial evidence or that its
conclusions are contrary to applicable law and jurisprudence. They have failed to discharge that burden.
WHEREFORE, the petition is hereby DENIED for lack of merit. The questioned decision of the Court of Appeals is
AFFIRMED.
SO ORDERED.

Republic of the Philippines


SUPREME COURT
Manila
THIRD DIVISION
G.R. No. 186983

February 22, 2012

MA. LOURDES S. FLORENDO, Petitioner,


vs.
PHILAM PLANS, INC., PERLA ABCEDE MA. CELESTE ABCEDE, Respondents.

168
DECISION
ABAD, J.:
This case is about an insureds alleged concealment in his pension plan application of his true state of health and its
effect on the life insurance portion of that plan in case of death.
The Facts and the Case
On October 23, 1997 Manuel Florendo filed an application for comprehensive pension plan with respondent Philam
Plans, Inc. (Philam Plans) after some convincing by respondent Perla Abcede. The plan had a pre-need price
of P997,050.00, payable in 10 years, and had a maturity value of P2,890,000.00 after 20 years.1 Manuel signed the
application and left to Perla the task of supplying the information needed in the application. 2Respondent Ma. Celeste
Abcede, Perlas daughter, signed the application as sales counselor.3
Aside from pension benefits, the comprehensive pension plan also provided life insurance coverage to
Florendo.4This was covered by a Group Master Policy that Philippine American Life Insurance Company (Philam
Life) issued to Philam Plans.5 Under the master policy, Philam Life was to automatically provide life insurance
coverage, including accidental death, to all who signed up for Philam Plans comprehensive pension plan. 6 If the
plan holder died before the maturity of the plan, his beneficiary was to instead receive the proceeds of the life
insurance, equivalent to the pre-need price. Further, the life insurance was to take care of any unpaid premium until
the pension plan matured, entitling the beneficiary to the maturity value of the pension plan. 7
On October 30, 1997 Philam Plans issued Pension Plan Agreement PP43005584 8 to Manuel, with petitioner Ma.
Lourdes S. Florendo, his wife, as beneficiary. In time, Manuel paid his quarterly premiums. 9
Eleven months later or on September 15, 1998, Manuel died of blood poisoning. Subsequently, Lourdes filed a claim
with Philam Plans for the payment of the benefits under her husbands plan. 10 Because Manuel died before his
pension plan matured and his wife was to get only the benefits of his life insurance, Philam Plans forwarded her
claim to Philam Life.11
On May 3, 1999 Philam Plans wrote Lourdes a letter,12 declining her claim. Philam Life found that Manuel was on
maintenance medicine for his heart and had an implanted pacemaker. Further, he suffered from diabetes mellitus
and was taking insulin. Lourdes renewed her demand for payment under the plan 13 but Philam Plans rejected
it,14 prompting her to file the present action against the pension plan company before the Regional Trial Court (RTC)
of Quezon City.15
On March 30, 2006 the RTC rendered judgment,16 ordering Philam Plans, Perla and Ma. Celeste, solidarily, to pay
Lourdes all the benefits from her husbands pension plan, namely: P997,050.00, the proceeds of his term insurance,
and P2,890,000.00 lump sum pension benefit upon maturity of his plan; P100,000.00 as moral damages; and to pay
the costs of the suit. The RTC ruled that Manuel was not guilty of concealing the state of his health from his pension
plan application.
On December 18, 2007 the Court of Appeals (CA) reversed the RTC decision, 17 holding that insurance policies are
traditionally contracts uberrimae fidae or contracts of utmost good faith. As such, it required Manuel to disclose to
Philam Plans conditions affecting the risk of which he was aware or material facts that he knew or ought to know.18
Issues Presented
The issues presented in this case are:

169
1. Whether or not the CA erred in finding Manuel guilty of concealing his illness when he kept blank and did
not answer questions in his pension plan application regarding the ailments he suffered from;
2. Whether or not the CA erred in holding that Manuel was bound by the failure of respondents Perla and
Ma. Celeste to declare the condition of Manuels health in the pension plan application; and
3. Whether or not the CA erred in finding that Philam Plans approval of Manuels pension plan application
and acceptance of his premium payments precluded it from denying Lourdes claim.
Rulings of the Court
One. Lourdes points out that, seeing the unfilled spaces in Manuels pension plan application relating to his medical
history, Philam Plans should have returned it to him for completion. Since Philam Plans chose to approve the
application just as it was, it cannot cry concealment on Manuels part. Further, Lourdes adds that Philam Plans
never queried Manuel directly regarding the state of his health. Consequently, it could not blame him for not
mentioning it.19
But Lourdes is shifting to Philam Plans the burden of putting on the pension plan application the true state of
Manuels health. She forgets that since Philam Plans waived medical examination for Manuel, it had to rely largely
on his stating the truth regarding his health in his application. For, after all, he knew more than anyone that he had
been under treatment for heart condition and diabetes for more than five years preceding his submission of that
application. But he kept those crucial facts from Philam Plans.
Besides, when Manuel signed the pension plan application, he adopted as his own the written representations and
declarations embodied in it. It is clear from these representations that he concealed his chronic heart ailment and
diabetes from Philam Plans. The pertinent portion of his representations and declarations read as follows:
I hereby represent and declare to the best of my knowledge that:
xxxx
(c) I have never been treated for heart condition, high blood pressure, cancer, diabetes, lung, kidney or
stomach disorder or any other physical impairment in the last five years.
(d) I am in good health and physical condition.
If your answer to any of the statements above reveal otherwise, please give details in the space provided for:
Date of confinement : ____________________________
Name of Hospital or Clinic : ____________________________
Name of Attending Physician : ____________________________
Findings : ____________________________
Others: (Please specify) : ____________________________
x x x x.20 (Emphasis supplied)

170
Since Manuel signed the application without filling in the details regarding his continuing treatments for heart
condition and diabetes, the assumption is that he has never been treated for the said illnesses in the last five years
preceding his application. This is implicit from the phrase "If your answer to any of the statements above
(specifically, the statement: I have never been treated for heart condition or diabetes) reveal otherwise, please give
details in the space provided for." But this is untrue since he had been on "Coumadin," a treatment for venous
thrombosis,21 and insulin, a drug used in the treatment of diabetes mellitus, at that time. 22
Lourdes insists that Manuel had concealed nothing since Perla, the soliciting agent, knew that Manuel had a
pacemaker implanted on his chest in the 70s or about 20 years before he signed up for the pension plan. 23 But by its
tenor, the responsibility for preparing the application belonged to Manuel. Nothing in it implies that someone else
may provide the information that Philam Plans needed. Manuel cannot sign the application and disown the
responsibility for having it filled up. If he furnished Perla the needed information and delegated to her the filling up of
the application, then she acted on his instruction, not on Philam Plans instruction.
Lourdes next points out that it made no difference if Manuel failed to reveal the fact that he had a pacemaker
implant in the early 70s since this did not fall within the five-year timeframe that the disclosure contemplated. 24But a
pacemaker is an electronic device implanted into the body and connected to the wall of the heart, designed to
provide regular, mild, electric shock that stimulates the contraction of the heart muscles and restores normalcy to
the heartbeat.25 That Manuel still had his pacemaker when he applied for a pension plan in October 1997 is an
admission that he remained under treatment for irregular heartbeat within five years preceding that application.
Besides, as already stated, Manuel had been taking medicine for his heart condition and diabetes when he
submitted his pension plan application. These clearly fell within the five-year period. More, even if Perlas knowledge
of Manuels pacemaker may be applied to Philam Plans under the theory of imputed knowledge, 26 it is not claimed
that Perla was aware of his two other afflictions that needed medical treatments. Pursuant to Section 27 27 of the
Insurance Code, Manuels concealment entitles Philam Plans to rescind its contract of insurance with him.
Two. Lourdes contends that the mere fact that Manuel signed the application in blank and let Perla fill in the required
details did not make her his agent and bind him to her concealment of his true state of health. Since there is no
evidence of collusion between them, Perlas fault must be considered solely her own and cannot prejudice Manuel. 28
But Manuel forgot that in signing the pension plan application, he certified that he wrote all the information stated in
it or had someone do it under his direction. Thus:
APPLICATION FOR PENSION PLAN
(Comprehensive)
I hereby apply to purchase from PHILAM PLANS, INC. a Pension Plan Program described herein in accordance
with the General Provisions set forth in this application and hereby certify that the date and other information stated
herein are written by me or under my direction. x x x. 29 (Emphasis supplied)
Assuming that it was Perla who filled up the application form, Manuel is still bound by what it contains since he
certified that he authorized her action. Philam Plans had every right to act on the faith of that certification.
Lourdes could not seek comfort from her claim that Perla had assured Manuel that the state of his health would not
hinder the approval of his application and that what is written on his application made no difference to the insurance
company. But, indubitably, Manuel was made aware when he signed the pension plan application that, in granting
the same, Philam Plans and Philam Life were acting on the truth of the representations contained in that application.
Thus:

171
DECLARATIONS AND REPRESENTATIONS
xxxx
I agree that the insurance coverage of this application is based on the truth of the foregoing representations and is
subject to the provisions of the Group Life Insurance Policy issued by THE PHILIPPINE AMERICAN LIFE
INSURANCE CO. to PHILAM PLANS, INC.30 (Emphasis supplied)
As the Court said in New Life Enterprises v. Court of Appeals: 31
It may be true that x x x insured persons may accept policies without reading them, and that this is not negligence
per se. But, this is not without any exception. It is and was incumbent upon petitioner Sy to read the insurance
contracts, and this can be reasonably expected of him considering that he has been a businessman since 1965 and
the contract concerns indemnity in case of loss in his money-making trade of which important consideration he
could not have been unaware as it was precisely the reason for his procuring the same. 32
The same may be said of Manuel, a civil engineer and manager of a construction company.33 He could be expected
to know that one must read every document, especially if it creates rights and obligations affecting him, before
signing the same. Manuel is not unschooled that the Court must come to his succor. It could reasonably be
expected that he would not trifle with something that would provide additional financial security to him and to his wife
in his twilight years.
Three. In a final attempt to defend her claim for benefits under Manuels pension plan, Lourdes points out that any
defect or insufficiency in the information provided by his pension plan application should be deemed waived after the
same has been approved, the policy has been issued, and the premiums have been collected. 34
The Court cannot agree. The comprehensive pension plan that Philam Plans issued contains a one-year
incontestability period. It states:
VIII. INCONTESTABILITY
After this Agreement has remained in force for one (1) year, we can no longer contest for health reasons any claim
for insurance under this Agreement, except for the reason that installment has not been paid (lapsed), or that you
are not insurable at the time you bought this pension program by reason of age. If this Agreement lapses but is
reinstated afterwards, the one (1) year contestability period shall start again on the date of approval of your request
for reinstatement.35
1wphi1

The above incontestability clause precludes the insurer from disowning liability under the policy it issued on the
ground of concealment or misrepresentation regarding the health of the insured after a year of its issuance.
Since Manuel died on the eleventh month following the issuance of his plan, 36 the one year incontestability period
has not yet set in. Consequently, Philam Plans was not barred from questioning Lourdes entitlement to the benefits
of her husbands pension plan.
WHEREFORE, the Court AFFIRMS in its entirety the decision of the Court of Appeals in CA-G.R. CV 87085 dated
December 18, 2007.
SO ORDERED.

172

173

CHAPTER VII
Republic of the Philippines
SUPREME COURT
Manila
EN BANC
G.R. No. L-20853

May 29, 1967

BONIFACIO BROS., INC., ET AL., plaintiffs-appellants,


vs.
ENRIQUE MORA, ET AL., defendants-appellees.
G. Magsaysay for plaintiffs-appellants.
Abad Santos and Pablo for defendant-appellee H. E. Reyes, Inc.
J. P. Santilla and A. D. Hidalgo, Jr. for other defendant-appellee.
CASTRO, J.:
This is an appeal from the decision of the Court of First Instance of Manila, Branch XV, in civil case 48823, affirming
the decision of the Municipal Court of Manila, declaring the H.S. Reyes, Inc. as having a better right than the
Bonifacio Bros., Inc. and the Ayala Auto Parts Company, appellants herein, to the proceeds of motor insurance
policy A-0615, in the sum of P2,002.73, issued by the State Bonding & Insurance Co. Inc., and directing payment of
the said amount to the H. Reyes, Inc.
Enrique Mora, owner of Oldsmobile sedan model 1956, bearing plate No. QC- mortgaged the same to the H.S.
Reyes, Inc., with the condition that the former would insure the automobile with the latter as beneficiary. The
automobile was thereafter insured on June 23, 1959 with the State Bonding & Insurance Co., Inc., and motor car
insurance policy A-0615 was issued to Enrique Mora, the pertinent provisions of which read:
1. The Company (referring to the State Bonding & Insurance Co., Inc.) will, subject to the Limits of Liability,
indemnify the Insured against loss of or damages to the Motor Vehicle and its accessories and spare parts
whilst thereon; (a) by accidental collision or overturning or collision or overturning consequent upon
mechanical breakdown or consequent upon wear and tear,
xxx

xxx

xxx

2. At its own option the Company may pay in cash the amount of the loss or damage or may repair,
reinstate, or replace the Motor Vehicle or any part thereof or its accessories or spare parts. The liability of
the Company shall not exceed the value of the parts whichever is the less. The Insured's estimate of value
stated in the schedule will be the maximum amount payable by the Company in respect of any claim for loss
or damage.
1wph1.t

174
xxx

xxx

xxx

4. The Insured may authorize the repair of the Motor Vehicle necessitated by damage for which the
Company may be liable under this Policy provided that: (a) The estimated cost of such repair does not
exceed the Authorized Repair Limit, (b) A detailed estimate of the cost is forwarded to the Company without
delay, subject to the condition that "Loss, if any is payable to H.S. Reyes, Inc.," by virtue of the fact that said
Oldsmobile sedan was mortgaged in favor of the said H.S. Reyes, Inc. and that under a clause in said
insurance policy, any loss was made payable to the H.S. Reyes, Inc. as Mortgagee;
xxx

xxx

xxx

During the effectivity of the insurance contract, the car met with an accident. The insurance company then assigned
the accident to the Bayne Adjustment Co. for investigation and appraisal of the damage. Enrique Mora, without the
knowledge and consent of the H.S. Reyes, Inc., authorized the Bonifacio Bros. Inc. to furnish the labor and
materials, some of which were supplied by the Ayala Auto Parts Co. For the cost of labor and materials, Enrique
Mora was billed at P2,102.73 through the H.H. Bayne Adjustment Co. The insurance company after claiming a
franchise in the amount of P100, drew a check in the amount of P2,002.73, as proceeds of the insurance policy,
payable to the order of Enrique Mora or H.S. Reyes,. Inc., and entrusted the check to the H.H. Bayne Adjustment
Co. for disposition and delivery to the proper party. In the meantime, the car was delivered to Enrique Mora without
the consent of the H.S. Reyes, Inc., and without payment to the Bonifacio Bros. Inc. and the Ayala Auto Parts Co. of
the cost of repairs and materials.
Upon the theory that the insurance proceeds should be paid directly to them, the Bonifacio Bros. Inc. and the Ayala
Auto Parts Co. filed on May 8, 1961 a complaint with the Municipal Court of Manila against Enrique Mora and the
State Bonding & Insurance Co., Inc. for the collection of the sum of P2,002.73 The insurance company filed its
answer with a counterclaim for interpleader, requiring the Bonifacio Bros. Inc. and the H.S. Reyes, Inc. to interplead
in order to determine who has better right to the insurance proceeds in question. Enrique Mora was declared in
default for failure to appear at the hearing, and evidence against him was received ex parte. However, the counsel
for the Bonifacio Bros. Inc., Ayala Auto Parts Co. and State Bonding & Insurance Co. Inc. submitted a stipulation of
facts, on the basis of which are Municipal Court rendered a decision declaring the H.S. Reyes, Inc. as having a
better right to the disputed amount and ordering State Bonding & Insurance Co. Inc. to pay to the H. S. Reyes, Inc.
the said sum of P2,002.73. From this decision, the appellants elevated the case to the Court of First Instance of
Manila which the stipulation of facts was reproduced. On October 19, 1962 the latter court rendered a decision,
affirming the decision of the Municipal Court. The Bonifacio Bros. Inc. and the Ayala Auto Parts Co. moved for
reconsideration of the decision, but the trial court denied the motion. Hence, this appeal.
The main issue raised is whether there is privity of contract between the Bonifacio Bros. Inc. and the Ayala Auto
Parts Co. on the one hand and the insurance company on the other. The appellants argue that the insurance
company and Enrique Mora are parties to the repair of the car as well as the towage thereof performed. The
authority for this assertion is to be found, it is alleged, in paragraph 4 of the insurance contract which provides that
"the insured may authorize the repair of the Motor Vehicle necessitated by damage for which the company may be
liable under the policy provided that (a) the estimated cost of such repair does not exceed the Authorized Repair
Limit, and (b) a detailed estimate of the cost is forwarded to the company without delay." It is stressed that the H.H.
Bayne Adjustment Company's recommendation of payment of the appellants' bill for materials and repairs for which
the latter drew a check for P2,002.73 indicates that Mora and the H.H. Bayne Adjustment Co. acted for and in
representation of the insurance company.
This argument is, in our view, beside the point, because from the undisputed facts and from the pleadings it will be
seen that the appellants' alleged cause of action rests exclusively upon the terms of the insurance contract. The
appellants seek to recover the insurance proceeds, and for this purpose, they rely upon paragraph 4 of the

175
insurance contract document executed by and between the State Bonding & Insurance Company, Inc. and Enrique
Mora. The appellants are not mentioned in the contract as parties thereto nor is there any clause or provision
thereof from which we can infer that there is an obligation on the part of the insurance company to pay the cost of
repairs directly to them. It is fundamental that contracts take effect only between the parties thereto, except in some
specific instances provided by law where the contract contains some stipulation in favor of a third person. 1 Such
stipulation is known as stipulation pour autrui or a provision in favor of a third person not a pay to the contract.
Under this doctrine, a third person is allowed to avail himself of a benefit granted to him by the terms of the contract,
provided that the contracting parties have clearly and deliberately conferred a favor upon such
person.2 Consequently, a third person not a party to the contract has no action against the parties thereto, and
cannot generally demand the enforcement of the same.3 The question of whether a third person has an enforcible
interest in a contract, must be settled by determining whether the contracting parties intended to tender him such an
interest by deliberately inserting terms in their agreement with the avowed purpose of conferring a favor upon such
third person. In this connection, this Court has laid down the rule that the fairest test to determine whether the
interest of a third person in a contract is a stipulation pour autrui or merely an incidental interest, is to rely upon the
intention of the parties as disclosed by their contract.4 In the instant case the insurance contract does not contain
any words or clauses to disclose an intent to give any benefit to any repairmen or materialmen in case of repair of
the car in question. The parties to the insurance contract omitted such stipulation, which is a circumstance that
supports the said conclusion. On the other hand, the "loss payable" clause of the insurance policy stipulates that
"Loss, if any, is payable to H.S. Reyes, Inc." indicating that it was only the H.S. Reyes, Inc. which they intended to
benefit.
We likewise observe from the brief of the State Bonding & Insurance Company that it has vehemently opposed the
assertion or pretension of the appellants that they are privy to the contract. If it were the intention of the insurance
company to make itself liable to the repair shop or materialmen, it could have easily inserted in the contract a
stipulation to that effect. To hold now that the original parties to the insurance contract intended to confer upon the
appellants the benefit claimed by them would require us to ignore the indespensable requisite that a stipulation pour
autrui must be clearly expressed by the parties, which we cannot do.
As regards paragraph 4 of the insurance contract, a perusal thereof would show that instead of establishing privity
between the appellants and the insurance company, such stipulation merely establishes the procedure that the
insured has to follow in order to be entitled to indemnity for repair. This paragraph therefore should not be construed
as bringing into existence in favor of the appellants a right of action against the insurance company as such
intention can never be inferred therefrom.
Another cogent reason for not recognizing a right of action by the appellants against the insurance company is that
"a policy of insurance is a distinct and independent contract between the insured and insurer, and third persons
have no right either in a court of equity, or in a court of law, to the proceeds of it, unless there be some contract of
trust, expressed or implied between the insured and third person." 5 In this case, no contract of trust, expressed or
implied exists. We, therefore, agree with the trial court that no cause of action exists in favor of the appellants in so
far as the proceeds of insurance are concerned. The appellants' claim, if at all, is merely equitable in nature and
must be made effective through Enrique Mora who entered into a contract with the Bonifacio Bros. Inc. This
conclusion is deducible not only from the principle governing the operation and effect of insurance contracts in
general, but is clearly covered by the express provisions of section 50 of the Insurance Act which read:
The insurance shall be applied exclusively to the proper interests of the person in whose name it is made
unless otherwise specified in the policy.
The policy in question has been so framed that "Loss, if any, is payable to H.S. Reyes, Inc.," which unmistakably
shows the intention of the parties.

176
The final contention of the appellants is that the right of the H.S. Reyes, Inc. to the insurance proceeds arises only if
there was loss and not where there is mere damage as in the instant case. Suffice it to say that any attempt to draw
a distinction between "loss" and "damage" is uncalled for, because the word "loss" in insurance law embraces injury
or damage.
Loss in insurance, defined. The injury or damage sustained by the insured in consequence of the
happening of one or more of the accidents or misfortune against which the insurer, in consideration of the
premium, has undertaken to indemnify the insured. (1 Bouv. Ins. No. 1215; Black's Law Dictionary;
Cyclopedic Law Dictionary, cited in Martin's Phil. Commercial Laws, Vol. 1, 1961 ed. p. 608).
Indeed, according to sec. 120 of the Insurance Act, a loss may be either total or partial.
Accordingly, the judgment appealed from is hereby affirmed, at appellants' cost.
Concepcion, C.J., Reyes, J.B.L., Dizon, Regala, Makalintal, Bengzon, J.P., Zaldivar, Sanchez and Castro, JJ.,
concur.

Republic of the Philippines


SUPREME COURT
Manila
FIRST DIVISION

177
G.R. No. L-44059 October 28, 1977
THE INSULAR LIFE ASSURANCE COMPANY, LTD., plaintiff-appellee,
vs.
CARPONIA T. EBRADO and PASCUALA VDA. DE EBRADO, defendants-appellants.

MARTIN, J.:
This is a novel question in insurance law: Can a common-law wife named as beneficiary in the life insurance policy
of a legally married man claim the proceeds thereof in case of death of the latter?
On September 1, 1968, Buenaventura Cristor Ebrado was issued by The Life Assurance Co., Ltd., Policy No.
009929 on a whole-life for P5,882.00 with a, rider for Accidental Death for the same amount Buenaventura C.
Ebrado designated T. Ebrado as the revocable beneficiary in his policy. He to her as his wife.
On October 21, 1969, Buenaventura C. Ebrado died as a result of an t when he was hit by a failing branch of a tree.
As the policy was in force, The Insular Life Assurance Co., Ltd. liable to pay the coverage in the total amount of
P11,745.73, representing the face value of the policy in the amount of P5,882.00 plus the additional benefits for
accidental death also in the amount of P5,882.00 and the refund of P18.00 paid for the premium due November,
1969, minus the unpaid premiums and interest thereon due for January and February, 1969, in the sum of P36.27.
Carponia T. Ebrado filed with the insurer a claim for the proceeds of the Policy as the designated beneficiary therein,
although she admits that she and the insured Buenaventura C. Ebrado were merely living as husband and wife
without the benefit of marriage.
Pascuala Vda. de Ebrado also filed her claim as the widow of the deceased insured. She asserts that she is the one
entitled to the insurance proceeds, not the common-law wife, Carponia T. Ebrado.
In doubt as to whom the insurance proceeds shall be paid, the insurer, The Insular Life Assurance Co., Ltd.
commenced an action for Interpleader before the Court of First Instance of Rizal on April 29, 1970.
After the issues have been joined, a pre-trial conference was held on July 8, 1972, after which, a pre-trial order was
entered reading as follows:
+.wph!1

During the pre-trial conference, the parties manifested to the court. that there is no possibility of
amicable settlement. Hence, the Court proceeded to have the parties submit their evidence for the
purpose of the pre-trial and make admissions for the purpose of pretrial. During this conference,
parties Carponia T. Ebrado and Pascuala Ebrado agreed and stipulated: 1) that the deceased
Buenaventura Ebrado was married to Pascuala Ebrado with whom she has six (legitimate)
namely; Hernando, Cresencio, Elsa, Erlinda, Felizardo and Helen, all surnamed Ebrado; 2) that
during the lifetime of the deceased, he was insured with Insular Life Assurance Co. Under Policy No.
009929 whole life plan, dated September 1, 1968 for the sum of P5,882.00 with the rider for
accidental death benefit as evidenced by Exhibits A for plaintiffs and Exhibit 1 for the defendant
Pascuala and Exhibit 7 for Carponia Ebrado; 3) that during the lifetime of Buenaventura Ebrado, he
was living with his common-wife, Carponia Ebrado, with whom she had 2 children although he was
not legally separated from his legal wife; 4) that Buenaventura in accident on October 21, 1969 as
evidenced by the death Exhibit 3 and affidavit of the police report of his death Exhibit 5; 5) that
complainant Carponia Ebrado filed claim with the Insular Life Assurance Co. which was contested by

178
Pascuala Ebrado who also filed claim for the proceeds of said policy 6) that in view ofthe adverse
claims the insurance company filed this action against the two herein claimants Carponia and
Pascuala Ebrado; 7) that there is now due from the Insular Life Assurance Co. as proceeds of the
policy P11,745.73; 8) that the beneficiary designated by the insured in the policy is Carponia Ebrado
and the insured made reservation to change the beneficiary but although the insured made the
option to change the beneficiary, same was never changed up to the time of his death and the wife
did not have any opportunity to write the company that there was reservation to change the
designation of the parties agreed that a decision be rendered based on and stipulation of facts as to
who among the two claimants is entitled to the policy.
Upon motion of the parties, they are given ten (10) days to file their simultaneous memoranda from
the receipt of this order.
SO ORDERED.
On September 25, 1972, the trial court rendered judgment declaring among others, Carponia T. Ebrado disqualified
from becoming beneficiary of the insured Buenaventura Cristor Ebrado and directing the payment of the insurance
proceeds to the estate of the deceased insured. The trial court held:
+.wph!1

It is patent from the last paragraph of Art. 739 of the Civil Code that a criminal conviction for adultery
or concubinage is not essential in order to establish the disqualification mentioned therein. Neither is
it also necessary that a finding of such guilt or commission of those acts be made in a separate
independent action brought for the purpose. The guilt of the donee (beneficiary) may be proved by
preponderance of evidence in the same proceeding (the action brought to declare the nullity of the
donation).
It is, however, essential that such adultery or concubinage exists at the time defendant Carponia T.
Ebrado was made beneficiary in the policy in question for the disqualification and incapacity to exist
and that it is only necessary that such fact be established by preponderance of evidence in the trial.
Since it is agreed in their stipulation above-quoted that the deceased insured and defendant
Carponia T. Ebrado were living together as husband and wife without being legally married and that
the marriage of the insured with the other defendant Pascuala Vda. de Ebrado was valid and still
existing at the time the insurance in question was purchased there is no question that defendant
Carponia T. Ebrado is disqualified from becoming the beneficiary of the policy in question and as
such she is not entitled to the proceeds of the insurance upon the death of the insured.
From this judgment, Carponia T. Ebrado appealed to the Court of Appeals, but on July 11, 1976, the Appellate Court
certified the case to Us as involving only questions of law.
We affirm the judgment of the lower court.
1. It is quite unfortunate that the Insurance Act (RA 2327, as amended) or even the new Insurance Code (PD No.
612, as amended) does not contain any specific provision grossly resolutory of the prime question at hand. Section
50 of the Insurance Act which provides that "(t)he insurance shag be applied exclusively to the proper interest of the
person in whose name it is made" 1 cannot be validly seized upon to hold that the mm includes the beneficiary. The word
"interest" highly suggests that the provision refers only to the "insured" and not to the beneficiary, since a contract of
insurance is personal in character. 2 Otherwise, the prohibitory laws against illicit relationships especially on property and
descent will be rendered nugatory, as the same could easily be circumvented by modes of insurance. Rather, the general
rules of civil law should be applied to resolve this void in the Insurance Law. Article 2011 of the New Civil Code states:
"The contract of insurance is governed by special laws. Matters not expressly provided for in such special laws shall be

179
regulated by this Code." When not otherwise specifically provided for by the Insurance Law, the contract of life insurance
is governed by the general rules of the civil law regulating contracts. 3 And under Article 2012 of the same Code, "any
person who is forbidden from receiving any donation under Article 739 cannot be named beneficiary of a fife insurance
policy by the person who cannot make a donation to him. 4 Common-law spouses are, definitely, barred from receiving
donations from each other. Article 739 of the new Civil Code provides:
+.wph!1

The following donations shall be void:


1. Those made between persons who were guilty of adultery or concubinage at the time of donation;
Those made between persons found guilty of the same criminal offense, in consideration thereof;
3. Those made to a public officer or his wife, descendants or ascendants by reason of his office.
In the case referred to in No. 1, the action for declaration of nullity may be brought by the spouse of
the donor or donee; and the guilt of the donee may be proved by preponderance of evidence in the
same action.
2. In essence, a life insurance policy is no different from a civil donation insofar as the beneficiary is concerned.
Both are founded upon the same consideration: liberality. A beneficiary is like a donee, because from the premiums
of the policy which the insured pays out of liberality, the beneficiary will receive the proceeds or profits of said
insurance. As a consequence, the proscription in Article 739 of the new Civil Code should equally operate in life
insurance contracts. The mandate of Article 2012 cannot be laid aside: any person who cannot receive a donation
cannot be named as beneficiary in the life insurance policy of the person who cannot make the donation. 5 Under
American law, a policy of life insurance is considered as a testament and in construing it, the courts will, so far as possible
treat it as a will and determine the effect of a clause designating the beneficiary by rules under which wins are
interpreted. 6
3. Policy considerations and dictates of morality rightly justify the institution of a barrier between common law
spouses in record to Property relations since such hip ultimately encroaches upon the nuptial and filial rights of the
legitimate family There is every reason to hold that the bar in donations between legitimate spouses and those
between illegitimate ones should be enforced in life insurance policies since the same are based on similar
consideration As above pointed out, a beneficiary in a fife insurance policy is no different from a donee. Both are
recipients of pure beneficence. So long as manage remains the threshold of family laws, reason and morality dictate
that the impediments imposed upon married couple should likewise be imposed upon extra-marital relationship. If
legitimate relationship is circumscribed by these legal disabilities, with more reason should an illicit relationship be
restricted by these disabilities. Thus, in Matabuena v. Cervantes, 7 this Court, through Justice Fernando, said:
+.wph!1

If the policy of the law is, in the language of the opinion of the then Justice J.B.L. Reyes of that court
(Court of Appeals), 'to prohibit donations in favor of the other consort and his descendants because
of and undue and improper pressure and influence upon the donor, a prejudice deeply rooted in our
ancient law;" por-que no se enganen desponjandose el uno al otro por amor que han de consuno'
(According to) the Partidas (Part IV, Tit. XI, LAW IV), reiterating the rationale 'No Mutuato amore
invicem spoliarentur' the Pandects (Bk, 24, Titl. 1, De donat, inter virum et uxorem); then there is
very reason to apply the same prohibitive policy to persons living together as husband and wife
without the benefit of nuptials. For it is not to be doubted that assent to such irregular connection for
thirty years bespeaks greater influence of one party over the other, so that the danger that the law
seeks to avoid is correspondingly increased. Moreover, as already pointed out by Ulpian (in his lib.
32 ad Sabinum, fr. 1), 'it would not be just that such donations should subsist, lest the condition 6f
those who incurred guilt should turn out to be better.' So long as marriage remains the cornerstone

180
of our family law, reason and morality alike demand that the disabilities attached to marriage should
likewise attach to concubinage.
It is hardly necessary to add that even in the absence of the above pronouncement, any other
conclusion cannot stand the test of scrutiny. It would be to indict the frame of the Civil Code for a
failure to apply a laudable rule to a situation which in its essentials cannot be distinguished.
Moreover, if it is at all to be differentiated the policy of the law which embodies a deeply rooted
notion of what is just and what is right would be nullified if such irregular relationship instead of being
visited with disabilities would be attended with benefits. Certainly a legal norm should not be
susceptible to such a reproach. If there is every any occasion where the principle of statutory
construction that what is within the spirit of the law is as much a part of it as what is written, this is it.
Otherwise the basic purpose discernible in such codal provision would not be attained. Whatever
omission may be apparent in an interpretation purely literal of the language used must be remedied
by an adherence to its avowed objective.
4. We do not think that a conviction for adultery or concubinage is exacted before the disabilities mentioned in Article
739 may effectuate. More specifically, with record to the disability on "persons who were guilty of adultery or
concubinage at the time of the donation," Article 739 itself provides:
+.wph!1

In the case referred to in No. 1, the action for declaration of nullity may be brought by the spouse of
the donor or donee; and the guilty of the donee may be proved by preponderance of evidence in the
same action.
The underscored clause neatly conveys that no criminal conviction for the offense is a condition precedent. In fact, it
cannot even be from the aforequoted provision that a prosecution is needed. On the contrary, the law plainly states
that the guilt of the party may be proved "in the same acting for declaration of nullity of donation. And, it would be
sufficient if evidence preponderates upon the guilt of the consort for the offense indicated. The quantum of proof in
criminal cases is not demanded.
In the caw before Us, the requisite proof of common-law relationship between the insured and the beneficiary has
been conveniently supplied by the stipulations between the parties in the pre-trial conference of the case. It case
agreed upon and stipulated therein that the deceased insured Buenaventura C. Ebrado was married to Pascuala
Ebrado with whom she has six legitimate children; that during his lifetime, the deceased insured was living with his
common-law wife, Carponia Ebrado, with whom he has two children. These stipulations are nothing less thanjudicial
admissions which, as a consequence, no longer require proof and cannot be contradicted. 8 A fortiori, on the basis of
these admissions, a judgment may be validly rendered without going through the rigors of a trial for the sole purpose of
proving the illicit liaison between the insured and the beneficiary. In fact, in that pretrial, the parties even agreed "that a
decision be rendered based on this agreement and stipulation of facts as to who among the two claimants is entitled to
the policy."
ACCORDINGLY, the appealed judgment of the lower court is hereby affirmed. Carponia T. Ebrado is hereby
declared disqualified to be the beneficiary of the late Buenaventura C. Ebrado in his life insurance policy. As a
consequence, the proceeds of the policy are hereby held payable to the estate of the deceased insured. Costs
against Carponia T. Ebrado.
SO ORDERED.

181

Republic of the Philippines


SUPREME COURT
Manila
EN BANC

G.R. No. L-28093 January 30, 1971


BASILIA BERDIN VDA. DE CONSUEGRA; JULIANA, PACITA, MARIA LOURDES, JOSE, JR., RODRIGO,
LINEDA and LUIS, all surnamed CONSUEGRA, petitioners-appellants,
vs.
GOVERNMENT SERVICE INSURANCE SYSTEM, COMMISSIONER OF PUBLIC HIGHWAYS, HIGHWAY

182
DISTRICT ENGINEER OF SURIGAO DEL NORTE, COMMISSIONER OF CIVIL SERVICE, and ROSARIO
DIAZ,respondents-appellees.
Bernardino O. Almeda for petitioners-appellants.
Binag and Arevalo, Jr. for respondent-appellee Government Service Insurance System.
Office of the Solicitor General for other respondents-appellees.

ZALDIVAR, J.:
Appeal on purely questions of law from the decision of the Court of First Instance of Surigao del Norte, dated March
7, 1967, in its Special Proceeding No. 1720.
The pertinent facts, culled from the stipulation of facts submitted by the parties, are the following:
The late Jose Consuegra, at the time of his death, was employed as a shop foreman of the office of the District
Engineer in the province of Surigao del Norte. In his lifetime, Consuegra contracted two marriages, the first with
herein respondent Rosario Diaz, solemnized in the parish church of San Nicolas de Tolentino, Surigao, Surigao, on
July 15, 1937, out of which marriage were born two children, namely, Jose Consuegra, Jr. and Pedro Consuegra,
but both predeceased their father; and the second, which was contracted in good faith while the first marriage was
subsisting, with herein petitioner Basilia Berdin, on May 1, 1957 in the same parish and municipality, out of which
marriage were born seven children, namely, Juliana, Pacita, Maria Lourdes, Jose, Rodrigo, Lenida and Luz, all
surnamed Consuegra.
Being a member of the Government Service Insurance System (GSIS, for short) when Consuegra died on
September 26, 1965, the proceeds of his life insurance under policy No. 601801 were paid by the GSIS to petitioner
Basilia Berdin and her children who were the beneficiaries named in the policy. Having been in the service of the
government for 22.5028 years, Consuegra was entitled to retirement insurance benefits in the sum of P6,304.47
pursuant to Section 12(c) of Commonwealth Act 186 as amended by Republic Acts 1616 and 3836. Consuegra did
not designate any beneficiary who would receive the retirement insurance benefits due to him. Respondent Rosario
Diaz, the widow by the first marriage, filed a claim with the GSIS asking that the retirement insurance benefits be
paid to her as the only legal heir of Consuegra, considering that the deceased did not designate any beneficiary with
respect to his retirement insurance benefits. Petitioner Basilia Berdin and her children, likewise, filed a similar claim
with the GSIS, asserting that being the beneficiaries named in the life insurance policy of Consuegra, they are the
only ones entitled to receive the retirement insurance benefits due the deceased Consuegra. Resolving the
conflicting claims, the GSIS ruled that the legal heirs of the late Jose Consuegra were Rosario Diaz, his widow by
his first marriage who is entitled to one-half, or 8/16, of the retirement insurance benefits, on the one hand; and
Basilia Berdin, his widow by the second marriage and their seven children, on the other hand, who are entitled to
the remaining one-half, or 8/16, each of them to receive an equal share of 1/16.
Dissatisfied with the foregoing ruling and apportionment made by the GSIS, Basilia Berdin and her children 1 filed on
October 10, 1966 a petition for mandamus with preliminary injunction in the Court of First Instance of Surigao, naming as
respondents the GSIS, the Commissioner of Public Highways, the Highway District Engineer of Surigao del Norte, the
Commissioner of Civil Service, and Rosario Diaz, praying that they (petitioners therein) be declared the legal heirs and
exclusive beneficiaries of the retirement insurance of the late Jose Consuegra, and that a writ of preliminary injunction be
issued restraining the implementation of the adjudication made by the GSIS. On October 26, 1966, the trial court issued
an order requiring therein respondents to file their respective answers, but refrained from issuing the writ of preliminary

183
injunction prayed for. On February 11, 1967, the parties submitted a stipulation of facts, prayed that the same be admitted
and approved and that judgment be rendered on the basis of the stipulation of facts. On March 7, 1967, the court below
rendered judgment, the pertinent portions of which are quoted hereunder:

This Court, in conformity with the foregoing stipulation of facts, likewise is in full accord with the
parties with respect to the authority cited by them in support of said stipulation and which is hereinbelow cited for purposes of this judgment, to wit:
"When two women innocently and in good faith are legally united in holy matrimony to the same
man, they and their children, born of said wedlock, will be regarded as legitimate children and each
family be entitled to one half of the estate. Lao & Lao vs. Dee Tim, 45 Phil. 739; Estrella vs. Laong
Masa, Inc., (CA) 39 OG 79; Pisalbon vs. Bejec, 74 Phil. 88.
WHEREFORE, in view of the above premises, this Court is of the opinion that the foregoing
stipulation of facts is in order and in accordance with law and the same is hereby approved.
Judgment, therefore, is hereby rendered declaring the petitioner Basilia Berdin Vda. de Consuegra
and her co-petitioners Juliana, Pacita, Maria Lourdes, Jose, Jr., Rodrigo, Lenida and Luis, all
surnamed Consuegra, beneficiary and entitled to one-half (1/2) of the retirement benefit in the
amount of Six Thousand Three Hundred Four Pesos and Fourty-Seven Centavos (P6,304.47) due to
the deceased Jose Consuegra from the Government Service Insurance System or the amount of
P3,152.235 to be divided equally among them in the proportional amount of 1/16 each. Likewise, the
respondent Rosario Diaz Vda. de Consuegra is hereby declared beneficiary and entitled to the other
half of the retirement benefit of the late Jose Consuegra or the amount of P3,152.235. The case with
respect to the Highway District Engineer of Surigao del Norte is hereby ordered dismissed.
Hence the present appeal by herein petitioners-appellants, Basilia Berdin and her children.
It is the contention of appellants that the lower court erred in not holding that the designated beneficiaries in the life
insurance of the late Jose Consuegra are also the exclusive beneficiaries in the retirement insurance of said
deceased. In other words, it is the submission of appellants that because the deceased Jose Consuegra failed to
designate the beneficiaries in his retirement insurance, the appellants who were the beneficiaries named in the life
insurance should automatically be considered the beneficiaries to receive the retirement insurance benefits, to the
exclusion of respondent Rosario Diaz. From the arguments adduced by appellants in their brief We gather that it is
their stand that the system of life insurance and the system of retirement insurance, that are provided for in
Commonwealth Act 186 as amended, are simply complementary to each other, or that one is a part or an extension
of the other, such that whoever is named the beneficiary in the life insurance is also the beneficiary in the retirement
insurance when no such beneficiary is named in the retirement insurance.
The contention of appellants is untenable.
It should be noted that the law creating the Government Service Insurance System is Commonwealth Act 186 which
was enacted by the National Assembly on November 14, 1936. As originally approved, Commonwealth Act 186
provided for the compulsory membership in the Government Service Insurance System of all regularly and
permanently appointed officials and employees of the government, considering as automatically insured on life all
such officials and employees, and issuing to them the corresponding membership policy under the terms and
conditions as provided in the Act.2
Originally, Commonwealth Act 186 provided for life insurance only. Commonwealth Act 186 was amended by
Republic Act 660 which was enacted by the Congress of the Philippines on June 16, 1951, and, among others, the
amendatory Act provided that aside from the system of life insurance under the Government Service Insurance

184
System there was also established the system of retirement insurance. Thus, We will note in Republic Act 660 that
there is a chapter on life insurance and another chapter on retirement insurance. 3 Under the chapter on life
insurance are sections 8, 9 and 10 of Commonwealth Act 186, as amended; and under the chapter on retirement
insurance are sections 11, 12, 13 and 13-A. On May 31, 1957, Republic Act 1616 was enacted by Congress,
amending section 12 of Commonwealth Act 186 as amended by Republic Act 660, by adding thereto two new
subsections, designated as subsections (b) and (c). This subsection (c) of section 12 of Commonwealth Act 186, as
amended by Republic Acts 660, 1616 and 3096, was again amended by Republic Act 3836 which was enacted on
June 22, 1963. The pertinent provisions of subsection (c) of Section 12 of Commonwealth Act 186, as thus
amended and reamended, read as follows:
lwph1.t

(c) Retirement is likewise allowed to a member, regardless of age, who has rendered at least twenty
years of service. The benefit shall, in addition to the return of his personal contributions plus interest
and the payment of the corresponding employer's premiums described in subsection (a) of Section 5
hereof, without interest, be only a gratuity equivalent to one month's salary for every year of service,
based on the highest rate received, but not to exceed twenty-four months; Provided, That the retiring
officer or employee has been in the service of the said employer or office for at least four years,
immediately preceding his retirement.
xxx xxx xxx
The gratuity is payable by the employer or office concerned which is hereby authorized to provide
the necessary appropriation to pay the same from any unexpended items of appropriations.
Elective or appointive officials and employees paid gratuity under this subsection shall be entitled to
the commutation of the unused vacation and sick leave, based on the highest rate received, which
they may have to their credit at the time of retirement.
Jose Consuegra died on September 26, 1965, and so at the time of his death he had acquired rights under the
above-quoted provisions of subsection (c) of Section 12 of Com. Act 186, as finally amended by Rep. Act 3836 on
June 22, 1963. When Consuegra died on September 26, 1965, he had to his credit 22.5028 years of service in the
government, and pursuant to the above-quoted provisions of subsection (c) of Section 12 of Com. Act 186, as
amended, on the basis of the highest rate of salary received by him which was P282.83 per month, he was entitled
to receive retirement insurance benefits in the amount of P6,304.47. This is the retirement benefits that are the
subject of dispute between the appellants, on the one hand, and the appellee Rosario Diaz, on the other, in the
present case. The question posed is: to whom should this retirement insurance benefits of Jose Consuegra be paid,
because he did not, or failed to, designate the beneficiary of his retirement insurance?
If Consuegra had 22.5028 years of service in the government when he died on September 26, 1965, it follows that
he started in the government service sometime during the early part of 1943, or before 1943. In 1943 Com. Act 186
was not yet amended, and the only benefits then provided for in said Com. Act 186 were those that proceed from a
life insurance. Upon entering the government service Consuegra became a compulsory member of the GSIS, being
automatically insured on his life, pursuant to the provisions of Com. Act 186 which was in force at the time. During
1943 the operation of the Government Service Insurance System was suspended because of the war, and the
operation was resumed sometime in 1946. When Consuegra designated his beneficiaries in his life insurance he
could not have intended those beneficiaries of his life insurance as also the beneficiaries of his retirement insurance
because the provisions on retirement insurance under the GSIS came about only when Com. Act 186 was amended
by Rep. Act 660 on June 16, 1951. Hence, it cannot be said that because herein appellants were designated
beneficiaries in Consuegra's life insurance they automatically became the beneficiaries also of his retirement
insurance. Rep. Act 660 added to Com. Act 186 provisions regarding retirement insurance, which are Sections 11,

185
12, and 13 of Com. Act 186, as amended. Subsection (b) of Section 11 of Com. Act 186, as amended by Rep. Act
660, provides as follows:
(b) Survivors benefit. Upon death before he becomes eligible for retirement, his beneficiaries as
recorded in the application for retirement annuity filed with the System shall be paid his own
premiums with interest of three per centum per annum, compounded monthly. If on his death he is
eligible for retirement, then the automatic retirement annuity or the annuity chosen by him previously
shall be paid accordingly.
The above-quoted provisions of subsection (b) of Section 11 of Commonwealth Act 186, as amended by Rep. Act
660, clearly indicate that there is need for the employee to file an application for retirement insurance benefits when
he becomes a member of the GSIS, and he should state in his application the beneficiary of his retirement
insurance. Hence, the beneficiary named in the life insurance does not automatically become the beneficiary in the
retirement insurance unless the same beneficiary in the life insurance is so designated in the application for
retirement insurance.
Section 24 of Commonwealth Act 186, as amended by Rep. Act 660, provides for a life insurance fund and for a
retirement insurance fund. There was no such provision in Com. Act 186 before it was amended by Rep. Act 660.
Thus, subsections (a) and (b) of Section 24 of Commonwealth Act 186, as amended by Rep. Act 660, partly read as
follows:
(a) Life insurance fund. This shall consist of all premiums for life insurance benefit and/or earnings
and savings therefrom. It shall meet death claims as they may arise or such equities as any member
may be entitled to, under the conditions of his policy, and shall maintain the required reserves to the
end of guaranteeing the fulfillment of the life insurance contracts issued by the System ...
(b) Retirement insurance fund. This shall consist of all contributions for retirement insurance
benefit and of earnings and savings therefrom. It shall meet annuity payments and establish the
required reserves to the end of guaranteeing the fulfillment of the contracts issued by the System. ...
Thus, We see that the GSIS offers two separate and distinct systems of benefits to its members one is the life
insurance and the other is the retirement insurance. These two distinct systems of benefits are paid out from two
distinct and separate funds that are maintained by the GSIS.
In the case of the proceeds of a life insurance, the same are paid to whoever is named the beneficiary in the life
insurance policy. As in the case of a life insurance provided for in the Insurance Act (Act 2427, as amended), the
beneficiary in a life insurance under the GSIS may not necessarily be a heir of the insured. The insured in a life
insurance may designate any person as beneficiary unless disqualified to be so under the provisions of the Civil
Code.4 And in the absence of any beneficiary named in the life insurance policy, the proceeds of the insurance will go to
the estate of the insured.
Retirement insurance is primarily intended for the benefit of the employee to provide for his old age, or incapacity,
after rendering service in the government for a required number of years. If the employee reaches the age of
retirement, he gets the retirement benefits even to the exclusion of the beneficiary or beneficiaries named in his
application for retirement insurance. The beneficiary of the retirement insurance can only claim the proceeds of the
retirement insurance if the employee dies before retirement. If the employee failed or overlooked to state the
beneficiary of his retirement insurance, the retirement benefits will accrue to his estate and will be given to his legal
heirs in accordance with law, as in the case of a life insurance if no beneficiary is named in the insurance policy.

186
It is Our view, therefore, that the respondent GSIS had correctly acted when it ruled that the proceeds of the
retirement insurance of the late Jose Consuegra should be divided equally between his first living wife Rosario Diaz,
on the one hand, and his second wife Basilia Berdin and his children by her, on the other; and the lower court did
not commit error when it confirmed the action of the GSIS, it being accepted as a fact that the second marriage of
Jose Consuegra to Basilia Berdin was contracted in good faith. The lower court has correctly applied the ruling of
this Court in the case of Lao, et al. vs. Dee Tim, et al., 45 Phil. 739 as cited in the stipulation of facts and in the
decision appealed from.5 In the recent case of Gomez vs. Lipana, L-23214, June 30, 1970, 6 this Court, in construing the
rights of two women who were married to the same man a situation more or less similar to the case of appellant Basilia
Berdin and appellee Rosario Diaz held "that since the defendant's first marriage has not been dissolved or declared
void the conjugal partnership established by that marriage has not ceased. Nor has the first wife lost or relinquished her
status as putative heir of her husband under the new Civil Code, entitled to share in his estate upon his death should she
survive him. Consequently, whether as conjugal partner in a still subsisting marriage or as such putative heir she has an
interest in the husband's share in the property here in dispute.... " And with respect to the right of the second wife, this
Court observed that although the second marriage can be presumed to be void ab initio as it was celebrated while the first
marriage was still subsisting, still there is need for judicial declaration of such nullity. And inasmuch as the conjugal
partnership formed by the second marriage was dissolved before judicial declaration of its nullity, "[t]he only lust and
equitable solution in this case would be to recognize the right of the second wife to her share of one-half in the property
acquired by her and her husband and consider the other half as pertaining to the conjugal partnership of the first
marriage."
WHEREFORE, the decision appealed from is affirmed, with costs against petitioners-appellants. It is so ordered.

Republic of the Philippines


SUPREME COURT
Baguio City
FIRST DIVISION
G.R. No. 171406

April 4, 2011

ASIAN TERMINALS, INC., Petitioner,


vs.
MALAYAN INSURANCE, CO., INC., Respondent.
DECISION
DEL CASTILLO, J.:
Once the insurer pays the insured, equity demands reimbursement as no one should benefit at the expense of
another.
This Petition for Review on Certiorari1 under Rule 45 of the Rules of Court assails the July 14, 2005 Decision 2and
the February 14, 2006 Resolution3 of the Court of Appeals (CA) in CA G.R. CV No. 61798.
Factual Antecedents
On November 14, 1995, Shandong Weifang Soda Ash Plant shipped on board the vessel MV "Jinlian I" 60,000
plastic bags of soda ash dense (each bag weighing 50 kilograms) from China to Manila. 4 The shipment, with an

187
invoice value of US$456,000.00, was insured with respondent Malayan Insurance Company, Inc. under Marine Risk
Note No. RN-0001-21430, and covered by a Bill of Lading issued by Tianjin Navigation Company with Philippine
Banking Corporation as the consignee and Chemphil Albright and Wilson Corporation as the notify party.5
On November 21, 1995, upon arrival of the vessel at Pier 9, South Harbor, Manila, 6 the stevedores of petitioner
Asian Terminals, Inc., a duly registered domestic corporation engaged in providing arrastre and stevedoring
services,7 unloaded the 60,000 bags of soda ash dense from the vessel and brought them to the open storage area
of petitioner for temporary storage and safekeeping, pending clearance from the Bureau of Customs and delivery to
the consignee.8 When the unloading of the bags was completed on November 28, 1995, 2,702 bags were found to
be in bad order condition.9
On November 29, 1995, the stevedores of petitioner began loading the bags in the trucks of MEC Customs
Brokerage for transport and delivery to the consignee.10 On December 28, 1995, after all the bags were unloaded in
the warehouses of the consignee, a total of 2,881 bags were in bad order condition due to spillage, caking, and
hardening of the contents.11
On April 19, 1996, respondent, as insurer, paid the value of the lost/ damaged cargoes to the consignee in the
amount of P643,600.25.12
Ruling of the Regional Trial Court
On November 20, 1996, respondent, as subrogee of the consignee, filed before the Regional Trial Court (RTC) of
Manila, Branch 35, a Complaint13 for damages against petitioner, the shipper Inchcape Shipping Services, and the
cargo broker MEC Customs Brokerage.14
After the filing of the Answers,15 trial ensued.
On June 26, 1998, the RTC rendered a Decision16 finding petitioner liable for the damage/loss sustained by the
shipment but absolving the other defendants. The RTC found that the proximate cause of the damage/loss was the
negligence of petitioners stevedores who handled the unloading of the cargoes from the vessel. 17 The RTC
emphasized that despite the admonitions of Marine Cargo Surveyors Edgar Liceralde and Redentor Antonio not to
use steel hooks in retrieving and picking-up the bags, petitioners stevedores continued to use such tools, which
pierced the bags and caused the spillage.18 The RTC, thus, ruled that petitioner, as employer, is liable for the acts
and omissions of its stevedores under Articles 217619 and 2180 paragraph (4)20 of the Civil Code.21Hence, the
dispositive portion of the Decision reads:
WHEREFORE, judgment is rendered ordering defendant Asian Terminal, Inc. to pay plaintiff Malayan Insurance
Company, Inc. the sum of P643,600.25 plus interest thereon at legal rate computed from November 20, 1996, the
date the Complaint was filed, until the principal obligation is fully paid, and the costs.
The complaint of the plaintiff against defendants Inchcape Shipping Services and MEC Customs Brokerage, and the
counterclaims of said defendants against the plaintiff are dismissed.
SO ORDERED.22
Ruling of the Court of Appeals
Aggrieved, petitioner appealed23 to the CA but the appeal was denied. In its July 14, 2005 Decision, the CA agreed
with the RTC that the damage/loss was caused by the negligence of petitioners stevedores in handling and storing
the subject shipment.24 The CA likewise rejected petitioners assertion that it received the subject shipment in bad

188
order condition as this was belied by Marine Cargo Surveyors Redentor Antonio and Edgar Liceralde, who both
testified that the actual counting of bad order bags was done only after all the bags were unloaded from the vessel
and that the Turn Over Survey of Bad Order Cargoes (TOSBOC) upon which petitioner anchors its defense was
prepared only on November 28, 1995 or after the unloading of the bags was completed. 25 Thus, the CA disposed of
the appeal as follows:
WHEREFORE, premises considered, the appeal is DENIED. The assailed Decision dated June 26, 1998 of the
Regional Trial Court of Manila, Branch 35, in Civil Case No. 96-80945 is hereby AFFIRMED in all respects.
SO ORDERED.26
Petitioner moved for reconsideration27 but the CA denied the same in a Resolution28 dated February 14, 2006 for lack
of merit.
Issues
Hence, the present recourse, petitioner contending that:
1. RESPONDENT-INSURER IS NOT ENTITLED TO THE RELIEF GRANTED AS IT FAILED TO
ESTABLISH ITS CAUSE OF ACTION AGAINST HEREIN PETITIONER SINCE, AS THE ALLEGED
SUBROGEE, IT NEVER PRESENTED ANY VALID, EXISTING, ENFORCEABLE INSURANCE POLICY OR
ANY COPY THEREOF IN COURT.
2. THE HONORABLE COURT OF APPEALS ERRED WHEN IT OVERLOOKED THE FACT THAT THE
TOSBOC & RESBOC WERE ADOPTED AS COMMON EXHIBITS BY BOTH PETITIONER AND
RESPONDENT.
3. CONTRARY TO TESTIMONIAL EVIDENCE ON RECORD, VARIOUS DOCUMENTATIONS WOULD
POINT TO THE VESSELS LIABILITY AS THERE IS, IN THIS INSTANT CASE, AN OVERWHELMING
DOCUMENTARY EVIDENCE TO PROVE THAT THE DAMAGE IN QUESTION WERE SUSTAINED WHEN
THE SHIPMENT WAS IN THE CUSTODY OF THE VESSEL.
4. THE HONORABLE COURT OF APPEALS ERRED WHEN IT ADJUDGED HEREIN DEFENDANT LIABLE
DUE TO [THE] FACT THAT THE TURN OVER SURVEY OF BAD ORDER CARGOES (TOSBOC) WAS
PREPARED ONLY AFTER THE COMPLETION OF THE DISCHARGING OPERATIONS OR ON
NOVEMBER 28, 1995. THUS, CONCLUDING THAT DAMAGE TO THE CARGOES WAS DUE TO THE
IMPROPER HANDLING THEREOF BY ATI STEVEDORES.
5. THE HONORABLE COURT OF APPEALS ERRED IN NOT TAKING JUDICIAL NOTICE OF THE
CONTRACT FOR CARGO HANDLING SERVICES BETWEEN PPA AND ATI AND APPLYING THE
PERTINENT PROVISIONS THEREOF AS REGARDS ATIS LIABILITY.29
In sum, the issues are: (1) whether the non-presentation of the insurance contract or policy is fatal to
respondents cause of action; (2) whether the proximate cause of the damage/loss to the shipment was the
negligence of petitioners stevedores; and (3) whether the court can take judicial notice of the Management
Contract between petitioner and the Philippine Ports Authority (PPA) in determining petitioners liability.
Petitioners Arguments

189
Petitioner contends that respondent has no cause of action because it failed to present the insurance contract or
policy covering the subject shipment.30 Petitioner argues that the Subrogation Receipt presented by respondent is
not sufficient to prove that the subject shipment was insured and that respondent was validly subrogated to the
rights of the consignee.31 Thus, petitioner submits that without proof of a valid subrogation, respondent is not entitled
to any reimbursement.32
Petitioner likewise puts in issue the finding of the RTC, which was affirmed by the CA, that the proximate cause of
the damage/loss to the shipment was the negligence of petitioners stevedores. 33 Petitioner avers that such finding is
contrary to the documentary evidence, i.e., the TOSBOC, the Request for Bad Order Survey (RESBOC) and the
Report of Survey.34 According to petitioner, these documents prove that it received the subject shipment in bad order
condition and that no additional damage was sustained by the subject shipment under its custody.35Petitioner
asserts that although the TOSBOC was prepared only after all the bags were unloaded by petitioners stevedores,
this does not mean that the damage/loss was caused by its stevedores. 36
Petitioner also claims that the amount of damages should not be more than P5,000.00, pursuant to its Management
Contract for cargo handling services with the PPA.37 Petitioner contends that the CA should have taken judicial
notice of the said contract since it is an official act of an executive department subject to judicial cognizance. 38
Respondents Arguments
Respondent, on the other hand, argues that the non-presentation of the insurance contract or policy was not raised
in the trial court. Thus, it cannot be raised for the first time on appeal.39 Respondent likewise contends that under
prevailing jurisprudence, presentation of the insurance policy is not indispensable. 40 Moreover, with or without the
insurance contract or policy, respondent claims that it should be allowed to recover under Article 1236 41 of the Civil
Code.42 Respondent further avers that "the right of subrogation has its roots in equity - it is designed to promote and
to accomplish justice and is the mode which equity adopts to compel the ultimate payment of a debt by one who in
justice, equity and good conscience ought to pay."43
Respondent likewise maintains that the RTC and the CA correctly found that the damage/loss sustained by the
subject shipment was caused by the negligent acts of petitioners stevedores. 44 Such factual findings of the RTC,
affirmed by the CA, are conclusive and should no longer be disturbed. 45 In fact, under Section 146 of Rule 45 of the
Rules of Court, only questions of law may be raised in a petition for review on certiorari. 47
As to the Management Contract for cargo handling services, respondent contends that this is outside the operation
of judicial notice.48 And even if it is not, petitioners liability cannot be limited by it since it is a contract of adhesion. 49
Our Ruling
The petition is bereft of merit.
Non-presentation of the insurance contract or policy is not fatal in the instant case
Petitioner claims that respondents non-presentation of the insurance contract or policy between the respondent and
the consignee is fatal to its cause of action.
We do not agree.
First of all, this was never raised as an issue before the RTC. In fact, it is not among the issues agreed upon by the
parties to be resolved during the pre-trial.50 As we have said, "the determination of issues during the pre-trial
conference bars the consideration of other questions, whether during trial or on appeal." 51 Thus, "[t]he parties must

190
disclose during pre-trial all issues they intend to raise during the trial, except those involving privileged or
impeaching matters. x x x The basis of the rule is simple. Petitioners are bound by the delimitation of the issues
during the pre-trial because they themselves agreed to the same." 52
Neither was this issue raised on appeal.53 Basic is the rule that "issues or grounds not raised below cannot be
resolved on review by the Supreme Court, for to allow the parties to raise new issues is antithetical to the sporting
idea of fair play, justice and due process."54
Besides, non-presentation of the insurance contract or policy is not
necessarily fatal.55 In Delsan Transport Lines, Inc. v. Court of Appeals,56 we ruled that:
Anent the second issue, it is our view and so hold that the presentation in evidence of the marine insurance policy is
not indispensable in this case before the insurer may recover from the common carrier the insured value of the lost
cargo in the exercise of its subrogatory right. The subrogation receipt, by itself, is sufficient to establish not only the
relationship of herein private respondent as insurer and Caltex, as the assured shipper of the lost cargo of industrial
fuel oil, but also the amount paid to settle the insurance claim. The right of subrogation accrues simply upon
payment by the insurance company of the insurance claim.
The presentation of the insurance policy was necessary in the case of Home Insurance Corporation v. CA (a case
cited by petitioner) because the shipment therein (hydraulic engines) passed through several stages with different
parties involved in each stage. First, from the shipper to the port of departure; second, from the port of departure to
the M/S Oriental Statesman; third, from the M/S Oriental Statesman to the M/S Pacific Conveyor; fourth, from the
M/S Pacific Conveyor to the port of arrival; fifth, from the port of arrival to the arrastre operator; sixth, from the
arrastre operator to the hauler, Mabuhay Brokerage Co., Inc. (private respondent therein); and lastly, from the hauler
to the consignee. We emphasized in that case that in the absence of proof of stipulations to the contrary, the hauler
can be liable only for any damage that occurred from the time it received the cargo until it finally delivered it to the
consignee. Ordinarily, it cannot be held responsible for the handling of the cargo before it actually received it. The
insurance contract, which was not presented in evidence in that case would have indicated the scope of the
insurers liability, if any, since no evidence was adduced indicating at what stage in the handling process the
damage to the cargo was sustained.57 (Emphasis supplied.)
In International Container Terminal Services, Inc. v. FGU Insurance Corporation, 58 we used the same line of
reasoning in upholding the Decision of the CA finding the arrastre contractor liable for the lost shipment despite the
failure of the insurance company to offer in evidence the insurance contract or policy. We explained:
Indeed, jurisprudence has it that the marine insurance policy needs to be presented in evidence before the trial
court or even belatedly before the appellate court. In Malayan Insurance Co., Inc. v. Regis Brokerage Corp., the
Court stated that the presentation of the marine insurance policy was necessary, as the issues raised therein arose
from the very existence of an insurance contract between Malayan Insurance and its consignee, ABB Koppel, even
prior to the loss of the shipment. In Wallem Philippines Shipping, Inc. v. Prudential Guarantee and Assurance, Inc.,
the Court ruled that the insurance contract must be presented in evidence in order to determine the extent of the
coverage. This was also the ruling of the Court in Home Insurance Corporation v. Court of Appeals.
However, as in every general rule, there are admitted exceptions. In Delsan Transport Lines, Inc. v. Court of
Appeals, the Court stated that the presentation of the insurance policy was not fatal because the loss of the cargo
undoubtedly occurred while on board the petitioners vessel, unlike in Home Insurance in which the cargo passed
through several stages with different parties and it could not be determined when the damage to the cargo occurred,
such that the insurer should be liable for it.

191
As in Delsan, there is no doubt that the loss of the cargo in the present case occurred while in petitioners custody.
Moreover, there is no issue as regards the provisions of Marine Open Policy No. MOP-12763, such that the
presentation of the contract itself is necessary for perusal, not to mention that its existence was already admitted by
petitioner in open court. And even though it was not offered in evidence, it still can be considered by the court as
long as they have been properly identified by testimony duly recorded and they have themselves been incorporated
in the records of the case.59
Similarly, in this case, the presentation of the insurance contract or policy was not necessary. Although petitioner
objected to the admission of the Subrogation Receipt in its Comment to respondents formal offer of evidence on the
ground that respondent failed to present the insurance contract or policy,60 a perusal of petitioners Answer61and PreTrial Brief62 shows that petitioner never questioned respondents right to subrogation, nor did it dispute the coverage
of the insurance contract or policy. Since there was no issue regarding the validity of the insurance contract or
policy, or any provision thereof, respondent had no reason to present the insurance contract or policy as evidence
during the trial.
Factual findings of the CA, affirming the RTC, are conclusive and binding
Petitioners attempt to absolve itself from liability must likewise fail.
Only questions of law are allowed in petitions for review on certiorari under Rule 45 of the Rules of Court. Thus, it is
not our duty "to review, examine, and evaluate or weigh all over again the probative value of the evidence
presented,"63 especially where the findings of both the trial court and the appellate court coincide on the matter.64 As
we have often said, factual findings of the CA affirming those of the RTC are conclusive and binding, except in the
following cases: "(1) when the inference made is manifestly mistaken, absurd or impossible; (2) when there is grave
abuse of discretion; (3) when the findings are grounded entirely on speculations, surmises or conjectures; (4) when
the judgment of the [CA] is based on misapprehension of facts; (5) when the [CA], in making its findings, went
beyond the issues of the case and the same is contrary to the admissions of both appellant and appellee; (6) when
the findings of fact are conclusions without citation of specific evidence on which they are based; (7) when the [CA]
manifestly overlooked certain relevant facts not disputed by the parties and which, if properly considered, would
justify a different conclusion; and (8) when the findings of fact of the [CA] are premised on the absence of evidence
and are contradicted by the evidence on record."65 None of these are availing in the present case.
Both the RTC and the CA found the negligence of petitioners stevedores to be the proximate cause of the
damage/loss to the shipment. In disregarding the contention of petitioner that such finding is contrary to the
documentary evidence, the CA had this to say:
ATI, however, contends that the finding of the trial court was contrary to the documentary evidence of record,
particularly, the Turn Over Survey of Bad Order Cargoes dated November 28, 1995, which was executed prior to the
turn-over of the cargo by the carrier to the arrastre operator ATI, and which showed that the shipment already
contained 2,702 damaged bags.
We are not persuaded.
Contrary to ATIs assertion, witness Redentor Antonio, marine cargo surveyor of Inchcape for the vessel Jinlian I
which arrived on November 21, 1995 and up to completion of discharging on November 28, 1995, testified thatit
was only after all the bags were unloaded from the vessel that the actual counting of bad order bags was
made, thus:
xxxx

192
The above testimony of Redentor Antonio was corroborated by Edgar Liceralde, marine cargo surveyor
connected with SMS Average Surveyors and Adjusters, Inc., the company requested by consignee Chemphil
Albright and Wilson Corporation to provide superintendence, report the condition and determine the final outturn of
quantity/weight of the subject shipment. x x x
xxxx
Defendant-appellant ATI, for its part, presented its claim officer as witness who testified that a survey was conducted
by the shipping company and ATI before the shipment was turned over to the possession of ATI and that the Turn
Over Survey of Bad Order Cargoes was prepared by ATIs Bad Order (BO) Inspector.
Considering that the shipment arrived on November 21, 1998 and the unloading operation commenced on
said date and was completed on November 26, 1998, while the Turn Over Survey of Bad Order Cargoes,
reflecting a figure of 2,702 damaged bags, was prepared and signed on November 28, 1998 by ATIs BO
Inspector and co-signed by a representative of the shipping company, the trial courts finding that the damage to
the cargoes was due to the improper handling thereof by ATIs stevedores cannot be said to be without
substantial support from the records.
We thus see no cogent reason to depart from the ruling of the trial court that ATI should be made liable for the 2,702
bags of damaged shipment. Needless to state, it is hornbook doctrine that the assessment of witnesses and their
testimonies is a matter best undertaken by the trial court, which had the opportunity to observe the demeanor,
conduct or attitude of the witnesses. The findings of the trial court on this point are accorded great respect and will
not be reversed on appeal, unless it overlooked substantial facts and circumstances which, if considered, would
materially affect the result of the case.
We also find ATI liable for the additional 179 damaged bags discovered upon delivery of the shipment at the
consignees warehouse in Pasig. The final Report of Survey executed by SMS Average Surveyors & Adjusters, Inc.,
and independent surveyor hired by the consignee, shows that the subject shipment incurred a total of 2881
damaged bags.
The Report states that the withdrawal and delivery of the shipment took about ninety-five (95) trips from November
29, 1995 to December 28, 1995 and it was upon completion of the delivery to consignees warehouse where the
final count of 2881 damaged bags was made. The damage consisted of torn/bad order condition of the bags due to
spillages and caked/hardened portions.
We agree with the trial court that the damage to the shipment was caused by the negligence of ATIs stevedores and
for which ATI is liable under Articles 2180 and 2176 of the Civil Code. The proximate cause of the damage (i.e., torn
bags, spillage of contents and caked/hardened portions of the contents) was the improper handling of the cargoes
by ATIs stevedores, x x x
xxxx
ATI has not satisfactorily rebutted plaintiff-appellees evidence on the negligence of ATIs stevedores in the handling
and safekeeping of the cargoes. x x x
xxxx
We find no reason to disagree with the trial courts conclusion. Indeed, from the nature of the [damage] caused to
the shipment, i.e., torn bags, spillage of contents and hardened or caked portions of the contents, it is not difficult to
see that the damage caused was due to the negligence of ATIs stevedores who used steel hooks to retrieve the

193
bags from the higher portions of the piles thereby piercing the bags and spilling their contents, and who piled the
bags in the open storage area of ATI with insufficient cover thereby exposing them to the elements and [causing] the
contents to cake or harden.66
Clearly, the finding of negligence on the part of petitioners stevedores is supported by both testimonial and
documentary evidence. Hence, we see no reason to disturb the same.
Judicial notice does not apply
Finally, petitioner implores us to take judicial notice of Section 7.01, 67 Article VII of the Management Contract for
cargo handling services it entered with the PPA, which limits petitioners liability to P5,000.00 per package.
Unfortunately for the petitioner, it cannot avail of judicial notice.
Sections 1 and 2 of Rule 129 of the Rules of Court provide that:
SECTION 1. Judicial notice, when mandatory. A court shall take judicial notice, without the introduction of
evidence, of the existence and territorial extent of states, their political history, forms of government and symbols of
nationality, the law of nations, the admiralty and maritime courts of the world and their seals, the political constitution
and history of the Philippines, the official acts of the legislative, executive and judicial departments of the
Philippines, the laws of nature, the measure of time, and the geographical divisions.
1avvphi1

SEC. 2. Judicial notice, when discretionary. A court may take judicial notice of matters which are of public
knowledge, or are capable of unquestionable demonstration or ought to be known to judges because of their judicial
functions.
The Management Contract entered into by petitioner and the PPA is clearly not among the matters which the courts
can take judicial notice of. It cannot be considered an official act of the executive department. The PPA, which was
created by virtue of Presidential Decree No. 857, as amended, 68 is a government-owned and controlled corporation
in charge of administering the ports in the country.69 Obviously, the PPA was only performing a proprietary function
when it entered into a Management Contract with petitioner. As such, judicial notice cannot be applied.
WHEREFORE, the petition is hereby DENIED. The assailed July 14, 2005 Decision and the February 14, 2006
Resolution of the Court of Appeals in CA-G.R. CV No. 61798 are hereby AFFIRMED.
SO ORDERED.

You might also like